You are on page 1of 255

A Complete Book of Logical Reasoning

Join channel @cetexamgroup

p
ou
gr
m
xa
te
ce
@

1
Join channel @cetexamgroup
Adda247 Publications For any detail, mail us at
Publications@adda247.com
Join channelA Complete Book of Logical
@cetexamgroup
Reasoning

01 Statement and Conclusion


Introduction: Dictionary meaning of conclusion is “a proposition concluded or inferred from
the premises of an argument.”

p
Conclusions rely on the facts of a situation to make a determination that is not implicitly stated
or implied by the information. Essentially, a conclusion is the next logical step in an information
series. A statement requires two conditions to serve as a conclusion. First, it must be a logically

ou
derived statement from the available information. Second, it must not be stated or inferred from
the available information.
For instance, if you know that Kareena’s current purse looks discolored and damaged, and she
has enough money to buy a new purse, and that she is in the purse aisle of a store, you can
conclude that she will buy a new purse.

gr
Process of drawing a conclusion

Collects Evaluates
information m Information

Evaluates Logical Draws Conclusions


xa
Conclusions (Using Logic)

1. Getting facts: This step includes asking question like “who”, “how”, “when”, “what”,
“where”.
2. Evaluating the facts: This steps include analyzing questions like Is this fact relevant or
significant?, Does it support the conclusion?,
te

3. Drawing a conclusion: To avoid error ask questions like “Is the conclusion valid or
consistent with the given information” and “Are there any logical flaws in the conclusion?”.
4. Evaluating a conclusion: This step include asking question “Is the conclusion drawn is fair
and logical?”.
ce

In this type of questions, One statement is given which is followed by two or more
conclusions. The Candidates are required to find out which of the conclusion, logically follow
from the given statement.
Examples:
1. Statement : Now-a-days, the sale of television sets of company X has increased.
@

Conclusion: I. The sale of television sets of other companies has decreased.


II. The sale of television sets of company X was nil in the past.
Explanations: In the given statement, nothing is given about the sale of television sets of
other companies. Hence, conclusion I is not valid while the second conclusion is not related
to the statement. hence, it is also not valid.

2 Adda247 Publications For any detail, mail us at


Publications@adda247.com
A Complete Book of Logical Reasoning

2. Statement: If you are skillful IT engineer, we want to engage you in our organization - an
advertisement of company X.
Conclusions: I. The company X thinks that an engineer is a good worker.
II. The company X is in need of engineers.
Explanations: Both the conclusion are valid because the company thinks that an engineer is
a good worker and also also in need of engineers, because it is given in advertisement.

p
ou
Points to Remember
(1) Consider Only the matter which is given in the statement. Do not add anything in the statement from your
side.
(2) You should avoid the presumption and it should be minded that the conclusion may not be converted into a

gr
course of action.
(3) Generally the past statement is not valid.
(4) If some law or any correction is talked about in the statement then things related to it will be taken as
conclusion because the idea of making a law or correction is that people will follow it. But mind it that
conclusion should directly be connected to the statement.
m
(5) If in conclusion the words, like : DEFINITELY, QUICKLY, CENT-PERCENT, ONLY ONE, ONLY FOREVER,
ALL, ALWAYS, EVERY etc. , are linked then these are not considered. But if the conclusion is the direct result
of the statement, then it is considered.
xa
Exercise

1. Study the following information in which (b) As the climate gets warmer and
a statement is followed by five frequency of rains reduces, such
conclusions, read carefully and answer spurts in coarse particles making
te

the question below- breathing difficult will become a


Other than being an essential source of new normal and the government is
not waking up to the alarm.
water for Indian agriculture, the
(c) Both the periodicity and duration of
monsoon plays a critical role in flushing dry spells in the country were rising
ce

out pollutants over Asia. However, as total rainfall events in a year had
recently increased pollution — fallen even though the average
particularly from coal burning — could rainfall in a year has not changed
potentially weaken this ability of the much, a direct consequence of
monsoon. climate change.
Which of the following can be deduced (d) The annual average rainfall has
@

from the given statement? remained the same because the


(a) The unusually high concentration of frequency of heavy downpours
particulate matter in the last few increased in the past two decades.
days in India clearly shows that air (e) The air quality in the region
deteriorated because of dust storms
pollution is emerging as a big in western India, particularly
problem. Rajasthan.
3 Adda247 Publications For any detail, mail us at
Publications@adda247.com
A Complete Book of Logical Reasoning

2. In the questions given below, a effects of rising temperature are waiting


statement is given, followed by two for some far-flung future. They’re
conclusions. You have to read the happening right now. Signs are
statement and decide which of the given appearing all over, and some of them are
conclusions can be drawn from the given surprising. The extreme weather events,
statement. the historic drought conditions over
Statement: More houses in city X have large parts of the world, the fast melting

p
lifts in them as compared to the houses of Greenland’s ice surface and the
in city Y. intensity of Hurricane Sandy are
I. People of city X are too lazy. examples of the changes to global

ou
II. People of city X are richer than the weather patterns that can be expected
people of city Y. from an overall rise in Earth’s surface
(a) Only I (b) Only II temperature.
(c) Both I and II (d) Either I or II Which of the following conclusions can
(e) None of these be drawn from the above passage?
(a) The effects of global warming are
3. Statement: Start to think of travelling by

gr
very slow.
train for a holiday. A train journey can (b) Extreme climatic condition on earth
give one a better view of places on the is the ill-effect of global warming.
way which an air journey cannot give. (c) Man-made reasons are responsible
You can walk around whenever you for ill-effect of global warming.
want, meet other travelers and locals,
relax and watch the landscape go
m (d) Global warming is a phenomenon
which can’t be stopped.
peacefully by. Train travel is both a (e) Developed countries of the world
beautiful and affordable way to see the should contribute to reducing
country, and no trip is more scenic than global warming in the world.
xa
the trip by trains. 5. Statement:
Conclusions: I. Persons of modest means try to
I. While going for a holiday, people have a house of their own.
want to enjoy the view of the places II. Since buying or constructing a
on the way. house is an expensive affair, they
te

II. People should not travel by air try to save money in all possible
when they are going for a holiday. ways.
Which of the following can be concluded III. After years of saving, they realize
from the given statement? that although they have saved the
ce

(a) Only I follows amount they had planned to save, it


(b) Only II follows is not sufficient now for
(c) Both I and II follow constructing a house.
(d) None follows Conclusion: Growing rates of land and
(e) Either I or II follows building material shatter their dream.
The conclusion drawn is
@

4. The planet is warming, from north pole (a) follows from the given statements
to south pole, and everywhere in (b) does not follow from the given
between. Globally, the mercury is statements
already up by more than 1 degree (c) is irrelevant
Fahrenheit (0.8 degree Celsius) or even (d) is probably false
more in sensitive polar regions. The (e) is probably true

4 Adda247 Publications For any detail, mail us at


Publications@adda247.com
A Complete Book of Logical Reasoning

6. Statement: private houses and public houses. These


I. The no-confidence motion is often children certainly would pose a threat to
allowed to be debated upon. society, if they are not treated equal and
II. There was a no-confidence motion given opportunities to develop towards
last week. the best of their potential despite being
Conclusion: No confidence motion was poorly educated. The existence of child
debated upon by the parties concerned. labour in India is a complex reality.

p
The conclusion drawn is However, it is a symptom, not the
(a) definitely true (b) probably false disease.
(c) definitely false (d) Can’t say Which of the following conclusions can

ou
(e) probably true be drawn from the given passage?
(a) Child labour can be reduced by
7. Statement:
I. Only those young men who smoke providing skill development to the
do not go to colleges. people.
II. Only those young men go to (b) Child labour has created the wrong
colleges who are not smart. image of India in international

gr
Conclusion: Smart young men do not arena.
smoke. (c) The condition of child labour has
The conclusion drawn is not improved in India and is
(a) probably true (b) definitely true causing threat to the society.
(d) Child labour is reducing the quality
(e) can’t say
m
(c) probably false (d) definitely false
of education.
(e) None of these
8. Statement:
I. Dogs do not bark on the arrival of 10. In question below is given a statement
xa
friends of the family. followed by two conclusions numbered I
II. When A entered B’s house, B’s dog and II. You have to assume everything in
started barking. the statement to be true, then consider
Conclusion: A is B’s enemy. the two conclusions together and decide
The conclusion which of them logically follows beyond a
(a) follows from the given statements reasonable doubt from the information
te

(b) is probably true given in the statement.


(c) is probably false Give answer
(d) does not follow from the given (a) If only conclusion I follows
statements (b) If only conclusion II follows
ce

(e) can’t say (c) If either conclusion I or conclusion


9. Study the following information carefully II follows
and answer the questions. (d) If neither conclusion I nor
Too little has changed with the practice conclusion II follows
of child labour in our country, despite (e) If both conclusions I and II follow
@

tall promises made by our leaders. A Statement: The eligibility for admission
good proportion of children throughout to the course is minimum second class
the world, especially in India, form a part Master’s degree. However, the
of the working labour force. Millions of candidates who have appeared for the
children work in fields and factories, on final year examination of Master’s
street corners and in garbage dumps, in degree can also apply.

5 Adda247 Publications For any detail, mail us at


Publications@adda247.com
A Complete Book of Logical Reasoning

Conclusions: 13. Statement: During 1997-98 the total loss


I. All candidates who have yet to get incurred by the Ill Public Sector Units was
their Master’s degree will be there to the tune of Rs 6809 crate which was
in the list of selected candidates. converted into paid capitals by the
II. All candidates having obtained Government of its total investment of Rs.
second class Master’s degree will be 5129 crore.
there in the list of selected Conclusions:

p
I. The Government is left with only
candidates.
one option, that is, to privatize
Direction (11-13): In each question below is these units.
II. The Government did not take care in

ou
given a statement followed by two conclusions
numbered I and II. You have to assume the matter of investments in these
everything in the statement to be true, then public sector units.
consider the two Conclusions together and 14. The argument for liberalization which
decide which of them logically follows beyond answers the worries of the Left parties
a reasonable doubt from the information about the possible trade deficits created

gr
given in the statement. Give answer by the opening up of the Indian economy
(a) if only conclusion I follows goes thus, "In today's economic scenario,
(b) if only conclusion II follows where there are many trading countries,
(c) if either I or II follows the trade between two specific countries
(d) if neither I nor II follows need not be balanced. The differing
(e) If both I and II follow
m demands of goods and services and the
differing productive capabilities of the
11. Statement: Although we have rating same among different countries will
agencies like Crisil, ICRA, there is demand cause a country like India to have trade
to have a separate rating agency of IT deficits with some countries and
xa
companies to protect investors. surpluses with other countries. On the
Conclusions: whole, the trade deficits and surpluses
I. Assessment of financial worth of IT will balance out in order to give a trade
Companies calls for separate, set of balance".
skills, insight and competencies. Which of the following conclusions best
te

II. Now the investors investing in IT summarizes the argument presented in


the passage above?
companies will get protection of
(a) India's trade deficits and surpluses
their investment.
with other countries always,
12. Statement: Company "Y" will improve the balance out.
ce

manufacturing facilities for the production (b) Left parties need not to worry
of shaving kits as a result of which about trade deficits in India since
capacity would increase and cost would its trade will always be in balance
be reduced. -- A spokesperson of the even though it runs a deficit with a
Company "Y" Conclusions: single country.
@

(c) The Left parties in India should not


Conclusions:
be concerned about India's trade
I. The products of Company "Y" will deficits with specific countries
compete the market norms in the because they will balance out in the
quality and cost factor. long run.
II. There will be increase in demand of (d) All of the above.
shaving kits of Company "Y" (e) None of the above.
6 Adda247 Publications For any detail, mail us at
Publications@adda247.com
A Complete Book of Logical Reasoning

15. In a famous experiment at the IISC Give answer (b): If only conclusion II follows.
campus, when a cat smelled milk, it Give answer (c): If either I or II follows.
salivated. In the experiment, a bell was Give answer (d): If neither I nor II follows
rung whenever food was placed near the Give answer (e): If both I and II follow
cat. After a number of trials, only the bell 17. Statements: In a one day cricket match,
was rung, whereupon the cat would the total runs made by a team were 200.
salivate even though no food was Out of these 160 runs were made by

p
present. Such behaviour has been spinners.
observed in other animals such as dogs, Conclusions:
monkeys, etc. and is a vital input for I. 80% of the team consists of spinners.

ou
training domesticated animals. II. The opening batsmen were spinners.
Which of the following conclusions may
be drawn from the above experiment? 18. Statement: The government run
(a) The ringing of a bell was associated company had asked its employees to
with food in the mind of the cat. declare their income and assets but it
(b) Cats and other animals can be easily has been strongly resisted by employees

gr
tricked. union and no employee is going to
(c) A conclusion cannot be reached on declare his income.
Conclusions:
the basis of one experiment.
I. The employees of this company do
(d) Two stimuli are stronger than one.
not seem to have any additional
(e) None of the above.
16. Statement: Through interest rate
m undisclosed income besides their
salary.
subvention scheme government of India II. The employee’s union wants all
will provide crop loan to farmers at just senior officers to declare their
4%. income first.
xa
Which of the following can be concluded 19. Statement: The Official Secrets Act
from the above statement? (O.S.A) enacted by the XYZ Government
(a) Financial condition of farmers will during the war seems to be the source of
improve much corruption in the country ‘P’.
(b) Repayment of loan will be done Conclusions:
te

timely by the farmers. I. The Official Secrets Act has to be


(c) Small and marginal farmers will be abolished immediately to stop
most benefited from this scheme. corruption in country ‘P’.
(d) Farmers will have to pay less II. The XYZ government wanted to
ce

interest on their loan. encourage corruption in the


(e) Only (a) and (d) government Offices.
Directions (17-21): In each question below 20. Statement: After collision of two vessels
is given a statement followed by two in the sea all the crewmen and
conclusions numbered I and II. You have to passengers are declared as missing. – A
@

assume everything in the statement to be news report.


true, then consider the two conclusions Conclusions:
together and decide which of them logically I. No one from the two vessels has
follows beyond a reasonable doubt from the survived after the collision.
information given in the statement. II. A few persons from the two vessels
Give answer (a): If only conclusion I follow. may have survived and are missing.

7 Adda247 Publications For any detail, mail us at


Publications@adda247.com
A Complete Book of Logical Reasoning

21. Statement: Good health is a luxury in Conclusions:


country ‘P’ where rate of death is very I. The Career power Institute is one of
high compared to other nations of that the leading institutes of the world.
region. II. Whatever is being followed by
Conclusions: world’s leading institutes will
I. People in country ‘P’ cannot afford definitely be good and useful.
to have many luxuries of life. 25. Statement: Smoking is one of those

p
II. Good health is a gift of the nature. human weaknesses which tend to test
Directions (22–23): Study the following the will power of the smoker to the edge.
instructions carefully and then answer the Conclusions:

ou
questions that follow. I. it is very difficult for the smokers to
(a) If only conclusion I follows. give up smoking even if they want
(b) If only conclusion II follows. to do so.
(c) If either I or II follows. II. Human beings have other
weaknesses as well.
(d) If neither I nor II follows.
(e) If both I and II follow. 26. Statement: Sealed tenders are invited

gr
from competent contractors experienced
22. Statement: Book your flat before 15
in executing construction jobs.
June and take benefit of interest free Conclusions:
loan from builders – An advertisement. I. Tenders are invited only from
Conclusions: experienced contractors.
No flat will be booked later.
m
After 15 June no loan will be available.
II. It is difficult to find competent
tenderers in construction jobs.
23. Statement: Due to jump in price of Directions (27-29): In each question below
international oil, government will is given a statement followed by two
xa
discuss on the present oil strategy. conclusion, numbered I and II. You have to
Conclusions: assume everything in the statement to be
After jump in price of international oil true, then consider the two conclusions
govt. will increase the price of disel. together and decide which of them logically
Although there is a jump in price of follows beyond a reasonable doubt from the
international oil govt. won’t increase the information given in the statement.
te

price of disel. Give answer:


(a) if only conclusion I follows;
Directions (24-26): In each of the following
(b) if only conclusion II follows;
questions, a statement is given followed by
(c) if either I or II follows;
two conclusions numbered I and II. Give
ce

(d) if neither I nor II follows


answer- (e) if both I and II follow.
(a) If only conclusion I follows.
(b) If only conclusion II follows. 27. Statement: The distance of 900 km by
(c) If either I or II follows. road between Bombay and Jafra will be
(d) If neither I nor II follows. reduced to 280 km by sea. This will lead
@

(e) If both I and II follow. to a saving of Rs. 7.92 crores per annum
on fuel.
24. Statement: ‘We follow some of best and Conclusions:
effective teaching learning practices I. Transportation by sea is cheaper
used by leading institutes all over the than that by road.
world’. – A statement of a Professor of II. Fuel must be saved to the greatest
Career power Institute. extent.
8 Adda247 Publications For any detail, mail us at
Publications@adda247.com
A Complete Book of Logical Reasoning

28. Statement: Good voice is a natural gift 32. Statement: The Prime Minister
but one has to keep practicing to emphatically stated that his government
improve and excel well in the field of will make every possible effort for the
music. upliftment of poor farmers and
Conclusions: farmhands.
I. Natural gifts need nurturing and Conclusions:
care. I. Except poor farmers and

p
II. Even though your voice is not good, farmhands, all others have got
one can keep practicing. benefits of fruits of development.
II. No serious efforts have been made
29. Statement: Domestic demand has been

ou
increasing faster than the production of in the past for upliftment of any
indigenous crude oil. section of the society.
Conclusions: 33. Comparative constitutional law, which
I. Crude oil must be imported. involves the study of constitutional
II. Domestic demand should be reduced. jurisprudence and its relative

gr
Directions (30-32): In each question below application to political institutions in
is given a statement followed by two different countries, has in recent times
conclusions, numbered I and II. You have to emerged as an important field of
assume everything in the statement to be examination. But much of the academia
true, then consider the two conclusions involved in these comparative studies
m
together and decide which of them logically has focused its attention on the
follows beyond a reasonable doubt from the constitutions of Western democracies.
information given in the statement. Even when constitutions of developing
Give answer: countries are considered, the
xa
(a) if only conclusion I follows; comparison often features their
(b) if only conclusion II follows; structures as those contained in the
(c) if either I or II follows; constitutions of supposedly more
(d) if neither I nor II follows; and sophisticated Western societies.
(e) if both I and II follow. Which of the following can be concluded
from the passage above?
te

30. Statement: The secret of success is


(a) Despite the significant differences
constancy of purpose.
in history and politics, developing
Conclusions:
nations share more than just
I. Constant dripping wears the stone. geography.
ce

II. Single-minded devotion is


(b) Comparative constitutional studies
necessary for achieving success.
available today are highly lopsided.
31. Statement: The percentage of the (c) The challenges faced by
national income shared by the top 10 per constitutional democracies in
cent of households in India is 35. developing countries are unfairly
@

Conclusions: evaluated with reference to the


I. When an economy grows fast, developments in Western countries.
concentration of wealth in certain (d) The need of the hour is
pockets of population takes place. constitutional studies written in the
II. The national income is unevenly correct perspective
distributed in India. (e) None of these

9 Adda247 Publications For any detail, mail us at


Publications@adda247.com
A Complete Book of Logical Reasoning

Directions (34-35): In each question below I. China wants to resolve territorial


is given a statement followed by two disputes through negotiations and
conclusions number I and II. You have to consultations between the two
assume everything in the statement to be countries.
true, then consider the two conclusions II. Analysts say the bridge will ensure
together and decide which of them logically swift movement of Indian troops in
follows beyond a reasonable doubt from the Arunachal Pradesh, which,

p
information given in the statement. therefore, will bolster India's
Given answer defence along the China border.
(a) If only conclusions I follows Which of the following can be concluded

ou
(b) If only conclusion II follows from given statement?
(c) If either conclusion I or conclusion II (a) Only I (b) Only II
follows (c) None is true (d) Both are true
(d) If neither conclusion I nor conclusion II (e) Either I or II
follows 37. Police in Jupiter, Florida, released dash-
(e) If both conclusions I and II follow cam footage Wednesday of the arrest of

gr
34. Statement: Nobody advises motorcyc- Tiger Woods for driving under the
lists and pillion-riders in Britain, influence two days earlier. The videos
Holland, Italy and other countries to show the golf superstar unsteady,
wear helmets. It is a matter of sheer disoriented and struggling to comply
common sense and
m
elementary with sobriety tests. Woods claimed that
he was suffering from "an unexpected
prudence.
reaction to prescription medications”.
Conclusions
Which of the following conclusions can
I. The citizens of Britain, Holland and
be made from Wood’s statement?
xa
Italy are literate.
(i) He is lying and wrongly trying to
II. The citizens of Britain, Holland and
prove his innocence.
Italy are conscious regarding their
(ii) He is trying to divert the attention
safety.
of his fans.
35. Statement: The complaint of poor (iii) He is trying to blame medical
te

customer service is partly correct and it science for this incident.


certainly needs to be improved. (a) Both (i) and (ii)
Conclusions (b) Only (i)
I. Poor customer service cannot be (c) Only (iii)
ce

improved. (d) All (i), (ii) and (iii)


II. The speaker knows about the poor (e) None of these
customer service of the company. 38. Born in a poor family in Andhra
36. Statement-"We hope India adopts a Pradesh's Srikakulam district Nandini
cautious and restrained attitude on the KR, the 26-year-old civil engineer topped
@

issue before the final settlement of the the Civil Services Examination. But
border issue with China to jointly control success for the government school
disputes, safeguard peace and tranquility teacher's daughter in Karnataka Kolar
in the border areas," China warns India district, famous for its gold mines, came
over Dhola-Sadiya bridge in Arunachal after three failures. This was her fourth
Pradesh. shot at the examination

10 Adda247 Publications For any detail, mail us at


Publications@adda247.com
A Complete Book of Logical Reasoning

Which of the following statements can be Which of the following can be deduced
concluded from Nandini’s performance from the given statement?
and success after her fourth attempt? (i) There cannot be absolute
(i) Consistency in efforts and comparison between the rankings
perseverance is required to be of 2016 and for 2017 since the
successful in life. parameters have been tweaked.
(ii) Some luck is required to be (ii) All the other cities do not have the

p
successful. potential to be cleanest cities.
(iii) One cannot succeed at one’s first (iii) Other cities are not following Swach
attempt. Bharat Abhiyan.

ou
(iv) Poverty motivates one to strive (a) Only I and II (b) Only III
towards greatness. (c) Only II and III (d) All of the above
(a) Both (i) and (ii) (e) None of these
(b) Only (i)
(c) Only (iii) 41. Statement- An off-duty policeman, who
(d) All (i), (ii) and (iii) was one of the first on the scene of the
London Bridge terror attack, was

gr
(e) None of these
stabbed after tackling one of the three
39. Senior BJP leaders LK Advani, Union attackers. The officer - who is a rugby
Minister Uma Bharti and Murli Manohar player - is in a critical condition after
Joshi were charged with criminal sustaining knife injuries. Police
conspiracy by a court in Lucknow today
in the 1992 Babri mosque demolition
m commissioner Cressida Dick praised the
"extraordinarily brave actions by officers
case. Uma Bharti, told reporters: "I don't
on and off duty first on the scene" who
consider myself an accused...There was
"ran towards the danger".
no conspiracy, it was an open movement
Which of the following deductions can be
xa
like it happened against the Emergency."
made after reading the above statement?
Which of the following deductions can be
(i) Most of the officials of London
made from Uma Bharti’s statement
following the court’s verdict. Police department likes the game of
(a) It was a conspiracy of the Rugby.
opposition ruled government to (ii) Playing rugby remove the fear of
te

frame her. death from one’s mind.


(b) Uma Bharti and other BJP leaders (iii) The stabbed policeman was also
are the culprits in the Babri careless as he should have been
demolition case. equipped and used his gun instead
ce

(c) Lucknow court is not afraid of the of tackling the terrorist.


ruling party. (a) Only (iii)
(d) She will approach higher judiciary (b) Both (i) and (ii)
for the justice. (c) All (i), (ii) and (iii)
(e) None of these (d) Only (ii)
(e) None follows.
@

40. Indore and Bhopal, both in Madhya


Pradesh, have emerged as the cleanest 42. Statement-Borders more secure now,
cities in the country as per a massive no scope for infiltration, Centre working
cleanliness survey in 2017, to improve financial condition of
commissioned by the Union Urban farmers, says Union Home Minister
Development Ministry. Rajnath Singh.

11 Adda247 Publications For any detail, mail us at


Publications@adda247.com
A Complete Book of Logical Reasoning

(I) Borders were not secure before and (II) Jadhav, 45, was sentenced to death
they were needed to be revamped. by a Pakistani military court in
(II) Government has a commitment April.
towards the nation’s development (III) Pakistan's military establishment
with secure borders and focus on announced that Jadhav would be
improving the lot of agriculturalists. hanged.
(III) In the previous UPA regime Which of the following would be the

p
farmers’ condition was far better correct statement for the given
than the present one. conclusion?
Which of the following can be (a) Only II and III (b) Only I and III

ou
deduced from the given statement? (c) Only II (d) Only I
(a) Only (I) (e) None of these
(b) Both (I) and (III)
(c) Only (II) 45. Statement-Mark Zuckerberg Wants To
(d) Both (II) and (III) "Bring The World Closer Together"
(e) None of these Through Facebook's New Message.
Which of the following can be concluded

gr
43. Statement- Drinking in moderation can from the given statement?
help our health, some research has (I) Mark Zuckerberg wants to make
showed. Many doctors recommend a more users of facebook to show the
glass of wine or beer a night as part of world the power of technology.
diet plans such as the Mediterranean
diet and the DASH diet, which have been
m (II) “Making the world more open and
connected,” means Facebook says
proven to keep your heart and brain
its mission is “To give people the
healthy. However, a new study suggests
power to build community and
that even moderate drinking may not be
bring the world closer together.”
xa
great for your brain.
(III) Facebook will now focus more on
Which of the following can be deduced
its image as a place for status
from the above statement?
(i) Scientific researches can contradict updates and photos.
at times. (a) Only II (b) Only I and III
(ii) Moderate intake of wine is less (c) Only II and I (d) Only II and III
te

harmful than cigarette. (e) None of these


(iii) Something which is not good for 46. Statement-Reserve Bank of India has
brain does not mean it cannot be widened the scope of its banking
healthy. ombudsman platform for addressing
ce

(a) Only (i) mis-sell of third party products, and


(b) Both (i) and (ii) customer grievances on and mobile
(c) Only (iii) banking and electronic banking issues,
(d) Both (ii) and (iii) recognising customers' plight in dealing
(e) None of these. with such troubles.
@

44. Conclusion-ICJ rejected India’s request Which of the following could be deduced
to delay Jadhav’s case, says Pakistan. from the above statement?
(I) India had asked the ICJ to offer it (I) Under the amended ombudsman
time till December to file pleadings scheme, a customer would also be
in the Jadhav case, however, "the able to lodge a complaint against
court has dismissed their request”. the bank for its non-adherence to

12 Adda247 Publications For any detail, mail us at


Publications@adda247.com
A Complete Book of Logical Reasoning

RBI instructions with regard to banking, the branch still continues to


mobile banking and electronic dominate banking channels in India.
banking services which are Which of the following can be deduced
important means for digital from the given statement?
banking. (I) Retail banking customers used to
(II) The pecuniary jurisdiction of the visit the branch/store at least once
banking ombudsman to pass an in the past 12 months.

p
award has been increased from (II) There is great potential for banks to
existing Rs 10 lakh to Rs 20 lakh. move more into the digital space.
(III) The deficiencies arising out of sale Only 51 per cent of retail banking

ou
of insurance, mutual fund other customers have a reliable online
third party investment products banking experience with their main
will be increased. financial institution.
(a) Only II (b) Only III and I (III) Digital banking is not yet a
(c) Only III (d) Only I pervasive experience in India.
(e) None of these (a) Only (II)

gr
(b) Only (III)
47. Statement:- Amidst the increasing (c) Only (I) and (II)
tension between the two countries, (d) Both (II) and (III)
China on Wednesday said that it was (e) All of the above
open to discussing with India "the
possibility of alternative arrangements
through other routes for Indian official
m 49. Statement: - A Conservative MP has
been suspended from the party after it
yatris who had planned to visit Kailash emerged she used a racist expression
and Manasrovar via Nathu La Pass in during a public discussion about Brexit.
Which of the following can be deduced
xa
next year".
Which of the following can be concluded from the above statement?
from the above statement? (i) She later apologized and said- “The
(i) China stalled this year's pilgrimage comment was totally unintentional.
I apologize unreservedly for any
from Sikkim and through the Nathu
offence caused.”
La pass.
te

(ii) Racist language has absolutely no


(ii) An Indian solider was released by
place in politics or in today's
China recently.
society.
(iii) There are no other routes to Kailash
(iii) She used the word “nigger” which
and Mansarovar except Nathu La
ce

originated in the American Deep


Pass.
South in the mid-19th Century and
(a) Only (i) is thought to have referred to slaves
(b) Only (ii) having to conceal themselves as
(c) Only (ii) and (iii) they sought to flee north and secure
(d) Only (i) and (iii) their freedom.
@

(e) None of these (a) Only (iii)


48. Over 90% Indian customers still prefer (b) Only (i)
branch over online banking: Report said. (c) Only (ii)
While the government has been pushing (d) Only (i) and (iii)
people to go online when it comes to (e) None of these

13 Adda247 Publications For any detail, mail us at


Publications@adda247.com
A Complete Book of Logical Reasoning

50. "Normal visits are less important than attitude towards Muslim
state visits and at a time when the community.
President of the USA has policies that (ii) London is facing an economic crisis
many in our country disagree with, I am and it will be economically unviable
not sure it is appropriate for our to roll out a red carpet to welcome
government to roll out the red carpet," another country’s president.
London Mayor Sadiq Khan told in an
(iii) People of London are not happy

p
interview Monday.
Which of the following can be deduced with the USA Policies.
from the above statement? (a) Only (i) (b) Only (ii)

ou
(i) London Mayor does not like US (c) Only (iii) and (i) (d) Only (iii)
President as he has a negative (e) None of these

Statement and Conclusion : Solutions

gr
1. (a); Only statement (a) can be deduced 5. (a); People are unable to have their own
from the given statement as by house due to rising cost of land and
burning the coal the concentration construction material.
of its particles is increased which 6. (d); Data in the given statement is
lead to excessive amount of air
m
pollution. But for others (b) and (e)
insufficient to draw given
conclusion so we can’t say anything
can be assumed but cannot be about it.
deduced. While for (c) and (d) the
amount of average rainfall and its 7. (d); According to given statement
xa
conclusion drawn is definitely false.
consistency in not directly given in
the statement. 8. (b); Conclusion probably follow from
the given statement.
2. (e); More lifts in city X than city Y could
be due to the differences in the 9. (c); Increased child labour is creating
average heights of the buildings or social evil.
te

the difference in the total number of 10. (d); Neither of the conclusions logically
houses of the two cities. So, neither follows beyond a reasonable doubt.
I nor II can be concluded from given It is said that the candidate who
statement. have appeared for the final year
ce

3. (d); I is an assumption. II is a suggestion examination of master’s degree can


also apply, but their selection
but not a conclusion. Hence, neither
subject to the certain condition.
I nor II follows.
Therefore, Conclusion I is not valid.
4. (b); Statement (b) clearly shows the The eligibility for admission to the
@

conclusion of the given passage. course is minimum second-class


Extreme climatic condition on earth master’s degree. This does not
is the effect of the increasing global imply that all candidates who have
warming. While (a) negates the obtained second class will be
statement and (c), (d) and (e) are selected. Second class master’s
not mentioned in the statement. degree is the minimum required
criterion.
14 Adda247 Publications For any detail, mail us at
Publications@adda247.com
A Complete Book of Logical Reasoning

Direction (11-13): 20. (b); Only conclusion II follows.


11. (a); II may be an assumption of the
speaker. But certainly, it is not a 21. (d); Neither of the conclusions logically
conclusion. follows from the given statements.
12. (a); Improvement in the manufacturing Direction (22-23):
facilities will automatically enhance
the quality of its product and 22. (d); Flat will be given later also and

p
reduce the cost. These two things interest will be available also after
are important to compete in the 15th June. Advertisement is only for
market. Hence I follow. II may be an benefits of interest free loan. So

ou
assumption but it not a conclusion. both the conclusions are not true.
13. (d); I is extreme case. Privatization is
not the only option. II is very 23. (c); It is possible that government will
generalize statement, it cannot be increase price in oil only in case of
concluded. rising price in international market.
Or it is also possible in that case

gr
14. (b); The paragraph states that
individual trade deficits are not a also government won’t increase
problem, since the overall position price.
gets balanced.
Direction (24-26):
15. (a); The ringing to the bell become
m
associated with food means the cat 24. (b); The statement hints at following
the practices used by leading
relates food with the bell in its mind
and whenever the bell rung, it institutes as a positive feature of
seems there was food. Hence, the the Career power Institute. So, only
xa
cat began to salivate. So option (a) II follows while I does not.
is the correct answer.
25. (e); The statement mentions that a very
16. (e); The subvention scheme will lead to
less interest. Hence d follows. If the strong will power is required to
burden of interest lessen, farmers give up smoking and so it is very
te

will prosper. Hence a follows. difficult. Thus, I follows. Also, it is


Direction (17-21): mentioned that smoking is one of
the human weaknesses. This means
17. (d); According to the statement, 80% of that there are other human
the total runs were made by
ce

spinners. So, I does not follow. weaknesses as well. So, II also


Nothing about the opening batsman follows.
is mentioned in the statement. So, II 26. (a); According to the statement, tenders
also does not follow.
are invited from contractors
18. (d); Neither of the conclusions logically experienced in executing
@

follows from the given statements. construction jobs. So, conclusion I


19. (a); Only I follow and II does not follow follow. The availability of
because no government wants or competent tenderers in
intends to encourage corruption in construction is not mentioned. So,
the government offices. So it’s not
conclusion II does not follow.
valid.

15 Adda247 Publications For any detail, mail us at


Publications@adda247.com
A Complete Book of Logical Reasoning

Direction (27-29): Direction (34-35):


27. (b); According to the statement, sea 34. (b); The matter of literary or illiteracy
transport is cheaper than road can’t be determined from the
transport in the case of Bombay to statement. Hence, statement I does
Jafra, not in all the cases. So, not follow. Statement II follow from
conclusion I does not follow. The the tone of statement.
statement stresses on the saving of 35. (b); From the statement it is clear that

p
fuel. So, conclusion II follows. the speaker knows about the poor
28. (a); Clearly, I follows directly from the service and need of improvement in
the customer service. Hence II

ou
given statement. However, II is not
related to the given statement and follows.
so does not follow. 36. (a); In the above statement we have to
29. (c); The statement mentions that give the conclusion of the above
statement.
demand for oil is increasing faster
For Statement I. Right, from the
than the production. So, either the

gr
above statement it is clear that
demand. Thus, either I or II follows.
India and China have border issue
Direction (30-32): and they should resolve it.
For Statement II. Right, from above
30. (e); Both I and II directly follow from statement that the bridge will
the given statement.
31. (b); Nothing about the growth of
m provide a support or strengthen
India’s defence.
economy is mentioned in the 37. (e); None follows. (i) There is not
statement. So, I does not follow. enough evidence for us to decide if
Also, it is given that 35 per cent of he is lying or not. (ii) His statement
xa
national income is shared by 10 per is directly related to the issue
cent of households. This indicates therefore it cannot be said that he is
unequal distribution. So, II follows. trying to divert attention. (iii) As
32. (d); No other section of society except the reaction was unexpected it
farmers has been talked about in means he is unsure about the cause
te

the statement So, neither I nor II of reaction. So, (iii) is not true.
follows. 38. (b); (i) The fact that she got success in
her fourth attempt clearly suggests
33. (b); The passage states that the frame of
that she did not give up and so,
reference of these studies is always
ce

perseverance is definitely one of the


to Western countries. So option (b)
reasons for her success.
is a conclusion that can be drawn (ii), (iii) and (iv) are vague and
from this —these studies are tilted nothing of the sort can be
towards Western democracies. concluded from the given facts
'Differences in history and politics'
@

and 'sharing' in option (a) find no 39. (e); (a) She never mentioned the role of
support in the passage. Option (c) is opposition in the statement. So,
inapt because the passage makes no this option is incorrect.
reference to evaluation of (b) This may or may not be true,
challenges. (d) is beyond the scope but the question asked is
of the passage. concerned with her statement
only. So, this option is incorrect.

16 Adda247 Publications For any detail, mail us at


Publications@adda247.com
A Complete Book of Logical Reasoning

(c) There is no information given in 44. (d); In the above question we have to
the statement to make this find the statement which
deduction. completely follows the given
(d) Even though one can assume conclusion.
that she is unhappy with the For I-This statement will be the
court’s verdict there is no correct one because In the
additional information given to conclusion we can see that ICJ

p
make that assumption. rejected India’s request in
Kulbhushan Jadhav’s case and this
40. (e); Only I cannot be concluded from statement describes about India’s
the above statement as it is not

ou
demand over it. So this will be the
related to the statement. Also, II and statement for the given conclusion.
III are vague as it is not mentioned For II- This may not be the
about the other cities in the given statement for the given conclusion
statement. because It states only about
41. (e); The data given in the statement is Pakistan’s take over Jadhav’s case.

gr
not sufficient enough to conclude (i) For III-This also may not be the
and (ii). The fact that the policeman statement because it does not relate
was off-duty negates conclusion to the given conclusion as It defines
(iii). the Pakistan Military’s views on this
case which has no connection with
42. (c); In the above question we have to
find which statement can be
m ICJ’s decision.
45. (a); In the above question we have to
concluded from the given
find which statement can be
statement.
concluded from the given
For I- False, because It cannot be
xa
statement.
said from the given statement about
For I-It is not true because it is not
the security of Borders previously.
given in the statement that he
For II-True, as it is clear from the
wants to increase the number of
above statement that the
users of facebook.
government is working towards
For II-This is true because this
te

development of borders and statement describes the message


working for better condition of given by facebook which implies
farmers. that it wants to make people
For III- False, because it is not clear socially connected to each other.
ce

from the above statement about the For III- This is also not true because
farmers’ condition in UPA regime. it is not given in the statement that
43. (a); (i) “However a new study” implies facebook will now focus more on
that there might have been some providing the feature of images in
studies regarding this in the past place of status. So it cannot be
concluded from the given
@

and “proven to be healthy”. So, (i) is


definitely true. (ii) cannot be statement.
deduced as smoking is not 46. (d); In the above question we have to
mentioned in the statement. (iii) is find which statement can be
vague and not in context with the concluded from the given
statement. statement.

17 Adda247 Publications For any detail, mail us at


Publications@adda247.com
A Complete Book of Logical Reasoning

For I- This is true as it is given in about banks potential to turn into


the above statement that after digital space and about the
amendment in baking ombudsman percentage of customers who have
scheme a customer will be able to a reliable online banking
file complaint against mobile and experience.
electronic banking issues. So this For III-This can be deduced from
can be deduced from the above the given statement as it is clearly

p
statement. mentioned in the given statement
For II-This statement cannot be that 90% Indian customers still
deduced from the given one prefer branch over online banking

ou
because nothing is mentioned about which implies that digital banking is
pecuniary jurisdiction in the given still not widely accepted by group
statement. of people.
For III-It is also not mentioned in
49. (c); In this question we have to choose
the given statement that the
an option which can be concluded
problems regarding sale of mutual
from the given statement.

gr
fund, insurance will be increased or
Option (i) in not the conclusion but
not. So this statement cannot be
the aftermath of her remark.
deduced from the given statement.
Option (ii) can be concluded as it is
47. (a); In this question we have to choose given in the statement that she was
an option which can be concluded
from the facts given in the
m fired for her remark.
Option (iii) cannot be concluded as
statement. it is only given in the statement that
Statement (i) can definitely be she used a racist word. Its
concluded as it is given in the impossible to conclude which word
xa
statement that China is willing to was it from the facts given in the
discuss the alternative routes for statement.
Indian pilgrims and there was an
50. (d); In this question, we have to choose
increase in tension between the two
an option which can be concluded
countries.
from the facts given in the
te

Statement (ii) cannot be concluded


statement.
as it is not related to the statement.
Option (i) cannot be true as
Statement (iii) cannot be concluded
personal disliking cannot play a
as it is given in the statement that
factor in deciding the degree of
ce

China is willing to discuss


hospitality to the President of
alternative routes.
another country. Moreover, it is
48. (b); In the above question we have to nowhere mentioned in the
find which statement concluded statement that US President has a
from the given statement. hostile attitude towards a particular
@

For I-This cannot be deduced from community.


the given statement as it is nowhere Option (ii) cannot be deduced as it
mentioned in the given statement. is nowhere mentioned that London
For II- This cannot be deduced from is facing an economic crisis.
the given statement as it cannot be Option (iii) can be deduced because
determine from the given statement it is directly given in the statement.

18 Adda247 Publications For any detail, mail us at


Publications@adda247.com
Join channel @cetexamgroup
A Complete Book of Logical Reasoning

p
ou
gr
m
xa
te
ce
@

Join channel @cetexamgroup


1 Adda247 Publications For any detail, mail us at
Publications@adda247.com
A Complete Book of Logical Reasoning

02 Statement and Assumption


Introduction: The questions in this chapter consists of a statement (which consists of facts,
observations, discussions etc) and followed by assumptions, of which the validity is to be

p
checked.
What is an Assumption? An assumption is that hidden part of the statement which is assumed

ou
/supposed and taken for granted. Something that is not clearly mentioned in the statement, but
is an integral part of it.
For eg: Let’s take an example of a five storey building made of glass and steel pillars. Now, the
glass, the steel pillars can be clearly seen, but the foundation or base of the building is hidden or
not clearly seen.

gr
This analogy can be used to explain the questions type. The glass, pillars which can be clearly
seen are parts of the building. This building is the statement of the question. On the other hand,
the foundation is the hidden part, not clearly seen, which is the assumption. So, the assumption
is the hidden or the implicit part of the statement without which the statement cannot exist.
m
Ex.1 Statement:Amitabh Bachchan says, “Today, I have money, fame, property, bank balance.
What do you have?” Shashi Kapoor says, “I have my mother.”
Assumption: Mother is above all materialistic pleasures of life. This assumption is valid as
without assuming it, Shashi Kapoor wouldn’t have concluded what he said.
xa
To check whether an assumption is implicit or not, keep the following points in
mind.
(1) The Assumption should be in the domain of the statement, i.e., it should be directly related
to the statement. If the assumption talks about any point, not mentioned in the statement, it
has to be out rightly rejected.
te

Eg. 2 Statement:“All the sweets available in our shop are made from pure ingredients.” The
banner outside a sweet shop
Assumption-1: People can spend any amount of money to buy sweets made from pure
ce

ingredients. Invalid Assumption: The owner of the sweet shop may have
thought about the money factor associated with the sweets, but the assumption
cannot be accepted as it is not mentioned in the statement.
(2) Any assumption can be accepted if it is:
(a) Root Cause of a statement, or
@

(b) Desired effect of a statement. For the statement discussed above


Assumption-2: People want sweets made from pure ingredients.
→ Valid assumption: The assumption is the root cause of the statement. People want sweets
made from pure ingredients, that is why the banner was put up.

2 Adda247 Publications For any detail, mail us at


Publications@adda247.com
A Complete Book of Logical Reasoning

Assumption-3: The owner of the shop expected that people will get attracted from the banner
and his sales will increase. Valid Assumption: This is the desired effect of the
statement.
Important Notes:
(1) Always check whether an assumption is implicit or not, by “Keeping yourself in the shoes of
the subject”. Think from the perspective of the person saying the line in the statement, the

p
person giving the advertisement, the person advising someone etc. As in the example above,
check the assumptions from the perspective of owner of the shop, not yourself.
(2) Always be careful of the extreme words used in the sentence, such as, most, only, all, best,

ou
definitely etc. the statement are supposed to be read carefully to pick the right assumption.
Statement: The country’s overall development has struggled since in dependence.
Government should make every possible step to eradicate corruption.
Assumption-1: Eradicating corruption is the only solution for country’s overall development.
Assumption-2: Eradicating corruption is the best solution for country’s overall development.

gr
Assumption-3: Eradicating corruption is the definite solution for country’s overall
development.
Assumption-4: Eradicating corruption would probably help in overall development. In 1, 2, 3
the words only, best, definite are extreme words with no proof from statement.
m
Whereas “probably” is acceptable in context of the statement.
xa

Points to Remember
te

(1) Always remember that Assumption is always indefinite and positive.


(2) Some words like only, each, any, every, all, Question indicating words (why, these, what), Answer indicating
words (therefore), Definitely, But, Certainly exist in the assumption and that assumption will always be
ce

explicit (False).
(3) Some words like some, to large extent, many, much, exist in the assumption and that assumption will always
be implicit (True).
(4) Any assumption that is conveying the message of advertisement, notice and appeal, that assumption will
always be implicit (True).
(5) Any assumption that is talking about the social welfare (positive), govt. policies that assumption will always
@

be implicit (True).
(6) If any assumption is talking about past and future that assumption will always be explicit (False).
(7) If any assumption showing the word like suggestion, order, request that will always be implicit (True).
(8) Remember that restatement is never implicit.
(9) Comparison are always wrong.
(10) By keeping all above points you can solve questions of assumptions easily.

3 Adda247 Publications For any detail, mail us at


Publications@adda247.com
A Complete Book of Logical Reasoning

Exercise

1. Study the following information in which Which of the following can be assumed
a statement is followed by five from the given statement?
assumptions, read carefully and answer (a) Both I and II follow

p
the question below- (b) Neither II nor I follow
The banking sector is facing tough times. (c) Only II follow
But the sector is facing only short-term (d) Only I follow

ou
pain as the rise in non-performing assets (e) Either I or II follow
(NPAs) is proving a key challenge for the
3. Study the following information in which
industry at large. The regulator wanted
a statement is followed by five
some stringent measures to be taken,
assumptions, read carefully and answer
and the government is coming out with
the question below-
so many reforms and policies.
“If we slightly increase the price of our

gr
Which of the following can be assumed
product then we can improve the quality
from the given statement?
of our product to increase the number of
(a) NPA was 2.4% more in last two
customers from our only rival company
years as compared to the present
V,”-A statement by owner of company U.
one.
Which of the following can be assumed
(b) The steps taken by the government
are not enough to revamp the
m from the given statement?
(a) Without increasing the price, the
banking sector.
quality of the product offered by
(c) Manufacturing sector is performing
company U can’t be improve.
xa
far better than the banking sector
(b) Quality offered by company V of the
for last five years.
product is better than the quality
(d) After the implementation of
offered by company U.
suitable expedient, the things are
(c) Improvising the quality is the only
likely to be changed for banking
way to attract more number of
sector.
te

customers.
(e) The increase in NPA is the only
(d) The price offered by the rival
factor responsible for the difficult
company V is already higher than
time of banking industry.
the price offered by company U.
2. Study the following information in which (e) Superior quality even for bit higher
ce

a statement is followed by five price will definitely increase the


assumptions, read carefully and answer demand of the product in the
the question below- market.
Statements: All the population of village
4. Study the following information in which
A is relocating to city B, near to village A.
a statement is followed by some
@

Assumption:
assumptions, read carefully and answer
(I) City B is rich in terms of resources
the question below-
and employment opportunities.
National Crime Records Bureau (NCRB)
(II) A critical virus has been spread in
Director Ish Kumar on Thursday
village A and more than 10 people
requested the government to provide
died from it.
limited access to the Aadhaar card
4 Adda247 Publications For any detail, mail us at
Publications@adda247.com
A Complete Book of Logical Reasoning

database, which would help the (a) All Follows (b) Both I and III
investigating agencies in conducting a (c) Only III (d) Both I and II
probe into inter-State criminals more (e) None of these
effectively with the help of fingerprints. 6. Study the following information in which
Assumption- a statement is followed by some
I. The Aadhar card database consist of assumptions, read carefully and answer
basic information of a citizen the question below-

p
including his fingerprints. “Children are biologically programmed
II. If the number of explorers to a to grow best in the care of a parent
confidential database increases, its figure and when that bond is broken

ou
authenticity will be subjected through long and unexpected
questionable. separations, children respond at the
III. NCRB do not have its own deepest physiological and emotional
maintained record containing the levels which triggers high levels of
realizable information of criminals. anxiety, which make them more
Which of the following can be assumed susceptible to physical and emotional
from the given statement?

gr
illness, and damage their capacity to
(a) Only I and II (b) Only II and III manage their emotions, trust people, and
(c) All of the above (d) Only II focus their attention on age-appropriate
(e) None of these activities.”
5. Study the following information in which Which of the following can be assumed
a statement is followed by some
m
assumptions, read carefully and answer
from the given statement?
I. Parents are essential to young
the question below- children’s survival, providing
“Amid public outcry over Trump’s family nutrition, warmth, and access to
separation practice, Medical profess- shelter and medical care.
xa
ionals and lawyers who visited the sites II. For regulating children’s stress and
said they were safe and clean. But they emotional reactivity, parents are an
described the children detained in the integral piece of this powerful
‘Tender age shelters’ as “hysterical.” system.
Which of the following can be Assumed III. Parent is really in many ways an
te

from the given statement? extension to the child’s biology


I. The separation from their families especially when the child is
at tender age will lead to an developing.
irreversible trauma both physically (a) All Follows (b) Both I and III
and emotionally to the children. (c) Only III (d) Both I and II
ce

II. well-trained clinicians, and those (e) None of these


shelters meet state licensing 7. Statement-The menacing spread,
standards for child welfare starting last Friday, of the malicious
agencies, and they’re staffed by software Wanna Cry, is a frightening
people who know how to deal with reminder of the vulnerabilities of a
@

the needs. connected world.


III. The administration’s policy of Which of the following can be postulated
separating children from their from the given statement?
families is not only needless and (I) It is more than obvious now that
cruel, it threatens the mental and
cyber vulnerabilities have massive
physical health of both the children
and their caregivers. global implications.

5 Adda247 Publications For any detail, mail us at


Publications@adda247.com
A Complete Book of Logical Reasoning

(II) The state of preparedness is a cause Assumption:


for worry, after the likely origin of (I) SSC has not taken any corrective
Wanna Cry. measures to stop paper leak.
(III) The hackers have used cyber (II) There will be no paper leak in case
weapons and believed to have of banking examination.
linkage with terrorist organizations. (III) There is a great future in banking
(a) Only (I) (b) Only (III) field.

p
(c) Only (II) (d) Both (I) and (II) (a) Only II follows
(e) None of these (b) Both I and III follow
8. In the following question a statement is (c) Only III follows

ou
given followed by three assumptions (d) Both I and II follow
numbered as I, II and III. You have to (e) None follows
read all the statements and decide which 10. The argument about whether
of them follows the argument mentioned demonetization was good or bad for the
in statement. economy refuses to die down even a year
Rooftop solar power growth has after the event. While one can endlessly

gr
demonstrated an overall positive trend, debate, with very little data, on whether
But this will need to be scaled up the ban on high-value currency notes
massively to achieve the national target. dealt a body blow to terrorism,
Assumption:
corruption and counterfeiting, one area
(I) With ongoing improvements to
in which its impact can be quantified
m
solar cell efficiency and battery
technology, rooftops will only get
with data is tax compliance. In FY18, net
collections increased by 17.1%.
more attractive in the future.
Which of the following can be postulated
(II) Major solar projects that connect to
from the given statement?
the grid often face the challenge of
xa
land acquisition and transmission I. Counterfeiting of notes was one of
connectivity. the major concern for Government
(III) A survey helps determine usable of India.
rooftops, separating them from II. Many economists has termed it
green spaces, and analyses the wrong as it has impact badly on the
weaker section of the society.
te

quality of the solar resource.


(a) Only II follows III. There has been revamp in the
(b) Both II and III follows percentage increase of direct tax
(c) Both I and III follows with respect to indirect tax.
(d) Only I follows (a) Only III (b) Both I and III
ce

(e) Both I and II follows (c) Both II and III (d) Only II
(e) None of these
9. In the following question a statement is
given followed by three more statements 11. Statement: “Good news for all, Vikalp is
numbered as I, II and III. You have to available and every passenger will get
read both the statement and decide reserve seats. Waiting status will be
@

which of the given assumptions follows omitted from reservation chart” quoted
the statement. by Railways Ministry. Spokesperson of
Statement: “SSC aspirants are fade up railways has said if confirm seat is not
with Paper leak and most of them have available in desired train, option for
decided to start preparing for banking confirm seat in other train will be given
field.”- An article published in The Hindu. at the time of reservation.

6 Adda247 Publications For any detail, mail us at


Publications@adda247.com
A Complete Book of Logical Reasoning

Which of the following can be assumed 13. Which is the following is an assumption
from the given statement? that the researchers apparently made in
(I) Fairs of reserved seats will be this study?
increased by Railways department. (a) The women who participated in the
(II) Commuters would have faced study were sufficiently represent-
issues regarding confirm seats in tative of modern women in general.
railways. (b) Male faces are, in general, attractive

p
(III) There will be increase in number of to women.
persons travelling through (c) Visual images are important to
railways. women.

ou
(a) Only I (b) Only II and III (d) It is impossible to predict what
(c) Only I and III (d) Only I and II features an ideal face would have.
(e) None of these (e) Women in previous ages would
have preferred more masculine
12. Statement: Mr. X, asked to his friend Mr.
men.
Y why you have not change your
company since a long time. Mr. Y replied 14. It is a truism of military science that

gr
with smile that I am enjoying my work “Generals always prepare for the last
here, doing my work with honesty and war.” In the same way, public officials
company is also rewarding honestly. generally spend their efforts on
Which of the following can be problems that were resolved- one way or
hypothesized from the above statement?
m
(I) Mr. Y has created a comfort zone in
another –years before. By the time a
public issue reaches the consciousness of
his current company and don’t enough of the citizenry to become a high
want to move from his comfortable priority of our elected leaders, the
area. problem is usually past the point at
xa
(II) Mr. Y is getting good salary in his which government efforts can
current company and he is happy significantly affect it.
with it. All of the following are assumed in the
(III) Mr. X changes company very argument above except-
frequently. (a) Most of the public problems tend to
te

(a) Only I and II (b) Only II and III evolve towards a point at which the
(c) Only II (d) Only I and III government can do little to control
(e) None of these them.
(b) Political and military leaders are
Directions (13): Read the following
ce

both prone to react tardily to


information carefully and answer the
changes in their fields.
questions which follow.
(c) Issues attain importance for public
In a recent study of responses to visual officials when large number of
images, researchers found that women most citizens are concerned about them.
frequently gave the rating ‘most attractive’ to (d) Planning policies, solely based on
@

images of male faces that were more feminine past experiences are likely to be in
in contour, and rated more masculine faces, effective.
on average, ‘less attractive’. The researchers (e) The government officials can
concluded that modern women prefer men generally do little to influence
who are less obviously masculine in their directly the course of public policy.
facial features.

7 Adda247 Publications For any detail, mail us at


Publications@adda247.com
Join channel @cetexamgroup
A Complete Book of Logical Reasoning

15. Statement: Wars must be discouraged The argument above is based on which
vehemently even though majority of the of the following assumptions?
victims might have been a nuisance to (a) For every disease there is only one
peace loving people. strategy that can prevent its
Assumptions: occurrence.
I. Wars kill majority of wicked people. (b) In the future, genetics will be the
II. Innocent people are also killed in only medical specialty of any

p
wars. importance.
III. Vehement opposition to wars may (c) All human sicknesses are in part the
have some desirable impact. result of individuals’ genetic

ou
(a) Only I and II are implicit susceptibilities.
(b) Only III is implicit (d) All humans are genetically
(c) Only III and either I or II are implicit susceptible to some diseases.
(d) All are implicit (e) None of these.
(e) None of the above
18. Unlike other retail outlets, where items
16. Cases of food poisoning have been
are purchased in any number of units the

gr
reported from village X. After a dinner
customer wants, in super-markets items
party arranged for 100 people, 68 were
are grouped in bulk packages. This bulk
admitted to the hospital and 36 were
buying offers saving to the customer.
reported to be out of danger. The food,
which was cooked and stored in an open The option to buy at wholesale prices by
space for almost 12 h, was served after
reheating. Investigation is going on ‘A
m buying in bulk makes super-market a
practical choice for budget-conscious
news report.’ consumers.
Which of the following can be Which of the following assumption may
be derived from the above information?
xa
hypothesized from the above
information? (a) Super-markets often have greater
(a) Cases of food poisoning need to be buying power and lower overhead
handled carefully costs, so they can offer a greater
(b) State food is likely to be the cause of variety of products than regular
food poisoning retail outlets.
te

(c) Late night dinner parties for a large (b) Super-markets are often more
number of people result in food conveniently located and have
poisoning better parking facilities.
(d) Cases of food poisoning are not (c) The emergence of super-markets
ce

reported in urban dinner parties has caused many small retail stores
(e) None of the above to close down and thus eliminate
17. Within 20 year it will probably be competitions.
possible to identify the genetic (d) It is economically wise to buy single
susceptibility an individual may have items since bulk packages seldom
offer significant savings.
@

toward any particular disease.


Eventually, effective strategies will be (e) The financial savings from
discovered to counteract each such purchasing bulk packages may
susceptibility. Once these effective outweigh the inconvenience of
strategies are found, therefore, the being unable to purchase in any
people who follow them will never get number of units that suits the
sick. customers’ need.

8 Adda247 Publications For any detail, mail us at


Publications@adda247.com
A Complete Book of Logical Reasoning

19. The Government has appealed to all 21. Statement: The government is likely to
citizens to use potable water judiciously put some money in your bank account
as there is an acute shortage in supply. every month by deducting a portion of
Excessive use may lead to huge scarcity the subsidy each family receives through
in future months. various government welfare schemes
Which of the assumption is implicit in and putting an equivalent amount of
the above statement? cash in the account of the beneficiary.

p
(An assumption is something supposed Which of the following has been
or taken for granted) assumed in the above statement? (An
(a) People may ignore the appeal and assumption is something that is not

ou
continue using water as per their directly stated but is supposed or taken
consideration. for granted.)
(b) Government may be able to tap (a) Demand for goods and services will
those who do not respond to the go up.
appeal. (b) Subsidy pilferage will stop.
(c) Government may be able to put in (c) There will be a decrease in

gr
place alternate sources of water in corruption.
the event of a crisis situation. (d) Banks’ profit will increase.
(d) Large number of people may (e) Bank account holders will maintain
positively respond to the a minimum balance in their
Government’s appeal and help tide accounts.
over the crisis.
m 22. Basmati rice exporters are gearing up to
(e) Only poor are going to suffer from
top the market in China which recently
this shortage of water supply.
decided to allow imports from India.
20. Statements: “Although no war was Which of the following can be a possible
xa
witnessed during the tenure of prime assumption in the given statement? (An
ministership of Mr. X, a large number of assumption is something supposed or
top bravery medals were conferred upon taken for granted)
many cops in the name of curbing (a) Exporters believe, it may take a
terrorism and eliminating terrorists of couple of years to develop this new
te

different organizations belonging to our market, which largely consumes the


states”- view of a citizen. glutinous sticky rice.
Assumptions: (b) China presents a new market for
I. It is disgraceful of cops to name the Indian exporters, who have seen a
ce

act of killing of terrorists of our own surge in volumes of basmati in


country as an act of bravery. recent years.
II. A war is less harmful for a country (c) China’s market has monopoly in
than menace of terrorism. competition market.
Which of the following assumption(s) (d) Basmati rice is high in demand all
is/are implicit in contrast of the over the world.
@

statement? (e) None of these.


(a) Only I is implicit
23. Statement: The biggest story of this
(b) Only II is implicit
Budget is a major change in the way the
(c) Either I or II is implicit
Union Government spends its money.
(d) Neither I nor II is implicit
Earlier this week, Finance Minister Arun
(e) Both I and II are implicit

9 Adda247 Publications For any detail, mail us at


Publications@adda247.com
A Complete Book of Logical Reasoning

Jaitley announced that the government (a) Home-appliance usage would not
had accepted the recommendations of increase along with the energy
the 14th Finance Commission and raised efficiency of the appliances.
the States’ share in the net proceeds of (b) It would not be expensive to
union tax revenues from 32 per cent to produce home appliances that are
42 percent. With more of its pie going energy-efficient.
directly to States to spend as they like, (c) Home-appliance manufacturers
now have the technology to

p
the Union Government would begin
produce appliances that are twice
reducing its allocations to the State plan as energy-efficient as those
and wind up some Centrally Sponsored currently available.

ou
Schemes. The Central Government has (d) The cost of energy to the consumer
directed the State Governments to would rise with increases in the
reduce government expenditure in view energy efficiency of home
of the serious resource crunch and it appliances.
may not be able to sanction any (e) None of these
additional grant to the states for the next
25. The Govt. has decided to auction

gr
six months. construction of highways to private
Assumptions: entities in several blocks across the
I. The State Governments are totally country on build operate transfer basis.
dependent on Central Government Which of the assumption(s) is/are
for its expenditures. implicit in the above statement?
II. The Central Government has
m I. An adequate number of private
reviewed the expenditure account entities may not respond to the
of the State Government. Government’s auction notification,
III. The State Governments will abide II. Many private entities in the country
xa
by the directives of the central are capable of construction
highway within reasonable time.
Government.
III. The Govt.’s proposal of build-
You have to assume everything in the
operate-transfer may financially
statement to be true, and then decide benefit the private entities.
which of the three given suggested (a) I and II are implicit
te

assumptions logically follows. (b) II and III are implicit


(a) None is implicit (c) Only II is implicit
(b) Only II and III are implicit (d) I and III are implicit
(c) Only III is implicit (e) None of these
ce

(d) All are implicit 26. Radio stations with radio data system
(e) None of these (RDS) technology broadcast special
24. Twenty percent of all energy consumed program that only radios with an RDS
in the country is consumed by home feature can receive. Between 1994 and
1996, the number of RDS radio stations
appliances. If appliances that are twice
in Verdland increased from 250 to 600.
@

as energy-efficient as those currently However, since the number of RDS


available are produced, this figure will equipped radios in Verdland was about
eventually be reduced to about ten the same in 1996 as in 1994, the number
percent. of Verlanders receiving the special
The argument above requires which of program information probably did not
the following assumptions? increase significantly.

10 Adda247 Publications For any detail, mail us at


Publications@adda247.com
A Complete Book of Logical Reasoning

Which of the following assumption can II. Tetracycline is not rendered


be drawn from the above passage? ineffective as an antibiotic by
I. Few if any of the RDS radio stations exposure to the process involved in
that began broadcasting in making bread and beer.
Verdland between 1994 & 1996 III. Typhus cannot be transmitted by
broadcast to people with RDS- ingesting bread or beer
equipped radios living in areas not contaminated with the infectious
agents of this disease.

p
previously reached by RDS stations.
II. In 1996 most Verdlanders who IV. Bread and beer were the only items
lived within the listening area of an in the diet of the ancients Nubians
which could have contained

ou
RDS station already had a radio
tetracycline.
equipped to receive RDS.
V. Typhus is generally fatal.
III. Equipping a radio station with RDS (a) None follows except II
technology does not decrease the (b) Only III & V follow
station's listening area. (c) Only III follows
IV. In 1996 Verlanders who did not (d) All follow except I

gr
own radios equipped to receive RDS (e) None of these
could not receive any programming
from the RDS radio stations that 28. Statement: By asking five prominent
began broadcasting in Verdland hospitals in the national capital to
after 1994. deposit nearly Rs. 600 crore to
compensate for their failure to treat poor
V. The RDS radio stations in Verdland
m
in 1996 did not all offer the same
patients, the Delhi government has
drawn attention to the social obligation
type of programming. of healthcare providers in the corporate
(a) Only II & III follow sector as well as the need for timely
(b) All follow except III
xa
enforcement of applicable regulations.
(c) None follows except I According to the Delhi government,
(d) Only V & II follow trusts and registered societies to which
(e) none of these public land was allotted to establish
27. The ancient Nubians inhabited an area in hospitals were required to earmark a
which typhus occurs, yet surprisingly percentage of their medical facilities and
te

few of their skeletons show the usual services for indigent patients.
evidence of this disease. The skeletons Assumptions:
I. This strengthens the case for
do show deposits of tetracycline, an
private hospitals to dedicate a part
antibiotic produced by a bacterium
of their services to those who
ce

common in Nubian soil. This bacterium cannot afford treatment.


can flourish on the dried grain used for II. Social responsibility of hospitals
making two staples of Nubian diet, beer must be monitored and central
and bread. Thus, tetracycline in their government should ensure
food propably explains the low incidence compliance by the corporate
of typhus among ancient Nubi
@

hospitals.
Which of the following is an assumption III. There is a great need for the
can be drawn from the passage? government to monitor and enforce
I. Infectious diseases other than “health services” so that the poor
typhus to which the ancient who cannot afford modern health
Nubians were exposed are facilities that are extremely costly-
unaffected by tetracycline. are able to be benefited.

11 Adda247 Publications For any detail, mail us at


Publications@adda247.com
A Complete Book of Logical Reasoning

Which of the following 31. Statement: In order to prevent


assumption(s) is/are implicit from catastrophic damages to its assets,
the given statement? Railways is going for offshore
(a) All are implicit procurement of high-powered sensor-
(b) Only III is implicit based equipment, a first-of-its-kind for
the public transporter, to detect defects
(c) Only I is implicit
on tracks and rolling stock.
(d) Only I and II are implicit

p
Which of the following has been
(e) None is implicit assumed in the given information? (An
29. Statement : India’s economic growth assumption is something that is
supposed or taken for granted.)

ou
has come at a terrible price of increased
(a) Accidents will be minimized with
industrial and vehicular pollution.
the regular maintenance of tracks.
Assumptions: (b) The safety aspect of track and
I. Pollution is a part of industrial rolling stock will be enhanced.
society. (c) Railways will be able to know the
II. India economic growth is based on exact condition of tracks through

gr
only industrial growth. micro-analysis of data and this will
III. A country desires economic growth reduce the possibility of sudden
with manageable side effects. catastrophic failures.
(a) Only I is implicit (d) Railways will prepare a predictive
maintenance schedule for its tracks.
(b) Only II is implicit
(c) Only III is implicit
m (e) Only (a) and (b)
(d) Only I and III are implicit Directions (32-33): In each question below
(e) None of these is given a statement followed by two
assumptions numbered I and II. An
xa
30. Statement: According to the manpower assumption is something supposed or taken
Employment Outlook Survey, globally, for granted. You have to consider the
Indian employers are the most upbeat on statement and the following assumption and
hiring plans in the fourth quarter ending decide which of the assumption is implicit in
December 2015. the statement.
te

Which of the following can be an Give answer (a) If only assumption I is


assumption in the given information? implicit.
(An assumption is something that is Give answer (b) If only assumption II is
implicit.
supposed or taken for granted)
Give answer (c) If either assumption I or
ce

(A) Decline in the profit margins of the assumption II is implicit.


company Give answer (d) If neither assumption I nor II
(B) Establishment of more branches by is implicit.
the MNCs in India Give answer (e) If both assumption I and II
(C) Maintenance of positive growth are implicit.
@

rate by the MNCs in India. 32. Statement: Mohan tells Nita, “Let us
(D) Increase in the growth of service meet over lunch tomorrow.”
sector in June 2015. Assumptions:
(a) Only A and C (b) Only B and D I. Lunch timings are known to both.
(c) Only A and B (d) Only C and D II. Both are aware of the venue for
(e) Only B and C lunch.

12 Adda247 Publications For any detail, mail us at


Publications@adda247.com
A Complete Book of Logical Reasoning

33. Statement: The movie is a super-duper Assumptions:


hit and has broken all the records. I. People know about K-series engine.
Assumptions: II. Engine type/series is important for
I. There is an authentic criterion to buyers.
judge a hit or a flop.
II. The performance of earlier movies 37. Statement:
is known. The two countries have signed a fragile
pact, but the vital sovereignty issue

p
Directions (34–36): In each question below remains unresolved.
is given a statement followed by two Assumptions:
assumptions numbered I and II. An I. The two countries cannot have

ou
assumption is something supposed or taken permanent peace pact.
for granted. You have to consider the II. The two countries may become
statement and the following assumption and hostile again after a short spell of
decide which of the assumption is implicit in time.
the statement. (a) Only assumption I is implicit
Give answer (a) If only assumption I is

gr
(b) Only assumption II is implicit
implicit.
(c) Either I or II is implicit
Give answer (b) If only assumption II is
(d) Neither I nor II is implicit
implicit.
(e) Both I and II are implicit
Give answer (c) If either assumption I or
assumption II is implicit. 38. In each question below is given a
m
Give answer (d) If neither assumption I nor II statement followed by two assumptions
is implicit. numbered I and II. You have to consider
Give answer (e) If both assumption I and II the statement and the following
are implicit. assumptions and decide which of the
xa
34. Statement: Job rotation helps assumptions is implicit in the statement.
employees to get an overview of the Statement:
organization. A's advice to B - "Go to Jammu via
Assumptions: Amritsar - the shortest route".
I. Job rotations is only method to get Assumptions:
te

an overview of the organisation. B wishes to go to Jammu.


II. It is required to have an overview of A gives advice to everybody.
the organisation. (a) Only assumption I is implicit
(b) Only assumption II is implicit
35. Statement: Let us appoint Ms. X as the
(c) Either I or II is implicit
ce

CEO of our company so that the


(d) Neither I nor II is implicit
company’s products are also perceived
to be genuine. (e) Both I and II are implicit
Assumptions: Directions (39-43): In each question below
I. CEO can changes the perception of are given a statement followed by two
products.
@

assumptions numbered I and II. An


II. Perception is same as the actual assumption is something supposed or taken
reality. for granted. You have to consider the
36. Statement: An advertisement. The new statement and the following assumptions and
model has been launched with K – series decide which of the assumptions is implicit in
engine. the statement.

13 Adda247 Publications For any detail, mail us at


Publications@adda247.com
A Complete Book of Logical Reasoning

Give answer- Assumptions:


(a) If only assumption I is implicit. I. Length of service or seniority does
(b) If only assumption II is implicit. not alone reflect merit of an
(c) If either I or II is implicit. employee.
(d) If neither I nor II is implicit. II. It is possible to determine and
(e) If both I and II are implicit. measure merit of an employee.
43. Statement: Highly brilliant and
39. Statement: ‘You are expected to be

p
industrious students do not always excel
frank and objective while writing your
in the written examination.
self-appraisal report’. –An instruction for Assumptions:
writing self-appraisal report.

ou
I. The written examination is good
Assumptions: mainly for mediocre students.
I. Unless cautioned, people may tend II. The brilliant and industrious
to be little shy and less objective students cannot always write good
while writing their self-appraisal answers in the exam.
report.
Directions (44-47): In each question below
II. Every self-appraisal report helps

gr
is given a statement followed by two
the person in his further assumptions numbered I and II. An
development. assumption is something supposed or taken
40. Statement: The higher echelon of any for granted. You have to consider the
organization is expected to be models of statement and the following assumptions and
observational learning and should not be
m decide which of the assumptions is implicit in
the statement.
considered as merely sources of reward
Give answer-
and punishments.
(a) If only assumption I is implicit
Assumptions: (b) If only assumption II is implicit
xa
I. Employees are likely to be sensitive (c) If either I or II is implicit
enough to learn by observing the (d) If neither I nor II is implicit
behavior of their bosses. (e) If both I and II are implicit
II. Normally bosses are considered as
sources of reward and punishment. 44. Statements Instead of burning the
leaves, bury them in compost pits, by
te

41. Statement: ‘But, out of A, B, C and D which it gets converted to natural


products, you buy ‘B’ which alone is manure, making it beneficial for the soil.
based on the international technology’- A A notice issued in public interest by
shopkeeper tells a customer. Department of Environment
Assumptions
ce

Assumptions:
I. The customers normally accept the I. Whenever leaves are burnt in the
recommendation of the open, the air gets laden with tiny
shopkeeper. particulate matter which raises air
pollution to alarming levels, which
II. Use of international technology is
cause severe respiratory disorders
supposed to ensure better quality
@

and eye infections to those exposed


standards. to it.
42. Statement: The organization should II. Benefits gained from ashes of leaves
promote employees on the basis of merit burnt are not as much as the
alone and not on the basis of length of benefits gained from the natural
service or seniority. manure obtained from leaves by
burring it.

14 Adda247 Publications For any detail, mail us at


Publications@adda247.com
A Complete Book of Logical Reasoning

45. Statements “A tempting cup of garma 48. Clive is a great fan of P.G. Wodehouse, a
Georgia now awaits you at every street great writer of humorous novels, and he
corner. So, no matter whom you are or has read all the novels written by the
where you go, a Georgia Vending author. It shows that Clive is a jovial
Machine will hand you the same clean, person and does not take things very
delicious cup of tea in Regular, Adrak,
seriously in his personal life.
Elaichi and Masala and if you’re looking
Which of the following must be the most

p
for a change try the Regular, Mocha and
Cappuccino coffee. One sip will make you important assumption in drawing the
realize why every other alternative is a above conclusion?

ou
mere compromise!” ____ An advertisement (a) P.G. Wodehouse does not write
Assumptions serious novels.
I. Most of the people need delicious (b) Literary choices reflect the personal
cup of tea or coffee with a change in attitude of a person.
taste. (c) Clive has not met with any major
II. Every person is addicted to either adversities in his personal life.

gr
tea or coffee. (d) The society in which Clive lives
46. Statements “If you ask me about the likes his jovial nature.
daunting challenges that I face, I would (e) None of these
say that my government’s first priority is
to improve the existing law and order 49. The number of surgeons in the country
situation. Then follows the issue of
m has registered a two-fold increase in the
prices of commodities.” ___ Mr. ROY, a past three years. However due to the
newly appointed PM of country advent of non-invasive medical
CLINTON treatments, the total number of
xa
Assumptions surgeries performed has also come down
I. If a citizen of country CLINTON can drastically. This is bound to lead to a
sleep peacefully, he/she can then significant decline in surgical skills of
think of providing food to his/her surgeons.
family, ponder over education and
Which of the following is an assumption
move about freely in the country.
te

II. Prices of the commodities affect the in the above argument?


common man greatly. (a) Of late people prefer non-invasive
medical treatments to surgeries.
47. Statements “If you have any unresolved (b) Surgeons by and large are reluctant
consumer disputes, do not feel that you
ce

to use methods other than surgery,


are helpless, do not hesitate to assert
your rights. Approach the District for treating the patients.
Consumer Forum for speedy redressal.” (c) The general health condition of the
____ Department of Consumer Affairs people has improved so much that
Assumptions diseases or physical disabilities
@

I. People don’t want to approach warranting surgical operations


consumer forum due to the red- have come down in number
tapism in procedure adopted by the (d) Constantly performing surgeries
forum. alone can keep the surgical skills of
II. Speedy redressal will attract more surgeons in tact.
unresolved consumer disputes. (e) None of these

15 Adda247 Publications For any detail, mail us at


Publications@adda247.com
A Complete Book of Logical Reasoning

50. Statement: Govt. has taken decision in construction work of highway


many blocks of country that the within given time period.
construction work of highway is given to III. It may be that private forums get
the private forums of builder operator financial profit due to govt. builder
transfer on the basis of action.
operated proposed schemes.
Assumptions:
I. It may be possible that a sufficient (a) Only I and II are implicit.

p
number of private forums not (b) Only II and III are implicit.
answer in the respect of Govt. (c) Only II is implicit.
action.
(d) Only I and III are implicit.

ou
II. Many private forums of our country
are capable to complete the (e) None of these.

Statement and Assumption : Solutions

gr
1. (d); All the statement except (d), cannot convenient. All other option except
be assumed from the given (e) cannot be assumed quality and
statement as data provided in the price offered by company V for the
statement is not enough to assume product is not mention in the given
the increase or decrease in NPA statement. And also quality is only
m
from the previous year also it is not parameter to increase the number
clear whether NPA is the only cause of customer or it is the only way
of pain to banking sector. Further also cannot be hypothesized from
we also cannot assume from the the given statement.
xa
given statement that steps taken by
4. (a); Only I and II can be assumed from
the government are not enough or
the given statement as I is directly
some other sector is performing
given in the statement that
better than it. But it can be clearly
investigating agencies are going to
assumed from the given statement
use the Aadhar database for their
te

that after the proper


investigations which includes the
implementation of the policies and
fingerprints of criminals. And II can
reforms there will be some
also be assumed from the given
improvement in the condition of the
statement which states that limited
banking sector.
ce

access leads to assurance and


2. (e); Either (I) or (II) can be assumed clearly with increase in access,
from the given statement as better vulnerability also increases. While
opportunities or virus can be the III cannot be assumed as it is not
reason behind the major step taken mentioned in the statement.
@

by the population of village A.


5. (a); The statement I talks about that in
3. (e); Statement (e), can be assumed from the tender age the separation of
the given statement as better children from their parents will
quality is always a major factor of result in negative impact on a
concern for customers and for it child’s growth which can be
paying a bit higher amount is assumed from the given statement

16 Adda247 Publications For any detail, mail us at


Publications@adda247.com
A Complete Book of Logical Reasoning

and In statement II, it talks about 9. (d); For I: Yes, it can be assumed as it is
the well trained clinicians and staff mentioned clearly in the statement
at the shelters who are licensed to that SSC aspirants have decided to
take care of the children which is move towards banking field after
also can be assumed from the given getting fade up by paper leak of SSC,
statement as it states that lawyers it means clearly that SSC
who visited the sites said they were management has not taken any

p
safe and clean. Also the statement proper action.
III is criticizing the policy as For II: Yes, it can be assumed as
needless and cruel which is also aspirants move to banking field by

ou
stated in the statement that the supposing that there will be no
condition of children after paper leak in banking examination.
separation is Hysterical. For III: No, It cannot be assumed
because this is regarding career in
6. (a); All the statement can be assumed as banking field which is not discussed
because these statements talks in statement.

gr
about the parents playing an
10. (e); For I: Yes, about counterfeiting is
integral role in the child’s growth
mentioned, whether ban on high
both physically and mentally how currency notes gets success in
they shape their child’s future so terrorism, corruption and
that he/she became a better person counterfeiting. So, we can easily
for his /her family and for the
m assume that Govt. was concerned
society. about the counterfeiting of notes.
For II: No, as according to statement
7. (d); In the above question we have to
demonetization is good or bad is a
find which statement can be
xa
matter of debate, but its impact on
assumed from the given statement. weaker section is not mentioned.
For I- True, because cyber-attacks For III: No, from statement we can
had a tremendous impact on all the conclude that there is revamp in tax
countries as given in statement that compliance but to classify its
it infected thousands of computer increment into direct and indirect
te

system. tax, it’s not possible to assume


For II-True, as it is clear from the according to statement.
above statement that it is a 11. (e); For I: True, as per vikalp plan it is
frightening reminder all over the certain everyone will get confirm
ce

world. seats, means facilities for


For III- False, because it is not clear passengers will be increased. Then
from the above statement about its it may be assumed that fares will
linkage with any terrorist get increased.
organizations For II: True, as vikalp plan has come
@

to resolve the waiting status, it


8. (d); Only (I) can be assumed from the means commuters would have
given statement as it is mentioned faced issues in past.
in the given statement that rooftop For III: True, as there is certainty of
solar power technology is showing confirm seats, Then it may be that
growth. number of commuters will increase.

17 Adda247 Publications For any detail, mail us at


Publications@adda247.com
A Complete Book of Logical Reasoning

12. (c); For I: False, as Mr. Y has expressed for almost 12 hr. and was served
his feelings with confidence that he after reheating. It may be cause of
is enjoying his work in his current food poisoning. But late night
company but in assumption it is dinner or state food can’t be reason
assumed that he is nervous to for food poisoning. And it is
change his company as he don’t nowhere in information mentioned
want to move out from his comfort in urban dinner parties, this case

p
zone. doesn’t occur.
For II: True, as Mr. Y has said in
17. (c); The argument states that people
affirmation that company is also
who follow the appropriate

ou
rewarding honestly.
strategies for counteracting their
For III: False, as Mr. X has only
genetic susceptibilities to disease
asked to Mr. Y so we can’t say
will never get sick. How can it be
anything about Mr. X
that they would never get sick? The
13. (a); The conclusion refers to “modern argument requires an assumption
women” a fact that suggests the about the cause of all sickness that

gr
authors think their work applies to every known disease of humans
all modern women. Hence they do must correspond to some genetic
assume what is stated in (a). susceptibilities. So, (c) is the correct
option.
14. (e); The paragraph is really an
argument for the position that
m 18. (e); Buying in wholesale and bulk
government leaders should act depend upon the choice of the
more quickly to address problems. customer’s choice and may cause of
So, the passage is committed to the financial saving.
xa
idea that public policy is under the
19. (d); Option (d) is an assumption. Any
control of the government. It just
appeal has some effects and people
has not been effectively used in the
generally respond positively to any
past.so (e) cannot be the valid
appeal.
argument.
20. (d); I is not valid and II is also cannot be
te

15. (a); The statement mentions that ‘…


assumed from the given statement.
majority of the victims might have
been a nuisance to peace loving 21. (b); The transition to cash subsidy has
people. This implies that majority of been made with the assumption
ce

the victims are wicked people that pilferage that takes place due
though some victims are innocent to middlemen will stop.
people too. So, both I and II are
22. (e); All the statements have the
implicit. Further, the statement
additional information which can’t
advocates vehement opposition of
be generally assumed.
wars. So, III is not implicit.
@

23. (b); Nothing about the sources of


16. (e); None of the above can be
income of the State Governments is
hypothesized from the above
mentioned in the statement. So, I is
information. Option (a) is a
not implicit. Since the Central
preventive action. And it is clearly Government has directed the State
mention that food was kept in open
Governments to reduce

18 Adda247 Publications For any detail, mail us at


Publications@adda247.com
A Complete Book of Logical Reasoning

expenditure, so II is implicit. other than typhos. statement III is


Further, since the Central also not valid because Typhus is a
Government has refused any disease and the statement talks
further grant to the State about the contamination of a
Governments for the next six disease by another disease which is
months, it is quite clear that state wrong in the context of this
Governments will abide by the passage. Statement IV & V are

p
directives of central Government. invalid as No information is given in
So, III is also implicit. the passage that bread & beer were
their only items in the diet and also
24. (a); This argument is based on the

ou
about Typhus is fatal or not.
assumption that if the appliances
become more efficient, then more 28. (c); Such strict actions of Delhi
homes would not be tempted to government made private hospitals
increase their utility/consumption to serve their services to poor
in the household. people. So I is implicit. But nothing
is mentioned about its motorization

gr
25. (b); Only II and III assumptions are in the statement. So, II and III are
implicit because these two are not implicit.
relevant with situation while I is not
29. (d); The statement mentions that India
relevant for the statement. had to pay the price of increased
pollution level to earn its economic
because the number of RDS radio
m
26. (c); Statement I is the valid assumption
growth. So, both I and III are
increased from 1994 to 1996 so the implicit. However, this does not
people with RDS-equipped radios imply that only industrial growth
has brought about India’s economic
could use RDS technology.
xa
growth. So, II is not implicit.
Statement II is not valid as it says
that Most Verdlanders had a radio 30. (e); Decline in profit will lead to closure
equipped to receive RDS and of the company, so option a) is not
nothing is said about this in the valid. Establishment of new
passage. Statement III& V are not branches by the MNCs will lead to
upsurge in the hiring activities. At
te

valid because nothing is said about


the same time, maintenance of
station’s listening area and type of
positive growth rate by the Indian
programming. Statement IV can be MNC companies will help them
the conclusion of the passage but expand their business. So only B)
ce

not a valid assumption. and C) follow.


27. (a); Statement II is the valid assumption 31. (e); With the proper maintenance of
as tetracycline is produced by a tracks the safety of tracks will be
bacterium which is found in Nubian enhanced and the no. of accidents
soil and the bacterium can develop will be minimized. So only a) and b)
@

on the grain which is used in follow.


making bread & beer.so tetracycline Directions (32-33):
is formed through the process of 32. (e); By close observing we can say that
making of bread & beer. Statement I both I and II are implicit.
is not valid because nothing is said
33. (b); By close observation we can say
in the passage about the disease that only II assumption is implicit.

19 Adda247 Publications For any detail, mail us at


Publications@adda247.com
A Complete Book of Logical Reasoning
34. (b); ‘Only’ word makes weak the 42. (e); Both assumptions I and II are
assumption. Only IInd assumption implicit in the statement. The
is implicit. statement implies clearly that
35. (a); Assumption I implicit while IInd is length of service or seniority does
not. not alone reflect merit of an
employee. uses the word ‘alone’ in
36. (e); Both I and II are implicit because the assumption. If it is not possible
people know well about K-series to measure the merit of an

p
engine according to statement also employee why such statement has
it is important for buyers. been made.
37. (b); From the fact that the present pact

ou
43. (b); Only assumption II is implicit in the
is not a lasting one, the possibility
of a permanent pact cannot be statement.
ruled out. So, I is not implicit. The Directions (44-47):
statement mentions that the
present pact is a 'fragile' one and 44. (b); Assumption I goes very deep. Hence
the vital sovereignty issue still it is not implicit. But assumption II
is implicit.

gr
remains unresolved. So the same
issue may rise again in the future. 45. (a); Assumption I is implicit.
Thus, II is implicit.
46. (d); I is not implicit. II is not implicit
38. (a); A has advised B the route to Jammu. because the PM only assumes that
This means that B wishes to go to law and order affects the common
Jammu. So, I is implicit. The
m
statement mentions only A's advice
man more than prices do.
to B. So, II is not implicit. 47. (b); Assumption II is implicit.
Directions (39-43): 48. (b); if the nature of the novels preferred
39. (a); Only assumption I is implicit. The
xa
by a person reflects his nature then
instruction was issued assuming the conclusion is properly drawn.
that people tend to be little shy and (b) is the correct choice. (a) and (c)
less objective while writing their cannot be assumed. (d) is irrelevant
self-appraisal report if not so to the argument.
instructed. Assumption II is not
49. (d); As it is suggested that surgical skills
te

properly related with the


statement. will suffer a decline on account of
the fall in the number of surgeries,
40. (e); Both the assumptions I and II are so (d) is the correct answer because
implicit in the statement. If constant practice is the way for
employees do not learn by
ce

continuous growth. (a) concern


observing the behavior of their
non- invasive methods. (b) is not an
bosses the statement would not
have made again it has been assumption. As the arguments is
highlighted that bosses should not not about why the number of
be considered as merely source of surgeries has come down, (c) is
reward and punishment. It implies incorrect.
@

that bosses are generally


considered as source of reward and 50. (b); Only II & III are implicit. It may be
punishment. Hence assumption II is possible that private forums can
implicit. complete the work of highway in a
given time and also get financial
41. (e); Both assumptions I and II are profit.
implicit in the statement.

20 Adda247 Publications For any detail, mail us at


Publications@adda247.com
A Complete Book of Logical Reasoning

p
ou
gr
m
xa
te
ce
@

1 Adda247 Publications For any detail, mail us at


Publications@adda247.com
A Complete Book of Logical Reasoning

03 Course of Action
Indroduction: A course of action is ‘a step or administrative decision to be taken for
improvement, follow-up or further action with regard to the problem, policy etc. on the basis of

p
the information given in the statement’. The question in this section, thus involve finding the
appropriate course of action, considering the problem or policy being talked about in the

ou
statement.
In this type of question, a statement is given followed by two courses of action numbered I and
II. The candidate is required to grasp the statement, analyze the problem or policy it mention
and then decide which of the course of action logically follow(s).
Format of the problem: Normally, the directions of the questions will be long and reading them
in the examination hall is time consuming. Hence, understanding the question properly and

gr
quickly is more important.
Key points to keep in mind with respect to courses of action.
(1) Course of action should be RELEVANT: The problem or situation at hand should be
directly related to the course of action and it should not overshadow the given situation.
m
(2) Course of action should have IMMEDIATE IMPACT: The course of action should be such
that there should be immediate action involved in it to solve the problem.
Immediate action does not mean that problem is immediately solved, which is not possible
xa
in most of the cases. It refers to take immediate action which can take some time to handle
the situation.
Statement: The city has been troubled with high pollution levels for the past ten years.
Course of Action: The polluting factories should be shifted outside the city.
The suggested course of action does not refer to shift the factories immediately. It mean to
te

pass the suitable law and give the factories due time to shift. The course of action follows.
(3) LONG TERM AND SHORT TERM Courses of Action: The problems in the statement can be
generally divided into two categories: Short term and Long term. The problems related to
poverty, pollution, corruption, unemployment, epidemic etc are long term. They can be
ce

followed by long term courses of action. Short term problems are generally not followed by
long term courses of action.
Statement: There has been a drop of 10% literacy rate as compared to 2001 as mentioned
in the latest census report.
Course of action: The government should allocate special funds for development of
@

education in the yearly budgets and five year pl


Being a long term problem, course of action can be long term.
(4) The Course of Action should be taken with reasonable proof.
Statement: The Chief Minister is suspected to have given illegal contracts of mining to
family members.

2 Adda247 Publications For any detail, mail us at


Publications@adda247.com
A Complete Book of Logical Reasoning

Course of Action: The minister should be forced to step down as the Chief Minister. This
Course of Action is taken on the basis of suspicion; hence, it cannot be justified. The Course
of action does not follow.
Statement: The investigation bureau evidently proved cases of illegal mining against the
State’s Chief Minister.
Course of Action: The minister should be forced to step down as the Chief Minister.

p
This Course of action follows, as it is justified, with the evidences from the bureau.
(5) The Course of action cannot be illegal.
All the actions, no matter how big the problem is, should be under the legal jurisdiction.

ou
Even if the problem can be solved, no illegal means can be adopted.
Statement: In the year before elections, at least ten high profile cabinet ministers were
convicted of corruption charges causing a loss of crores of rupees to the central
government.
Course of Action: All the convicted ministers should be killed by hanging them publically.

gr
This course of action doesn’t follow. No doubt with the killings, the corrupt minister will be
removed forever and many will be afraid to do such acts in the future. But, it is an
undemocratic and illegal act to follow.
(6) The Course of Action should not be EXTREME:
m
Let us study a course of action for the statement in the previous point.
Course of Action: The political parties which the ministers were representing, should
never be allowed to contest for elections in the future.
xa
This course of action cannot follow as it is an extreme step to take the contesting right away
from all the party members.
Let study another example:
Statement: In the past few months, many cases have been reported against private courier
companies for delays in scheduled delivery timing.
te

Course of Action: Such private courier companies should be banned.


It is an extreme course of action, as services given to millions of consumers would be
disrupted and it would lead to bigger problems for the government.
(7) The course of action cannot follow for one off cases. (i.e. rules are not changed for
ce

exceptions)
Statement: The student entered the class when half of the lecture was already delivered.
He requested the lecturer to repeat what he has already taught, as he met with an accident
and couldn’t reach on time.
@

Course of Action: The lecturer should start the lecture again.


This course of action cannot follow as rules cannot be changed for exception. Other students
cannot be made to suffer because of one student.
(8) A Course of Action cannot be repeated if it has previously failed. We cannot accept the
optimism, that the previously failed step would be successful if implemented again.

3 Adda247 Publications For any detail, mail us at


Publications@adda247.com
A Complete Book of Logical Reasoning

(9) A Course of Action should be PREVENTIVE in nature. No matter, how convincing or


relevant a course of action appears, it should have a tendency to reduce/remove the
problem in future or handle a situation in hand.
Statement: A group of school students was reported to be enjoying at a picnic spot during
school hours.
Course of action:

p
I. The principal of the school should contact the parents of those students and inform
them with a real warning for future.
II. Some disciplinary action must be taken against those students for the awareness of all

ou
the other students.
Both the courses of action are relevant to problem and can be immediately
implemented. Along with it, these actions tend to control the problem from occurring in
the future, hence preventive in nature.
(10) OPINIONS (irrelevent) are not Course of Action: A valid course of action comprises of an

gr
act which is practical, feasible and falls within the domain of legal jurisdiction. Any action
violating the above mentioned characteristics is an opinion, hence, not practically
applicable.
Statement: A large number of people visiting India from Country X have been tested
m
positive for carrying viruses of a killer disease.
Course of Action: The government of India should immediately put a complete ban on
people coming from Country X including those Indians who are settled in Country X.
This course of action is neither practical nor feasible, being solely an opinion. It cannot
xa
follow.
Important note: While approaching a question set, read the statement carefully and
understand the domain and extremity of the statement. Observe key words like, “all”,
“immediate”, “impossible”, “completely” etc. for better understanding.
te

Always keep aside your personal opinion about the statement.


Read the Course of action carefully and try to correlate it with the statement, ignoring “any”
other course of action in your mind that is not mentioned in the question.
• Questions are of the following format
ce

Directions: In the following questions a fact situation is given followed by two suggested
courses of action. Read the situation and then decide which of the given courses of action
follows.
Give answer
(1) if only course of action I follows
@

(2) if only course of action II follows


(3) if either course of action I or course of action II follows.
(4) if neither course of action I nor course of action II follows.
(5) if both courses of action I and II follow.
• Some examples based on the above concept:-
4 Adda247 Publications For any detail, mail us at
Publications@adda247.com
A Complete Book of Logical Reasoning

(1) Statement: Drinking water supply is getting polluted due to leakage in pipes.
Courses of action:
I. The government should order an enquiry into the matter.
II. The civic body should set up a fact-finding team assess the damage and take effective
step.
Explanation: Considering the nature of problem, course of action I is not necessary. Again, it
is mentioned that water supply is disrupted due to loss of water owing to leakage in pipes

p
supplying owing to leakage in pipes supplying water. Thus, reason for disruption of water
supply is known. Then, what is the need of setting-up of fact-finding mission,.
So, both course of action do not follow.

ou
(2) Statement:- There is an alarming increase in the number of people suffering from malaria in
many parts of the city.
Courses of action:
I. The municipal corporation has advised all the government hospitals to store adequate
supply of malaria drugs.
II. The municipal corporation has urged people to use mosquito repellants and keep their

gr
premises clean.
Explanations: Both the courses of Action are suitable for pursuing. In case of outbreak of
malaria adequate supply of drugs is necessary. Again the people should use mosquito
repellants and keep their premises clean to check breeding of mosquitoes.
m
(3) Statement: Many people have encroached into the government property and built their
houses and business establishments.
Courses of action:
I. The government should take immediate steps to remove all unauthorized constructions
on government land.
xa
II. All the encroachers should immediately be put behind bars and also be slapped with a
heavy fine.
Explanation: Only course of Action I seems to be suitable for pursuing. Course of Action II is
too harsh to be suitable.
(4) Statement: People residing in some tribal areas are far from education.
te

Courses of action:
I. Schools for children and adults should be opened there.
II. Social workers should be entrusted with the job of educating them.
Explanation: Both course of actions are suitable.
ce

(5) Statement:- Youngsters are often found starring at obscene posters.


Courses of action:
I. Children should be punished and penalized if they are found doing so.
II. Any display of such banners should be banned.
Explanation: 1st action is not a judicious step in dealing such problem. But putting a
@

complete ban can be helpful in solving the problem by way of preventing from such
exposures.
Hence only II course of action follows.
• Hope, you have understand the detailed concept of courses of action.
For more speed, accuracy and clarification practice our Assignments……………

5 Adda247 Publications For any detail, mail us at


Publications@adda247.com
A Complete Book of Logical Reasoning

Points to Remember
(1) Evaluating courses of action is a major area of logical reasoning. This type of questions is intended to
scrutinise the decision-making skills of the candidate.

p
(2) You should take that course of action which are of solving, minimizing or reducing type nature.
(3) Remember that course of action should be practical in nature.
(4) Some suggested course of action may indeed solve a problem but in practical life it may not be advisable or

ou
possible. If it is so, then that course of action is rejected.
(5) Never allow your personal perceptions to come into play while evaluating the suggested courses of action.
(6) Courses of action should be impartial and impersonal.
(7) Try to analyze the given courses of action in your first attempt. so, that this habbit will help you to solve these
type of questions in less time.

gr
Exercise

1. Over the past few years, there has been a (a) Country A should stop all its
sharp rise in the number of accidents imports from country B.
involving cyclists with the four wheelers.
m (b) Heavy taxes should be implemented
Which of the following is the most by country A on the beer that is
effective as well as an appropriate imported from country B.
course of action in light of the above (c) People should be advised to cut
xa
statement? down on their beer consumption.
(a) A separate track should be
(d) The samples of beer imported from
developed for the cyclists.
(b) A ban should be implemented on country A should be sent for
the sale and the use of cycles. chemical analysis.
(c) Fares of public transport must be (e) A ban should be imposed on the
te

reduced. sales of beer imported from country


(d) All the drivers of four wheelers B until the matter is investigated.
involved in the accidents with the 3. Statement: With the record output,
cyclists should be penalized.
sugar prices have dropped from an
(e) Free helmets should be provided to
ce

average of Rs 37 a kg in the previous


all the cyclists in the city.
season to Rs 26 in the current season
2. Statement: In county A, there have been which results in the sugar mills’ dues to
numerous reports over the past three farmers stands at Rs 22,000 crore.
months about people falling sick after Which of the following is an immediate
consuming the beer imported from
@

course of action for the given statement?


country B. The beer manufacturers in I. The cabinet committee should
county B are claiming a conspiracy by
allocate Rs 7000 crore package to
the local manufacturers of country A.
Which of the following is a most sugar sector with a special Cess
appropriate course of action in light of under GST to alleviate distress
the above statement? among sugarcane farmers.

6 Adda247 Publications For any detail, mail us at


Publications@adda247.com
A Complete Book of Logical Reasoning

II. The government should procure true, and then decide which of the suggested
sugar from mills at a fixed courses of action logically follow(s) for
minimum price to help them clear pursuing.
dues to farmers. Give answer:
III. The government should levy (a) if only I follows.
minimum prices and offer special (b) if only II follows.
incentives to increase the (c) if either I or II follows.

p
production of molasses and ethanol. (d) if neither I nor II follows.
(a) Only I and III (b) Only II and III (e) if both I and II follow.
(c) All of the above (d) Only I and II 5. Statement: Some workers of the

ou
(e) None of these company were caught smoking in the
company lawn during office time.
Direction (4): Study the following
Course of action:
information in which a statement is followed
I. The management of the company
by some course of action read carefully and should immediately fire those
answer the question below- employees who were caught
smoking during office hours.

gr
The Uttarakhand high court has banned
paragliding and white-water rafting and other II. Warning should be issued to those
water sports. This move aimed at employees who were caught
safeguarding both the environment and smoking during office hours.
people engaging in these activities. The 6. Statement: A huge truck collided with

tourism and water sports in the Himalayan


m
court’s decision comes as a jolt to adventure an elephant trying to cross the highway
road running across the forest causing a
state. heavy traffic jam.
Course of action:
Course of Action- I. High voltage electric wires should
xa
I. An appeal should be filed against this be used to fence all the state
decision as It’s an industry that provides highways.
indirect jobs to no less than 10,000 II. People should be advised to avoid
families. using the route effected by collision
II. The tourism must be promoted but it is till the highway is cleared.
required to be regulated and for this the
te

Direction (7-8): In the following question a


State government should be directed to statement is given followed by two
prepare the transparent policy. statements numbered as I and II. You have to
III. All the villages and cities near the banks read all the statements and decide which of
of the rivers should be ceased to reduce the following is the suitable course of action
ce

the pollution in the rivers. for the mentioned statement above.


Which of the following is an immediate course
of action for the given statement? 7. To provide proper education, we need to
(a) Only I and III (b) Only II and III improve our education system. To
improve education system, we need
(c) All of the above (d) Only II
good teachers. To provide good teachers,
(e) None of these
@

we again need good education. In India


Directions (5-6): In each question below, are there are some good institutes too but
given a statement followed by two courses of most of the students opt for a job rather
action numbered I and II. On the basis of the than trying the career in teaching field
information given in the statement, you have which requires post-graduation as
to assume everything in the statement to be minimum eligibility criteria because they
do not get a good salary there.
7 Adda247 Publications For any detail, mail us at
Publications@adda247.com
A Complete Book of Logical Reasoning

Course of Action- III. Both the leak papers of class 12th and
(I) The Salary of the teachers should be class 10th should be cancelled by the
hiked to encourage them for opting CBSE and fresh and fair exam should be
teaching. conducted.
(II) The minimum eligibility criterion to be a (a) All follows
teacher should be graduate rather than (b) Only (I) and (III) follows
post-graduation and PhD to become a (c) Only (II) and (III) follows
teacher as students can get a job even

p
(d) Only (I) and (II) follows
after graduation. (e) Only (I) follows
(a) Only II follows
(b) Only I follows 10. In a survey which was done by employee

ou
(c) Either I or II follows union, it has been found that
(d) Neither I nor II follows productivity of work differs according to
(e) Both I and II follows changes in working environment of
8. The cost of treatment and proper health office. Cool, healthy and positive
care is unaffordable by the already environment escalates the performance.
needy masses. Which of the following course of action

gr
Course of action: has been taken to nullifies the facts
(I) The treatment for all should be free which comes in survey?
of cost of every disease. I. Management has tried to convince
(II) The government should establish employee that at the end of day
itself in manufacturing and productivity depends upon
m
distribution of medical equipment. determination and hard work.
(a) Only II follows II. Focus, honesty and hunger of
(b) Only I follows growth is key mantras which every
(c) Either I or II follows employee should follow for the
xa
(d) Neither I nor II follows expansion of company--statement
(e) Both I and II follows by HR.
Direction (9): In the following question a III. Casual leave for all employee has
statement is given followed by two or three been increased to 25 from 15 from
statements numbered as I, II and III. You have new financial year.
to read all the statements and decide which of (a) Only III (b) Both II and III
te

the following is the suitable course of action (c) Both I and III (d) Only II
for the mentioned statement above. (e) None of these
Statement: The Central board of secondary
11. There are many online food delivery
education (CBSE) faces a serious erosion of
companies like Swiggy, Uber Eats,
ce

credibility with the leak of annual


examination papers on economics for class Zomato, Food Panda, Faasoos etc. which
12th and mathematics for class 10th. are delivering food online at door in less
Course of Action: time which makes life easy in this fast
I. The HRD minister should conduct a phase of life. But on the other hand,
thorough inquiry to get the facts and small businessman who runs small
@

initiate remedial steps without any restaurants who are not registered with
delay. these companies are on the road and
II. The HRD minister should remove all the struggling for bread.
officers of CBSE from their respective What should be the effective course of
offices and conduct an inquiry against action by Food and Consumer ministry
them. to eradicate this problem?

8 Adda247 Publications For any detail, mail us at


Publications@adda247.com
A Complete Book of Logical Reasoning

(I) Ministry should guide and provide 13. Statement: There are more than 200
assistance for all those villages in the hill area of Uttar Pradesh
businessmen to start their new which are severely damaged due to
business in other segments. cyclone and it causes an extra burden of
(II) Ministry should order for closing Rs 200 crore on State Government for
these companies slowly as relief and rehabilitation work.
nowadays they are like hurdle for Course of Action:

p
small businessman and they are I. People of hill area should be shifted
making people lazy which are to other safer places.
increasing health issue also. II. State Government should ask more

ou
(III) Ministry should pass a proposal in financial support from Central
which every small restaurant Government.
should have tie-up with big III. Government should levy relief tax
restaurants. And on every order, to the corporate sector to ease the
there should be right contribution additional burden.
and have percentage on profit for (a) None follows

gr
both big and small restaurants. (b) Only I and II follow
(a) Only I and III (b) Only III (c) Only II and III follows
(c) Only I and II (d) Only II and III (d) Only I and III follow
(e) None of these (e) None of these
14. Statements: The state government has
12. Statement: In a recent survey of India
Today it has been found that business of
m decided to declare “Kala Azar” as a
notifable disease under the Epidemics
1000 crore has been done in one month
act, 1987. Family members or
just by selling copy of branded shoes, t-
neighbours of the patient are liable to be
shirts, jeans, shirts, bags etc. Branded
xa
punished in case they do not inform the
companies like Adidas, Puma, Nike
state authorities.
estimates loss of 500 thousand crore
I. Efforts should be made to
yearly by sale of copy item of their effectively implement the act.
brand. II. The cases of punishment should be
Which of the following action can be propagated through mass media so
te

taken to solve this issue? that more people become aware of


(I) On every sale of copy item, 30% of the stern action.
profit should be transferred to Which of the following course of action
owner company. does logically follow?
ce

(II) Selling of copy items on the name of (a) If only I follows


branded companies should be (b) None follows except II
banned. (c) If either I or II follows
(III) Loss of Branded companies should (d) If neither I nor II follows
bear by Government as copy item (e) If both I and II follow
has been sold by middle or lower-
@

class businessman and for the sake Direction (15): Study the following
of them Government should take information and answer the questions that
this step. follow:
The planet is warming, from north pole to
(a) Only I and II (b) Only II and III
south pole, and everywhere in between.
(c) Only II (d) Only I and III
Globally, the mercury is already up by more
(e) None of these

9 Adda247 Publications For any detail, mail us at


Publications@adda247.com
A Complete Book of Logical Reasoning

than 1 degree Fahrenheit (0.8 degree Celsius) production of the ‘king of the spices’
or even more in sensitive polar regions. The crop from Indian root stocks, are
effects of rising temperature are waiting for farming better by adopting modern
some far-flung future. They’re happening cultivation practices.
right now. Signs are appearing all over, and Courses of action:
some of them are surprising. The extreme I. India should immediately stop
weather events, the historic drought supplying root stocks of black

p
conditions over large parts of the world, the pepper to other countries.
fast melting of Greenland’s ice surface and the II. India should adopt modern
intensity of Hurricane Sandy are examples of technology for cultivating black

ou
the changes to global weather patterns that pepper to complete in the
can be expected from an overall rise in Earth’s International market.
surface temperature. III. India should reduce the price of its
What course(s) of action should people take black pepper to remain competitive
to reduce global warming? in the world market.
I. Increase afforestation in the world (a) All follow
(b) Only II follows

gr
II. Increase the use of recyclable products
rather than disposable products (c) Only I follows
III. Less use of CFC producing products (d) Only III follows
IV. Increase mining activities (e) None of the above
(a) Only I 18. Statement: A blast was triggered off
(b) Only land II
(c) Only I, II& III
m injuring many, when the night shift
workers at an ordinance factory were
(d) Only III handling ‘fox signaling explosive’.
(e) Only II, III &IV Courses of action:
I. The factory management should
xa
16. Statement: An increasing number of
train its staff as regards to the
farmers prefer to avail loans from local
safety aspects of handling such
moneylenders instead of the banks
explosive materials.
owing to complicated paperwork
II. The service of the supervisor in
involved in banks. charge of the night shift should be
Which of the following actions should be
te

terminated immediately.
taken by the government? III. The factory should immediately
(a) Local moneylenders who charge stop carrying out such exercises at
interest rates lower than the banks night.
should be punished. (a) None follows (b) All follow
ce

(b) Banks should simplify the (c) Only I follows (d) I and II follow
procedure to avail loans so as to (e) None of these
suit the farmers.
(c) The amount of loan should be 19. Statement: The prices of vegetables and
increased for the farmers. other food articles have decreased in the
(d) Both (a) and (b) recent months raising hope among
@

(e) Both (b) and (c) policy planners that the RBI's (Reserve
Bank of India) tight grip on supply of
17. Statement: India’s pre-eminent position liquid money in the market for
in the world black pepper production controlling inflation may be eased.
and trade is in danger as some of the Which of the following may be a possible
countries, which recently started action of above situation?

10 Adda247 Publications For any detail, mail us at


Publications@adda247.com
A Complete Book of Logical Reasoning

(a) The Reserve Bank of India may not Which of the following statements can be
reduce its key interest rates in near an effective course of action?
future. (a) The Internet Cafe should be
(b) The Government may step in and converted into coffee cafe.
make certain concessions to the (b) The Internet should be closed and
common people on various issues. new project should be started there.
(c) The Reserve Bank of India may (c) More funds should be invested on
the Internet Cafe.

p
consider lowering certain rates to
bring in more liquidity in the (d) Internet Cafe should be banned.
market. (e) None of these.
(d) The RBI may wait for at least

ou
22. Statement: As transporters nationwide
another year before taking any step. strike entered the fifth day, industry
(e) The RBI may collect more data from chamber Assocham said prices of
the market and wait for another essential commodities such as fruits,
four months to ensure they take the vegetables and milk may increase by up
correct step. to 15 per cent if the stir continues.
Besides, the strike has adversely

gr
20. Statement: A severe cyclonic storm hit
the Eastern coastline last month impacted the government’s revenue to
resulting in huge loss of life and property the tune of Rs. 36 crore. "With over one
on the entire east coast and Government crore trucks and tempos on strike since
the past few days, it has led to shortage
had to disburse a considerable amount
of essential commodities like milk, fruits
administration machineries.
m
for relief activities through the district
and vegetables and their costs are likely
to increase by about 10-15 per cent per
Which of the following may possibly be a
day if the strike continues".
follow up measure to be taken up by the Courses of Action:
Government?
xa
I. The transporters' association
(a) The Government should handled all should be ordered by the
the post relief work to the state government to immediately
government for proper implement- withdraw strike call.
tation. II. The government should immedia-
(b) The Government may set up a tely make alternative arrangements
te

committee for proper disbursement to ensure adequate supply of


of relief supplies in future essential commodities in the
(c) The Government may empower the market.
District Magistrates to make all III. The government should immedia-
future disbursements of relief. tely declare the strike illegal and
ce

(d) The Government may send relief put all those responsible for the
supplies to the affected people in strike behind the bars.
future only after proper assessment You have to assume everything in the
of the damage caused by such statement to be true, and then decide
calamities. which of the three given suggested
@

(e) The Government may not need to courses of action logically follows for
activate any follow up measure. pursuing.
(a) Only I and II follow
21. The Internet Cafes are incurring heavy
(b) Only II and III follow
loss these days as people prefer to use
(c) Only I and III follow
Internet in their mobiles PCs and
(d) All follow
Laptops.
(e) None of these

11 Adda247 Publications For any detail, mail us at


Publications@adda247.com
A Complete Book of Logical Reasoning

23. Statement As soon as summer has non-serious approach to the provision of


begun, complaints of water shortage universal health coverage to all its
have started pouring from all parts of the citizens. India’s health system is one of
city. the most privatized in the world, poorly
Courses of action regulated and accessible only to those
(I) Water tankers should be made with income levels well above the
available by the Water Board. average.

p
(II) The Water Board should claim that Which of the following courses of action
they have the problem under should be taken by the government to
control. make reliable changes in this poorly

ou
(III) Round-the-clock water service regulated health system?
should be made available by the I. Raising government expenditure on
Water Board. health, in conjunction with the
(a) Only III follows
states, should be a considerable
(b) Only I follows
change in this system.
(c) Either I or III follows
II. The government should shrink

gr
(d) Only II follows
(e) I and II follow their responsibility to provide free
and easily accessible health care to
24. Statement: At least 80 people have been poor and middle classes.
killed and over 350 wounded in a suicide
III. The government should make more
car bombing in Kabul's diplomatic
quarter this morning, a few hundred
m allocations on public health system
and make it accessible to the
metres from the Indian embassy. The
area where the explosion took place is deserving people.
close to the presidential palace and (a) All of the above (b) All except I
xa
several foreign embassies. External (c) All except II (d) All except III
Affairs Minister Sushma Swaraj has (e) None of the above
tweeted that Indian officials are safe.
26. Statement: Rivers in western Uttar
Which of the following is an appropriate
course of action as a fallout of the above Pradesh like Kali, Krishna and Hindon
terrorist attack? have been polluted to dangerous levels
te

(a) India should call back its diplomats because of the industrial waste released
and officials immediately. illegally into them by the industries.
(b) India should proceed to Which of the following course of action
International Court of Justice should be taken by the government to
ce

against the Afghan government. make the river clean?


(c) India should ask the Afghanistan (a) Heavy fine should be slapped on the
Government to ramp up the industries polluting the river water.
security of its officials. (b) Industrial waste should not be
(d) Officials in Afghanistan should be allowed to flow into the river.
advised to remain indoors as far as
@

(c) Industries should be shifted to


possible.
(e) None of these some other locations of the state.
(d) All the industries located on the
25. Statement: The National Health bank of the river should be closed
Accounts data for 2013-14 present fresh down.
evidence that India continues to have a (e) Only (b) and (c)

12 Adda247 Publications For any detail, mail us at


Publications@adda247.com
A Complete Book of Logical Reasoning

Directions (27–29): In each question below 29. Statement: Many management


is given a statement followed by three courses institutes in the city have enrolled a
of action numbered I, II and III. A course of large number of students for
action is a step or administrative decision to management courses which are not
be taken for improvement, follow-up or recognized either by the local university
further action in regard to the problems, or by the department of technical
policy etc. on the basis of the information education.

p
given in the statement, you have to assume Courses of Action:
everything in the statement to be true, then (I) All these management institutes
decide of the suggested courses of action should immediately derecognized

ou
logically follows for pursuing. by the university and the
department of technical education.
27. The major road connecting the two main (II) All these management institutes
parts of the city is inundated due to should be asked to refund fees to all
heavy rains during past two days. students and enroll them only for
Courses of Action: recognized courses.
(I) The govt. should immediately send

gr
(III) All students should be advised to
a relief team to the affected area. switch over to the recognized
(II) The municipal authority should courses in other institutes.
make immediate effort to pump out (a) Only I follows
water from the road. (b) Only I and II follow.
(III) The municipal authority should
m
advise the general public to stay
(c) Only I and III follow.
(d) Only II and III follow.
indoors till the water is cleared. (e) None of these
(a) Only I follows
(b) Only II follow. Direction (30): In given question below are
xa
(c) Only I and III follow. given a statement followed by two or three
(d) Only II and III follow. courses of action numbered I, II and III. You
(e) None of these have to assume everything in the statement to
be true and on the basis of the information
28. The prices of essential commodities have given in the statement, decide which of the
gone up substantially during the past suggested courses of action logically follows
te

few weeks. for pursuing.


Courses of Action: Give answer-
(I) The government should set up an (a) If only I follows
expert committee to study the trend (b) If only II follows
of prices.
ce

(c) If either I or II follows


(II) The govt. should immediately (d) If neither I nor II follows
abolish taxes on essential (e) If both I and II follow.
commodities.
(III) The govt. should advise the general 30. Statement:
public to refrain from purchasing There has been large number of cases of
@

essential commodities for few days. internet hacking in the recent months
(a) None of these creating panic among the internet users.
(b) Only I follows. Courses of Action:
(c) Only II follows. I. The government machinery should
(d) Only III follows. make an all-out effort to nab those
(e) Only I and II follow. who are responsible and put them
behind bars.
13 Adda247 Publications For any detail, mail us at
Publications@adda247.com
A Complete Book of Logical Reasoning

II. The internet users should be Courses of action


advised to stay away from using I. One should not pay attention to
internet till the culprits are caught. fashion
II. One should pay attention to fashion
31. Statements: Number of dropouts from III. Books on self-development should
the municipal schools have significantly be encouraged.
increased after withdrawal of mid-day (a) Only I follows

p
meal scheme. (b) Only II follows
Courses of action (c) Only III follows
I. The government should reconsider (d) Only I and III follow

ou
its decision of withdrawal of (e) None of these
midday meal scheme.
34. Statements: There have been quite a
II. The government should close down few incidents of highway robbery on the
some of the municipal schools. super expressway between cities A and
III. The government should carry out a B during recent months.
detailed study to find out the Course of action

gr
reasons for school dropouts. I. The local administration should
(a) None follows immediately set up police force
(b) Only I follows along the expressway to prevent
(c) Only I and III follow robbery
(d) Only II and III follow II. The local administration should
(e) All follow
m immediately close down the
expressway till the robbers are
32. Statements: In the city, over 75 percent apprehended.
of the people are living in slums and sub- III. More and more people should be
xa
standard houses which is a reflection on given training on how to tackle with
the housing and urban development the robbers.
policies of the Government. (a) Only I follows
Courses of action (b) Only I and II follow
I. There should be a separate (c) Only I and III follow
department looking after housing (d) All follow
te

and urban development. (e) None of these


II. The policies in regard to urban
35. Statement: It has been reported by one
housing should be reviewed. of the TV channels that the answer
III. The policies regarding rural papers of Board examination of one State
ce

housing should also be reviewed so are evaluated by students studying in the


that such problems could be same standard with the help of model
avoided in rural areas. answers as instructed by the examiners.
(a) Only I follows Courses of action
(b) Only I and II follow I. All such examiners should be
@

(c) Only II follow immediately suspended from their


(d) Either II or III follows official positions.
(e) Only II and III follow II. All such papers evaluated by the
students should be immediately
33. Statement: Besides looks and confiscated and got evaluated by
appearances, it is also important to qualified teachers.
develop oneself from within.
14 Adda247 Publications For any detail, mail us at
Publications@adda247.com
A Complete Book of Logical Reasoning

III. The board should explore 2016-17 was just 6.1%, all of 1.8
possibilities even though they are percentage points lower than the 7.9%
remote, of getting the answer recorded in its first (which decelerated
papers of this examination to 7.5% and 7% in the second and third
evaluated by computerized quarters, respectively).
machines. Which of the courses of actions by the
(a) Only I follow government can help improve the

p
(b) Only II follows current situation?
(c) Only III follows (i) The government should
(d) Only I and II follow reintroduce banned currency.

ou
(e) All follow (ii) Those spreading pessimism about
Directions (36-42): In each question below the current government should be
is given a statement, followed by two/three punished.
courses of action numbered I, II and III. You (iii) The government should give
have to assume everything in the statement to rebates and incentives like tax cuts
be true, and then decide which of the three to companies and advise RBI to

gr
given suggested courses of action logically lower interest rates.
follows for pursuing. (a) Both (ii) and (iii)
(b) Only (iii)
36. Statement: There have been quite a few (c) (i), (ii) and (iii)
incidents of highway robbery on the (d) Both (i) and (iii)
super expressway between cities A and
B during recent months.
m (e) None of these.
Courses of action: 38. Statement: In one of the worst accidents
I. The local administration should in railway level crossing, fifty people
immediately set up police along the died when a bus carrying them collided
xa
expressway to prevent robbery. on to a running train.
II. The local administration should Courses of action:
immediately close down the I. The train driver should
expressway till the robbers are immediately be suspended.
apprehended. II. The driver of the bus should be
te

III. More and more people should be tried on court for negligence on his
given training on how to tackle with part.
the robbers. III. The railway authority should be
(a) Only I follow asked to man all its level crossings.
(b) Only I and II follows
ce

(a) None follows


(c) Only I and III follow (b) Only I and II follow
(d) All follow (c) Only III follow
(e) None of these (d) Only II and III follow
37. Statement: A top government economist (e) None of these
has lashed out at ‘messiahs of doom’ who
@

39. Statement: In a bizarre incident, the 50-


had earlier predicted a 2% decline in year-old wife of a former Davis Cup
growth due to the Centre’s tennis player overpowered and stabbed
demonetization of currency notes last to death an intruder who had sneaked
November. But scratch deeper, those into her Sainik Farms with the intention
naysayers don’t appear to be too far off of committing a theft.
the mark. Growth in the final quarter of

15 Adda247 Publications For any detail, mail us at


Publications@adda247.com
A Complete Book of Logical Reasoning

Courses of action: 42. Statement: It is reported that though


I. An enquiry should be made to look Vitamin E present in fresh fruits and
into the matter and examine its fresh vegetables is beneficial for human
veracity. body, capsule Vitamin E does not have
II. The lady should be awarded for her same effect on human body.
bravery and excellent courage. Courses of Action:
(a) If only I follows I. The sale of capsule Vitamin E

p
(b) If only II follows should be banned.
(c) If either I or II follows II. People should be encouraged to
(d) If neither I nor II follows. take fresh fruits and fresh

ou
(e) If both I and II follow vegetables to meet the body
40. Statement: There are repeated requirement of Vitamin E.
complaints by tourists of misbehavior by (a) If only I follows
CISF (Central Industry Security Force) (b) If only II follows
men deployed at heritage sites like Taj (c) If either I or II follows
Mahal in the name of ticket – checking, (d) If neither I nor II follows.

gr
etc. (e) If both I and II follow.
Courses of Action:
Directions (43-47): In each question below
I. A letter should be issued instructing
is given a statement followed by two courses
the DG of CISF to tell his men to
avoid any direct contact with the of action numbered I and II. A course of action
tourists during duty.
m is a step or administrative decision to be
taken for improvement, follow up or further
II. The responsibility to check tickets
etc. should be entrusted to the men action in regard to the problem, policy etc. On
of ASI (Archaeological Survey of the basis of the information given in the
xa
India) statement, you have to assume everything in
(a) If only I follows the statement to be true and then decide
(b) If only II follows which of the suggested courses of action
(c) If either I or II follows logically follows for pursuing. Given answer
(d) If neither I nor II follows. 43. Statement: A large number of students
(e) If both I and II follow
te

are reported to be dropping out of school


41. Statement: Traffic rules are being in villages as their parents want their
violated by lawmakers and those children to help them in farms.
machineries who have been entrusted Course of Action:
with the responsibility to make common I. The government should
ce

people abide by them. immediately launch a programme


Courses of action: to create awareness among the
I. Majority of those among the farmers about the value of
lawbreakers are the lawmakers. education.
II. No fine is imposed when a traffic II. The government should offer
@

policeman violates traffic rules. incentives to those farmers whose


(a) If only I follows children remain in schools.
(b) If only II follows III. Education should be made
(c) If either I or II follows compulsory for all children up to
(d) If neither I nor II follows. the age of 14 and their employment
(e) If both I and II follow. banned.

16 Adda247 Publications For any detail, mail us at


Publications@adda247.com
A Complete Book of Logical Reasoning

(a) Only I and II follow was found in 50% of patients. A large


(b) Only II and III follow number of infants were dying due to
(c) Only I and III follow infections that did not respond to
(d) All follow treatment.
(e) None of these Courses of action-
I. The revision of antibiotics classes
44. Statement: Without the active
by the World Health Organisation in
cooperation between the proprietor and

p
its list of essential medicines is a
the employees of the mill, it cannot welcome step in the global initiative
remain a profitable concern for long. to push back against antimicrobial
Course of Action:

ou
resistance, the phenomenon of
I. The mill should be closed down. bacteria becoming resistant even to
II. The workers should be asked to the most potent drugs.
cooperate with the owners. II. Close scrutiny of these by national
III. The owners should be asked to stewardship programmes such as
cooperate with the employees. those initiated by the ICMR is
(a) None follows needed.

gr
(b) Only I and II follow Which of the following will be the course
(c) All follow of action?
(d) Only II and III follow (a) Only I (b) Both I and II
(d) None of these (c) Either I or II (d) Only II
(e) None of these
45. Statement: The air and rail services
have been severely disrupted due to
m 47. Statement: Any further increase in the
thick fog in the northern part of the pollution level in the city by way of
country. industrial effluents and automobile
Course of Action: exhaustions would pose a severe threat
xa
I. The rail and air services should be to the inhabitants.
temporarily suspended in the Course of Action:
region. I. All the factories in the city should
II. People should be advised to make immediately be closed down.
their travel plan keeping in mind II. The automobiles should not be
te

the probable disruption resulting in allowed to ply on the road for more
delay or cancellation of services. than four hours a day.
III. The government should III. The Government should restrict the
immediately install modern issue of fresh licenses to factories
machines which will enable it to and automobiles.
ce

guide the rail and air services even (a) None follows (b)Only II follows
if the thick fog develops. (c) Only III follows (d) All follows
(a) Only II follows (e) None of these
(b) Only III follows 48. Statement: The Officer In charge of a
(c) Only II and III follow Company had a hunch that some money
@

(d) All follow was missing from the safe.


(e) None of these Courses of action:
46. Statement-The Indian Council of I. He should get it recounted with the
Medical Research issued a warning two help of the staff and check it with
years ago, based on studies conducted in the balance sheet.
hospitals that resistance to antibiotics II. He should inform the police.

17 Adda247 Publications For any detail, mail us at


Publications@adda247.com
A Complete Book of Logical Reasoning

(a) Only I follow. 50. Statement-“The surveillance system put


(b) Both I and II follow. in place by the Health Ministry
(c) Only II follow. succeeded in identifying three adults
(d) Either I or II follow. infected with Zika virus between
(e) Neither I nor II follow. November 2016 and February 2017 in
Gujarat.
49. Statement: The ground water in the
Course of Action-
locality has been found to contain high

p
I. Over 34,000 human samples and
level of arsenic making it dangerous to
nearly 13,000 mosquito samples
drink.
were tested for the presence of the
Courses of action:

ou
Zika virus.
I. The people living in the area should
II. The government machinery should
be shifted to another area to avoid a
be on overdrive to educate and
catastrophic situation.
increase awareness about ways to
II. The government should make
avoid infection.
arrangements for supply of safe
III. India should inform the WHO and
drinking water.

gr
the global community.
(a) Only I follow.
Which of the following will be the course
(b) Both I and II follow.
of action?
(c) Only II follow.
(a) Only I (b) Only II and III
(d) Either I or II follow.
(c) Only I and II (d) Only II
(e) Neither I nor II follow.
m (e) None of these

Course of Action : Solutions


xa
1. (a); (a) Is both effective and 2. (e); It is not appropriate to ban all the
appropriate in reducing the imports from country B which
number of accidents involving might include some essential
cyclists and four wheelers. commodities so (a) not correct.
(b) A ban will be the most effective
te

Option (b) is not appropriate as


course of action in reducing the implementing heavy taxes might
number of accidents involving reduce the consumption of beer
cyclists but not an appropriate imported from country B but the
course of action as the people
consumption by even one person
ce

should have the right to choose.


can be dangerous to him. Similarly
(c) It is not clear whether people
are using cycles to save money we don’t know the quantity of beer
so it is not an appropriate consumed by people who fell sick
course of action. after drinking the beer imported
(d) It is not necessary that only from country B so (c) is not
@

four wheelers are responsible appropriate. Option (d) is irrelevant


for all the accidents with as nothing has been mention in the
cyclists, sometimes it can be a statement about the imports from
fault of cyclists too. country A. Option (e) will be most
(e) This will only reduce the effective as a corrective and
number of injuries but not the preventive measure.
number of accidents.
18 Adda247 Publications For any detail, mail us at
Publications@adda247.com
A Complete Book of Logical Reasoning

3. (d); Statement I is the immediate course equipment the government can


of action as it will help the sugar provide affordable treatment. But
mills to clear off the outstanding (I) does not follow as it is not a
arrears of the sugarcane farmers. practical solution.
Statement II is also an immediate
9. (b); For I: Yes, it is an immediate course
course of action as it is talking
of action because HRD minister
about a fixed minimum price of the
should conduct the thorough

p
sugar which will help the farmers
inquiry to protect its credibility and
and in the statement III is talking
about to increase the production of stop any further leak of papers.
For II. No, it is not the immediate

ou
molasses and ethanol which is not
an immediate course of action. course of action as it is a harsh
course of action against all the CBSE
4. (d); Only II course of action follows as officers inspite of punishing those
safety of the people and who found guilty after the inquiry.
environment protection require For III. Yes, it is an immediate
transparent reforms and policies. course of action as the leak papers

gr
But I and III does not follow as even of class 12th and class 10th should
by considering the fact that this be conducted fairly with the view to
tourism industry provide uphold the sanctity of Board
employment to so many people but
examinations and in the favour of
the lives of so many others cannot
m
be put at stake for it. And also, III is
all the hard-working students.
not feasible as we cannot relocate 10. (e); For I: Yes, it nullifies the fact in
app. lakhs of people to reduce the survey as management is trying to
pollution level near rivers. convince that productivity depends
xa
5. (b); Action I is not appropriate because upon determination and hard work.
it is too harsh as a course of action. For II: Yes, it nullifies as HR is
Action II is appropriate in order to trying to motivate and trying to
maintain discipline. keep focus of employee towards
only work by encouraging and
6. (b); Action I is not appropriate because
te

counselling them.
of the word ‘all’. Action II is helpful For III: No, as it is in favour of
in reducing the inconvenience employee because by getting more
caused to the public. casual leave Employee will be able
Direction (7-8): to relax more and can keep them
ce

7. (e); Both I and II follows as salary is an refreshed to work with more


important factor for encouraging enthusiasm.
students to opt teaching as a career
11. (b); For I: False, Changing the segment
option rather than job and changing
is not the solution
the eligibility criterion to
For II: False, closing of companies is
@

graduation for being a teacher as


generally the students choose their also not a solution. It is like to stop
career option after graduation. innovation.
For III: True, It is like a problem
8. (a); Statement (II) follows as by solving proposal in which small
engaging itself in manufacturing businessman will get opportunity
and distribution of medical to work and chances to grow.
19 Adda247 Publications For any detail, mail us at
Publications@adda247.com
A Complete Book of Logical Reasoning

12. (c); For I: False, this is not a feasible 18. (c); Action I is the right course of action
course of action. because training to the staff as to
For II: True, as this is the solution safety aspects of handling explosive
which can completely eradicate this material will reduce the chances of
problem as selling of copy items is such accidents in future. Action II
without rights is illegal. does not follow.
For III: False, Government can help 19. (c); Option (c) may be a possible action

p
those businessmen in many ways because interest rates plays a key
but which is illegal, why role in controlling inflation. Its
government will bear such cost. obvious that when interest rates

ou
gets lowered, there will be more
13. (e); Only I course of action follows liquidity in the market.
because it is mentioned in
statement that hill area is severely 20. (b); Option (b) may be a follow up
damaged. II and III, course of action measure as it will help in proper
do not follow because it is not disbursement of relief supplies.
mention in the statement that the 21. (e); It can’t be a better course of action

gr
state Govt. is not capable of tackling to convert the Internet Cafe into
Rs. 200 cr. burden and govt. can’t Coffee Cafe. Also a new project at
levy relief tax to the corporate the place of internet cafe after
sectors to ease burden. diminishing it can’t be a rational
course of action. More investment
14. (e); When the government takes such
m
action it is necessary that people
in loss making project is also not
rational without a strong strategy,
are made aware of the and the 4th statement is illogical –
consequences they would face if not a good course of action.
they do not obey the directives.
xa
22. (a); Clearly, the situation demands that
Hence II follows. I is obvious.
strike be called off, either through
15. (c); I, II & III all can be the course of warning or negotiations, and till
action to reduce global warming. By then alternative arrangements be
increasing mining activities the made to retain normalcy in supply
global warming cannot be reduced of essential commodities. So, both I
te

so statement IV is not a proper and II follow. Taking extreme steps


course of action. (as getting the striking transporters
arrested) at the first stage doesn't
16. (b); Only II followed as by simplifying seem proper. So, III does not follow.
the procedure more farmers will
ce

23. (b); I is the only practical step. Tankers


prefer to avail loan from bank
can carry required water to the
instead of moneylenders. I is not
needy one. II is not a course of
feasible and III is also not action which is needed, while III is
appropriate course of action. clamoring for the impossible.
17. (b); Only better quality can put India
@

24. (c); (a) Calling back officials on the


back in the competitive field of basis of this attack will send a
black pepper production. So, India strong message to the
should go for modern technology international community and
for cultivating black pepper to might even show that terrorist
compete in the present have been successful in their
international market. purpose.

20 Adda247 Publications For any detail, mail us at


Publications@adda247.com
A Complete Book of Logical Reasoning

(b) Proceeding ICJ is absurd as it is 29. (a); Such institutes should immediately
not known if the Afghan be derecognized by the university.
government was actually Hence I course of action follows. It
involved in the attack. is not possible to refund fees to all
(c) Asking the Afghanistan students and enrol them only for
Government to ramp up the recognized courses. All students
security of its officials as this could not be advised to switch over

p
will help ensuring the security to the recognized courses in other
of its officials. institutes.
(d) Advising the officials to stay

ou
30. (a); Clearly, internet users should not
indoors is not a right way to suffer on account of certain
respond to the terrorist attack individuals who indulge in internet
and shows a sense of hacking. However, such wrong-
helplessness. doers ought to be penalized so that
25. (c); It is not practically feasible to there are no hassles in the use of

gr
provide free health facility to all internet. So, only course I follows.
poor and middle classes. So, II is not 31. (c); Clearly, the government ought to
an effective course of action. find out the reason behind the
Whereas raising expenditure in increase in number of dropouts,
conjunction with the states and
providing more allocations on
m and also the extent to which the
withdrawal of mid-day meal
public health system will be an scheme is responsible for the same.
effective course of action. So, both I and III follow, II appears
xa
26. (b); Only b) will be a sensible course of to be vague.
action taken by the government to 32. (b); The statement talks of housing
keep the river clean. a), c) and d) conditions in urban areas only. So,
can be ruled out because these are III does not follow. Also, to improve
extreme courses of action. the deteriorating housing
te

27. (b); II course of action logically follow conditions, the urban housing
from the statement, while III course policies need to be studied and the
of action does not explicitly tell that lackenings removed by a team of
whether the water is cleared by the efficient personnel deployed for the
ce

municipal authority or as a natural same. So, both I and II follow.


process and I is not appropriate. 33. (c); The statement stresses the need for
28. (c); The courses of action suggested in I all-round personality development
statement is absurd as it talks about of an individual. So, only III follows.
the trend not the remedial 34. (a); Clearly, incidents of robbery can be
@

measures to bring down to price practically averted only by


similarly the purchasing and tightening security arrangements
consumption of essential and increasing vigilance by police.
commodities cannot be postponed So, I follows. Neither II nor III
for a few days, hence statement III follows as none of them is
is also absurd. practically viable.
21 Adda247 Publications For any detail, mail us at
Publications@adda247.com
A Complete Book of Logical Reasoning

35. (d); Student who work hard all the year 42. (b); I is illogical because Vitamin E
round to perform well at the Board capsule is also useful even if less
examination deserve to be effective. Action II is more useful
evaluated correctly by exports, and and effective.
not mechanically or by
inexperienced people. Besides, 43. (d); All courses of action follow because
examiners who shirk their duty of creating awareness among farmers

p
evaluating answer papers ought to about the education is necessary
be punished. So, both I and II follow, and by offering incentives to
while III does not. farmers, govt. can help farmer

ou
36. (a); Clearly, incidents of robbery can be financially. And making education
practically averted only by compulsory is an important course
tightening security arrangements of action.
and increasing vigilance by police. 44. (d); Only II and III follow because for
So, I follows. Neither II nor III
better functioning and for making
follows as none of them is

gr
profit from mill is necessary that
practically viable.
both owner and worker should
37. (b); (i) is not an appropriate course of cooperate each other.
action. (ii) is not justified as
everyone has a right to express 45. (c); Only II and III follow. 2nd course of
his/her views. (iii) is an
appropriate course of action as it
m action is preventive in nature for
passengers.
will bring money back in to the
46. (b); Course of action I follows because
markets which is lying stagnant
with the banks. with a graded approach to the use
xa
of antibiotics, under which some
38. (c); The problem discussed in the medicines are reserved for the most
statement is not regarding the resistant microbes, the WHO list
mentioned accident, but to do can stop their misuse as broad-
something to avert such mishaps.
spectrum treatments. Course of
Accidents at railway crossings can
te

action II also follows because a


be averted by deploying men to
regulate traffic and installing close inspection is also needed of
barriers to check traffic movement the national supervisory
when a train passes by. So, only III programmes to help people to get
ce

follows. rid out of it.


39. (a); I is advisable as it will ensure that 47. (c); Only 3rd course of action follows. I
there were no ulterior motives and 2nd are extreme course of
behind stabbing the person to action. So, these two do not follow
death. II is not advisable until I and 3rd one is preventive course of
@

makes things clear. action.


40. (a); I is advisable because less direct 48. (a); Clearly, suspicion first needs to be
contact will reduce the gravity of confirmed and only when it is
the problem.
confirmed, should an action be
41. (d); Neither I nor II is a course of action. taken. So, only course I follows.

22 Adda247 Publications For any detail, mail us at


Publications@adda247.com
A Complete Book of Logical Reasoning

49. (c); Clearly, the contamination of the number of samples taken for
ground water doesn’t provide test of Zika virus.
sufficient grounds to shift people to For Statement II- True, In II
other areas. The problem demands statement it has been said that the
creating awareness among people government should aware the
of the dangers of drinking people about the virus and the
contaminated water and arranging
ways to prevent it .So it is true.

p
to provide them safe drinking
water. Thus, only course II follows. For Statement III-True, In this
statement it has been said that it is
50. (b); In the above question we have to

ou
India’s responsibility to inform
find the course of action for the
World Health Organization and
above statement.
For Statement I- False, In the above other global community to hinder it
statement it is not mentioned about from spreading elsewhere.

gr
m
xa
te
ce
@

23 Adda247 Publications For any detail, mail us at


Publications@adda247.com
A Complete Book of Logical Reasoning

p
ou
gr
m
xa
te
ce
@

1 Adda247 Publications For any detail, mail us at


Publications@adda247.com
A Complete Book of Logical Reasoning

04 Cause and Effect


Cause is an event that leads to another event, which is its effect. An event or action that causes
something else to happen. “Becuase” is a clue word that helps identify the cause. It answer the

p
question “Why”
→ An event or action that happened as a result of another event or action. It answers the

ou
question “what happened”.
In short: A cause is why something happens. An effect is What happens. Look for clue words,
such as if,then, becuase, since and so.
There are two methods which are usually ask in paper
(i) By using two statement

gr
(ii) By using given passage.
(i) By using two statement: There are two statements are given we have to find which one
cause and which one effect and is they relate to each other or not.
Their directions are
m
(a) Statement I is cause and II its effect
(b) Statement II is cause and I its effect
(c) Both are independent causes.
(d) Both statements are effects of independent cause.
xa
(e) Both statements are effects of common cause.

(ii) By using given passage: Their is any economic, any company data or scheme given we are
having four statement as options and we have to find out according to question which one
effect or cause of above passage.
te

for example: (i) The prices of petroleum products drop marginally last week.
(ii) The state government reduced the tax on petroleum products last week.
Solution: This is a direct releationship where statement II is the cause and statement I its
ce

effect. The common phrases in both the statements “last week” and “petroleum products”
justify the relationship. Also it a commn economic trend that when the taxes are reduced
prices fall. Hence answer (b).
The points which you have to remember for findout cause and effect which are following.
@

(i) First find out that both statements are having similar situation.

(ii) Find out which statement have occur firstly then corelated this statement to another one
cheek that it is its effect or not.
In Second type their is a passage given which related to any scheme, economic survey in that
situation we have to find out reason or effect of any line.

2 Adda247 Publications For any detail, mail us at


Publications@adda247.com
A Complete Book of Logical Reasoning

Points to Remember
(1) A cause is "WHY" something happens.
(2) An effect is "WHAT" happens.

p
(3) Cause and effect are combination of action and reaction.
(4) Remember that no effect can occur without a cause.
(5) No effect can be its own cause.

ou
(6) Every cause must produce an effect.
(7) Cause and effect cannot happen simultaneously.

Exercise

gr
1. In each of the following questions, two 2. In the following question, two statements
statements numbered I and II are given. numbered I and II are given. There may
There may be cause and effect be cause and effect relationship between
relationship between the two statements.
These two statements may be the effect of
m the two statements. These two statements
the same cause or independent causes. may be the effect of the same cause or
These statements may be independent independent causes. These statements
causes without having any relationship. may be independent causes without
xa
Read both the statements in each having any relationship.
question and mark your answer as- Statement I. Unemployment situation in
(a) If statements I is the cause and Country X has worsened over the past
statement II is its effect.
couple of years.
(b) If statements II is the cause and
te

statement I is its effect. Statement II. The government of country


(c) If both the statements I and II are X has been promoting automation in
independent causes. various sectors to achieve higher growth
(d) If both the statements I and II are rates.
ce

effects of independent causes. (a) Statement I is the cause and


(e) If both the statements I and II are statement II is its effect;
effects of some common cause.
(b) Statement II is the cause and
(I) DMRC's 9,000-strong workforce of statement I is its effect;
non-executive employees has been (c) Both the statements I-and II are
@

taking part in sit-in protests at


independent causes;
different metro stations.
(II) Demand of Delhi metro employees is (d) Both the statements I and II are
a revision of salaries and pay-grades effects of independent causes;
and payment of arrears, apart from (e) Both the statements I and II are
other demand like the right to form a effects of some common cause.
union.
3 Adda247 Publications For any detail, mail us at
Publications@adda247.com
A Complete Book of Logical Reasoning

3. Honorable Supreme Court has made Direction (5-6): Study the following
amendments in SC/ST act that there will information and answer the questions that
be no instant arrest in case of harijan act follow:
and case will be investigated by DSP rank
The planet is warming, from north pole to
police officer instead of Inspector and
south pole, and everywhere in between.
then only it will be decided by DSP rank
Globally, the mercury is already up by more

p
officer to continue investigation in it.
than 1 degree Fahrenheit (0.8 degree Celsius)
What may be the effect after this
or even more in sensitive polar regions. The
amendment?
effects of rising temperature are waiting for

ou
I. Oppression towards harijan may
some far-flung future. They’re happening
increases and equality in the society
right now. Signs are appearing all over, and
will be disturbed.
some of them are surprising. The extreme
II. Leaders of backward class and harijan
weather events, the historic drought
may call Bhart bandh against the
conditions over large parts of the world, the

gr
amendments made by supreme court
fast melting of Greenland’s ice surface and the
of India.
intensity of Hurricane Sandy are examples of
III. Leaders of some parties may go on
the changes to global weather patterns that
strike to put pressure on Central Govt.
can be expected from an overall rise in Earth’s
to abolish reservation system as this
m
is the right to put pressure on Central
surface temperature.

Govt. 5. Which of the following would be the ill


(a) Both II and III (b) Only I effect(s) of the increasing global warming
on the earth?
xa
(c) Both I and II (d) All of the above
(e) None of these I. Increase in sea level on the earth.
II. Mangrove forests will get submerged
4. Which of the following may be
under water.
cause/effect in respect to amendments in
III. Low-lying areas of the world would
Public Premises act?
te

be under water.
(I) New ministers and parliamentarians
IV. Floods and droughts would become
will get accommodation on right time.
more common.
(II) Hunger strike is called by India
(a) Only I (b) Only I and II
ce

against corruption leader Anna


(c) Only I, II, III (d) Only I, III and IV
Hajare.
(e) All of these.
(III)A RTI was filed by congress MP 2
years ago who had not get Govt. 6. Which of the following is/are the main
Bungalow for 5 months after cause of increasing global warming in the
@

becoming MP. world?


(a) I is cause and II is effect (a) Increasing deforestation in the world
(b) I and II is effect (b) Ozone layer depletion
(c) I is cause and III is effect (c) Increasing world population
(d) III is cause and I is effect (d) Increasing pollution in the world
(e) None of these (e) Can’t be inferred from the passage

4 Adda247 Publications For any detail, mail us at


Publications@adda247.com
A Complete Book of Logical Reasoning

Direction (7–8): Study the following (a) If statement I is the cause and the
information carefully and answer the statement II is its effect.
questions. (b) If statement II is the cause and the
statement I is its effect.
Too little has changed with the practice of
(c) If both the statements I and II are
child labour in our country, despite tall
independent causes.
promises made by our leaders. A good
(d) If both the statements I and II are effects
proportion of children throughout the world,

p
of independent causes.
especially in India, form a part of the working
(e) If both the statements I and II are effects
labour force. Millions of children work in
of some common cause.
fields and factories, on street corners and in

ou
garbage dumps, in private houses and public 9. (I) The bank has provided a link on its
houses. These children certainly would pose a website to obtain feedback from
threat to society, if they are not treated equal customers.
and given opportunities to develop towards (II) Customers have been complaining
the best of their potential despite being about poor services in the bank’s
poorly educated. The existence of child labour branches.

gr
in India is a complex reality. However, it is a
10. (I) The production of pulses has
symptom, not the disease.
dropped for the third consecutive
7. Which of the following is the main cause year.
of the rising child labour in India? (II) India has decided to import pulses
(a) Extreme poverty
(b) Unstable and poor income
m this year.
11. (I) Budgetary allocation for building a
(c) Parental illiteracy
better railway network, e.g.
(d) More number of dependent children
constructing new railway lines, has
(e) All of these
xa
increased.
8. Which of the following can be the main (II) There has been a substantial drop
effect of the increasing child labour in in the number of passengers and
India? they are opting for air travel.
Increasing number of crimes in India
12. Read the following statements and
Increasing unemployment
te

answer the question.


Increased number of unskilled labour
(I) Finally, the government has decided
force
to sell off its sick industrial units,
Increase in illiteracy rate
which have become a burden to it.
(a) Only I (b) Only I and II
(II) Corruption, inefficiency and red-
ce

(c) Only II and III (d) Only III and IV


tapism have led to heavy losses to
(e) Only III
the government owned industrial
Directions (9–11): Each of the questions units.
below consists of two statements labeled I Which of the following is true with
and II. These statements may be effects of respect to the above statements?
@

independent causes or of a common cause. (a) Statement I is the cause and


One of these statements may be the effect of Statement II is its effect
the other statement. Read both the (b) Statement II is the cause and
statements and decide which of the following Statement I is its effect
answer choices correctly depicts the (c) Both statement I and II are
relationship between these two statements. independent causes

5 Adda247 Publications For any detail, mail us at


Publications@adda247.com
A Complete Book of Logical Reasoning

(d) Both statements I and II are effects of (c) If both the statements I and II are
independent causes independent causes
(e) Both statement I and II are effect of (d) If both the statements I and II are
the same cause effects of independent causes and
13. The power distribution companies (e) If both the statements I and II are
(Discos) are set to conduct 12-hour load effects of some common cause
shedding in urban areas during summer 14. The power distribution companies

p
season as the gap between electricity (Discos) are set to conduct 12-hour load
demand and generation is expected to shedding in urban areas during summer
exceed 7,000MW after mid of April. The

ou
season as the gap between electricity
loadshedding in rural areas would be
demand and generation is expected to
more than 14 hours in scorching heat,
exceed 7,000MW after mid of April. The
they said quoting the Discos’ report sent
loadshedding in rural areas would be
to the Ministry of Water and Power on
more than 14 hours in scorching heat,
expected power cuts. The electricity
demand is considered to touch more they said quoting the Discos’ report sent

gr
than 21,000MW after a month, but its to the Ministry of Water and Power on
generation from hydel, thermal and IPPs expected power cuts. The electricity
components would remain below demand is considered to touch more
15,000MW which includes around than 21,000MW after a month, but its
6,000MW from hydel and 9,000MW from generation from hydel, thermal and IPPs
Wapda-owned Gencos
m
(generation components would remain below
companies) and private IPPs 15,000MW which includes around
(independent power producers). 6,000MW from hydel and 9,000MW from
In the given question, two statements Wapda-owned Gencos (generation
xa
numbered I and II are given based on the companies) and private IPPs
given passage. There may be cause and (independent power producers).
effect relationship between the two In the given question, two statements
statements. These two statements may numbered I and II are given based on the
be the effect of the same cause or given passage. There may be cause and
independent causes. These statements
te

effect relationship between the two


may be independent causes without statements. These two statements may
having any relationship. Read both the be the effect of the same cause or
statements in question based on the independent causes. These statements
given passage and mark your answer as- may be independent causes without
ce

Statements:
having any relationship. Read both the
I. Rural areas in the country have
statements in each question based on the
been suffering due to load shedding
given passage and mark your answer as-
for quite some time.
II. If the Government is not able to Statements:
overcome the power crisis, load I. Rural areas in the country have
@

shedding will be extended even to been suffering due to load shedding


the urban areas. for quite some time.
(a) If statement I is the cause and II. If the Government is not able to
statement II is its effect overcome the power crisis, load
(b) If statement II is the cause and shedding will be extended even to
statement I is its effect the urban areas.

6 Adda247 Publications For any detail, mail us at


Publications@adda247.com
A Complete Book of Logical Reasoning

(a) If statement I is the cause and Directions (17-18): Below in each question
statement II is its effect are given two Statement I and II. These
(b) If statement II is the cause and statements may be either independent causes
statement I is its effect or may be effects of independent causes or a
(c) If both the statements I and II are common cause. One of these statements may
independent causes be the effect of the other statement. Read both
(d) If both the statements I and II are the statements and decide which of the

p
effects of independent causes and following answer choice correctly depicts the
(e) If both the statements I and II are relationship between these two statements.
effects of some common cause
Give answer

ou
15. Effect- The price of food grains and (a) If Statement I is the cause and Statement
vegetables have increased by about 30% II is its effect.
in the past three months. Which of the (b) If Statement II is the cause and
following can be a probable cause of the Statement I is its effect.
above effect? (c) If both the Statements I and II are
(a) The farmers have decided to change independent causes.

gr
their farming style. (d) If both the Statements I and II are effects
(b) The prices of other products have of independent causes.
increased more than 30%.
(e) If both the Statement I and II are effects
(c) The number of farmers has
of some common causes.
reduced.
m
(d) Occupation of farming has not been 17. I. The government has marginally
viewed as a reputed work. increased the procurement price of
(e) None of the above wheat for the current crop.
16. Cause- The income tax authorities II. The current wheat crop is expected
xa
carried out raids at three different to be twenty per cent more than the
business houses in the city last week. previous wheat crop.
Which of the following can be a possible 18. I. The braking system of the tourist
effect of the above statement? bus carrying 40 passengers failed
(a) The three business houses are
while negotiating a stiff climb on a
te

regular defaulters in payment of


hilly road.
their income tax.
II. The tourist bus fell into the gorge
(b) The income tax department has
received a tip off about the illegal killing at least ten passengers and
activities going on in the three seriously injuring all the remaining.
ce

business tax. Directions (19-21): In each of the following


(c) The government decided to look questions, two statements numbered I and II
into the matter and has appointed are given. There may be cause and effect
an inquire committee. relationship between the two statements.
(d) The business houses took These two statements may be the effect of the
@

immediate action to clear off all


same cause or independent causes. These
their income tax dues, in order to
statements may be independent causes
avoid raid on their establishments.
without having any relationship. Read both
(e) The authorities intend to conduct
raids in several other business the statements in each question and mark
houses in the vicinity. your answer accordingly. Give answer-

7 Adda247 Publications For any detail, mail us at


Publications@adda247.com
A Complete Book of Logical Reasoning

(a) If statement I is the cause and statement (a) If statement I is the cause and statement
II is its effect; II is its effect;
(b) If statement II is the cause and statement (b) If statement II is the cause and statement
I is its effect; I is its effect;
(c) If both the statements I and II are (c) If both the statements I and II are
independent causes; independent causes;
(d) If both the statements I and II are effects (d) If both the statements I and II are effects

p
of independent causes; of independent causes;
(e) If both the statements I and II are effects (e) If both the statements I and II are effects
of some common cause.
of some common cause.

ou
22. I. The staff of Airport authorities
19. I. The local traders’ association urged
called off the strike they were
all its members to close down their observing in protest against
shops for a day to protest against privatization.
the government’s new tax policy. II. The staff of Airport authorities went
II. Many shopkeepers decided to close on strike anticipating a threat the

gr
down their shops for the day and their jobs.
gave a day off to their employees.
23. I. Many people visited the religious
20. I. The prices of petroleum products place during the weekend.
dropped marginally last week. II. Few people visited the religious
II.
m
The State Government reduced the
tax on petroleum products last
place during the week days.
Directions (24-25): Given below are pairs of
week. events ‘A’ and ‘B’. You have to read both the
21. I. The university authority has events ‘A’ and ‘B’ and decide their nature of
xa
decided to conduct all terminal relationship. You have to assume that the
examinations in March/April every information given in ‘A’ and ‘B’ is true and you
year to enable them to declare will not assume anything beyond the given
information in deciding the answer. Mark
results in time.
answer
II. There has been considerable delay
(a) If ‘A’ is the effect and ‘B’ is its immediate
te

in declaring results in the past due and principal cause.


to shortage of teachers evaluating (b) If ‘A’ is the immediate and principal
the answer sheets of the cause and ‘B’ is its effect.
examination conducted by the (c) If ‘A’ is an effect but ‘B’ is not its
ce

university. immediate and principal cause.


Directions (22-23): In each of the following (d) If ‘B’ is an effect but ‘A’ is not its
questions, two statements numbered I and II immediate and principal cause.
(e) None of these
are given. There may be cause and effect
relationship between the two statements. 24. Statement (A): The government has
@

These two statements may be the effect of the decided to roll back the hike in the prices
same cause or independent causes. These of cooking gas and kerosene.
statements may be independent causes Statement (B): Some ministers had
without having any relationship. Read both resigned in protest against the hike in
the statements in each question and mark prices of cooking gas and other
your answer accordingly. Give answer- petroleum products.

8 Adda247 Publications For any detail, mail us at


Publications@adda247.com
A Complete Book of Logical Reasoning

25. Statement (A): A traveler found Which of the following may be


cockroaches in the crevices of his seat cause/effect among the above
and also in the toilets of India’s most statements?
prestigious train, the Rajdhani Express. (a) Statement II will be cause and
Statement (B): He drew the attention of Statement I and III will be it’s
catering manager, Western Railway, to effects.
the insects. (b) Statement II and III will be cause

p
and I will be it’s effect.
Directions (26): Read the following (c) Statement III is cause and
information carefully to answer the given Statement I is it’s effect and

ou
questions. Statement II is effect of
26. Read the following statements and independent cause.
answer the question. (d) Statement II is cause and Statement
I. Finally, the government has decided I is it’s effect and Statement III is
to sell off its sick industrial units, effect of independent cause.
which have become a burden to it. (e) Statement III is cause and

gr
II. Corruption, inefficiency and red- Statement II is it’s effect and
tapism have led to heavy losses to Statement I is effect of independent
the government owned industrial cause.
units.
Which of the following is true with 28. Statement I- The Indian Army plans to
respect to the above statements?
m recruit women in combat roles,
especially with regard to the operations
(a) Statement I is the cause and
in Jammu & Kashmir, Army Chief General
Statement II is its effect
Bipin Rawat said at the Passing Out
(b) Statement II is the cause and
Parade at the Indian Military Academy
xa
Statement I is its effect
(IMA).
(c) Both statement I and II are
Statement II- Many a time jawans in
independent causes
Kashmir feel hesitant about dealing with
(d) Both statements I and II are effects
women when they are in the front lines.
of independent causes Statement III- The situation in Kashmir
te

(e) Both statement I and II are effect of has been volatile for decades, flaring up
the same cause for worse since last summer. In the last
27. Statement I-Plan of the AAP led Delhi four days alone, the army has killed 13
government’s plan to install CCTV suspected militants.
ce

cameras in all Delhi Transport Which of the following may be


Corporation (DTC) and cluster buses cause/effect among the above
moved a step closer to reality after the statements?
Cabinet approved the proposal. (a) Statement II will be cause and
Statement II-AAP Party had announced Statement I and III will be it’s
effects.
@

in their manifesto before election to


install CCTV cameras in all Delhi (b) Statement II and III will be cause
transport. and I will be it’s effect.
Statement III-This measure will impart a (c) Statement III is cause and
feeling of security to women using public Statement I is it’s effect and
transport. Statement II is effect of
independent cause.

9 Adda247 Publications For any detail, mail us at


Publications@adda247.com
A Complete Book of Logical Reasoning

(d) Statement II is cause and Statement Statement I- Johnny Depp has apologized
I is it’s effect and Statement III is for a remark he made about
effect of independent cause. assassination. "I apologize for the bad
(e) Statement III is cause and joke I attempted last night in poor taste
Statement II is it’s effect and about President Trump," he said in a
Statement I is effect of independent statement made exclusively to People
cause. magazine.

p
29. In the following question, two Statement II-Jonny made a joke about
statements numbered I and II are given. the US president speaking to a crowd on
There may be cause and effect Thursday. "When was the last time an

ou
relationship between the two actor assassinated a President?"
statements. These two statements may Statement III- The actor was at the
be the effect of the same cause or Glastonbury Festival in the UK when he
independent causes. These statements made his remarks, which were recorded.
may be independent causes without Which of the following may be
having any relationship. Read both the cause/effect among the above

gr
statements in each question and mark statements?
your answer as- (a) Statement II will be cause and
Statement I-Pakistan Air Force has Statement I will be it’s effects.
operationalized all its forward bases (b) Statement I will be cause and II will
even as reports spoke of a Pakistani be it’s effect.
fighter jet flying over Siachen.
m (c) Statement III is cause and
Statement II-Indian counterpart asking Statement I is it’s effect and
all Indian Air Force officers to be ready Statement II is effect of
for operations at a very short notice as independent cause.
xa
there is an ever persistent sub- (d) Statement II is cause and Statement
conventional threat from Pakistan. I is it’s effect and Statement III is
(a) If statements I is the cause and effect of independent cause.
statement II is its effect. (e) Statement III is cause and
(b) If statements II is the cause and Statement II is it’s effect and
statement I is its effect.
te

Statement I is effect of independent


(c) If both the statements I and II are cause.
independent causes.
(d) If both the statements I and II are Direction (31-35): Each of the questions
effects of independent causes. below consists of two statements labeled I
ce

(e) If both the statements I and II are and II. These statements may be effects of
effects of some common cause. independent causes or of a common cause.
One of these statements may be the effect of
30. In the following question, three the other statement. Read both the
statements numbered I, II and III are statements and decide which of the following
given. There may be cause and effect answer choices correctly depicts the
@

relationship between the three relationship between these two statements.:


statements. These three statements may (a) If statement I is the cause and the
be the effect of the same cause or statement II is its effect.
independent causes. These statements
(b) If statement II is the cause and the
may be independent causes without
statement I is its effect.
having any relationship.

10 Adda247 Publications For any detail, mail us at


Publications@adda247.com
A Complete Book of Logical Reasoning

(c) If both the statements I and II are (a) If statement I is the cause and the
independent causes. statement II is its effect.
(d) If both the statements I and II are effects (b) If statement II is the cause and the
of independent causes. statement I is its effect.
(e) If both the statements I and II are effects (c) If both the statements I and II are
of some common cause. independent causes.
(d) If both the statements I and II are effects

p
31. I. Computer education has been made of independent causes.
compulsory for all the classes by (e) If both the statements I and II are effects
many schools. of some common cause.

ou
II. The current job market prefers
36. (I) The bank has provided a link on its
computer literate workforce.
website to obtain feedback from
32. I. The standard of education in customers.
evening colleges of the State has (II) Customers have been complaining
been deteriorating. about poor services in the bank’s
branches.

gr
II. The standard of school education
has been fast deteriorating in the 37. (I) The production of pulses has
State. dropped for the third consecutive
33. I. All domestic airlines increased the year.
(II) India has decided to import pulses
effect.
m
fare of all its classes with immediate
this year.
II. Railways increased the fare of all its 38. (I) Budgetary allocation for building a
classed with immediate effect. better railway network, eg
constructing new railway lines, has
xa
34. I. The prices of fruits and vegetables increased.
fell substantially over the last few (II) There has been a substantial drop
decades. in the number of passengers opting
II. The quantity of fruits and for air travel.
vegetables improved considerably
39. (I) Indian citizens are willing to incur
te

over the last few days.


the cost of using environment-
35. I. Recent floods in the area changed friendly technology.
the nutritional content of the soil. (II) Many countries are taking steps to
II. Farmers in the area switched over cut their carbon emissions.
ce

to cultivating rice instead of wheat. 40. (I) The government has amended tax
Directions (36-40): Each of the questions laws to boost exports.
below consists of two statements labeled I (II) The export sector has been passing
and II. These statements may be effects of through difficult times due to heavy
tax burdens.
@

independent causes or of a common cause.


One of these statements may be the effect of Direction (41-42): In each of the following
the other statement. Read both the questions, two statements numbered I and II
statements and decide which of the following are given. There may be cause and effect
answer choices correctly depicts the relationship between the two statements.
relationship between these two statements.: These two statements may be the effect of the

11 Adda247 Publications For any detail, mail us at


Publications@adda247.com
A Complete Book of Logical Reasoning

same cause or independent causes. These Which of the following may be


statements may be independent causes cause/effect among the above
without having any relationship. Read both statements?
the statements in each question and mark (a) Statement I will be cause and
your answer as- Statement II and III will be it’s
(a) If statements I is the cause and effects.
statement II is its effect. (b) Statement II and III will be cause

p
(b) If statements II is the cause and and I will be it’s effect.
statement I is its effect. (c) Statement III is cause and
(c) If both the statements I and II are Statement I is it’s effect and

ou
independent causes. Statement II is effect of
(d) If both the statements I and II are effects
independent cause.
of independent causes.
(d) Statement II is cause and Statement
(e) If both the statements I and II are effects
I is it’s effect and Statement III is
of some common cause.
effect of independent cause.
41. I. A mere 35.25% students passed (e) Statement III is cause and

gr
their class XII examination in 2017 Statement II is it’s effect and
in the Bihar School Examination Statement I is effect of independent
Board (BSEB) . cause.
II. Stringent steps were taken this year
after BSEB's Class XII results in 44. Statement I-The challenge for banks
2016 were marred by allegations
m under GST is to register in each state,
of brazen favouritism to help unlike the current tax system. They will
undeserving students top the exam. have to maintain state-wise revenue data
42. I. The government wants to exit Air and operate IT solutions to raise invoices
xa
India completely. for their business clients.
II. Niti Aayog has proposed total Statement II-Banks would need to
privatization of the national carrier customise the frontend IT ecosystem to
in a report to the Prime Minister’s recognise and deal with the state-centric
Office. compliances mandated under GST.
te

Statement III-Banks will be required to


43. Statement I-Finance Minister Arun
provide statewise data and compute
Jaitley has demanded reforms to
state-wise tax payments, which they
International Monetary Fund’s
controversial quota system, shedding didn’t have to do previously.
ce

light on the problems facing the Bretton Which of the following is cause/effect
Woods institution in today’s global among the given statement?
economy. (a) Statement I will be cause and
Statement II-Mr. Jaitley announced that Statement II and III will be it’s
India is seeking $2 billion from the New effects.
(b) Statement II and III will be cause
@

Development Bank.
Statement III-Quotas determine the size and I will be it’s effect.
of contingency funds at the disposal of (c) Statement III is cause and
the IMF to lend to countries in need of Statement I is it’s effect and
help, as well as the power of individual Statement II is effect of
countries to influence lending decisions independent cause.
and tap into the funds themselves.
12 Adda247 Publications For any detail, mail us at
Publications@adda247.com
A Complete Book of Logical Reasoning

(d) Statement II is cause and Statement effect relationship among the statements.
I is it’s effect and Statement III is These statements may be the effect of the
effect of independent cause. same cause or independent causes. These
(e) Statement III is cause and statements may be independent causes
Statement II is it’s effect and without having any relationship. Read all the
Statement I is effect of independent statements in each question and mark your
cause. answer as-

p
(a) Statement I will be cause and Statement
Direction (45-46): In each of the following
II and III will be it’s effects.
questions, two statements numbered I and II
(b) Statement II and III will be cause and I
are given. There may be cause and effect

ou
will be it’s effect.
relationship between the two statements.
(c) Statement III is cause and Statement I is
These two statements may be the effect of the
it’s effect and Statement II is effect of
same cause or independent causes. These
independent cause.
statements may be independent causes
(d) Statement II is cause and Statement I is
without having any relationship. Read both
it’s effect and Statement III is effect of
the statements in each question and mark

gr
independent cause.
your answer as-
(e) Statement II is cause and Statement III is
(a) If statements I is the cause and it’s effect and Statement I is an
statement II is its effect. independent cause.
(b) If statements II is the cause and
statement I is its effect. 47. Statement I- China has approved a plan
(c) If both the statements I and II are
m to build an underwater observation
independent causes. network across the disputed East and
(d) If both the statements I and II are effects South China seas, in yet another
of independent causes. provocative move that will anger
xa
(e) If both the statements I and II are effects Beijing's neighbors.
of some common cause. Statement II-China is likely to establish
additional military bases in Pakistan and
45. Statement I-Infosys, among India’s other countries with which it has
largest software services companies, longstanding friendly ties and similar
said it plans to hire 10,000 American strategic interests.
te

workers over the next two years. Statement III- China's military build-up
Statement II-In filling these jobs, Infosys initiative, along with regular naval vessel
will hire experienced technology visits to foreign ports, both reflects and
professionals and recent graduates from amplifies China's growing influence,
major universities, and local and
ce

extending the reach of its armed forces.


community colleges, to create the talent Which of the following may be
pools for the future. cause/effect among the above
46. Statement I- Aadhar has become statements?
mandatory for filing income tax returns 48. Statement I-International pop sensation
with PAN card.
@

Justin Bieber arrived in Mumbai on


Statement II-PAN has become more and Wednesday for his maiden concert in
more susceptible to duplicity and fraud. India.
Directions (47-48): In each of the following Statement II-A rage among the youth,
questions, three statements numbered I, II especially teenagers, Bieber is touring to
and III are given. There may be cause and promote his critically acclaimed album
‘Purpose.’
13 Adda247 Publications For any detail, mail us at
Publications@adda247.com
A Complete Book of Logical Reasoning

Statement III-Grammy Award winning (c) If both the statements I and II are
singer Justin Bieber fought back tears independent causes.
during a short speech, after he (d) If both the statements I and II are effects
performed acoustic versions of Love of independent causes.
Yourself and Cold water at the One Love (e) If both the statements I and II are effects
Manchester concert. of some common cause.
Which of the following may be
49. Statement I-The former UB group chief

p
cause/effect among the above
Mr. Mallya fled the country last year and
statements?
has unpaid debts close to the tune of Rs.
Directions (49-50): In each of the following 9000 crore taken as loans from various

ou
questions, two statements numbered I and II Indian banks.
are given. There may be cause and effect Statement II-Fugitive businessman Vijay
relationship between the two statements. Mallya was booed by a section of India
These two statements may be the effect of the supporters as he entered the Oval arena
same cause or independent causes. These to watch India’s crucial ICC Champions
statements may be independent causes Trophy encounter against South Africa.

gr
without having any relationship. Read both
50. Statement I-The National Council of
the statements in each question and mark
Educational Research and Training
your answer as-
(NCERT) will review all its books.
(a) If statements I is the cause and
Statement II-These books were created
statement II is its effect.
(b) If statements II is the cause and
m over a decade ago and need to be
updated, the books were created in
statement I is its effect.
2007.
xa
Cause and Effect : Solutions
1. (b); Clearly Statement I is the effect of For III: No, in this situation no
statement II as the Delhi metro political parties will show their
te

employees are protesting which is courage to put pressure on Govt. as


the effect and the cause of this they have to maintain their vote
protest is hike in the salaries and bank also.
pay grades. 4. (d); In this question we have to find out
ce

2. (b); Automation reduces the need of the cause and effect for making
amendments in Public Premises
manpower. So, statement-II is the
Act.
cause and statement-I is its effect.
For Statement I: It is clearly an
3. (c); For I: Yes, as this law was to effect of amendments in Public
@

prevent SC/ST from brutality but Premises Act.


this amendment may increase For Statement II: This cannot be
oppression towards backward cause or effect, as there is no role of
class. India against corrupt leader in this.
For II: Yes, they may call for Bharat For Statement III: It may be the
bandh to fight for their rights. cause for making amendments.

14 Adda247 Publications For any detail, mail us at


Publications@adda247.com
A Complete Book of Logical Reasoning

5. (d); I, III & IV all are the ill-effects of the 16. (d); Income tax authorities carried out
increasing global warming on the raids due to non-payment of tax or
earth. illegal money so as an effect
business houses took immediate
6. (e); The main cause of the increasing
action to clear off all their income
global warming can’t be inferred
tax dues in order to avoid raid.
from the given passage.
17. (e); Both Statements I and II are effects

p
7. (e); All these are responsible for
of some common causes.
increased child labour in India.
18. (a); Statement I is the cause while

ou
8. (d); Child labour will reduce skill
Statement II is its effects.
development. And since children
are working, it will also reduce the Directions (19-21):
literacy rate. 19. (a); Clearly, the association’s decision to
Direction (9-11): go for a ‘bandh’ on a massive scale
9. (b); The surge in complaints has let the to protest against the government’s

gr
banks to receive them in an new policy would have instigated
electronic mode. the shopkeepers to keep their
shops closed. Thus, I mentions the
10. (a); The shortage in production has led direct cause for the effect in II.
to the import decision
11. (e); Both are the effects of a common
m 20. (b); Clearly, a cut in the tax or duty on a
certain product is bound to cause a
cause. The Railway has gone in a reduction in the price of the
reviving mode. product. Thus, II is the cause of the
xa
12. (b); The government has decided to sell effect in I.
off its industrial units, because 21. (c); Clearly, each statement is self-
heavy loss was caused by sufficient in itself and stands
corruption, inefficiency and red- independent of the other. Thus, I
tapism. Hence statement II is the and II are independent causes.
te

cause and I is its effect.


22. (d); Clearly, calling off the strike and
13. (e); The facts given in both the going on strike are events that may
statements are clearly the result of not be backed by the same cause.
acute power shortage. Thus, they must have been effects
ce

14. (e); The facts given in both the triggered by separate independent
statements are clearly the result of causes.
acute power shortage. 23. (e); Clearly, lesser number of people
15. (b); The prices of food grains and visiting a place during the week
days and more people visiting
@

vegetables have increased by about


30% due to increase of more during the weekend, both imply
than30% in the price of other events that go together, and must
products, because farmers will need have happened due to a common
30% more funding for purchasing cause such as, it being a holiday
other products. during the weekend.

15 Adda247 Publications For any detail, mail us at


Publications@adda247.com
A Complete Book of Logical Reasoning

24. (a); Because of the statement (B), 30. (a); The reason why Depp had to
government has decided to roll apologize was the fact that he made
back the hike in the prices of “horrible” remark against the
cooking gas and kerosene. president. So, (ii) is the cause and
25. (b); As he found cockroaches in the (i) is its effect.
crevices, he immediately drew the Statement (iii) just gives the details
attention of the catering manager. about the actor’s whereabouts at

p
So statement (B) is the effect of the time when he made that remark
statement (A). so it is irrelevant to the question.
Direction (31-35):

ou
Directions (26-30):
31. (b); Job market is an important
26. (b); The government has decided to sell consideration in determining the
off its industrial units, because curriculum of schools.
heavy loss was caused by
corruption, inefficiency and red- 32. (e); There seems to be some common
tapism. Hence statement II is the cause that is leading to both kinds

gr
cause and I is its effect. of education.
27. (a); In the above question it is clear that 33. (e); Hike in fuel price seems to be
Statement II will be the cause and because of some common cause.
both Statement I and Statement III 34. (d); The two effects are unrelated.
mentioned in statement II that
m
will be its effects. Because it is
35. (a); The farmer’s decision is apparently
cabinet has taken a decision an effect of change in the soil.
towards women safety so this will
36. (b); The surge in complaints has let the
be cause and its effect will be Delhi
xa
banks to receive them in an
government’s plan to install CCTV
electronic mode.
and also to give feeling of security.
37. (a); The shortage in production has led
28. (d); In the above question it is clear that
to the import decision
Statement II will be the cause and
Statement I will be its effect. 38. (e); Both are the effect of a common
te

Because it is mentioned in cause. The Railway has gone in a


statement II that jawans have doubt reviving mode.
how to deal with the women so this
will be cause and its effect will be, 39. (e); Both are the effects of gearing up on
the environmental front.
ce

The Indian Army will recruit


women in combat roles to tackle 40. (b); The burden is sought to e reduced
such kind of situation. by the amended tax laws.
29. (a); As it is clear from the above 41. (b); In the above statement II is cause
statement that statement I is the and I is effect because after the
@

cause and statement II would be its


results of BSBE’s Class XII in 2016
effect as Pakistan had made all its
bases operative, its effect would be several steps were taken by the
that Indian air force has also made authority and it’s effect is only
their officers ready to give response 35.25% students passed in the
in an appropriate manner. examination.

16 Adda247 Publications For any detail, mail us at


Publications@adda247.com
A Complete Book of Logical Reasoning

42. (a); In the above statement it is clear vulnerable that is why Aadhar has
that I is cause and II is it’s effect become mandatory for filing
because the government will take Income Tax.
steps to make it a strong and viable
47. (e); In this question Statement II is the
airline and that is why Niti Aayog
cause and Statement III will be its
has come up with proposal of
effect because China is going to
privatization of Air India.
establish military bases in Pakistan

p
43. (c); In the above question we have to and other countries as its effect will
find the cause and effect among the be that the reach of its armed forces
given statements. is going to be enhanced. Statement I

ou
It is clear from the above will be a independent cause
statements that Statement III is the because it discusses about the plans
cause and Statement I is it’s effect of China regarding its disputed East
as the quota system influence the and South China seas.
lending system and because of that
48. (d); In the above question statement II
Mr. Jaitley has demanded reforms

gr
will be the cause and statement I
to it.
will be its effect as Justin Bieber is
Whereas Statement II has it’s own
touring to promote his album will
independent cause as it discusses
be the cause of his arrival to
about the loan India is taking from
Mumbai for his concert in India
NDB.
m which is its effect. And Statement III
44. (a); In the above statement we have to will be the effect of a different cause
find the cause and effect among as it discusses about him being
them. emotional at a concert.
xa
Statement I will be the cause and
49. (a); In the above question it is clear that
Statement II and III will be it’s
statement I will be the cause and
effects as banks would need to
statement II will be its effect.
change it’s IT system and will
Because Mallya has taken loan from
provide state wise data because of
various banks in India and has not
changes in banking system after
te

been able to repay it for some time.


GST invokes.
So this will be the reason of Vijay
45. (a); It is clear from the above that Mallya’s disapproval by the
statement I is the cause and II is it’s audience at the stadium
ce

effect as this will be the cause that


50. (b); It is clear from the above statement
Infosys is planning to hire workers
that statement II will be cause and
and for that it will hire graduates
statement I will be its effect.
and will create talent pools will be
Because books were created in
it’s effect.
2007 quite a while before it will be
@

46. (b); It is clear from the above statement a cause and it need to be updated
that II is cause and I is it’s effect now so it will be its effect.
because the PAN has become more

17 Adda247 Publications For any detail, mail us at


Publications@adda247.com
A Complete Book of Logical Reasoning

p
ou
gr
m
xa
te
ce
@

1 Adda247 Publications For any detail, mail us at


Publications@adda247.com
A Complete Book of Logical Reasoning

05 Statement and Inference


Introduction: Inference is an idea or conclusion that’s drawn from evidence and given fact. An
inference is an educated guess. Inference is totally based conclusion on the given passage and

p
information and it must be correct for that particular fact.
→ Inference is a part of critical reasoning. Which shows one’s thinking apart systematically.
→ Inference is a step of the mind, an intellectual act by which one concludes that something is

ou
true in light of something else’s being true or seeming to be true.
For example:
If you come at me with a knife in your hand, I probably would infer that you mean to do me
harm, so we can say Inferences can be accurate or inaccurate, logical or illogical, justified or
unjustified.
→ Here we see that it looks similar as an assumption but remember that both are different

gr
concept not similar.
→ An assumption is something we take for granted for granted or presuppose. Usually it’s
something we previously learned and donot question.
Note: Always remember that an Inference have work something similar as assumption, but an
m
assumption can never work any similarity with Inference.
→ We naturally and regularly use our beliefs as assumptions and make inferences based on
those assumptions. We must do so to make sense of where we are, what we are about and
what is happening.
→ Assumptions and Inferences permeate our lives precisely because we can’t act without
xa
them we make judgements if interpretations and come to conclusions bassed on the beliefs
we have formed.
For example:
We see dark clouds and inter rain. We hear the door slam and infer that someone has arrived.
→ Eventually we need to realize that the inferences we make are heavily influenced by our
te

point of view and the assumptions we have made about people and situations.
→ Note: Inference is a click think about situation on the spot and on the basis of inference we
think many more assumption.
For example:A man is lying in the gutter.
ce

Inference: That man is in need of help.


Assumption: Anyone lying in the gutter is in need of help.
Some tricky points you have to remember to find out the inference from the passage :
(1) Analyze scope: Inference junk answer will typically go outside the direct scope of the
passage. Be careful to look directly at the scope of the question. Inference answers must be
with in the scope of the passage. Your opinions or information outside of the passage are
@

always outside of the scope.


(2) Don’t jump into the assumption hunt these questions usually don’t carry much in the way of
glaring assumptions. Instead, these questions generally test your ability to derive a
conclusion from stated premises.
(3) Try to fully understand what the passage’s point is and the exact reasoning so that if the
question asks you to extend that reasoning, you are able to accurately do so.
2 Adda247 Publications For any detail, mail us at
Publications@adda247.com
A Complete Book of Logical Reasoning

(4) Knock out answers with extreme wording. Inference answer typically donot use "only",
always, never, best or any strong words that leave little wiggle room. The right answers on
inference question will generally use more qualifiers and less extreme language.
(5) Use the process of elimination - Infference questions typically have two or three good
answers that are semi-plausible (similarily). The best way to tackle these questions is to
gradually eliminate the possible answers until you have one or two and then choose the last
one by scope.

p
Question: In 2012, India used three-times as much energy from non- renewable energy by
2022, while using a larger amount of energy than in 2012.
Which of the following must happen for India’s plan to work.

ou
(a) By 2022, India will more than triple its use of energy sources.
(b) India will have to make a political effort to have a more snstainable energy economy.
(c) By 2022, India will have to decrease its dependency on non-renewable energy sources.
(d) By 2022, India will more than triple its uuse of renewable energy sources over 20012 levels.
(e) New technologies, must be developed to make the lost of renewable resources more

gr
competitive with renewables.
Answer and Explanation:
(a) It doesn’t implies because India does not need to triple its energy sources (just renewables).
(b) This statement is not relevant.
(c) India doesn’t need to decrease its use of non-renewables.
m
(d) This one is implies, India must at least triple its use of non-renewables.
(e) This may be the case, but there is nothing in the given passage question to make this point.
So, answer will be only (d).
Format of questions are
xa
Direction: Given will be a passage followed by several inference based upon it. You have to
examine the passage carefully and then decide the validity of each of the inference.
Mark answer
(a) If the inference is definitely true. (b) If the inference is probably true.
(c) If the data are Inadequate. (d) If the inference is probably false.
te

(e) If the inference is definitely false.


Probably/Definitely True of False questions belong to the Critical Reasoning category in the
Verbal Section. You are expected to assess the degree of truth or falsity of a set of statements in
the context of a given passage.
ce

(1) A statement is definitely true, if it follows directly from the data given in the passage. A
statement is definitely true if and only if it follows from the given passage without any
ambiguity, and is completely supported by the passage. For a statement to be definitely true
there should be no scope for any reasoning which suggests that the statement may not be
true
@

(2) A statement is probably true, if it is most likely but not definitely true. A statement is
probably true if it does not directly follow from the passage, but the data in the passage
suggests or indicates that it is likely to be true. In other words, the inference is such that
when one evaluates its degree of truth and falsity one finds that the degree of truth is higher
than the degree of falsity. For a statement to be probably true there is sufficient direct
evidence that makes it likely to be true, but not enough to make it definitely true.

3 Adda247 Publications For any detail, mail us at


Publications@adda247.com
A Complete Book of Logical Reasoning

(3) A statement is definitely false, if it contradicts the data that is given in the passage. A
statement is definitely false if and only if there is absolutely no chance that the statement
can be considered true according to the data given in the passage. For a statement to be
definitely false there should be no scope for any reasoning which suggests that the
statement may be true.
(4) A statement is probably false, if it is most likely but not definitely false. A statement is
probably false if according to the passage, there is a high chance that it is false though there

p
is no direct contradiction to the data in the passage that makes it definitely false. For a
statement to be probably false there should be no direct evidence that proves the statement
to be false. The evidence available in the passage forces us to evaluate its ‘probability’. On

ou
evaluating its probability, we find that the inference is ‘likely to be false’ rather than ‘likely
to be true’. In other words, though it ‘may be true’, it is less likely to be true. If one were to
think mathematically to understand this concept, if the chances are equal or more than
equal (50%), the inference is likely to be true or “probably true”, and if the chances are less
than 50%, it is likely to be false or ‘probably false’
(5) A statement is uncertain, if its truth or falsity cannot be ascertained to any degree from

gr
the given data. A statement is uncertain or ‘data inadequate’ if the given data is not
sufficient to evaluate the definiteness or probability of the inference. Taking the probability
example from the previous explanation, the data is uncertain when the probability of its
truth or falsity is 50%.
m
Points to Remember
xa
(1) Inference is a conclusion/deduction/proposition drawn from the given passage with the help of facts and
justifiable assumption.
(2) Inference must be from "the point of view" of the passage and not your personal point of view.
(3) Analyze the "scope" of passage. Inference must be with in the scope of passage.
(4) First look for definitely true and definitely false inferences. Then use ellinination method to determine
te

"Probably true" and "probably false" inferences.


(5) "Assumption" should be relevant, logical and universally acceptable.
ce

Exercise

1. Statement: A sting operation by a news Which of the following can be inferred from
organisation called Cobrapost claims to the above statement?
@

have revealed a deeply engrained bias (i) India’s leading media companies are
towards the ruling BJP within many of more concerned with TRP rather than
India's leading media groups, as well as a showing the truth.
willingness among some of the country's (ii) The ruling party is trying to manipulate
most senior media executives and the content provided by the media.
journalists to take money in return for (iii) Some of the journalists are corrupt.
pushing a political agenda.
4 Adda247 Publications For any detail, mail us at
Publications@adda247.com
A Complete Book of Logical Reasoning

(a) Both (i) and (ii) III. Question paper will be provided to
(b) Both (ii) and (iii) centres in the form of zip file which
(c) Only (ii) will be password protected.
(d) Only (iii) (a) Only I (b) Both I and III
(e) None of these (c) Both I and II (d) All of the above
Direction (2): Study the following (e) None of these
information in which a statement is followed

p
4. Begusarai, a district in Bihar which has
by some statement, read carefully and answer
seen the phase of kidnapping, extortion,
the question below-
A company X has decided to increase the HRA murder to “The Industrial city of Bihar”.

ou
allowances of its employees due to Nowadays there are 4 major industries
reallocation of the company to NCR region. in Begusarai. Growth rate of Begusarai is
Which of the following can be inferred from top amongst all districts of Bihar and 5th
the given statement? among all districts of India in year 2017.
(I) The HRA allowances directly depend on Which of the following can be inferred
the region of living of the employee. from the above statement?

gr
(II) The employees of the company X are on I. Opportunities of the employment
strike due to the reallocation of the has been increased in Begusarai.
company. II. There is no case of murder,
(III) The profit amount of company X has kidnapping and extortion in 2017.

amount of HRA of its employees.


m
increased so it has also increased the III. People of Begusarai are
hardworking and keen to go
(a) Both (I) and (II)
forward.
(b) Both (II) and (III)
IV. Top industrialists of India are
(c) Only (I)
xa
(d) Only (III) desirous to have their industry in
(e) None of these Begusarai.
(a) Only II (b) Only I and III
3. Facing flak over paper leaks, the Central (c) Only I (d) Only III and IV
Board of Secondary Education on
(e) None of these
Monday put in place a system of
te

“encrypted” question papers, which are Direction (5): Study the following
supposed to be printed by the schools. information in which a statement is followed
“CBSE has devised a mechanism of by some statement, read carefully and answer
providing encrypted question paper at the question below-
ce

the examination centres just before the There are some criteria which will be
commencement of examination,” considered during appraisal of employee-
Which of the following can be inferred Statement by manager of a company.
from the above statement?
Punctuality and sincerity are one of the key
I. CBSE has considered encrypted
points which will surely reviewed.
@

system as safe way to conduct a fare


exam. Hard work and dedication towards work will
II. There will be safe and secure room be applauded.
for installation of computers and Just to quantify an amount in the salary
printers which will have high speed package of employee with no reason will not
internet connection. be entertained.

5 Adda247 Publications For any detail, mail us at


Publications@adda247.com
A Complete Book of Logical Reasoning

5. Which of the following can be inferred Which of the following can be inferred
from the above statement? from the above statement?
I. Punching machine of office will be (I) Delhi Government has good
evaluated by HR. intentions to have peace and
II. There will be minimum 10% maintaining law and order in city.
increment for deserving employee (II) Kejriwal Government has built
based on their performance. many govt. schools with advance

p
III. Some of employees of company amenities to improve education
have done a tremendous job with system.
complete determination and (III) There is a difference between

ou
enthusiasm.
monopolies of Central and State
(a) Only I (b) Both II and III
Government.
(c) Both I and III (d) Only III
(a) Only III (b) Only I and II
(e) None of these
(c) Only I and III (d) Only I
6. Statement: Mr. Thompson- “People (e) None of these
generally carries grudge from their past

gr
relationship instead of being happy with Direction (8-9): In the following question a
sweet memories and learn from its statement is given followed by an inference.
experience. One can move with its all You have to read the statement and decide
learning and can bring positivity further which of the following option logically follows
in life”. among the given options.
Which of the following can be inferred
m Mr. Singh, a chief editor of The Times of India
from Mr. Thompson statement? writes DeMon demon: The most ill-
(I) Relationship is a mixture of grudge, considered economic move by the Modi
memories and learnings. government has been demonetization.
xa
(II) Positivity is just a myth, Grudge is Following the release of RBI’s annual report,
not just a bite of cake to digest it. it is clear Almost 99.3% of demonetized
(III) Grudge is hindrance in growth of currency was returned by its holders. From
life. an economic standpoint, it is the most ill-
(a) Only II (b) Only I and III advised move of the Narendra Modi
(c) Only II and III (d) Only I
te

government. Out of the Rs. 15.42-lakh crore,


(e) None of these Rs. 15.31 lakh crore has come back to the
7. Statement: Delhi chief minister Arvind banking system. The expectation that those
Kejriwal who is much concerned for the who had hoarded their ill-gotten wealth in
social security, education and health cash would fear depositing or exchanging
ce

system has visited Government hospital these at banks. Far from dealing a body blow
for the inspection of medicines and to black money, demonetization inflicted
availability of doctors. At the same time avoidable pain on farmers, daily wage
when Mr. Kejriwal was on inspection, a labourers and informal enterprises used to
dalit boy has been beaten by some goons transact in cash.
@

near a police station which was at 100 m


away from the place of incident. 8. Inference: A huge amount of black
Spokesperson of Delhi Government has money had been there which was
stated that Delhi police is under Central circulated in market.
Government, so we face hindrance to On the basis of statement, mark your
achieve social security. answer as:

6 Adda247 Publications For any detail, mail us at


Publications@adda247.com
A Complete Book of Logical Reasoning

(a) If the inference is definitely true these smartphones can walk in to the
(b) If the inference is probably true nearest Reliance Digital or Xpress Mini
(c) If the data provided is inadequate Store to get a new Jio SIM with 3 months
(d) If the inference is probably false of unlimited data, voice calling, and SMS,
(e) If the inference is definitely false as well as access to Jio apps and services
like Jio on Demand.
9. Inference: Reality about black money is
Which of the following can be inferred
not known but it’s a clear fact that

p
from the given statement? (An inference
normal people suffered a lot mostly daily
is something that is not directly stated
wage workers and farmers.
but can be inferred from the given
On the basis of statement, mark your

ou
information)
answer as:
I. These changes would improve the
(a) If the inference is definitely true economics of building up internet
(b) If the inference is probably true services.
(c) If the data provided is inadequate II. Reliance Jio is enhancing its cost
(d) If the inference is probably false base in mobile infrastructure.
(e) If the inference is definitely false

gr
(a) Only I (b) Only II
10. Statement: Every holy river in India is (c) Either I or II (d) Neither I nor II
going to lose its holiness and the (e) Both I and II
percentage of pollution is only 12. Statement: Persistent weakness in
increasing in spite of spending millions global demand and the lower value of oil
of rupees for its treatment by the
m products led India’s merchandise
government. exports to fall for the sixth straight
Which of the following can be an month in May, while a decline in gold
inference of the above statement? imports helped the trade deficit narrow
xa
(a) The government is not serious to a three-month low.
about cleaning rivers in India. Which of the following inference drawn
(b) Pollution divests rivers of their is probably false according to the above
holiness. statement?
(c) India is using polluted river water (a) There will be softening of oil
for drinking purposes. products in the global market.
te

(d) India is averse to seeking overseas (b) Rupee will appreciate against the
technical help for cleaning its rivers. dollar.
(e) None of these (c) There will be an increase in the
collection of indirect taxes in the
ce

11. Statement: Reliance Jio SIM is now said


month of April and May.
to be available for purchase by anyone
(d) There will be an increase in the
with a 4G phone. Multiple users on domestic demand of gold and
Twitter are reporting that they’ve been jewellery.
able to purchase a Reliance Jio SIM with (e) Only (b) and (d)
their non-Lyf/ Samsung/ LG phone,
@

though others claim that the Reliance 13. It is a popular misconception that
Digital officials are unaware of this new nuclear fusion power is free of
offer. It had emerged that Reliance Jio radioactivity; in fact the deuterium-
had opened its Jio Preview offer to more tritium reaction that nuclear scientists
Samsung smartphones and select LG are currently exploring, produces both
smartphones, which means owners of alpha particles and neutrons.

7 Adda247 Publications For any detail, mail us at


Publications@adda247.com
A Complete Book of Logical Reasoning

Which of the following can be inferred Which of the following can be inferred
from the above? from the above?
(a) Nuclear fusion does not involve (a) Beauvoir’s work had a distinct anti-
production of alpha particles, and woman stance.
neutrons. (b) Political conditions during
(b) Production of alpha particles and
Friedan’s times were more stable
neutrons constitutes radioactivity.

p
(c) Nuclear fusion does not result in than during Beauvoir’s times.
radioactivity. (c) The works of Friedan were more
(d) The deuterium-tritium reaction is appealing to the common people

ou
an example of nuclear fission. than those of Beauvoir.
(e) None of these (d) Friedan plagiarized the works of
14. The human body responds to a viral Beauvoir.
infection by producing antibodies. An (e) None of these
antibody can either interfere with a
16. Margaret: Earth is situated at such a
virus’ ability to bind to a cell, or can

gr
distance from the sun, that it is neither
prevent it from releasing its nucleic acid.
Unfortunately, the common cold, too near to be very hot nor too far to be
produced most often by rhino viruses, is very cold. Thus if another planet could
intractable to antiviral defense. Humans be present at almost the same distance
m
have difficulty resisting colds because
rhinoviruses are so diverse, including at
from the sun, it would definitely sustain
human life.
least 100 strains. Mary: You seem to assume that climate
Which of the following can be inferred is the only essential condition for life.
from the above passage?
xa
But if oxygen and water are not found in
(a) An antibody cannot attack all types
such a planet it cannot sustain human
of rhino viruses.
(b) Common cold producing rhino life.
viruses can easily be destroyed by Which of the following can by inferred
antibodies. from Mary’s reply?
te

(c) Common cold is not amenable to (a) Presence of oxygen and water is
any medical treatment. more important than climatic
(d) Rhinoviruses are capable of conditions to sustain human life.
destroying antibodies produced by (b) There is no possibility of such a
ce

human body. planet.


(e) None of these
(c) Human life could exist even in
15. Simone de Beauvoir’s work greatly planets which are not at a similar
influenced Betty Friedan’s — indeed distance from the sun as the earth
made it possible. Why then was it is; provided they have oxygen and
@

Friedan who became the prophet of water.


women’s emancipation in the United
(d) Oxygen and water are factors in
States? Political conditions, as well as a
certain anti-intellectual bias, prepared addition to proper climate to
Americans and the American media to support human life in a planet.
receive Friedan better than Beauvoir. (e) None of these

8 Adda247 Publications For any detail, mail us at


Publications@adda247.com
A Complete Book of Logical Reasoning

Directions (17): Study the following What can be inferred from Modi’s
information carefully and answer the remark “meant for each other” to
questions given below. Merkel?
The centre reportedly wants to continue (i) He has grown fond of Merkel
providing subsidy to consumers for cooking (ii) He made a slip-up
gas and kerosene for five more years. This is (iii) He meant to say that India needs
not good news from the point of view of help from Germany for

p
reining in the fiscal deficit. Mounting development.
subventions for subsidies means diversion of (a) Only (ii)
savings by the government from investment (b) Only (i)

ou
to consumption, raising the cost of capital in (c) Only (iii)
the process. (d) All (i), (ii) and (iii)
The government must cut expenditure on (e) None of these
subsidies to create more fiscal space for 19. India’s indigenous GSLV-Mark III makes
investments in both physical and social a bid to breach a heavy-lift rocket club
infrastructure. It should outline a plan for that can put four-tonne satellites into

gr
comprehensive reform in major subsidies space.
including petroleum, food and fertilizers and Which of the following can be inferred
set goals. from the given statement?
17. Which of the following is an inference (I) With the launch, ISRO has
demonstrated its mastery in
in the above paragraph?
m
which can be made from the facts stated
developing a cryogenic engine, a
technology denied to it years ago. It
(a) India’s fiscal deficit is negligible in
has also laid a strong foundation for
comparison to other emerging
its ambitious future projects,
economies in the world.
xa
including Chandrayan-II and a
(b) Subsidy on food and fertilizers are
manned mission, besides venturing
essential for growth of Indian
into the global heavy payload
economy. market.
(c) Reform in financial sector will (II) With this launch India will become
weaken India’s position in the most powerful country in the world.
te

international arena. (III) India has become more powerful


(d) Gradual withdrawal of subsidy is than china with this heavy GSLV-
essential for effectively managing Mark III satellite.
fiscal deficit in India. (IV) The launch proves the homegrown
ce

(e) None of these launch vehicle's capability of


18. On Tuesday, Germany PM Merkel met hurling up to four tonne payload
with Indian Prime Minster Narendra into higher orbits.
Modi. The two leaders signed eight (a) Only I and II (b) Only III
agreements pertaining mostly to (c) Only II and IV (d) Only III and II
(e) None of these
@

cooperation on job skills development


and environmental issues. This meeting 20. Statement- Indian Prime Minister
could not have been more positive and Narendra Modi has vowed his country
cordial. will go "above and beyond" the 2015
"We are meant for each other," Modi told Paris accord on combating climate
Merkel, smiling widely. change.

9 Adda247 Publications For any detail, mail us at


Publications@adda247.com
A Complete Book of Logical Reasoning

Which of the following can be inferred (I) The Academy has imparted free
from the above statement? magic training for the children with
(i) India is not in a condition to accept an objective of creating confidence
the Paris accord. and self-esteem among them under
(ii) India has made a significant a special programme titled “M-
progress since 2015.
Power”.
(iii) India understand the impact of

p
(II) This programme also aims at
carbon emissions on climate change
and is willing to do more on this reducing the barriers in the path of
issue. a differently-abled person and

ou
(a) Both (i) and (iii) creating conducive environment for
(b) Both (ii) and (iii) his/her holistic development
(c) (i), (ii) and (iii) (III) Government will ensure that such
(d) Only (iii) children have access to school so
(e) None of these. that they can live their life in a more

gr
21. Statement-The "kidnapping" case of a desirable manner.
retired Pakistan army officer in Nepal Which of the following can be inferred
could not be taken to the ICJ or the UN from given statement?
on the basis of "assumptions" as there is (a) Only II and III (b) Only I and III
no "concrete evidence" suggesting (c) Only II (d) Only I and II
m
India's hand in it, a minister has told (e) None of these
Parliament.
Which of the following can be inferred 23. Statement- The banks of Ganga to go
from the given statement? eco-friendly, 'Green Crematoriums' to be
xa
(I) The issue appeared to have a link set up in villages. This 'ecologically-
with the conviction of Kulbushan responsible cremation' will safeguard
Jadhav. the environment by putting in best
(II) It can be said that Indian practices during cremations.
intelligence agency RAW is behind (I) It may help in a bid to use less wood
te

the kidnapping. and protect the Ganga and other


(III) The FO (foreign officers) was trying
rivers from pollution.
to obtain solid evidence and the
(II) Zero cremations in the open would
Nepalese government had been
ce

asked to get in touch with three ensure that no ash and no carbon
Indians who had received the emissions are dumped into the
Pakistan army officer at the airport. river that ends up getting polluted
(a) Only (I) in the normal course.
(b) Only (III) Which of the following can be inferred
@

(c) Only (II) from the given statement?


(d) Both (II) and (III) (a) Only I
(e) None of these (b) Only II and I
22. Statement-Vice-President Hamid Ansari (c) None of the above
launches M-power to empower special (d) Either I or II
kids through magic training. (e) All of the above
10 Adda247 Publications For any detail, mail us at
Publications@adda247.com
A Complete Book of Logical Reasoning

24. Statement:- Some people like to start (III) China is keenly watching the
slow when they pick up a new sport. defence agreements set to be inked
Others like to take on the 49-time by Indian Prime Minister Narendra
undefeated champion. That’s what mixed Modi and US President Donald
martial arts champ Conor McGregor Trump in Washington, with
plans to do this August when he enters particular attention on the deal for
the boxing ring to face off against Floyd surveillance drones that would

p
Mayweather. A gimmick? Possibly. A boost India's capabilities in the
blockbuster money-maker? Almost Indian Ocean.
definitely. (a) Only II and I (b) Only III and I

ou
Which of the following can be inferred (c) Only II and III (d) All of the above
from the above statement? (e) None of these
(I) It is not advisable to pick up a new
sport. 26. Statement-“Killing in name of cow
(II) Gimmick sports events are able to protection is unacceptable, it’s not
draw people’s attention. something Mahatma Gandhi would

gr
(III) It is highly unlikely for McGregor to approve”:- says Indian PM Modi.
win against Floyd. Which of the following can be inferred
(a) Only (III) from the given statement?
(b) Only (I) and (II) (I) India and our freedom fighters
(c) Only (III) and (I) would be proud of having cows in
(d) Both (II) and (III)
(e) All (I), (II) and (III)
m our country.
(II) Mahatma Gandhi was a big
supporter of non-violence .He
25. China has confirmed that it shut down
always resist killing of innocents.
the Nathu La pass in Sikkim after the
(III) Cow is India’s national animal so we
xa
obstruction by Indian guards and denied
should protect it.
entry to Kailash Mansoravar (located in
(a) Only (III)
Tibet) pilgrims because of an on-going
stand-off between Indian and Chinese (b) Only (II)
troops at the border and on account of (c) Only (I)
"security reasons". (d) Both (II) and (III)
te

Which of the following can be assumed (e) None of these


from the given statement? 27. Statement:- Days after news of the
(I) The Indian border guards in Sikkim massive IS recruitment drive from north
section crossed the border into the Kerala's villages in mid-2016 shocked
ce

territory of China, obstructed the the state, Kerala police launched a silent
Chinese border troops from but strategic de-radicalisation drive
conducting normal activities. China named Operation Pigeon for youths,
has taken corresponding measures. aimed at preventing a repeat of a
(II) The Chinese side asked the Indian previous year situation.
side to respect the border treaty,
@

Which of the following can be inferred


respect the territorial sovereignty from the given statement?
of China, carry out the immediate (i) All districts reported footprints of
withdrawal of border officials and IS recruiters who had made more
thoroughly investigate the matter, than preliminary contacts with
to maintain peace and tranquility in
them.
the Chinese-Indian border.
11 Adda247 Publications For any detail, mail us at
Publications@adda247.com
A Complete Book of Logical Reasoning

(ii) The police organised individual and Which of the following can be inferred
collective counselling sessions for from the given statement?
the youth. The sessions were (I) Republic Day 2018 will be the first
handled by specially-trained time ever that so many leaders will
personnel from NIA and IB. together be chief guests at the
(iii) Kerala's youth have been parade which showcases India's
vulnerable to intelligence agencies, military might.

p
who have been sending letters to (II) India is the only country which
them through pigeons. celebrates republic day among
(a) Only (i) and (iii) Southeast Asian Nations.

ou
(b) All (i), (ii), (iii) (III) India and Asean are actively
(c) Only (ii) and (iii) engaged in comprehensive
(d) Only (i) and (ii) economic partnership', which is
(e) None of these expected to emerge as the largest
regional trading arrangement.
28. Statement:- Amidst the increasing (a) Only (I) (b) Only (II) and (I)

gr
tension between the two countries, (c) Only (III) and (II) (d) Only (II)
China on Wednesday said that it was (e) None of these
open to discussing with India "the
possibility of alternative arrangements 30. Statement:- India's rivers Ganga and
through other routes for Indian official Yamuna cannot be viewed as living
entities, the Supreme Court has ruled. It
yatris who had planned to visit Kailash
and Manasrovar via Nathu La Pass".
m overruled an order made in March by the
High Court in Uttarakhand state, which
Which of the following can be inferred
said that the two rivers had the same
from the above statement?
legal status as human beings.
(i) China stalled this year's pilgrimage
xa
Which of the following could be inferred
from Sikkim and through the Nathu
from the above statement?
La pass.
(i) The Supreme Court is not serious
(ii) An Indian solider was released by about the cleansing of the rivers.
China recently. (ii) It was legally unsustainable to
(iii) There are no other routes to Kailash consider a river as a living entity.
te

and Mansarovar except Nathu La (iii) Digitalization and modernization


Pass. rather than worshipping rivers is
(a) Only (i) the right approach towards growth.
(b) Only (ii) (a) Only (i)
ce

(c) Only (ii) and (iii) (b) Only (ii)


(d) Only (i) and (iii) (c) Only (ii) and (iii)
(e) None of these (d) Only (i) and (iii)
29. Statement-India will first time invite (e) None of these
heads of 10 Asean nations for Republic 31. Statement:- The Supreme Court on
@

Day 2018 celebrations. Asean is the Monday allowed IITs, NITs and other
Association of Southeast Asian Nations. engineering colleges to go ahead with
It has as members Brunei, Cambodia, their admission process on the basis of
Indonesia, Laos, Malaysia, Myanmar, JEE merit list.
Philippines, Singapore, Thailand and Which of the following could be inferred
Vietnam. from the above statement?

12 Adda247 Publications For any detail, mail us at


Publications@adda247.com
A Complete Book of Logical Reasoning

(i) The apex court had earlier observed 33. Statement:- The Republican Party's
some faults in the entrance process efforts to gut former President Barack
and stayed the ongoing counseling Obama's legacy health care law came to
and admission in IITs, NIITs, IIITs
an abrupt -- if temporary – halt last
and all government-funded
engineering colleges. Monday night.
(ii) Supreme Court has no right to Which of the following can be inferred
interfere and cause delay in the from the above statement?

p
admission of the candidates as their (i) This halt is a setback for
future career depends on it. Republicans who have been against
(iii) There is a lack of sufficient evidence

ou
the law.
to decide if there were some
discrepancies in the admission (ii) It is now apparent that the effort to
process which had motivated the repeal and immediately replace the
Supreme Court to put a stay on the Obama care laws will not be
ongoing admission process earlier. successful.
(a) Only (iii) (iii) Americans will have to cope up

gr
(b) Only (ii) with Obama’s health care laws
(c) Only (ii) and (iii)
forever.
(d) Only (i) and (iii)
(e) None of these (a) Only (i)
(b) Only (ii) and (i)
32. The US Transportation Security
Administration said on Monday it had
m (c) Only (iii) and (i)
(d) Only (ii) and (iii)
lifted a ban on passengers on Saudi
Arabian Airlines carrying large (e) None of these.
electronics like laptops onboard US-
34. Statement: Pakistan cricketer Kamran
xa
bound flights, the last carrier under the
restrictions. Akmal, on Thursday, accused former
Which of the following can be inferred national coach Waqar Younis of causing
from the given statement? damage to Pakistan cricket. The
(I) US officials had earlier imposed wicketkeeper-batsman slammed Waqar
restrictions on passengers carrying for not being able to guide Pakistan to
te

laptops and other large electronic


success during his two separate tenures
gear.
(II) US officials announced new security as head coach in 2010-2011 and 2014-
requirements for all airlines rather 2016.
than an expansion of the laptop ban Which of the following can be inferred
ce

and have been dropping the from the above statement?


restrictions from airlines as they (i) Pakistan was unable to win any
boosted security. match during the tenure of Waqar.
(III) The US Department of Homeland
Security issued a revised directive (ii) Kamran was kept out of the team by
Waqar.
@

to airlines around the world in


response to security measures. (iii) Kamran wants to apply as a coach
(a) Only (III) for Pakistan team.
(b) Only (II) (a) Only (i) (b) Only (ii)
(c) Only (I) (c) Only (iii) (d) Only (i) and (iii)
(d) All (I),(II) and (III)
(e) None of these
(e) None of these

13 Adda247 Publications For any detail, mail us at


Publications@adda247.com
A Complete Book of Logical Reasoning

35. Statement: England boss Mark Sampson (ii) England will meet France in the
says France coach Olivier Echouafni is finals of the tournament.
"wet behind the ears when it comes to (iii) Mark is more experienced than
tournament football" as the teams Olivier, when it comes to
prepare to meet in the last eight of tournament football.
Women's Euro 2017. (a) Only (i)
Which of the following can be inferred (b) Only (ii)

p
from the above statement? (c) Only (iii)
(i) Betting odds are in favour of (d) Only (i) and (ii)
England to win the tournament. (e) None of these

ou
Statement and Inference : Solutions
1. (d); Statement (i) cannot be inferred as 3. (d); For I: Yes, as according to statement
nothing relating to the TRP is CBSE has issued instruction to

gr
mentioned in the statement. centres regarding encrypted
Statement (ii) cannot be inferred as systems it means CBSE has
the statement is only concerned considered it as safe measures.
with the bias of media executives For II: Yes, as it is mentioned in
statement, encrypted paper are
party which does not necessarily
m
and journalists towards a particular
supposed to be printed by schools
and as it’s a matter of safety of
mean that the ruling party is
paper so its obvious there will be
involved in it. safe and secure room for
xa
The fact that some of the senior installation and its sure that high
media executives and journalists speed internet will be needed to
were willing to take money in decrypt question paper.
return for pushing a political For III: Yes, Question paper in the
agenda clearly points to their form of encryption it means that
te

corrupt intentions. So statement question paper will be in the form


(iii) can be inferred. of zip file and as it is a matter of
safety to prevent it from leak and it
2. (c); Only statement I can be inferred is obvious that it will be password
from the given statement as HRA protected.
ce

allowance increase after


4. (b); For I: Yes, as it is mentioned in
reallocation which clearly indicates
statement that nowadays there are
that both are directly related to
4 major industries in Begusarai,
each other. But II and III cannot be then it is obvious employment will
inferred as we cannot state that increase.
@

strike is the reason behind the rise For II: No, we cannot say that there
in allowance. And also increase in is no case of murder, kidnapping or
profit cannot be a factor to increase extortion, we can only say it has
the allowance which the statement been reduced. But this reduction
also states that the decision has has touch the figure of zero, it
been taken after the reallocation. cannot be inferred.

14 Adda247 Publications For any detail, mail us at


Publications@adda247.com
A Complete Book of Logical Reasoning

For III: Yes, as it is mentioned Govt. has clearly mention the


growth rate of Begusarai is top in concern of Delhi Government. It
Bihar and 5th among all districts in shows their intention.
India, and Begusarai has moved For II: False, it is clear Delhi
from the phase of darkness. It Government is much concerned
clearly indicates that people of about education system, but it
Begusarai are hardworking and cannot be inferred that Govt

p
keen to move forward. schools has been built.
For IV: No, it has been mentioned For III: False, there is a difference
that nowadays Begusarai has 4 between cooperation of State

ou
major industries but we cannot say Government and Central
that top industrialists have desire Government, but we cannot
to have their industry in Begusarai. inferred about monopolies.
Direction (5): Direction (8-9):
5. (a); For I: Yes, it can be inferred from 8. (b); The inference is probably true
the given statements as it is clearly because according to the statement

gr
mentioned punctuality and it is likely to be true that there had
sincerity are one of the key points been black money which was to be
which will surely reviewed. recovered, it cannot be definitely
For II: No, it is clear from the given true because out of the Rs. 15.42-
statements that there will be
m
increment but percentage cannot be
lakh crore, Rs. 15.31 lakh crore has
come back to the banking system.
inferred.
For III: No, as it is mentioned by 9. (a); The inference is definitely true as
manager that hard work and certainty about black money is not
xa
dedication towards work will be known but it is clearly mentioned
applauded. But, Is there some that demonetization inflicted
employee who have done their avoidable pain on farmers, daily
work with complete determination wage labourers and informal
and enthusiasm, it cannot be enterprises used to transact in cash.
te

inferred. We can assume it but it 10. (b); Pollution divests rivers of their
cannot be inferred from it. holiness as it is given that every
6. (b); For I: True, according to Mr. holy river in India is losing its
Thompson, description of holiness due to pollution.
ce

relationship is given in sense of all


11. (e); First one can be directly concluded
three points i.e. Grudge, memories
from the statement. Second one can
and learnings.
For II: False, It is just opposite of be inferred from the statement as
thinking of Mr. Thompson. the main motive of Reliance Jio is
For III: True, Mr. Thompson the enhancement in the field of
@

indirectly wants to say by leaving mobile infrastructure which is there


grudge, one can bring positivity in in the statement in hidden manner
life. 12. (d); Note that there has been a decline
7. (d); For I: True, as we consider in gold imports so the demand of
statement 100% true and in the gold can’t be increase so it is
statement spokesperson of Delhi probably false.

15 Adda247 Publications For any detail, mail us at


Publications@adda247.com
A Complete Book of Logical Reasoning

13. (b); The deuterium—tritium reaction is 19. (e); In the above question we have to
given as example to prove that the find the statement which we can
popular misconception that nuclear drive from the above statement.
fusion does not involve For I-This statement states the
radioactivity is wrong. Hence (b) is ability of ISRO in developing
the correct alternative. (a) and (c) cryogenic engine, but it can’t be
contradict the passage. (d) cannot inferred from the given statement.

p
be inferred from the passage. For II-Not true, because it cannot be
14. (a); From the passage it can be inferred said that this launch will make India
that an antibody cannot most powerful nation .

ou
incapacitate rhinoviruses .so (a) is For III- Not true, because It cannot
the correct answer. (b) cannot be be concluded from the given
inferred from the passage. statement.
For IV- False, because It can’t be
15. (c); As the passage mentions about inferred from the given statement
“anti-intellectual bias”, (c) must be that ISRO has the capability of

gr
the correct alternative. (a) cannot throwing four tonne payload into
be inferred. (b) is not given in the higher orbits as it only states about
passage. (d) cannot be inferred the newly launched Mark III rocket.
from the passage.
20. (d); Here “above and beyond” is used as
16. (d); Mary’s reply suggest that oxygen
and water are also necessary in
m an idom which means doing more
than expectations and
addition to a conductive climate to
requirements. So, (iii) can be clearly
sustain human life. Mary does not
inferred. Whereas (i) is contrary to
talk about the relative importance
xa
Modi’s statement. (ii) Cannot be
of oxygen and water and climatic
inferred as there is nothing
conditions. (a) is inappropriate and
mentioned in the statement to
(b),(c) can’t be inferred from Mary’s
make this deduction.
reply.
(d) Gradual withdrawal of subsidy 21. (b); In the above question we have to
te

is essential for effectively managing find the statement which can be


fiscal deficit in India. inferred from the given statement.
17. (d); Gradual withdrawal of subsidy is Only (III) will be true because this
essential for effectively managing issue requires solid evidences and
ce

fiscal deficit in India. enquiry regarding Pakistan army


officer kidnapping. Both (I) and (II)
18. (e); Clearly the statement was made in are false because we cannot say
context of India’s relationship with that if this issue has a link with
Germany and mean that both Kulbushan Jadhav’s case or this
countries have a mutual interest to
@

issue has some involvement of


serve or help each other. So, both (i) RAW.
and (ii) can not be inferred.
(iii) cannot be inferred as the word 22. (d); In the following question we have
“each other” signifies that India too to find which of the following can
have something to offer to be inferred from the given
Germany. statement.

16 Adda247 Publications For any detail, mail us at


Publications@adda247.com
A Complete Book of Logical Reasoning

For I-This statement describes 25. (a); In the above we have to find which
about the motive of the M-Power of the following can be assumed
programme launched for the from the given statement.
differently abled children so this For I-This statement is true. It
can be inferred from the given describes that Indian border guards
statement. crossed Chinese border which
For II- This statement states about interrupts their normal activities

p
the importance of the programme which might have motivated China
to shut down the Nathu La pass.
and how it will going to help special
For II- This can also be assumed
children. So this can also be

ou
from the given statement. It
inferred from the given statement.
describes China’s take over this
For III-It is not mentioned in the whole issue as it said that India
given statement about weather should not go into Chinese-held
Government is planning to provide areas and should maintain peace
access to school to such children. So along the border.

gr
it cannot be inferred from the given For III-This statement cannot be
statement. assumed from the given statement
because it states about the defence
23. (a); In the above we have to find which
agreement signed between India
of the following can be indirectly
and US on deal regarding drones
stated from the given statement.
m
For I-This statement can be inferred
whereas the given statement
describes China’s stand on border
from the given statement because issue with India.
this initiative has been taken up to
26. (e); For I-This cannot be assumed from
xa
protect Ganga from pollution as it is
given that banks will go ecofriendly. the given statement as it is nowhere
mentioned that India and its
For II-This cannot be inferred from
freedom fighters are proud of cows.
the given statement because it is
For II-This cannot be inferred as it
nowhere mentioned that by the
is given in the above statement that
te

above step the pollution will end


up. Mahatma Gandhi would not support
“Killing in the name of cow” which
24. (d); There is not enough evidence in the presumably means killing of
statement to conclude (I). It is innocent people in the name of cow
ce

mentioned in the statement that the protection, whereas statement (II)


event is a gimmick but is also a
is only concerned with killing of
block buster money maker which
cows.
clearly suggest that such events are
For III-This also cannot be assumed
able to draw people’s attention, so,
from the given statement as it is not
@

(II) can also be inferred. As it is


given that cow is India’s national
given in the statement that Floyd is
animal.
a 49-time undefeated champion
whereas boxing is a new sport for 27. (d); In this question, we have to choose
McGregor, so (III) can also be an option which can be inferred
inferred. from the given statement.

17 Adda247 Publications For any detail, mail us at


Publications@adda247.com
A Complete Book of Logical Reasoning

Statement (i) can be inferred as it is For II-This statement cannot be


given in the statement that IS inferred from the given statement
recruiters have been active in the as it is not mentioned in the given
past in that state. statement.
Statement (ii) can also be inferred For III-This also cannot be inferred
as it is given in the statement that a as no information is given about
de-radicalization drive has been India and Asean’s economic

p
initiated. partnership.
Statement (iii) cannot be inferred as 30. (b); In this question we have to choose
nothing is mentioned about the an option which can be inferred

ou
medium of communication used or from the given statement.
any other investigating agency’s Statement (i) cannot be inferred as
involvement. Operation Pigeon is the Supreme Court only denied
just the name of the de- giving a living entity status to the
radicalization drive. river which does not mean it’s not
serious about the river pollution.

gr
28. (a); In this question we have to choose
an option which can be inferred Statement (ii) can be inferred as it
from the facts given in the was not practical and could lead to
statement. complicated legal situations, even
Statement (i) can definitely be claims against the rivers in cases of
flooding or drowning.
inferred as it is given in the
statement that China is willing to
m Statement (iii) is cannot be inferred
discuss the alternative routes for as nothing of the sort is mentioned
in the statement.
Indian pilgrims and there was an
increase in tension between the two 31. (d); In this question, we have to choose
xa
countries. an option which can be inferred
Statement (ii) cannot be inferred as from the given statement.
it is not related to the statement. Option (i) can be inferred as it given
Statement (iii) cannot be inferred as in the statement the Supreme Court
it is given in the statement that has allowed these institution to
te

China is willing to discuss carry on with the admission


alternative routes. process which implies that it must
have observed some faults and
29. (a); In the above question we have to stayed the admission process
find which statement gives the
ce

earlier.
indirect conclusion of the given Option (ii) cannot be inferred as it
statement. is nowhere mentioned in the
For I-This statement can be inferred statement that Supreme Court
from the given statement because it deliberately interfered with the
describes that Republic day 2018 admission process or had any
@

will witness so many leaders for the intention to cause any delays.
first time which is also mentioned Option (iii) can be inferred as lack
in the given statement that for the of sufficient evidence must be the
first time India will invite heads of only reason why Supreme Court has
10 Asean nations for Republic Day given a go ahead to the admission
2018 celebrations. process which it had stayed earlier.

18 Adda247 Publications For any detail, mail us at


Publications@adda247.com
A Complete Book of Logical Reasoning

32. (c); For I- This statement can be the law can indeed be scraped by
inferred from the given statement Republicans in the near future.
because it is given that US Security
34. (e); In this question, we have to choose
Administration had lifted a ban on
an option which can be inferred
passengers carrying large
from the facts given in the
electronics like laptops so it is clear
statement.
that US officials had earlier imposed
Option (i) cannot be assumed from

p
restrictions on passengers carrying
the statement as we don’t know the
laptops.
number of matches played and won
For II- This statement cannot be
by Pakistan.

ou
inferred from the given statement Option (ii) is also not a correct
because nothing is mentioned in the choice as we cannot assume that
given statement about the new the personal indifferences were the
security requirements announced cause for the criticism by Waqar.
by US officials. Option (iii) is also not a correct

gr
For III- This statement also cannot choice as nothing of the sort has
be inferred from the given been mentioned in the statement
statement because it is nowhere and it’s a far fledged assumption.
mentioned about the revised 35. (c); In this question, we have to choose
directive to airlines issued by US an option which can be inferred
Department of Homeland Security.
m from the above statement.
33. (b); In this question, we have to choose Option (i) cannot be inferred from
an option which can be inferred the statement, as difference
from the given statement. between the experiences of two
xa
Option (i) can be inferred as it is coaches does not necessarily
given in the statement that translate into likelihood of a
Republicans were trying to scrap particular team to win.
Obama’s health care laws. Option (ii) cannot be inferred, as it
Option (ii) can also be inferred as it is given in the statement that
England will meet France in the last
te

is given in the statement that the


efforts to gut Obama's health care eight which means quarterfinals.
law came to an abrupt and may be Option (iii) can be inferred as it is
temporary halt which means the given in the statement that Olivier
law cannot be replace immediately. Echouafni is "wet behind the ears
ce

Option (iii) cannot be inferred as when it comes to tournament


the word temporary in the football" which means the French
statement suggests that the halt coach does not have enough
may be not be permanent and that experience behind him.
@

19 Adda247 Publications For any detail, mail us at


Publications@adda247.com
A Complete Book of Logical Reasoning

p
ou
gr
m
xa
te
ce
@

1 Adda247 Publications For any detail, mail us at


Publications@adda247.com
A Complete Book of Logical Reasoning

06 Strength of Argument
Arguments is a logical statement that uses to prove and make strong to given statement. Or we
can say it is a season given in Proof or rebuttal for a given statement.

p
→ Always semesters Argument never analyse as wrong because in is a part of thinking. It may
be in favour or against in given data. Argument further classified in two types which are:

ou
(i) Strong Argument
(ii) weak Argument
(1) Strong Argument which in favour or support to the statement are called strong
argument.
(2) Weak argument which weakens or against to the statements.

gr
Strong Argument consists some facts which are given below:
(i) Universal facts
(ii) Analysed and experience based date
(iii) In favour people and country
m
(iv) Always Authrised by any govt. bodies like government, supreme covert and constitution.
(v) Logical predictions. Weak argument consists some facts which are given below:
(a) Ambigons date (unclear data) and Restatement.
xa
(b) option and comparison, irrelevant data.
(c) Against government and country.
Above all these Hints you have to follow some other points also to understand or easily
find out strong argument.
te

(i) The argument should not be just an opinion. It must answer one of the questions, why,
How, when, of the statements otherwise it is not an argument.
(ii) An argument also depends on the language used because by changing a few words we can
make a weak argument forceful.
ce

(iii) The argument must be an argument and not with what somebody or news paper says.
(iv) If some one is quoted in support of the statement it can’t be a forceful argument.
(v) The argument is based on an assumption and hence it cannot be forceful.
(vi) If the argument is in form of a simple sentence Lacking any facts any facts or established
@

notions is not ambignons is consider as weak argument.


(vii) If the argument the superfluous and only glances at the theme without making an indepth
analysis of the facts or information, the argument will be weak.
(vii) If any argument contain only, the best, will be, until, unless and definitely such word make
it weak.

2 Adda247 Publications For any detail, mail us at


Publications@adda247.com
A Complete Book of Logical Reasoning

Points to Remember
(1) Strength of argument are classified in two types:
(a) Strong Argument (b) Weak Argument

p
(2) Strong Argument are always in favour of the statements.
(3) Weak Argument are always in against of the statements.
(4) Remember that Universal facts are Strong Argument.

ou
(5) Some words like only, until, unless and definitely make argument weak.

Exercise

gr
1. In the following question a statement is (a) Only statement (I) strengthen but
given followed by three arguments statement (II) and (III) is the
numbered as I, II and III. You have to neutral statement.
read all three arguments and decide (b) Both statement (I) and statement
which of them strengthens or weakens
the statement.
m (III) strengthen the given statement
but statement (II) is neutral
Statement: Among the governments, statement.
there is a strong policy emphasis on
(c) Both statement (I) and statement
improving living conditions in urban
(II) are neutral statements and
xa
areas as more and more people are
looking to migrate to semi-urban or statement (III) strengthen the
rural areas. People are willing to statement.
compromise on their standard of living (d) Both statement (I) and (II) weakens
in order to live in a healthier/pollution but statement (III) strengthen the
free environment. given statement.
te

(I) Cities are economically vibrant (e) All statement (I), (III) and
spaces around the world and statement (II) weaken the given
drawing a large number of rural statement.
migrants looking for better
2. Today’s children are different from the
ce

prospects. This is a sustained trend,


particularly in developing countries children of previous generations. They
now a day. are more interested in playing video
(II) A fresh look at urban governance is games or sitting hours on the play
necessary as migration from rural stations than in playing outdoor games.
areas picks up pace. They are more technology oriented.
@

(III) The rate of migration has shown a These things are depriving the child
consecutive decline in the last three from developing their personality. They
Censuses. It was possible that this lack the stamina and strength of the
slow-down signalled the worsening previous generations.
conditions in urban slums, where Which of the following statement
most of the rural immigrants were weakens the given statement?
concentrated.
3 Adda247 Publications For any detail, mail us at
Publications@adda247.com
A Complete Book of Logical Reasoning

(I) The children of today are mentally 4. In the following question a statement is
active as this has made children given followed by some arguments. You
much more organized and mature have to read all the argument and decide
than in previous times. which of them weakens the statement.
(II) These gadgets are addicting. They The NITI Aayog released the results of a
have made the children so much study warning that now its “worst”
addicted to PC and PS that they do water crisis in history of India and that

p
not enjoy the outdoor games the demand for potable water will
anymore. outstrip supply by 2030, if steps are not
(III) The children of present time are not taken.

ou
physically active. These children are Which of the following most weakens the
not being a team player anymore, above statement?
they grow up to become shy and (a) Currently 600 million Indians face
introvert instead of being bold and high to extreme water shortage and
extrovert. about two lakh people die every
(a) Only I (b) Both III and I year due to inadequate access to

gr
(c) Only II (d) Both II and III potable water.
(e) All I, II and III (b) People may face water scarcity and
3. A survey conducted recently shows that the country will suffer a loss of up
religious people are more financially to 6 per cent of the GDP.
successful than atheists. This proves that (c) The country's nearly 70 per cent of
following religious practices or having
m water is contaminated, which puts
religious beliefs is not only important for India at the 120th position among
your spiritual health but also beneficial 122 countries in the water quality
in maintaining a healthy bank account. index.
xa
Which of the following, if true, would (d) The ranks of all states in the index
most seriously weaken the above on the composite water
conclusion? management, comprising 9 broad
(a) The survey does not tell us if those sectors with 28 different indicators
people became financially covering various aspects of ground
successful after they started
te

water, restoration of water bodies,


practicing religion or they started irrigation, farm practices, drinking
practicing religion after they water, policy and governance.
became successful. (e) The condition of availability of
(b) Some people embrace religion and water currently in India is better
ce

spirituality mainly because they than the water crisis which India
believe that somehow being had faced in 1960.
religious or spiritual can help them
achieve financial success. Direction (5): In the following question a
(c) Most of the poor people are too statement is given followed by two more
busy thinking about earning their arguments numbered as I and II. You have to
@

daily livelihood. read both the arguments and decide which of


(d) Percentage of atheists among the them strength which of them strengthens or
total population is highest in most weakens the statement.
of the developed countries. 5. Statement- As an emerging economic
(e) Spiritual and religious people lead a superpower, it is quite tragic that more
more disciplined life. than one in three children in India are

4 Adda247 Publications For any detail, mail us at


Publications@adda247.com
A Complete Book of Logical Reasoning

stunted. Stunting is associated with an Directions (7): Read the following


underdeveloped brain, with long-lasting information carefully and answer the
harmful consequences, and increased questions which follow.
risks of nutrition-related chronic In a recent study of responses to visual
diseases, such as diabetes, hypertension, images, researchers found that women most
and obesity in future. Thus, stunting can frequently gave the rating ‘most attractive’ to
be classified by itself as an illness as it images of male faces that were more feminine

p
leads to negatively affecting the present in contour, and rated more masculine faces,
and future of the child. on average, ‘less attractive’. The researchers
(I) Higher economic growth and concluded that modern women prefer men

ou
increased spending on nutrition who are less obviously masculine in their
access programme have not facial features.
resulted in proportionate decrease
7. The conclusion would be most severely
in child malnutrition or stunting.
weakened if which of the following were
(II) Multiple researchers have proven true?
the link between illness and

gr
(a) In a recent study, it is found that
malnutrition/ stunting in children. percentage of masculine faces men
(a) Both I and II strengthens in relationship is more in
(b) I strengthen while II weakens comparison to feminine faces.
(c) II strengthens while I weaken (b) The visual images were computer
(d) Statement I strengthen while
Statement II is neutral.
m generated composites
photographs and not pictures of
of

(e) Both I and II weakens. actual men.


(c) The rating scale was a ten point
6. Which of the following arguments is scale with most attractive scoring
xa
strong in respect of the given statement? 1-2 and least attractive scoring 8-
Statement: An apple a day, keeps a 10.
doctor away-Instruction given by Health (d) Most popular male actors have the
ministry of India. features that the study allocated to
Arguments: the more masculine category.
te

I. Every individual should consume (e) The faces with the more masculine
an apple daily to keep himself fit features were all significantly older
than those with the feminine
physically and mentally.
features.
II. Real juice of apple flavor should be
ce

banned as it is contaminated with 8. Secondary school graduates in Japan


harmful chemicals. score significantly higher on tests of
III. To consume an apple daily by every science and mathematics than do
individual is not possible as it is students at the same level in the United
States. Some educational reformers in
costly in summer season.
the United States attribute this
@

IV. An individual can be healthy by difference to the more rigid and rigorous
taking green vegetables and Japanese secondary school programme,
alternative cheap fruits. which emphasizes required courses, long
(a) I and III (b) I, II and III hours of study and homework, and
(c) I, III and IV (d)All I, II, III and IV memorization to a far greater degree
(e) None of these than do American schools.

5 Adda247 Publications For any detail, mail us at


Publications@adda247.com
A Complete Book of Logical Reasoning

Which of the following, if true, would 10. Statement: Some of the recent
most seriously weaken the conclusion researches points to the addictive nature
drawn by the above passage? of grains. Neurologist and author of
(a) The Japanese elementary school Grain Brain, Dr. David Perlmutter,
programme is far less rigid and discussed the addictive effects grains
structured than the elementary have on our brain in his book. While the
school programmes in most United book doesn’t directly say “carbs are

p
States schools. addictive,” it does say that when you eat
(b) Many Japanese parents and carbs you will crave more carbs. And
educators decry the Japanese absolutely, wheat for many people feels
educational system, saying that it

ou
like something they could never give up.
stifles independent thinking on the
Which of the following most strengthens
part of students.
the above statement?
(c) United States schools, in which
creativity and flexible student (a) Emotional eating doesn’t fix your
schedules are emphasized, usually emotional problems, in fact, it
produce students with science and usually makes them worse.

gr
maths scores equal to those of the (b) Grains are not meant for human
best Japanese students. consumption, a close look at our
(d) On an average, Japanese students digestive autonomy suggest that we
score lower than United States are “frugivores” or “fruit eaters”.
students on tests of logical thinking, (c) Many birds have a distinct organ,
language arts, and communications
m called the gizzard, which helps
skills. grind grains and seeds. Grains are
(e) A higher percentage of American meant to be bird food for parrots,
students go for higher education pigeons, and doves.
xa
than in Japan. (d) Our digestive tract cannot handle
9. An animal welfare agency has claimed the mixture of starch and sugar
that the carelessness by the zoo found in most recipes of cereals,
authorities in city X to be responsible for breads, bagels, etc. And as a
the death of four dolphins within a span consequence, most cooked grain we
te

of a week. eat tends to ferment inside of us


Which of the following supports the producing gas and alcohol.
claims made by the animal welfare (e) Most people could easily go for
agencies? months without eating certain
(a) There has been a dispute regarding fruits or vegetables and it wouldn’t
ce

sharing of water between city X and be a problem for them but that’s not
city Y since the past few months. the case with processed grains.
(b) Some organisms were found to be
infected with an unknown virus, 11. In the following question a statement is
which results spread of virus given followed by some arguments. You
have to read all the arguments and
@

among other organism in zoo.


(c) Some of the visitors have reported decide which of them strengthen the
seeing a group of zoo officials statement.
taking selfies with zebra. Out of all the major quadrennial
(d) A huge pile of garbage was international sporting events, the
discovered near the zoo premises. Football World Cup is the one that gives
(e) Both (d) and (c) Indians the purest pleasure. The Cricket

6 Adda247 Publications For any detail, mail us at


Publications@adda247.com
A Complete Book of Logical Reasoning

World Cup douses Indians in a broth of factors causing this reversal in progress,” said
tension. The Olympics continue to visit the report.
humiliation on the idea of India as an Which of the following statement strengthen
athletic nation. The FIFA jamboree, the give statement?
though, is devoid of any such freight. (a) The increasing impact of extreme events
Which of the following most strengthen related to a changing climate, economic
the above statement? losses attributed to disasters were

p
(a) Cricket has always been the first estimated at over $300 billion.
love of Indians. Indians breath (b) More than nine out of 10 people living in
Cricket and enjoy every bit of it. urban areas around the world are

ou
(b) Cricket is a religion here and it breathing polluted air, with southern
makes a lot of us happy. People Asia scoring the worst in this area.
need a Sachin in football. Indians (a) Only I
want to talk about the popularity of (b) Only II
football players in the national side (c) All of the above
as well. (d)Either I or II

gr
(c) With the people expressing their (e) None of these
support, this surely earmarks a 13. A statement by motivational speaker-
turning point for the rise of football Love is not an attachment, not a feeling,
in India and exploring the potential not a lust. It is not an emotion just to
of our youth and give impetus to make yourself cherish. It’s an admiration
the rising status of this sport.
m between two soul that connects them
(d) So far, Hockey is the superior sport. direct from heart to heart in which one
Its players took pride in 'their' way partner remains pleased in his/her
of playing - of their touch and partner’s delight.
xa
superb body language. Which of the following substantiates the
(e) The performance of Indian Women statement given by motivational
who reach the Olympics are thus speaker?
likely to already have exceptional I. A well renowned youth leader-
skills, relative to men in India. Love is a desire, passion, addiction,
excitement and intensity.
te

Therefore, India should field more


make an effort to field more women II. It’s just an emotional support
and give them better access to between two individuals.
trainers and facilities. III. It is a determination in which one
also become ready to sacrifice
ce

Direction (12): Study the following his/her joy to be in blessing of


information which is followed by some other.
statement read carefully and answer the (a) Only I and III (b) Only II and III
question below- (c) Only II (d) Only III
The number of hungry people in the world (e) All of the above
@

has risen for the first time in more than a Direction (14-15):
decade, according to a United Nations report There are some criteria which will be
released on Wednesday. After a prolonged considered during appraisal of employee-
decline, world hunger appears to be on the Statement by manager of a company.
rise again. Conflict, drought and disasters Punctuality and sincerity are one of the key
linked to climate change are among the key points which will surely reviewed.

7 Adda247 Publications For any detail, mail us at


Publications@adda247.com
A Complete Book of Logical Reasoning

Hard work and dedication towards work will 16. “Dowry” which is illegal in India but it is
be applauded. entertained very pleasantly by the family
Just to quantify an amount in the salary members and grooms itself. They take it
package of employee with no reason will not as their birth right. There is a mindset in
be entertained. society that women have to take care of
kitchen, babies and home and men have
14. Which of the following undermines the to see their office only. But the bitter

p
statement given by Manager? truth is that a woman gives many
I. Increment of 15% has been done sacrifice in comparison to men in
for all the employees whose 1 year persuasion of society.

ou
is completed. Which of the following is not in line
II. Performance chart has been according to above statement?
prepared by leaders of different I. There should be awareness
department based on their program about dowry which should
proficiency. be regulated in the society on
III. Relaxation time for late coming of timely basis.

gr
240 minutes in a year i.e. of 10 II. There should be law of death
minutes twice in a month is given to penalty for rape cases which is a
all employees. must do step in India.
(a) Only II (b) Both II and III III. It is also believed that women
should follow the customs that are
(c) Both I and III
(e) None of these
(d) Only I
m made by the society for better
civilization.
15. Many employees of a company whose (a) Only III (b) Both I and III
basic salary is 20k starts playing IPL (c) Both II and III (d) Only II
xa
game on Dreams11 in office premises. (e) None of these
Five employees have won 5k each as a 17. Statement: State X has increased his
prize money in a single day and its craze demand once again for special category
is increasing among other office status to state after demand of State Y
employees. But it is just like betting and has been rejected by Central
te

starting phase of compulsion. Government as State X has claimed,


Which of the following can be the fallout there has been a great loss to economy
of this activity? after division of state. And As a result of
I. There may be adverse impact on this state Y government party has break
ce

working in office premises. alliance with central Government party.


II. Income from Dream11 will boost What may be the fallout after breaking of
net earning up to 25k, which will be alliance of State Y government party
supplement for office employees. with Central government party?
III. Craving to earn money in short time (I) Central government will fall as
magic number will get decreased.
@

may give them wrong direction, and


its impact may be reflected to their (II) State X demand will be like extra
political pressure on central
earning.
government.
(a) Only III (b) Both I and III
(III) Government of State Y may go on
(c) Both II and III (d) Only II
protest at different venues and may
(e) None of these
call state bandh.

8 Adda247 Publications For any detail, mail us at


Publications@adda247.com
A Complete Book of Logical Reasoning

(a) Only II and III (b) Only III near a police station which was at 100 m
(c) Only I and II (d) Only II away from the place of incident.
(e) None of these Spokesperson of Delhi Government has
stated that Delhi police is under Central
Direction (18): Statement: Amazon’s
Government, so we face hindrance to
country manager for India has proposed a
achieve social security.
radical idea: “Log off; get a life.” In an email to
Which of the following abrogates the
his team earlier this month, Amit Agarwal

p
concern of spokesperson of Delhi
counseled colleagues to stop responding to
Government?
emails or work calls between 6 p.m. and 8
(I) “There was a heated argument
a.m. in the interest of “work-life harmony.” He

ou
between Delhi CM and Delhi Police
also talked about the importance of work
Chief and a Central minister 2 days
discipline and how to draw the line. The
ago.” – A Report.
leaked note has broken through the sleep-
(II) PM himself bothers about Delhi
deprived haze in the technology hub of
security and in his recent speech he
Bangalore and is a topic of heated discussions
told, will take all necessary steps to
on social networks and WhatsApp chat

gr
maintain law and order in state.
groups.
(III) Central government has decided to
18. Which of the following abrogates the give a special package to Delhi for
radical idea of Mr. Agarawal? the safety of women and to
(I) Psychologists, sleep laboratories maintain law and order.
and fertility clinics have raised
m (a) Only II (b) Only I and III
concerns about the mental and (c) Only II and III (d) Only III
physical toll wrought by the frenetic (e) None of these
work schedule.
20. In a live program of Republic TV from
xa
(II) Employees of many MNC
Jantar Mantar in which spokesperson of
companies are working late night
BJP, Congress, AAP and BSP was present
and they are getting a handsom
in which Mr. Arnab a chief reporter
salary package.
stated-
(III) Insomnia, depression and suicidal
“Campaign for 2019 Lok Sabha election
tendencies are rampant symptoms,
te

has been started by all the parties.


said Dr. S. Kalyanasundaram, a well-
Allegation, denouncement and criticizing
known psychiatrist who sees many
to each other is on peak. So, nation wants
technology workers in his thriving
to know, who will be the right PM of
south Bangalore practice.
ce

India in 2019 who can solve Kashmir


(a) Only II and III (b) Only I and III
issue, who can abolish women security
(c) Only I and II (d) Only II
issue, who can take country to the next
(e) None of these
level in prospect of economic growth”
19. Statement: Delhi chief minister Arvind Comments by different spokesperson: -
Kejriwal who is much concerned for the (I) Mr. Patra, a BJP spokesperson- In 4
@

social security, education and health years, BJP govt. has given 2 crores
system has visited Government hospital employment and curb corruption
for the inspection of medicines and through the demonetization and
availability of doctors. At the same time GST bills. We have signed many
when Mr. Kejriwal was on inspection, a deals to empower our country.
dalit boy has been beaten by some goons

9 Adda247 Publications For any detail, mail us at


Publications@adda247.com
A Complete Book of Logical Reasoning

(II) Mr. Sibal, a Congress spokesperson- Direction (22): Study the following passage
BJP has not clear his stand nor carefully to answer the given questions.
given any clarification on Rafel It was a step which was long overdue. The
issue. BJP govt. is against Muslim Union cabinet has cleared amendments to the
community. Public Premises Act to ensure that our
(III) Mr. Sindhe, a BSP spokesperson- parliamentarians and bureaucrats don’t use
GDP has fallen in comparison to their clout to flout accommodation norms.

p
previous govt. session. Our soldiers The amendments will ensure that ministers,
are killed daily on the border but Members of Parliament and bureaucrats do
not continue to overstay their welcome in
there is no intention of govt. to

ou
government bungalows even after their term
solve Kashmir issue. is over. They’ll also have to pay steep
Which of the following comments penalties. Anybody overstaying beyond five
according to the statement can be months, for instance, will have to pay a fine of
most effective for Indian people to Rs 10 lakh.
select right PM of India?
22. Which of the following negates the

gr
(a) Only I and III (b) Only III possible fallout of Public Premises Act?
(c) Only I and II (d) Only II and III (I) Fine of Rs. 10 lakh is for anybody
(e) None of these who overstay beyond five months.
(II) A meeting will be called by some
21. Statement: Two years back, Mr. prime
political parties to appeal for some
minister had started “Swachh Bharat
Abhiyan” and to a great extent he has
m changes in this new public premises
act.
succeed to keep India clean. But the (III) This amendment is for the all
point is: Will India be a clean country, ministers, all parliamentarians who
xa
without improving the thinking of are of ruling party or opposition
people. We are in 2018 and even today party.
also people judge girls on the basis of (a) Only I and III (b) Only I
length of their dresses. (c) Only II (d) Only II and III
Which of the following refutes the (e) All of these
te

concern which is raised in above Direction (23-24): Pollution control can no


statement? longer be viewed as a national problem to be
(I) Keeping women safety as top addressed by individual countries on the
priority, toilets are made in every basis of national sovereignty. As the
international effects of the Chernobyl nuclear
ce

home of 550 villages of India under


accident make clear, pollutants do not respect
Swachh Bharat Abhiyan.
political boundaries; thus every nation has a
(II) Education of Girls has been made legitimate stake in the environmental
free in all Government schools of practices and policies of its neighbours.
India.
(III) Recently, Five Southern states of 23. Which of the following would be the
@

most logical continuation of the


India has launched a program to
argument above?
improve mental thinking of people. (a) So, growth of the nuclear power
(a) Only II and III (b) Only I and II industry should be halted until
(c) Only III (d) Only I and III more rigorous safety procedures
(e) Only I have been developed.

10 Adda247 Publications For any detail, mail us at


Publications@adda247.com
A Complete Book of Logical Reasoning

(b) Hence, attempts made by one (a) There has been a slowdown in the
nation to impose its environmental rate of increase in world demand
policies on another should be for coal over the last 5 years from
resisted-if necessary, by force. 10% to 5%.
(c) Consequently, issues of pollution (b) It has been known for many years
should be handled by an that there are vast stocks of coal
international commission with the under Antarctica which are yet to
be economically exploited.

p
authority to set policies for all
(c) Oil is being used increasingly in
nations. place of Coal for many industrial
(d) Thus, every nation should pledge and domestic uses.

ou
itself to environmental policies that (d) As Coal resources are depleted
will minimize the danger to its more marginal supplies, which are
neighbours. more costly to produce and less
(e) As a result, only the ultimate efficient in use, are being mined.
emergence of a sovereign world (e) None of the above
government will resolve today’s
26. A government survey released today

gr
most pressing environmental shows that 80 % of the people who fly
dilemmas. are satisfied with the service they
24. Which of the following, if true, most receive from the airlines in this country
strongly supports the view expressed in .Three interviewers stood outside a
the passage? major airport and asked people leaving
(a) Acid rain from factories in the
m the terminal, “Do you have any
complaints about the flight you just got
midwestern United States pollutes
off?” Only 20 percent responded “yes!”
lakes in Canada.
Which of the following, if true, would
(b) Soviet leaders refused western most undermine the conclusion of the
xa
reporters access to safety records argument above?
after the Chernobyl accident. (a) Sixty percent of the people coming
(c) Neighbouring states within the out of the airline terminal were not
United States are often unable to people who had just gotten off a
agree on joint pollution control flight.
efforts. (b) One percent of the people
te

(d) Existing international bodies have approached by the interviewers


no authority to impose pollution refused to respond to their
control regulations on member- inquiries.
nations. (c) The interviewers began their
ce

(e) Fishers form Japanese fleets have inquiry just after passengers were
increasingly depleted fish supplies discharged from a flight that was 40
in United States’ territorial waters. minutes late.
(d) The interviewers were able to
25. Unless new reserves are found soon, the speak to only 70 percent of the
world’s supply of coal is being depleted people leaving the terminal, but
@

in such a way that with demand those people were selected at


continuing to grow at present rates, random.
reserves will be exhausted by the year (e) For six months following the day of
2050. the interviews, no official
Which of the following, If true, would complaints were filled by any
most weaken the above arrangement? passenger with the Federal agency
that regulated the airlines.
11 Adda247 Publications For any detail, mail us at
Publications@adda247.com
A Complete Book of Logical Reasoning

27. Landmark preservation laws unfairly (a) The untreated birds were confused
impinge on the freedom of owners to by the erratic flight patterns of the
develop their own property as they see surgically treated birds and failed
fit. In some cases, owners of hotels and to migrate successfully.
office buildings designated as landmarks (b) The surgically treated birds were
have been forbidden to make changes in able to follow their usual flight
the original facades or interiors, even patterns successfully by day, but

p
though they reasonably believe that the not by night.
changes would enhance the structures (c) The surgically treated birds were
and make them more valuable. able to migrate about as accurately

ou
Which of the following statements, if true, as the untreated birds.
seriously weakens the author’s argument? (d) The surgically treated birds were
(a) Altering the appearance of a able to migrate successfully only
historic structure sometimes does when closely following a group of
not enhance its beauty or value. untreated birds.
(b) In traditional legal doctrine,
(e) In addition to being impaired in

gr
ownership of a property implies the
their migration ability, the
right to alter it at will.
surgically treated birds suffered a
(c) Only buildings over 75 years old are
variety of other disabling effects.
normally affected by landmark
preservation laws. Direction (29–30): Read the questions
(d) Landmark designations must be
m carefully and answer the questions which
approved by a local regulatory body follow:
before taking effect.
(e) Historic buildings represent a 29. The earth’s resources are being depleted
xa
cultural heritage which the too much fast. To correct this, the United
community has a legitimate stake in States must reduce its resource
preserving. consumption at present levels for many
years to come.
28. Some scientists believe that, in certain Which of the following, if true, would
species of birds, actual particles of metal most strengthen the argument above?
te

within the brain react to the Earth’s (a) The United States has been
magnetic field in the same way as the conserving resources for several
needle in a compass. It is this mechanism years.
that is thought to underlie the birds’ (b) Other countries have agreed to hold
ce

amazing ability to navigate accurately


their resource consumption at
over distances of thousands of miles by
present levels.
day and night during migration. To test
(c) Other countries need economic
this theory, researchers surgically
development more than the United
removed the metal particles from the
States does.
heads of some birds and then released
@

them, along with a number of untreated (d) New resource deposits are
birds, at the usual time and place of their constantly being discovered.
annual winter migration. (e) The United States consumes one-
Which of the following results would most third of all resources used in the
seriously weaken the theory being tested? world.

12 Adda247 Publications For any detail, mail us at


Publications@adda247.com
A Complete Book of Logical Reasoning

30. Red is a colour which has powerful A. Gold prices are determined by
effects on human beings as well as international market prices.
animals. A group of psychologists carried B. Prices of gold in local market are on a
out an experiment which confirms the continuous decline
subconscious effects of this colour on C. Soaring price of gold is not a positive
human behaviour. They provided sign for an economy.
selected sports teams at school and D. The US economy is very speculative.

p
college level with either red or blue E. There are strong signs of decline in the
shorts and recorded the outcome of the price as it was in the case of petroleum
games. The teams wearing red won in a and crude oil five years ago. Later, the

ou
disproportionate number of matches. market did decline.
The psychologists suggested that either
the teams wearing red subconsciously 31. Which of the following statements (A),
felt themselves more powerful, or that (B), (C), (D) and (E) would weaken the
the non-red teams were subconsciously argument of the author?
intimidated by the red colour. (a) Only A (b) Only B
(c) Only D (d) Both B and C

gr
Which of the following, if true, would
most weaken the psychologists’ (e) Both A and E
suggestion? 32. Which of the following, if true, would
(a) Each team wore red in some strengthen the argument of the author?
matches and blue in others. (a) Only C (b) Only E
(b) The colour blue has
subconscious effect of making
m
the (c) Both A and C (d) Both B and C
(e) None of these
human beings less competitive.
(c) The effect was only observed if all Directions (33-34): Each question given
the team members wore white below consists of a statement, followed by
xa
shirts. three or four arguments numbered I, II, III
(d) Red signifies danger in some and IV. You have to decide which of the
cultures whereas white signifies arguments is/are 'strong' (arguments) and
happiness in others. which is/are 'weak' (arguments) and
(e) In a subsequent study, teams with accordingly choose your answer from the
te

all-red outfits were more likely to alternatives given below each question.
report that they thought they would
win no matter what colour the 33. Statement: A record of sort was created
opponents wore. after the UP government, which
advertised for 368 posts of peon in
ce

Directions (31–32): Read the following secretariat, got 23 lakh responses. The
paragraph carefully and answer the questions minimum qualification for the post of
that follow. peon is that the applicant should be a
After rising for several months, gold prices class five pass out and know how to ride
are likely to see a gradual decline in the short a bicycle. Grabbing the opportunity,
@

term on the back of strong global cues. With opposition parties in unison attacked the
the US economy showing signs of recovery, SP-led state government over its
domestic gold prices are expected to settle at promise of providing employment to
under Rs. 30000 per 10 grams by March, youths in the state. Of the total 368
although the decline may not be as steep as posts, 268 are for general category and
international prices. the rest are for SC, ST and other reserved

13 Adda247 Publications For any detail, mail us at


Publications@adda247.com
A Complete Book of Logical Reasoning

categories in the age group of 18 to 40 II. Yes, Illiterate people are less likely
years. "When we categorized the to make politically wiser decisions
applications viz. a qualification, we were of voting for a right candidate or
surprised to find that 255 doctorates party.
have applied for the job”, a senior III. No, Voting is the constitutional right
secretariat administration official said. of every citizen.
Should people with educational (a) None is strong

p
qualification higher than the optimum (b) Only I and II are strong
requirements be debarred from seeking (c) Only III is strong
jobs? (d) Only II and III are strong

ou
Arguments: (e) All are strong
I. No, It will further aggravate the
35. Statement: In a recent report, the gross
problem of educated
enrolment ratios at the primary level
unemployment.
that is, the number of children enrolled
II. Yes, It creates complexes among
in classes one to five as a proportion of
employees and affects the work
all children aged 6 to 10, were shown to

gr
adversely.
be very high for most states; in many
III. No, This goes against the basic
cases they were above 100 percent!
rights of the individuals.
These figures are not worth anything,
IV. Yes, This will increase productivity.
since they are based on the official
(a) Only I and III are strong
(b) All are strong
m enrolment data compiled from school
records. They might as well stand for
(c) Only II and IV are strong
‘gross exaggeration ratios’.
(d) Only III is strong
Which one of the following options best
(e) None of these
supports the claim that the ratios are
xa
34. Statement: Sociologists and political exaggerated?
observers are of the opinion that making (a) The definition of gross enrolment
education qualification mandatory for ratio does not exclude, in its
election candidates would be numerator, children below 6 years
discriminatory since that would or above 10 years enrolled in
te

naturally deprive citizens of India the classes one to five.


right to represent their people. (b) A school attendance study found
Institutional qualification, as they say, is that many children enrolled in the
different from political acumen and school records were not meeting a
ce

wisdom. In a country like India where minimum attendance requirement


about a quarter of 815 million eligible of 80 percent.
voters are illiterate, it would not be (c) A study estimated that close to 22
prudent to deny a politically aware and percent of children enrolled in the
socially conscious individual the election class were below 6 years of age and
ticket just because he/she doesn’t have a still to start going to school.
@

minimum qualification. (d) Demographic surveys show shifts in


Should "literacy" be the minimum the population profile which
criterion for becoming a voter in India? indicates that the number of
Arguments: children in the age group 6 to 10
I. No, Mere literacy is no guarantee of years is declining.
political maturity of an individual. (e) None of these

14 Adda247 Publications For any detail, mail us at


Publications@adda247.com
A Complete Book of Logical Reasoning

36. Prompted by a proposal to convert a (e) Workers laid off by Midas in


shipyard into a complex of Reverse Ltd. have been hired by a
condominiums with a full-service marina competitor who is taking an
and boat repair centre and by concern increasing share of the market.
about the proposal from local residents,
38. While Governor Verdant has been in
baymen, and environmentalists, the
office, the state’s budget has increased
town is considering a one-year building
by an average of 6 per cent each year.

p
moratorium for the waterfront area.
While the previous governor was in
Which of the following, if true, would
office, the state’s budget increased by an
most seriously weaken the proposal?
average of 11.5 per cent each year.

ou
(a) Condominiums would sell for
Obviously, the austere budgets during
$350,000 each.
Governor Verdant’s term have caused
(b) There is large demand for boat
the slowdown in the growth in state
repair services.
spending.
(c) A growing population results in the
Which of the following, if true, would
closure of shellfish.
most seriously weaken the conclusion

gr
(d) There are already 1200 moorings
drawn above?
on the waterfront.
(a) The rate of inflation in the state
(e) The shipyard may be sold for
averaged 10 per cent each year
another commercial use.
during the previous governor’s
m
37. Before the arrival of Joe, a new partner,
sales output in Bill’s company, Midas in
term in office and 3 per cent each
year during Verdant’s term.
Revere Ltd., had been rising by 10% per (b) Both federal and state income tax
year on average. Innovations by Joe rates have been lowered
included computerization of technical considerably during Verdant’s term
xa
processes and reductions in the work in office.
force, but annual sales output has only (c) In each year of Verdant’s term in
risen by 5% per year. It appears that office, the state’s budget has shown
Joe’s innovations have caused the some increase in spending over the
reduction in the annual growth rate. previous year.
te

Which of the following, if true, would (d) During Verdant’s term in office, the
most seriously weaken the conclusions state has either discontinued or
above? begun to charge private citizens for
(a) The investment in new machinery numerous services that the state
ce

entails a provision for depreciation offered free to citizens during the


of the cost of the fixed assets, which previous governor’s term.
causes a reduction in profit. (e) During the previous governor’s
(b) Midas in Reverse Ltd. Does not base term in office, the state introduced
increase in the selling price of its several so called "austerity"
products with costs. budgets intended to reduce the
@

(c) Joe’s innovations were intended as growth in state spending.


long-term investment and not made
39. The use of petroleum products in the
for short-term profit growth.
manufacture of plastics should be
(d) General demand for the product
regulated and limited by law. Our
manufactured by the company has
country’s need for petroleum for energy
declined.

15 Adda247 Publications For any detail, mail us at


Publications@adda247.com
A Complete Book of Logical Reasoning

production is more vital than our need Arguments:


for plastics, and our growing I. Yes. It will provide them some
dependence on foreign sources of monetary help to either seek
petroleum could have serious employment or to kick start some
consequences if, for example, a war cut ‘self-employment’ venture.
off our access to those imports. By II. No. It will dampen their urge to do
reducing our use of petroleum products something to earn their livelihood

p
in making plastics, we can take a major and thus promote idleness among
step towards national energy the unemployed youth.
independence and so enhance our 41. Statement: Should all the practicing

ou
country’s security. doctors be brought under Government
Which of the following, if true, would control so that they get salary from the
most greatly weaken the argument Government and treat patients free of
above? cost?
(a) Only a small fraction of petroleum Arguments:
products consumed in this country I. No. How can any country do such

gr
is used in making plastics. an undemocratic thing?
(b) New methods of plastics II. Yes. Despite many problems, it will
manufacture can somewhat reduce certainly help to minimize, if not
the amount of petroleum needed. eradicate, unethical medical
(c) The development of atomic energy practices.
as an alternative to petroleum-
based energy has been slowed by
m 42. Statement: Should there be
legitimate concerns over safety. concentration of foreign investment in
(d) In times of war, combatant nations only few states?
Arguments:
xa
would be seriously tempted to sieze
forcibly the territories of I. No. It is against the policy of overall
petroleum-producing nations. development of the country.
(e) Some plastic products, such as II. Yes. A large number of states lack
aircraft and motor vehicles’ parts, infrastructure to attract foreign
play vital roles in our nation’s investment.
te

growth. 43. Statement: Should India engage into a


Directions (40-44): Each question given dialogue with neighbouring countries to
below consists of a statement, followed by stop cross border tension?
two arguments numbered I and II. You have Arguments:
ce

to decide which of the arguments is a strong I. Yes, This is the way to reduce the
or weak. Give answer as: cross border terrorism and stop
(a) If only argument I is strong loss of innocent lives.
(b) If only argument II is strong II. No, Neighbouring countries cannot
(c) If either I or II is strong be relied upon in such matters, they
may still engage in subversive
@

(d) If neither I nor II is strong


(e) If both I and II are strong activities.

40. Statement: Should the educated 44. Statement: Should system of offering
unemployed youth be paid jobs only to the wards of government
“unemployment allowance” by the employees be introduced in all
Government? government offices in India?

16 Adda247 Publications For any detail, mail us at


Publications@adda247.com
A Complete Book of Logical Reasoning

Arguments: (c) Depletion of ground level in the


I. No. It denies opportunity to many country
deserving individuals and (d) Less plantation of tree in the
government may stand to lose in country
the long run. (e) Wastage of water by the people of
II. No. It is against the principle of the country
equality. Does not government owe

p
its responsibility to all its citizens? Directions (47-49): Study the following
instructions carefully and then answer the
45. Statement: A slump in home sales has questions that follow.
pushed builders’ inventory to

ou
“unsustainable levels” in the National In making decisions about important
Capital Region of Delhi, creating questions, it is desirable that a candidate
conditions for a real price correction that should be able to distinguish between ‘strong’
developers have so far avoided. and ‘weak’ arguments so far as, they relate to
Which of the following statements is the questions. ‘Weak’ arguments may not to
definitely true on the basis of the given be directly related to the trivial aspect of the

gr
statement? question. Each question given below is
(a) Demand for houses is quite low in followed by two arguments numbered I and
most of the big cities. II. You have to decide which of the arguments
(b) The high inventory level will are strong arguments and which is a ‘weak’
hamper the ability of launching new
projects by the builders.
m argument give you answer as:
(a) If only argument I is strong.
(c) Builders in the market who have (b) If only argument II is strong.
shown a good track record of (c) If either I or II is strong.
delivering on time and on their
xa
(d) If neither I nor II is strong.
promises have been able to garner (e) If both I and II are strong.
sales even in this slow market.
(d) Unless this unsold stock gets 47. Statement: Should sex education
absorbed, it will be difficult for promote in school?
consumer confidence to come back Arguments:
te

in this market. I. No, it will end the education system


(e) High prices have pushed most of set from ancient times.
the on-sale apartments beyond the II. Yes, it will give right information at
reach of average home buyers. right time. It will help children in
ce

fighting against problem in future.


46. Statement: The water situation is grim
as the total storage in 91 main reservoirs 48. Statement: Should overseas press enter
across the country has gone below the in India?
last 10-year average, says the report of Arguments:
the Central Water Commission (CWC). I. No, they do not give good
@

Which of the following may be the most information, and Indian press
probable reason for the fall in the level industry will destroy.
in reservoirs? II. Yes, they will give true and right
(a) Less rainfall across the country news receives good reactions and
(b) More use of water by the irrigation overseas press will help us to know
department truth.

17 Adda247 Publications For any detail, mail us at


Publications@adda247.com
A Complete Book of Logical Reasoning

49. Statement: Is globalization necessary 52. The earth’s resources are being depleted
for India? much too fast. To correct this, the United
Arguments: States must reduce its resource
I. Yes, we need latest technology to consumption at present levels for many
feed large population of India and years to come.
in a short time this need can be Which of the following, if true, would
fulfilled by globalization. most strengthen the argument above?

p
II. No, globalization can’t face poverty (a) The United States has been
and unemployment and this will conserving resources for several
increase problems because modern years.
technology need less investment

ou
(b) Other countries have agreed to hold
and human power. their resource consumption at
Directions (50–51): Study the following present levels.
instructions carefully and then answer the (c) Other countries need economic
questions that follow. development more than the United
In making decisions about important States does.

gr
questions, it is desirable that a candidate (d) New resource deposits are
should be able to distinguish between ‘strong’ constantly being discovered.
and ‘weak’ arguments so far as, they relate to (e) The United States consumes one-
the questions. ‘Weak’ arguments may not to third of all resources used in the
be directly related to the trivial aspect of the world.
question. Each question given below is
followed by two arguments numbered I and
m 53. Each question given below consists of a
statement, followed by three or four
II. You have to decide which of the arguments
arguments numbered I, II, and III. You
is a strong arguments and which is a s ‘weak’
have to decide which of the arguments
argument give you answer as:
xa
(a) If only argument I is strong. is/are 'strong' arguments) and which
(b) If only argument II is strong. is/are 'weak' arguments) and
(c) If either I or II is strong. accordingly choose your answer from
(d) If neither I nor II is strong. the alternatives given below each
(e) If both I and II are strong. question.
Statement:
te

50. Statement: Should doctors make an Should the income generated out of
organisation? agricultural activities be taxed?
Arguments: Arguments:
I. No, doctors are in top and ideal I. No. Farmers are otherwise suffering
ce

profession and to make from natural calamities and low


organisation is against the rules of yield coupled with low
jobs. procurement price and their
II. Yes. If you are not an organisation, income should not be taxed.
there will be no attention paid II. Yes. Majority of the population is
towards you small requirements. dependent on agriculture and hence
@

51. Statement: Should we end censor in their income should be taxed to


Cinema? augment the resources.
Arguments: III. Yes. Many big farmers earn much
I. No, It will increase vulgarity. more than the majority of the
II. Yes, control will prevent rise and service earners and they should be
development in art. taxed to remove the disparity.
18 Adda247 Publications For any detail, mail us at
Publications@adda247.com
A Complete Book of Logical Reasoning

(a) Only I is strong (b) It is possible that Philip’s signature


(b) Only I and II are strong on the petition was forged by a
(c) Only II and III are strong member of the Student Rights
(d) All are strong Coalition.
(e) None of these (c) Any member of the student body is
eligible to sign a petition dealing
54. Landmark preservation laws unfairly
with university affairs.
impinge on the freedom of owners to

p
(d) Philip may have resigned from the
develop their own property as they see
Student Rights Coalition after
fit. In some cases, owners of hotels and
signing the petition.
office buildings designated as landmarks

ou
(e) Some of those who signed the
have been forbidden to make changes in
petition may not be members of the
the original facades or interiors, even
Student Rights Coalition.
though they reasonably believe that the
changes would enhance the structures 56. The percentage of family income spent
and make them more valuable. on entertainment has remained almost
Which of the following statements, if the same over the past twenty years –

gr
true, seriously weakens the author’s about twelve per cent. When new forms
argument? of entertainment become popular, they
(a) Altering the appearance of a do not expand this percentage; instead,
historic structure sometimes does they take consumer spending away from
not enhance its beauty or value.
(b) In traditional legal doctrine,
m other forms of entertainment. Therefore,
film producers have observed the video
ownership of a property implies the boom with concern, knowing that every
right to alter it at will. dollar spent on rental of videos means a
(c) Only buildings over 75 years old are dollar less spent on movie theatre
xa
normally affected by landmark admissions.
preservation laws. Which of the following, if true, most
(d) Landmark designations must be forcefully undermines the argument of
approved by a local regulatory body the passage above?
before taking effect. (a) The cost of renting a video is
te

(e) Historic buildings represent a generally substantially less than the


cultural heritage which the price of a movie theatre admission.
community has a legitimate stake in (b) Most film producers receive a
preserving. portion of the income from the sale
ce

of video rights to their movies.


55. All the members of the Student Rights
(c) Fears of some film producers that
Coalition signed the petition calling for a
videos would completely supersede
meeting with the university trustees.
movies have not come to pass.
Philip must be a member of the Student
(d) Since the start of the video boom,
Rights Coalition since his signature
money spent on forms of
@

appears on the petition.


entertainment other than videos
Which of the following best states the
and movies has dropped.
central flaw in the reasoning above?
(e) Some movies that were
(a) Some members of the Student
unprofitable when shown in
Rights Coalition may not support all
theatres have become successful
of the organization’s positions.
when released in video form.

19 Adda247 Publications For any detail, mail us at


Publications@adda247.com
A Complete Book of Logical Reasoning

Stength of Argument : Solutions


1. (d); For I- Statement I weaken the given (c) poor people might not have
statement as according to it large enough time to engage in
number of people migrating to religious practices but it does
urban areas for better job not eliminate the possibility of
opportunities whereas the given them having religious beliefs.

p
statement suggests that people are (d) mentions a fact which is
willing to compromise on their irrelevant as the given
standard of living in order to live in statement talks about the

ou
a healthier/pollution free general population of
environment financially successful people
For II- Statement II weaken the which includes both developed
given statement as according to it and developing countries.
migration from the rural areas is (e) is irrelevant to the statement.

gr
increasing whereas according to the
4. (e); Except (e), all the statement
statement, more and more people
strengthens the given statement by
are willing to migrate to
pointing out the effects of water
rural/semi-urban areas.
scarcity India is currently facing
For III- Statement III strengthens
and problems related to it but
has been a decline in number of
m
the statement is it states that there
option (e) states that the crisis was
more in 1960 from now which
migrants but it is due to the poor
contradicts the given statement as
condition of urban slum areas,
it states that now its “worst” water
xa
mostly acquired by the migrants
crisis in history of India. So (e)
and because of the policies of the
weakens the given statement.
government.
5. (d); Statement I strengthen the given
2. (a); Only I weakens the given
statement as even the economic
statements as the statement
growth is higher the problem of
te

suggest that the today’s children


stunting is keep on increasing
are weaker than previous
among children. But statement II is
generation and are not as much
neutral statement.
active and also lack in developing
ce

their personality whereas the 6. (e); For I: Yes, It is strong argument as it


statement I states that today’s supports the statement that by
children are mentally more active consuming an apple a day any one
so it weakens the given statement. can keep himself fit and fine.
For II: No. It is not supporting the
3. (a); (a) mention a flaw in the survey
statement because argument II is
@

which weakens the conclusion


about real juice of apple flavor but
drawn from it.
in statement consuming of apple
(b) supports the statement by
fruit is instructed and it is also said
pointing out that some people
in argument to ban it which is not in
embrace religion to achieve
support of statement.
financial success.

20 Adda247 Publications For any detail, mail us at


Publications@adda247.com
A Complete Book of Logical Reasoning

For III: No, It is not strong because (d) Zoo officials could not be held
according to statement it’s an responsible for anything that
initiative from health ministry to happened outside zoo premises.
take an apple daily but in argument 10. (e); Argument (b), (c) and (d) talks
III, other side of statement is shown about the digestibility of grains or
which is not in support. their bad effects while the
For IV: No, it is not strong with statement is only concerned with

p
respect to statements as it’s an the addictive nature of grains.
alternative to be healthy but it is Argument (a) is irrelevant.
not in support of an initiative which Argument (e) points to a fact which

ou
is stated by health ministry. proves the addictive nature of
grains.
Directions (7):
11. (c); Only (c) strengthen the given
7. (a); The researchers are testing statement as it states that interest
preference for one feature: the of people of India is rising in
masculinity/femininity of the faces. football which will help the skilled

gr
If it is found that more masculine youth to indulge in this game. But
faces men are in relationship, then (a) and (b) suggest that Cricket is
it is obvious if there is attraction the most lovable game and football
then only there is relationship. require star players to become as
popular as cricket players among
8.
difference between the Maths and
m
(c); The passage claims that the critical Indi Further (d) is about the
extraordinary players of Hockey in
science scores of Japanese and India and (e) is about superiority of
American children is the discipline Indian Women players over Men
of the Japanese educational system.
xa
players in Olympics game
(c) Directly contradicts this claim. It
says, in essence, that this cannot be 12. (a); Statement I strengthen the given
the proper casual explanation statement as is states about the
amount invested on disasters and
because American students, in
climate related mishappenings
highly flexible environments,
which is also a cause for rising the
te

actually outscore the Japanese number of hungry people. But


students in their rigid statement II cannot be assumed
environments. from the given statement as it is
9. (b); (a) Water sharing between two about increase in pollution level.
ce

cities is out of the scope of the 13. (d); For I: No, Statement given by youth
zoo authorities. leader refutes the statement given
(b) It may be the reason for the by motivational speaker because
death of four dolphins which love has not been described as it is
could be noticed by zoo described by speaker.
@

officials. For II: No, It is not a just emotional


(c) Mentions an incident which support, as love has been described
describes an action, but it does by speaker very deeply.
not pointing towards careless- For III: Yes, It describes the depth
ness which may be the factor of as it is also stated in statement that
four dolphins death. one partner remains happy in
his/her partner delight.
21 Adda247 Publications For any detail, mail us at
Publications@adda247.com
A Complete Book of Logical Reasoning

14. (c); For I: Yes, it weakens the statement For III: It is a myth that should be
of manager as it is said by manager avoided or abolished but it is the
that increment will be based on part of the statement given above.
performance i.e. hard work and
17. (a); For I: False, We cannot consider this
determination. But 15% for all
point, because in this with
those employees who have
certainty, it is said that number will
completed 1 year undermines the
get decreased but according to

p
statement by manager.
statement we don’t know even the
For II: No, it strengthens the
magical number.
statement by manager as
For II: True, Demand of State X has

ou
performance chart has been
come after demand of State Y has
prepared and appraisal will be done
been rejected and state Y
accordingly.
government political party has also
For III: Yes, it weakens because it is
withdrawn their support.
mentioned that punctuality is one
For III: True, It may be the adverse
of the key factor and if those who is
result as demand of state Y has

gr
punctual and those who gets
been rejected by Central
relaxation time and both are
government.
considered as same, then it’s a
partiality for the punctual coming 18. (d); For I: False, It supports the above
employee. idea as it shows the mental and
m
15. (b); For I: Yes, in this question we have
physical concerns.
For II: True, It nullifies the above
to find the possible adverse impact
idea because it shows the another
of playing IPL game on Dream11.
angle that working upto late night
So, there may be wrong impact on
and they get ransom salary.
xa
working as concentration of
For III: False, It supports the above
employee may be deviated from
statement as some psychological
their office works.
issues are shown in last statement.
For II: No, as according to the
question we have to find the 19. (c); For I: False, It supports the concern
adverse impact but IInd statement of Spokesperson of Delhi
te

is a positive vibe for employees. Government.


For III: Yes, addiction of betting For II: True, It nullifies the concern
may impact on their basic earning as it shows PM himself bothers
which is coming from their hard about Delhi security.
ce

work. For III: True, it also nullifies the


concern as Central Government is
16. (d); In this question, we have to find
giving a special package to Delhi
which is not related to the above
Govt. for safety of women and to
statement.
maintain law and order.
For I: It is completely related to the
@

passage, there must be awareness 20. (b); For I: False, according to statement
program to make society awake. which we have to consider as base,
For II: This is must do step which and in comments Mr. Patra has not
should be taken by Government but discussed about points which is
it is not related to statement which mentioned in statement.
is mention above. Empowering the economy has been

22 Adda247 Publications For any detail, mail us at


Publications@adda247.com
A Complete Book of Logical Reasoning

discussed but we have to find most 24. (a); The author cite one example of
effective comment. international pollution, so another
For II: False, Congress has not even example would help strengthen his
discussed on a point which has claim. (a) provides such an
been mentioned by Mr. Aranab. example.
For III: True, Mr. Sinde has
discussed two points which is most 25. (e); statement (a) states that demand of
effective comments among all. coal is still increasing at 5% per

p
annum therefore the argument is
21. (b); We have to find which conclusion
nullifies the concern of statement. not weakened by the statement.
Statement (b) does not affect the

ou
For I: True, In statement I it is
concern that cleaning program had argument that stocks will be
been started in country but thinking depleted unless new reserves are
towards girls is not clean but in found because it considers the
conclusion it is trying to show some reserve in Antarctica. statement
steps has been taken for women (c)informs that there is an
safety. alternative to coal which is being

gr
For II: True, in conclusion II, it is used increasingly.(d) states an
shown same as conclusion I that economic fact of life that would
something is done for girls but by have to be faced if the statements in
this step concern of statement will the paragraph are true.so neither
not get solved.
For III: False, it supports the
m (a),(b),(c) or (d) is appropriate.
concern of statement as recently 26. (a); The question was put to people
steps has been taken to improve the who were coming out of the
mental thinking of people. terminal about the services aboard
xa
Direction (22): the flight. If it was shown that a
22. (a); In this question we have to choose majority of people coming out of
that particular statement which the terminal were not passengers,
nullifies the possible adverse effect Then the conclusion would really be
of Public Premises Act. jeopardized.(a) does just that.
For Statement I: Right, As there is
te

monetary punishment of Rs. 10 27. (e); The conclusion of the argument is


lakh for all so if any one does so, he that landmark preservation laws
will have to pay the declared deprive landlords of their right to
monetary punishment. use their own property. (e) comes
ce

For Statement II: Wrong, This will to grips with this assumption by
support the fallout of Public nothing that a landmark building
premise act. may not be purely private property
For Statement III: Right, as this and some part of building may
rule is for all so opposition cannot belong to the community at large.
raise voice against it. (a) does not valid as it represents
@

23. (c); The argument is leading up to call only a partial attack on


for some sort of international argument.(b) strengthens the
agency to control pollution, which, claims that landmark preservation
as the author claims, is an laws represent an unwanted
international problem. Thus (c) is interference with the rights of the
the best response. landlord.

23 Adda247 Publications For any detail, mail us at


Publications@adda247.com
A Complete Book of Logical Reasoning

28. (c); The question stem here asks for the place, it is the performance of the
evidence that would most weaken individual that matters and that
the theory that the metal particles makes him more or less wanted,
are important to the bird’s and not his educational
navigational mechanism. The best qualifications. So, neither I nor II
refutation of the theory would be a holds strong. Besides, the needs of a
finding that when the particles are job are laid down in the desired

p
removed, the birds continue to qualifications for the job. So,
migrate as effectively as before. (c) recruitment of more qualified
is tantamount to saying that this people cannot augment

ou
was the case. productivity. Thus, IV also does not
hold strong. However, it is the right
Direction(29-30):
of an individual to get the post for
29. (e); All other option are irrelevant
which he fulfills the eligibility
except option (e) which says that
criteria, whatever be his extra
because US consumes one-third of
merits. Hence, argument III holds
all world’s resource, it should keep

gr
strong.
its consumption under check as
they are depleting very fast. 34. (e); Clearly, illiterate people lack in
power and maturity in thoughts.
30. (b); The conclusion concerns the They may easily be misled into false
supposed “powerful effects” of red convictions or lured into
colour. However, if the observed
m temptations to vote for a particular
results are due to the “power” of group. So, argument II holds.
blue, the conclusion is not valid! However, a person is literate does
The experimental design is flawed not mean that he is conscious of all
xa
in that only two colours are used political movements, which
and the so-called positive effect of requires practical awareness of
red cannot be distinguished from a everyday events. Thus, I also holds
possible negative effect of blue. strong. Besides, Constitution has
extended the right to vote equally
Directions (31–32): to all its citizens. Hence, III also
te

31. (c); If an economy is speculative, it holds.


cannot be so trustworthy and hence 35. (c); From the statement, it is clear that
we can’t predict or determine the the word ‘exaggerated’ represents
prices of goods or commodity with something as better or worse than
ce

respect to that economy. Hence, it it really is. It is mention there that


weakens the argument of the the number of children enrolled in
author. classes one to five were shown to
be high for more states and in
32. (b); It clearly strengthens the argument
option (c), it is given that 22
of the author, who is optimistic percent children enrolled and still
@

about the decline in the price. going on. So, this option supports
33. (d); The issue discussed in the the best.
statement is nowhere related to 36. (e); If the shipyard is not covered into
increase in unemployment, as the the proposed complex, it might be
number of vacancies filled in will sold for another use, which could be
remain the same. Also, in a working more detrimental to the proposal.

24 Adda247 Publications For any detail, mail us at


Publications@adda247.com
A Complete Book of Logical Reasoning

37. (d); In order to shift the blame for the 41. (b); A doctor treating a patient
reduction in the annual rate of individually can mislead the patient
growth of sales output away from into wrong and unnecessary
Joe’s new regime, we must find treatment for his personal gain. So,
reason for the lack of sales. (d) argument II holds strong. Also, a
provides us with this alternative. policy beneficial to common people
cannot be termed ‘undemocratic’.

p
38. (a); Statement (b) deals with the So, I is not strong.
courses of government income, not
with government spending. The 42. (a); An equitable distribution of foreign
investment is a must for uniform

ou
accuracy of the six-percent figure
development all over the country.
given in the argument and on which
So, argument I holds. Also, no
the argument rests is not called into
backward state ought to be
question; so (c) does not weaken neglected; rather such states should
the conclusion. (d) tends to be prepared and shaped up to
strengthen, rather than weaken the attract foreign investment as well.

gr
position that Verdant’s budgets So, II does not hold.
have been austere. (e) is
inappropriate because the 43. (a); Clearly, peaceful settlement
through mutual agreement is the
conclusion is based on actual
best option, whatever be the issue.
avowed goals.
m
figures, bot descriptive phrases and
So, argument I holds strong.
Moreover, the problem indicated in
39. (a); In the passage it claims that in II can be curbed by constant check
order to ensure that we have and vigilance. So, 2nd one is not
xa
sufficient energy ,we should quit strong.
using petroleum to manufacture 44. (e); Merit, fair selection and equal
plastics. If we do so, we will have opportunities for all – these three
made a significant step towards factors, if taken care of, can help
energy independence. This government recruit competent
te

argument however assumes that we officials and also fulfill the


use a lot of petroleum in the objectives of the Constitution. Thus,
manufacture of plastics –an both the arguments hold strong
assumption questioned by (a). 45. (e); (a) does not follow because the NCR
ce

Directions (40-44): phenomenon can’t be extrapolated


40. (e); Young people, who do not get to “most of the big cities”. b) and d)
employment due to the large are probable consequences. c)
number of applicants in all fields, shows us the other side. e) follows
must surely be given allowance so from the mention of “slump in
@

that they can support themselves. home sales” and scope for “price
correction”.
So, argument I is strong. However,
such allowances would mar the 46. (a); The most probable reason for the
spirit to work, in them and make depletion in the reservoirs can be
them idle. So, argument II is also less rainfall in the country. So only
strong. option a) follows.

25 Adda247 Publications For any detail, mail us at


Publications@adda247.com
A Complete Book of Logical Reasoning

Directions (47-49): is strong. Besides, a major part of


47. (b); Sex education provided in school is the population is dependent on
true when it is provided in agriculture and such a large section,
adolescence age. They are confused if taxed even with certain
in so many things about sex. They concessions, would draw in huge
remain in tension, it will be better if funds, into the government coffers.
they are given information at right Also, many big landlords with

p
time. Education given in a substantially high incomes from
controlled environment does not agriculture are taking undue
affect ancient Indian system. Argue advantage of this benefit. So, both

ou
II is strong but I is not. arguments II and III hold strong.
48. (d); If we view closely none of argue 54. (e); The conclusion of the argument is
seems strong. This seems that it is that landmark preservation laws
just imagination. deprive landlords of their right to
49. (b); Today is time of machines and use their own property. (e) comes

gr
computers. Reality is that it needs to grips with this assumption by
very less man power. Hence nothing that a landmark building
argument II is strong and I is not. may not be purely private property
and some part of building may
50. (a); In reality doctor is considered as
belong to the community at large.
make organisation and to do
m
God. It is not expected from them to
(a) does not valid as it represents
only a partial attack on argument.
strikes. Because patients need them
anytime. If anything goes wrong (b) strengthens the claims that
with them. They can use another landmark preservation laws
xa
organisation to strong their voice. represent an unwanted
So argue I is strong and argue II is interference with the rights of the
not strong. landlord.

51. (a); If censor is finished vulgarity will 55. (e); In the above question it is asked to
increase. Under censor everything choose from a given option which
te

goes through a system. So, weaken the statement or make it


argument I is strong but II is not. imperfect so correct answer will be
(e).
52. (e); All other option are irrelevant
ce

except option (5) which says that 56. (d); The author argues for the following
because US consumes one-third of connection: videos take money
all world’s resource, it should keep away from movies. What choices
its consumption under check as (d) asserts, in effect, is that the
resources are depleting very fast. money spent on videos came from
@

53. (d); Clearly, the profit and losses both some other source. So, (d)
are to bear by the farmers which statement undermines the given
occur due to natural calamities. So, I passage.

26 Adda247 Publications For any detail, mail us at


Publications@adda247.com
A Complete Book of Logical Reasoning

p
ou
gr
m
xa
te
ce
@

1 Adda247 Publications For any detail, mail us at


Publications@adda247.com
A Complete Book of Logical Reasoning

Political Affairs Based Logical


07 Reasoning
Introduction: Political affairs based logical reasoning is commonly based on the political
happenings all around the world. All the questions in this are based on whatever is happening in

p
politics all over the world is comprised in this topic. All the latest political upheaval whether in
Indian politics or around the world is covered in it and the questions are entirely based on it. As
you all know that politics is very vast and also quite complicated, so one should be bit attentive

ou
while solving these types of questions.
As you all know that the basic rule of solving logical question is that the statement is the base of
the question and you have to consider it 100% true. Answers should be concluded on the basis
of statements. So, the one should not be biased in terms of favoritism towards any political
party, to not consider your own views towards any political issue or your own expectation from

gr
the leaders.
Political Chaos are generally seen in every nation’s political scenario. And the questions are
based on the current happenings, any leader’s statement, any political issue, elections, agendas
etc.
m
Syntax of the Political Affairs Based Logical Question

Example-1: In Europe, the politicians who welcome the refugees take a stand of basic human
decency. They believe that people are fleeing for their lives from terror and war and to deny
xa
them asylum would violate the most basic standards of compassion. But those who oppose the
refugees have their own valid points. They say that the new host countries are unable to commit
large social transfers to an unending, the essentially unlimited, number of new arrivals. That
would lead to a fiscal crisis.
Which of the following can be concluded from the passage above?
(a) Migration crisis in Europe is a complex issue.
te

(b) In Europe, accepting influx of refugees is really compassionate.


(c) Keeping doors wide open for refugees is suicidal for a country.
(d) Trying to forcibly stop the migrant influx would fail.
(e)Both (a) and (b)
ce

Brief for the question: In the given question the situation of Europe is mentioned, in which
some politicians welcome the refugees whereas some people are opposing the arrival of the
refugees. There are five options given below the statement and we have to find which of them is
concluded from the given statement.
@

Solution------
Explanation-1:
Ans. (a) Some politicians believe that denying asylum to refugees would violate the basic
standards of compassion. Others believe that host countries would face a financial crisis if they
allow an unending and unlimited number of refugees. The refugee problem in Europe is viewed

2 Adda247 Publications For any detail, mail us at


Publications@adda247.com
A Complete Book of Logical Reasoning

differently by two groups of politicians – one in favour of accepting them on humanitarian


ground and the other expressing their inability to accept them due to the financial crisis the
refugees would create. It is reasonable to conclude that the refugee crisis is a complex one: it is
inhuman to send them back, and at the same time accepting them would lead to a fiscal crisis. So
(a) is the right option. Option (b) contradicts the passage. (c) Misses one point and carries the
other too far. Option (d) is unrelated to the passage.

p
Example-2: In Delhi, The BJP won all seven seats during the Lok Sabha elections, but it got an
unexpected and unprecedented drubbing in the 2015 assembly polls. The party was able to win
only three out of seventy seats.

ou
Which of the following can be logically concluded from the passage above?
(a) Voters have learned to shock the party that has not kept its electoral promises.
(b) States can buck the national trend in an election.
(c) Voters have started electing their representatives as per the horses-for-courses principle.
(d) Political parties exploit the prevailing conditions in the state to win an election.
(e)Both (c) and (d)

gr
Brief for the question: In the given question the result of 2015 assembly polls is given, in which
the Lok Sabha election winner party BJP did not get enough seats. The question is followed by
five conclusions and we have to find which of them is concluded from the given statement.

Solution------
m
Explanation-2:
Ans.(c) The BJP won all seven seats during the 2014 Lok Sabha election, but it won only three
seats out of seventy in the 2015 assembly polls. The passage suggests that voters do not show
xa
the same inclination for a particular political party in Lok Sabha and assembly polls. Clearly,
they elect representatives based on something else. Option (a) is irrelevant to the passage as the
passage doesn’t make any reference to electoral promises. Option (b) is a general comment as it
is about reversing the national trend in an election, but the focus of the passage is on voters’
choice of party. Option (c) is a logical conclusion that can be drawn from the passage – voters
te

elect different representatives for different purposes, here for Lok Sabha and assembly. (d) is a
vague comment; in fact, nothing in the passage points to prevailing conditions.

Points to Remember----
I. Political affairs based logical questions are usually concern with the political issues,
ce

elections, disputes between political parties etc.


II. While solving such questions you have to be neutral and strictly inline with the given
statement.
III. Current Political scenario, past work of any political party, ruling party’s positives and
negatives etc. should not be considered while answering the question.
@

IV. You should not assume anything on your own while answering the questions.
V. While solving such question first completely go through the given statement and try to
understand what has been said in it and then go on with the options.

3 Adda247 Publications For any detail, mail us at


Publications@adda247.com
A Complete Book of Logical Reasoning

Exercise

Directions (1-2): In making decisions about 3. Statement: In the General Budget of


important questions, it is desirable to be able India we are always to see that the
to distinguish between “strong” arguments government makes deficit budget it is in

p
and “weak” arguments insofar as they relate the hand of the government to make the
to the question. “Strong” arguments are those budget surplus by reducing the planned
which are both important and directly related expenditures.

ou
to the question. “Weak” arguments are those Which of the following could be the
which are of minor importance and also may reason for making deficit budget?
I. If the budget will be surplus the
not be directly related to the question or may
economy will contract
be related to a trivial aspect of the question.
II. It helps in the economic
Each question below is followed by two development of the country.

gr
arguments numbered I and II. You have to III. The world bank will not sanction
decide which of the arguments is a “strong” loans if there is budget surplus
argument and which is a “weak” argument. IV. The government will have to
Give answer impose more taxes on the people.
(a) If only argument I is “strong”. (a) Only I. (b) Only II
(b) If only argument II is “strong”.
m (c) Only III
(e) All of these
(d) Only IV
(c) If either I or II is “strong”.
(d) If neither I nor II is “strong”. 4. Statement: Mrs Indira Gandhi
(e) If both I and II are “strong”. nationalised 20 banks of India – 14 in the
xa
year 1969 and 6 in the year 1980 –
1. Statement: Should Anti-piracy Bill be saying that the banks were not following
brought in the Parliament? the policies adopted by the country after
Arguments: Independence and forced them to come
I. Yes, This will help the film industry under government ownership.
te

to fight against piracy as a result of Which of the following is/are


which the industry will be assumption(s) in the above statement?
financially strong. I. Banks were nationalized because
II. No, We have already provisions they did not follow the policy of the
government.
ce

such as Copyright laws etc to fight


against piracy. II. India being a socialist country, it is
the duty of the banks to the welfare
2. Statement: Should Non-Resident of people.
Indians (NRIs) be entitled to dual III. The banks should be nationalized in
citizenship? socialist country.
@

Arguments: IV. It was declared by the first PM of


I. Yes, This will bolster the intimacy of the India that there will be nationalized
NRIs with the Indian soil. banks in India.
II. No, The move will not be fruitful for all (a) Only I (b) Only I and II
NRIs because there is lack of provisions (c) Only II and III (d) Only I, II and III
for dual citizenship in many countries. (e) All of these

4 Adda247 Publications For any detail, mail us at


Publications@adda247.com
A Complete Book of Logical Reasoning

5. Statement: A severe cyclonic storm hit (d) The local academic body made a
the Eastern coastline last month fervent appeal for the special status.
resulting in huge loss of life and property (e) None of the above.
on the entire east coast and Government
7. The RBI has raised the Interest rate
had to disburse a considerable amount
ceiling on NRI deposits in foreign
for relief activities through the district
currencies by up to 3%.
administration machineries.
Which of the following assumption can

p
Which of the following may possibly be a
be possible in the given statement? (An
follow up measure to be taken up by the
assumption is something supposed or
Government?
taken for granted)

ou
(a) The Government may ask the
(a) The RBI wants to attract inflows in
people to relocate to some other
view of the falling value of rupee.
place far away of the coastline.
(b) The Indian banks will be able to
(b) The Government may set up a
offer higher interest rates on NRI
committee for the estimation of the
deposits in foreign currency.
losses occur due to this
(c) RBI wants to create perfect

gr
mishappening.
competition.
(c) The Government may empower the
(d) The Indian banks have more
District Magistrates to make all
liquidity.
future disbursements of relief.
(e) None of these.
(d) The Government may send relief
m
supplies to the affected people in 8. Statement: Despite strong opposition,
future only after proper assessment the controversial Prohibition of Forcible
of the damage caused by such Conversion of Religion Bill was passed
calamities. by the Tamil Nadu Assembly with the
xa
(e) The Government may not need to AIDMK and the BJP outvoting the
activate any follow up measure. combined opposition of the DMK, the
Congress, the Pattali Makkal Katchi and
6. Statements: The State Government
the left parties – A news.
granted special status to the seminar on
Assumption
the importance of protecting the
I. Conversions create resentment
te

environment around us organized by the


among several sections and also
local academic body to create awareness
inflame religious passions, leading
among the citizens.
to communal clashes.
Which of the following may be a possible
II. Conversions only lead to the
ce

reason for the state Government's


isolation of the converted.
decision?
III. Strong opposition puts hindrance
(a) The Central Government has
before a bill in taking final shape.
accorded highest priority to issues
(a) II and III (b) I and II
related to protection of
(c) I and III (d) All of these
environment, over all other issues.
@

(e) None of these


(b) In the past the State Government
had granted special status to such 9. Prime minister Modi gave hard hitting
seminars. remarks at G20 summit in china “One
(c) The State Government has failed to nation in South Asia is spreading terror
implement measures for protecting in the region”. The growing forces of
the environment. violence and terror pose a fundamental

5 Adda247 Publications For any detail, mail us at


Publications@adda247.com
A Complete Book of Logical Reasoning

challenge. There are some nations that (III) Indian Para commandos are special
use it as an instrument of state policy. forces made for mainly special
India has a policy of zero tolerance to operations.
terrorism. Because anything less than (a) Only I (b) Only III
that is not enough, Mr. Modi said. “For us (c) Only I and II (d) Only I and III
a terrorist is a terrorist." (e) None of these
Which of the following statement can be
11. Which of the following can be courses of

p
inferred from above sentences?
action after surgical strike on POK?
(I) India had agonized a lot from
(I) Indian Govt. should evacuate
terrorist attack.
villages which are closer to LOC.

ou
(II) The country which is indicated by
PM Modi will take strong action (II) Army should be more attentive and
against terrorism. should increase security across
(III) Govt. of India is too much concern border.
about terrorist activity and cannot (III) Indian Govt. should give reward to
indulge terrorism. all those Para commandos who
made surgical strike successful.

gr
(a) Only I (b) Only III
(c) II and III (d) Only I and III (a) Only III (b) Only I and III
(e) None of these (c) All follows (d) Only II and III
(e) None of these
Directions (10-11): After running through a
variety of non-military responses to the 12. On the basis of given statement you have
m
terrorist strike at an Army camp in Uri, the to decide relation of cause and effect.
Centre on Thursday announced that Indian (I) Indian forces carried out surgical
forces had carried out “surgical strikes” strike on Thursday across POK with
across the Line of Control. The operation, that no injuries caused to Indian Para
xa
began and concluded in the early hours of commandos who went for strike.
Thursday, was claimed to be a military (II) Pakistan government on Saturday
success, with no injuries to the Indian Para- sent two fighter jets in India with an
commandos who went across the LOC into intention to cause damage.
Pakistan-occupied Kashmir to attack several (a) if statement I is the cause and II is
locations. The decision to strike in this
its effect.
te

manner was evidently taken after specific


(b) if statement II is the cause and I is
intelligence that terrorist groups were
its effect.
planning attacks in India. The terms “surgical
strike” and “pre-emptive strike” used by the (c) if both the statements I and II are
independent causes.
ce

Centre were intended to convey that this was


not an attack on Pakistan’s defence forces, but (d) if both the statements I and II are
a targeted action against terrorists poised to effect of independent causes.
wreak damage in India. (e) if both the statements I and II are
effects of some common causes.
10. Which of the following can be
postulated from the given statements? 13. With less than three weeks to go before
@

(I) There may be pressure of the election of president, the campaigns


opposition on ruling government saw explosive revelations about Mr.
after Uri attack. Trump and Ms. Clinton. This included
(II) Govt. of Pakistan was expecting disturbing evidence of Mr. Trump
action like surgical attack from boasting about groping women without
India after Uri attack. their consent, allegations of such

6 Adda247 Publications For any detail, mail us at


Publications@adda247.com
A Complete Book of Logical Reasoning

behaviour and sexual assault by at least (b) Quality of life will improve in rural
nine women, and an avalanche of and urban areas.
confidential emails of Ms. Clinton’s (c) Provision of roads and public
campaign released by WikiLeaks that transportation beside
underscored her proximity to deep- implementation of reforms in urban
pocketed Wall Street donors. governance would be the focus
Which of the following can be under the scheme.

p
hypothesized from the above passage? (d) Millions of jobs will be created and
(I) Mr. Trump and Ms. Clinton are scarcity of houses will be removed
candidates of opponent parties in urban and rural areas.

ou
which are fighting for presidential (e) All of the above.
posts.
15. Statement: Reinforcing its commitment
(II) Serious allegation against
to green initiatives, railways will explore
candidates may affect the polling
result. its future course of action to prevent
(III) Trade and economy of country pollution. Issues related to environment
like solid waste management, pollution

gr
depends on the selection of
Presidential candidates. control, water management and energy
(a) All I, II and III (b) Only I and III management will be discussed at a
(c) Only II and III (d) Only I and II workshop on ‘Environmental challenges
(e) None of these before Indian Railways and solutions’.

14. Statement: Prime Minister Narendra


m Which of the following points will
strengthen the decision taken by the
Modi launched three mega flagship Railways?
schemes aimed at transforming Urban I. Introduction of bio-fuel and solar
India, including the much talked about energy in train operation.
xa
Smart Cities proposal, with a whopping II. Installation of rain water harvesting
Rs. 4 lakh crore kitty having been lined facilities at major stations.
up for the projects. He also unveiled logo III. Waste water recycling plant is also
of the housing mission, which has his being planned in which recycled
personal touch in the finalization of its
water will be used for washing and
te

design.
horticulture purpose.
Speaking at the event, Modi said, “the
IV. Initiatives such as LED light fittings
decision to make cities smart will be
to reduce power consumption and
taken not by governments but by the
use of five percent bio-diesel in
people of the city, the local
ce

diesel locos.
administration. Let there be competition
between cities in development so that V. Introduction of water audit to
smart cities come up.” assess consumption and wastage at
Which of the following will be an effect major stations.
of the initiative taken by the (a) All (b) All except I
(c) All except I and V (d) All except V
@

government?
(a) Ensuring water supply to each (e) None of these
household, sewer connections, 16. Statement: Chinese Ministry of Foreign
sanitation and solid waste Affairs spokesperson said that the
management in rural and urban members of the Nuclear Suppliers Group
governance would be the focus (NSG), were holding discussions on the
under the scheme.
7 Adda247 Publications For any detail, mail us at
Publications@adda247.com
A Complete Book of Logical Reasoning

accession of all countries, which had not 17. Which of the following may be
signed the Nuclear Non-Proliferation anticipated reason for fallout of high
Treaty (NPT). The upcoming NSG voltage drama which is mentioned in the
Plenary meeting in Seoul will not cover passage?
the issue of NSG memberships for non- I. Remissness of Central Govt. to enact
signatories to the Non Proliferation OROP in the country for Indian
Treaty (NPT). Therefore, there is no Soldiers.

p
point talking about supporting or II. Central Govt. tried to embezzle
opposing the entry of a particular non- soldiers for their political gain
NPT country at the moment. behind the shade of success of

ou
Which of the following statements surgical strike.
substantiates views expressed in the III. Arrest of Deputy CM of Delhi was
above statement? illegitimate, he should not be
(a) The existing policy could not detained as he is deputy CM of Delhi
change through a “a change in and it is his responsibility to visit to
criteria” for admission. the family of victim.

gr
(b) The signatories of the NPT are not (a) Only I and III (b) only III
eligible for admission to the NSG. (c) Only I and II (d) Only I
(c) China has the right to take the (e) None of these
stand, which is in compliance with 18. What will be the probable repercussions
the rules of NSG.
(d) India always maintain strict rules
m after the suicide of former Army jawan
Ram Kishan Grewal?
and all the guidelines of NSG, hence I. There may be mass strike overall
it’s a SUO MOTO right of the nation country which may muddle the law
and order system in the country.
xa
to improve its supply to power
generation. II. Deputy CM Manish Sisodia will file
(e) None of these. defamation case against Delhi
Police as he was detained by Delhi
Directions (17-18): Police.
Delhi Chief Minister Arvind Kejriwal on III. Central Govt. will give compensa-
te

Wednesday targeted Prime Minister tion amount to the family of Former


Narendra Modi and the Central government Army Jawan Ram Kishan Grewal.
for the suicide committed by former Army (a) Only II (b) Only I and III
jawan Ram Kishan Grewal allegedly over the (c) Only III (d) Only II and III
ce

OROP issue. Deputy Chief Minister Manish (e) None of these


Sisodia and Aam Aadmi Party MLA from Delhi Directions (19-21):
Cantonment ''Commando'' Surender were It is almost four months since the unrest in
detained by the Delhi Police at the RML Kashmir began following the killing of Hizbul
hospital where they had gone to talk to the Mujahideen ‘commander’ Burhan Wani.
@

family of Grewal. Grewal’s family members Protests, intermittent violence and long
were detained by the police after they refused stretches of curfew have continued to put
to take the body home and threatened a stir at normal life on hold. Delegations of civil
the hospital. Mr. Kejriwal said the society representatives as well as politicians
government should stop trying to derive have attempted to reach out to separatists
political gains from the recent surgical strikes. and find a way to bring calm to the streets,

8 Adda247 Publications For any detail, mail us at


Publications@adda247.com
A Complete Book of Logical Reasoning

but to little avail. In fact, the opposite is II. Govt. of India should deploy proper
happening with increasing mindless arson army forces outside school
attacks on schools over the past two months. premises for their immediate start
By one count, 27 schools, most of them and smooth functioning.
government-run, have been set afire so far in III. Govt. of India should impose
the Valley over this period. No one has yet presidential rule in J&K for smooth
claimed responsibility for these attacks. The functioning of administration in

p
government has blamed the separatists for state.
encouraging the arson. In turn, the separatists
(a) Only II (b) Only I and III
charge the administration of failing to protect
(c) Only III (d) Only I and II

ou
the schools.
(e) None of these
19. Which of the following is not in line with
the given passage? 22. Statement: The government will
I. Blame game of separatists and Govt. consider merging some of its more than
are ruining the future of children. two dozen state-run banks (27 PSU
II. United Nations has deterrent to banks), as it seeks to improve efficiency

gr
state Govt. of Kashmir after the at the ailing lenders that dominate the
Pakistani terrorist attacks to make nation's banking sector. Bankers have
situation under control as soon as "strongly supported" the idea of
possible. consolidation, suggesting the

blocked which affects the trade


m
III. All the National Highways are being government form a panel of experts to
devise a merger strategy, Arun Jaitley
from J&K after heavy snowfall. said after a two-day annual
(a) Only I (b) Only II and III brainstorming event of industry leaders
(c) Only III (d) Only I and III
xa
and officials from the central bank and
(e) None of these finance ministry.
20. Which of the following can be inferred Should there be only few banks in place
from the above passage? of numerous smaller banks in India?
I. Making ferocity in Kashmir is a Arguments:
game plan of Hizbul Mujahideen to
te

I. Yes, This will help secure the


make pressure on State Govt. investor's money as these big banks
II. All the delegation have endeavored will be able to withstand
to bring tranquility in the area. intermittent market related shocks.
III. Most of the school premises have
ce

II. No, A large number of people will


been vandalized by the arsonist.
lose their jobs as after the merger
(a) Only I and III (b) Only II
many employees will be redundant.
(c) Only III (d) Only II and III
III. Yes, This will help consolidate the
(e) Only I
entire banking industry and will
21. Which of the following course of action lead to healthy competition.
@

Govt. of India should take to bring peace (a) None is strong


so that schools can run smoothly again. (b) Only I and II are strong
I. Govt. of India should give shoot at (c) Only II and III are strong
sight order for those who are (d) Only I and III are strong
responsible for savagery in the
(e) All are strong
valley.

9 Adda247 Publications For any detail, mail us at


Publications@adda247.com
A Complete Book of Logical Reasoning

23. Statement: Juvenile Justice (Care and vegetable crop at mandis governed by a 1963
Protection of Children) Act, 2015 has law on marketing farm produce is a bold and
been passed by Parliament of India. It laudable step. Chief Minister Devendra
aims to replace the existing Fadnavis has stood his ground against the
Indian juvenile delinquency law, Juvenile powerful lobby of middlemen, who shut shop
Justice (Care and Protection of Children) in protest, is even more commendable. The
Act, 2000, so that juveniles in conflict problem with the present regime, under

p
with Law in the age group of 16–18, which produce has to be sold through
involved in Heinous Offences, can be Agricultural Produce Market Committees, is
tried as adults. The Act came into force that farmers seldom benefit from price

ou
from 15 January 2016. It was passed on movements; traders rake in the upside that
7 May 2015 by the Lok Sabha amid consumers are forced to shell out.
intense protest by several Members of
24. Which of the following can be concluded
Parliament. It was passed on 22
from the given passage?
December 2015 by the Rajya Sabha.
(i) This will encourage the farmers to
What are the possible consequences of
grow more crops.

gr
the Juvenile Justice bill in India?
(ii) Similar steps will be implemented
(i) The bill aims to "consolidate and
in other produces.
amend the law relating to children
(iii) Now, the farmers will definitely get
alleged and found to be in conflict
a good price for their hard work.
with law and children in need of
care and protection by catering to
m (a) All except (i)
(c) All except (ii)
(b) All of the above
(d) Only (i)
their basic needs through proper
(e) None of the above
care, protection, development,
treatment, social re-integration, by 25. What can be the feasible problems of the
xa
adopting a child-friendly approach." farmers which are observed from the
(ii) The bill deals with adoption of passage?
children and lays down the (i) Money for the produce is never
eligibility criteria for adoptive given on the spot.
parents. A central adoptive (ii) The values are decided by the
“buyers” who are usually
te

resource agency will frame the


rules for adoption, which will be middleman, even if government
implemented by state and district system is involved.
level agencies. (iii) The inputs are under the clutches of
(a) All except (i) Multinational Corporations.
ce

(b) All of the above (a) All of the above (b) All except (iii)
(c) Both (i) and (ii) follows (c) All except (ii) (d) All except (i)
(d) All except (ii) (e) None of the above
(e) None of the above
26. Statement: Ordinarily, Ajit Jogi’s
Direction (24-25): Read the following decision to form a new political party in
@

passage and answer the given questions Chhattisgarh would not have caused as
according to the passage: much as a mild flutter in the Congress.
His reputation has been under a cloud
The Maharashtra government’s decision to
for a long while and his importance in
promulgate an ordinance to exempt farmers
the Congress’s scheme of things has
from having to mandatorily sell their fruit and
declined. Having become the first Chief

10 Adda247 Publications For any detail, mail us at


Publications@adda247.com
A Complete Book of Logical Reasoning

Minister of Chhattisgarh upon its (a) Only II (b) Only I and III
formation in 2000, he lost the state to (c) Only III (d) Only V
the BJP in 2003, which has held it since (e) Only III and II
then; meanwhile, the Congress has
28. Statement: The decision to pick Reserve
frittered an inordinate amount of
Bank of India Deputy Governor Urjit
political capital in trying to defend him
Patel as the successor to outgoing
in a string of controversies.
Governor Raghuram Rajan is a clear

p
Conclusions:
affirmation of the Centre’s commitment
I. Congress is in a catch-22 situation.
to ensure policy continuity at the central
It needs to promote competent
bank and to minimize the bad debts

ou
leaders in the states to revitalize
which brings banks on back foot
the party.
II. A solid step by the party could take previous year. That the man chosen for
to tackle the existential crisis head- the top job at the RBI is a person, who
on is to project Rahul Gandhi as CM helped formulate crucial changes in the
candidate for the upcoming UP monetary framework, including the
decision to target a specified inflation

gr
polls and for him to get into the
grind. level as the primary remit of the bank,
Which of the following can be concluded reflects the administration’s focus on
from the given statement? making price stability central to its
(a) Only I follows economic agenda.
(b) Only II follows
(c) Both I and II follow
m What will be the major challenge(s) for
Urjit Patel in near future?
(d) None follows I. The new governor may have to cut
(e) Either I or II follows rates sharply to let growth take
place in a broad-based manner.
xa
27. Statement: The committee on sexual II. Public sector banks reported heavy
discrimination in the workplace has bad debts under RBI’s asset quality
highlighted Mastero Company as a chief
review during previous financial
offender. Of the twenty senior executives
year. This must be a challenge for
in the firm, only one is a woman. And of
new governor.
the forty junior executives, only five are
te

III. Inflation will be a challenging factor


female.
for new governor. It has started
Maestro could best defend itself against
inching up, led by food prices even
the charges by showing that
as global commodity prices,
I. Male and female executives at the
ce

particularly that of oil has started


same level have the same
qualifications picking up.
II. They pay the same salary to senior (a) All of the above (b) All except II
men and senior women (c) Only I and II (d) Only I
III. Ten times more men than women (e) None of these
@

apply for jobs with the company 29. Statement: Prime Minister Narendra
IV. The work pressures and long hours Modi on Friday announced the setting up
make jobs with the company of a task force after failure of
unattractive to married women sportsperson in RIO olympics which will
V. All job applicants who were help to plan for the “effective
rejected had fewer qualifications participation” of Indian sportspersons in
than those accepted.
11 Adda247 Publications For any detail, mail us at
Publications@adda247.com
A Complete Book of Logical Reasoning

the next three Olympics, to be held in (c) The quality audit of all higher
2020, 2024 and 2028. education institutions, both private
He made the announcement at the and public, leads to improving the
monthly meeting of the Union Council of literacy rate of the country.
Ministers and said the structure of the (d) There will be strong focus on value
task force would be in place in next few education.
days. (e) None of these.

p
Which of the following is/are not in line
31. Statement: Brazil, just like India, is still
in contrast with the given statement?
a developing nation and the mammoth
I. The task force will prepare an
expenditure on the Football World Cup

ou
overall strategy for facilities,
led to wide scale protests against then
training, selection procedures and
Brazilian PM Dilma Rouseff. They spent
other related matters.
millions of dollars on single stadiums
II. The task force will comprise of
members who are in-house experts which would become useless after the
as well as those from outside. World Cup while their nation lacks

gr
III. Poor infrastructure and training proper Healthcare facilities like
facilities for athletes and political Hospitals.
interference in sports have been Similarly, their Olympics preparations
flagged as reasons for the poor are equally shabby with frequent reports
performance in current year. of athletes facing huge discomfort in the
(a) All except II (b) Only II
m Games Village. There were also reports
that Athletes in Water Surfing and
(c) All except I (d) All of the above
(e) None of these related events were asked to keep their
"mouths closed" due to contaminated
30. Statement: A five-member committee
xa
water.
has recommended that the government Should these big events like FIFA and
reinstate detention of students beyond Olympics be hosted in developing
Class V and also set up an all-India cadre countries?
of educational services on the lines of the I. Yes, Hosting a mega-event always
Indian Administrative Service (IAS) in its involves urban renewal and
te

report on the new education policy. The development.


committee has written strongly in favour II. No, Building new infrastructure in a
of remedial classes for such students city means destroying established
during holidays and after school hours. urban areas. When that happens,
ce

They will be given three attempts to local populations and communities


clear the examination in the same year. are often dispersed and displaced.
Which of the following can be III. Yes, Such events always promote
postulated from the given information? infrastructure redevelopment,
(a) Foreign universities are allowed to improved transportation systems,
set up campuses in India under a
@

economic growth and job creation.


strict regulatory framework and Which of the following argument(s)
thus quality of education will holds strong?
improve in the country. (a) Only II and III (b) Only I and II
(b) Under this new education policy, (c) Only II (d) All of the above
fewer students will move to higher (e) None of these
classes as compared to earlier.

12 Adda247 Publications For any detail, mail us at


Publications@adda247.com
A Complete Book of Logical Reasoning

32. Statement: India's Mars Orbiter Mission was not able to tackle the protests
marks the country's first mission to the and violence. Thus, it is neither a
Red Planet and first entry into win-win situation nor does it take
interplanetary spaceflight. The BJP on back foot.
unmanned Mars Orbiter Mission, or II. Yes, definitely. The instability in the
MOM, will map the surface of the Red state post the Patel upsurge, the
Planet and test technologies needed for deterioration in health and

p
future Indian space missions. The education sectors, the snail-slow
mission is a “technology demonstrator” employment rate are few of the
project to develop the technologies for many factors for her resignation.

ou
designing, planning, management and This will not just affect the BJP in
operations of an interplanetary mission. Gujarat but also around the
What are the advantages of Mars Orbiter country.
Mission for India? III. Yes, as the opposition especially the
pseudo seculars would say that
I. It has given a huge boost to the
Gujarat BJP lacks Women
image of India in the world. It will
Leadership.

gr
also generate interest among the
Which of the following arguments holds
youths of India and world over to strong?
pursue science and technology as (a) All of the above (b) All except I
career. Thus, it gives a moral boost. (c) All except II (d) All except III
II. This success will definitely bring (e) None of the above
more enthusiasm towards R&D
m 34. Statement: The fall of Dilma Rousseff
activity related to science and
technology. An increase in funding from being one of the most popular
for such activities is imminent in politicians to an impeached leader is
much more than a story of a “corrupt”
xa
the future.
President being stripped of her powers
(a) Only I (b) Only II
by a righteous legislature. The exit of
(c) Both I and II (d) Neither I nor II
Brazil’s first woman President brings the
(e) Either I or II 13-year rule of the left-leaning Workers’
33. Statement: Faced with infighting in the Party to an end. The charge levelled
te

party and huge disenchantment on the against her is that she used illegal
ground, beleaguered Gujarat Chief bookkeeping manoeuvres to hide a
Minister Anandiben Patel offered to growing deficit.
resign and asked the BJP to select a new Which of the following assumption(s)
is/are implicit from the given statement?
ce

face.
By offering to quit, she has pre-empted I. There were numerous marches in
any move to remove her. Her decision cities with very large turnouts
protesting her government.
comes at a time when the BJP’s long
II. The charges of her impeachment
dominance in the State appears to be
were a result of political and
under strain, just a year before the
@

economic maneuvering as opposed


Assembly elections. to corruption.
Will Anandiben Patel’s resignation take III. Ms Rousseff cooked the books to
BJP on back foot? improve the poor conditions of
I. No, it doesn’t because one of the under dimensional Brazil, its
important factors behind her welfare schemes for public housing,
resignation is age factor. Also, she sanitation and education.
13 Adda247 Publications For any detail, mail us at
Publications@adda247.com
A Complete Book of Logical Reasoning

(a) All of the above 37. According to budget 2016-17, 28.5 lakh
(b) Only II hectares will be brought under irrigation
(c) Only II and III system under Pradhan Mantri Krishi
(d) None of the above sinchai Yojana. What may be the effect of
(e) Only I and III above announcement?
35. Statement: To tone up the bureaucratic (a) The government increases the food
apparatus and weed out officials of production of our country.

p
doubtful integrity and efficiency, the (b) By increase irrigation programme,
government has asked all its the farmer can increase their
departments to identify such public income.

ou
servants and move proposals for their (c) Agriculture sector will boost by this
premature retirement. step at particular significance
Which of the following can be concluded improvement
from the above statement? (d) All may be effect
(a) There will be voluntary retirement (e) None of these
for the person who skips the office 38. Targeted Delivery of new Government

gr
for a long time. subsidies and benefits to ensure that
(b) Only Government service is made they reach the poor and the deserving.
for those who can perform. What will be the cause of above
(c) Negligence of work in the office will government budget?
lead to premature retirement. (a) Some portion was remain
(d) Efficiency of the government
service will increase.
m untouched from new subsidies and
other scheme.
(e) Only (c) and (d) (b) Some urban sector can get subsidy
36. Statement: The Modi government has scheme profit.
xa
rolled out an ambitious plan to top up (c) DBT ensures related information
the existing technical syllabus across about the poor people for subsidies.
industrial training institutes with (d) The transparency and clearance
modules aimed at making them ready for will be increased.
the modern workplace. (e) None of these
Which of the following will be an effect
te

39. The U.S. Greenhouse gas emission rates,


of the above statement? which caused due to increase amount of
(A) Demand for skilled labour will
CO2, increase with 0.2% in comparison
increase in India.
with last year. This increase amount of
(B) Millions of jobs will be created for
CO2 will affect the environment badly.
ce

skilled labour.
What will be possible effect of above
(C) People will be able to get job easily
phenomena?
due to proper training.
(a) Increase in Co2 levels has made the
(D) Upgradation in the modern
world’s oceans more acidic.
education system will encourage
(b) Co2 and other gases will be higher
entrepreneurship skills in the
@

amounts in the air and it effects the


country.
growth and nutrition in plants.
(a) Only (B) and (C)
(c) The U.S. government banned old
(b) Only (A) and (D)
(c) Only (C) and (D) industries which gives bad effect on
(d) Only (D) environment.
(e) Only (B) (d) All a, b, c
(e) None of these
14 Adda247 Publications For any detail, mail us at
Publications@adda247.com
A Complete Book of Logical Reasoning

40. Many countries stockpile nuclear (d) Global warming is an issue of


weapons in their armory. These concern to all —especially the
weapons are built to serve more as economically developed nations.
deterrents to enemy nuclear nations (e) None of these
than as potential weapons to be used in
case of a war erupting between two Directions (42-44): Read the following
nations possessing nuclear weapons. It is passage carefully and answer the questions
given below. The government's version of the

p
clear that no country possessing nuclear
weapons would risk using them against Lokpal Bill says that sitting prime ministers
an enemy nation having nuclear will not come under the ombudsman's lens.
weapons for fear of retaliation. Now, the Lokpal will have jurisdiction over

ou
Which of the following is an assumption ministries, but not the prime minister. It's a
in the conclusion that no nuclear country mistake. He is the head of the Cabinet and the
would use nuclear weapons in a war first among equals; as liable to scrutiny as his
against its enemy country for fear of peers. Ministries and various departments are
reprisal? the functioning arms of the government,
(a) In a war between two nuclear which enforce rules. The root of corruption

gr
nations, third nations possessing lies in how these rules and policies are
nuclear weapons will not interfere. administered.
(b) Nuclear weapons can cause more 42. Which of the following is the conclusion
havoc than traditional weapons. of the above discussion?
(c) No nation possesses a nuclear
device which when deployed can
m (a) The government has no will to fight
corruption.
destroy all the nuclear weapons of
(b) In case the prime minister is
another country at one stroke.
brought under the purview of the
(d) World opinion is always against the
Lokpal, his office may face frequent
xa
use of nuclear weapons in a war.
frivolous allegations.
(e) None of these
(c) Keeping the prime minister out of
41. Energy consumption involves release of the purview of the Lokpal will
greenhouse gases resulting in global render the institution impotent to
warming. Hence, it is the contention of check corruption in the country.
te

the under-developed nations that the (d) Ministries and departments are not
economically developed countries must under the effective control of the
make a higher contribution to funds government.
constituted towards taking measures to (e) None of these
fight global warming.
ce

Which of the following is an assumption 43. Which of the following can be the
in the above argument? assumption behind the above
(a) Developed nations are rich and discussion?
hence can contribute more liberally (a) The incumbent government wants
to funds required for fighting global to protect the prime minister from
facing corruption charges.
@

warming.
(b) Energy consumption varies directly (b) The prime minister wants to
as the economic status of a nation. maintain his supremacy over the
(c) Fighting global warming needs ministers.
huge funds which can be (c) The prime minister is a
contributed only by the developed constitutional position and it can't
nations. be brought under the Lokpal.

15 Adda247 Publications For any detail, mail us at


Publications@adda247.com
A Complete Book of Logical Reasoning

(d) Bringing ministers under the Assumptions


Lokpal is enough to check I. People don’t want to approach
corruption in high places. consumer forum due to the red-
(e) None of these tapism in procedure adopted by the
forum.
44. Which of the following is an inference of
II. Speedy redressal will attract more
the above passage? unresolved consumer disputes.
(a) There are fundamental flaws in how

p
(a) Only I (b) Only II
policies and rules are administered (c) Either I or II (d) Neither I nor II
my ministries and departments. (e) Both I and II

ou
(b) Ministries and departments are
incapable of enforcing rules and 47. Statement: The Government has
policies properly in lack of support decided to auction construction of
from ministers. highways to private entities in several
(c) Scrutiny of ombudsman is not a blocks across the country on build-
necessary factor for controlling operate-transfer basis.
Which of the following assumption(s)

gr
corruption.
(d) Only sitting prime ministers are is/are implicit in the above statement?
accused of corruption charges. Assumptions
(e) None of these I. An adequate number of private
entities may not respond to the
45. Statements “If you ask me about the
m
daunting challenges that I face, I would
Government’s auction notification.
II. Some private entities in the country
say that my government’s first priority is are capable of constructing
to improve the existing law and order highways within reasonable time.
situation. Then follows the issue of III. The Government’s proposal of build
xa
prices of commodities.” ___ Mr. ROY, a operate-transfer may financially
newly appointed PM of country benefit the private entities.
CLINTON (a) I and II are implicit
Assumptions (b) II and III are implicit
I. If a citizen of country CLINTON can (c) Only II is implicit
te

sleep peacefully, he/she can then (d) I and III are implicit
think of providing food to his/her (e) None of the above
family, ponder over education and 48. President Obama has described climate
move about freely in the country. change as one of the biggest challenges
ce

II. Prices of the commodities affect the facing America and has said he is open to
common man greatly. new ideas to address it. He can start by
(a) Only I (b) Only II supporting legislation to increase the
(c) Either I or II (d) Neither I nor II nation's hydropower capacity, one of
(e) Both I and II America's-vital renewable energy
@

46. Statements “If you have any unresolved resources. Producing electricity from the
consumer disputes, do not feel that you nation's 80,000 dams can avoid million
are helpless, do not hesitate to assert metric tons of carbon dioxide emissions
your rights. Approach the District each year.
Consumer Forum for speedy redressal.” All the following can be assumed from
____ Department of Consumer Affairs the above EXCEPT

16 Adda247 Publications For any detail, mail us at


Publications@adda247.com
A Complete Book of Logical Reasoning

(a) Hydropower harnesses the force of (c) The Finance Commission has been
flowing water to generate mandated to recommend the
electricity. quantum of sharing of central taxes
(b) Many of the dams are not utilized between the Centre and states.
for electricity generation in (d) The centre resorts to surcharges
America. and cesses only when the share of
(c) Hydropower doesn't cause carbon the states in central taxes is raised.

p
dioxide emissions. (e) None of these
(d) All the dams in America are safe.
50. Gunland, an island nation, has granted
(e) None of these

ou
13 million of its citizens gun licenses
49. The recommendation of the 14th during the last few years. Though the
Finance Commission to raise the share of number of guns in Gunland has
the States in the central taxes from 32
increased by more than 50 percent
per cent recommended by the 13th
Finance Commission to 42 per cent during this period, the gun homicide rate
underlines the need to do some hard in the country has not registered a

gr
thinking. It will put immense pressure corresponding increase.
on the Centre to resort to surcharges and Which of the following, if true, most
cesses to make up for the higher helps to resolve the paradox?
devolution burden. The percentage share (a) Licenses were granted on condition
m
of the States in the Centre’s tax revenue,
popularly called vertical sharing, should
that the license holders carry their
guns concealed.
not be left to the caprices of a Finance
Commission in the future. (b) Gun magazines holding more than
Which of the following is an assumption seven bullets are banned in
xa
on which the argument depends? Gunland.
(a) The Finance Commission will (c) A thorough background check was
increase the states’ share of central done before issuing gun licenses.
taxes in the future. (d) Many deaths occurred due to
(b) The recommendation of the 14'" accidents while handling guns.
te

Financial Commission surprised the


(e) None of these
central and state governments.
ce

Political Affairs Based Logical Reasoning : Solutions


Direction (1-2) restricted only because the move is
1. (a); Only I is strong. II is not strong not giving benefit to all of a kind.
because we do not know whether
3. (b); Only II is true because it will help in
@

the existing provisions are


sufficient or not. development.

2. (a); I is not strong because intimacy of 4. (b); I and II are implicit as “banks were
NRIs with Indian soil will be helpful not following the policies adopted
for Indian economy. II is a weak by the country after Independence.”
argument because a move can’t be

17 Adda247 Publications For any detail, mail us at


Publications@adda247.com
A Complete Book of Logical Reasoning

5. (d); Option (d) will be the correct option that there may be word-war
as after the proper analysis the between opposition and govt.
government should provide relief before surgical strike. And as Indian
supply. Option (a) is not a proper Para commandos are used in an
measure also (b) is vague and (c) is important operation “surgical
not a considerable measure. (e) is strike”. So they must be special task
absurd. force.

p
6. (a); Option (a) may be a probable 11. (c); All courses of action follows as after
reason for the state government’s surgical strike there may be chance
decision as granting special status that terrorist will attack in reverse

ou
to seminar on environment clearly or Pakistan Army may conduct
indicates that it is priority issue for some intentional strike across
the government. border of India. So, for safety of
people, villages should be
7. (b); From the statement II we can evacuated and Indian Army should
generally assume that the Indian increase security across border.

gr
banks have capacity to pay higher And Indian Govt. should give
interest rate on NRI deposits. reward as it was one of the most
8. (e); Only assumption III is implicit. Note important operation and Para
that the statement is a piece of Commandos made this successful.
And reward is nothing but a token
news. What the journalist assumes
m
about the conversion and its impact of appreciation and encouragement.
can’t be explicitly said. But the 12. (a); Clearly I is cause as India conducted
words ‘despite strong opposition’ a surgical strike and Pakistan sent
indicate that the journalist must be its two jets is its effect.
xa
assuming assumption III.
13. (d); From the behavior and sense of
9. (d); Only I and III can be inferred from passage we can clearly say that
above sentences. Tone of above both are candidates of opponent
sentence and statement of Indian parties and allegation may hinder
PM clearly indicates that India had their image and it may affect the
te

suffer a pain of terrorist attack. IInd polling result. Trade and Economy
one may be an assumption or is not discussed in above passage.so
course of action but it cannot be III cannot be assumed.
inferred that indicated country will
ce

Directions (14-16):
take stern action. From the
14. (c); Note that we are talking about
statement of PM that “India has a Smart Cities Mission. Thus anything
policy of zero tolerance” it can related to rural areas is beyond the
clearly inferred that India is too scope. Whereas, in option (c), we
much concern and cannot tolerate focus only on urban development.
@

terrorism. Hence, option (c) will be an effect of


10. (d); There may be pressure of the statement.
opposition on ruling govt. as Uri 15. (d); All the points in the options
attack was a major attack and it is promote green initiatives except
mentioned in first line that after auditing of water consumption as it
“non-military response”……it means is an examination of system records
18 Adda247 Publications For any detail, mail us at
Publications@adda247.com
A Complete Book of Logical Reasoning

and equipment that may be used to area. But it is given that most of the
identify, quantify and verify how schools have set on fire by the
much water passes through the unknown groups of terrorists, so
system and where it goes. we can’t inferred as all this done by
16. (e); If the existing policy could not arsonist or separatist.
change and if the signatories of the 21. (a); Shoot at sight is a harsh course of
NPT are not eligible, then it will action. And statement II is

p
always oppose the entry of a immediate course of action. And
particular non-NPT country. imposing presidential rule is not a
Options (c) and (d) are independent proper solution.

ou
statements which relates
22. (a); The security of the investor's
somewhere to the passage but not
money is not related to the size of
completely. Hence, none of the
the bank. Besides even after
statements substantiates the views
consolidation, the number of
expressed in the above passage.
investors, their amounts and hence

gr
Directions (17-18): the duties shall remain the same
17. (d); Only I may be the possible reason and so no employees will be
for high voltage drama as main root redundant. Reducing the number of
for all this happening is OROP and smaller banks will also not affect
due to ignorance of Central Govt. to the mutual competition among the
implement OROP make Ram Grewal
m banks. Thus, none of the arguments
to suicide. Arrest of Deputy CM is holds strong.
also a high voltage drama but in
23. (c); The parliament of India has passed
statement it is mentioned it is
Juvenile Justice Act. Under this act,
xa
illegitimate, which is wrong so we
care and protection of children is
cannot consider this point.
also a major issue. Both (i) and (ii)
18. (b); Only I and III may be the possible deals with care and protection of
effect as by mass strike, people will children. So, these both may be the
try to make pressure on Central possible consequences of the
te

Govt. to implement OROP. And by Juvenile Justice bill.


giving compensation to family of
24. (d); The government’s decision will
victim, motive of central govt. will
definitely encourage the farmers to
to make people calm.
grow more crops as they can sell
ce

Directions (19-21): their fruit and vegetable crops


19. (b); Only II and III are not in line of the directly at mandis. The given
above passage. As, there is nothing passage deals with only agricultural
mentioned about intervention of produces. So, we can’t say anything
UN and there is nowhere discussed about other produces. The word
@

in the passage about trade from ‘definitely’ in (iii) makes it vague.


J&K. So, (ii) and (iii) cannot be
20. (b); Only II is inferred from the above concluded.
passage as politicians and 25. (b); It is mention in the passage that
delegation of civil society have farmers seldom benefitted from
attempted to bring peace in the price movements and it is also

19 Adda247 Publications For any detail, mail us at


Publications@adda247.com
A Complete Book of Logical Reasoning

given that the Chief Minister is hence it is clear that the


against the powerful lobby of government focus on value
middleman. So, (i) and (ii) are the education. So only (d) is a valid
feasible problems of the farmers. option.
Whereas we can’t say anything
31. (e); The main concern of the given
about its clutching power.
statement is that by hosting big
26. (a); I follows as for the revitalization of events like FIFA and Olympics,

p
Congress party, some competent developing countries will have to
leaders are needed. II may be face so many difficulties. So, we
inferred from the statement but have to proceed in that direction as

ou
does not follow as nothing is per the given statement. But none
mention there in the statement of the given argument talks about
about UP polls. providing healthcare benefits to
people over organizing Olympics or
27. (d); To defend itself against a charge of
FIFA.
discrimination the company will

gr
have to show its selections which 32. (c); Other than just the science though,
were made on the basis of merit not space missions are fantastic
sex. The best way to do that is to catalysts for scientific and
show that none of the women who technological advances in the entire
were rejected had better nation. India will learn how to
qualifications than the men who
m reliably get things in space, a skill
were appointed. Only option (e) that will become more and more
implies that and hence is the important. Space exploration
correct answer. excites people and encourages them
xa
to study STEM topics and pursue as
28. (a); The government and the industry
a carrier. As per regarding R&D
body want a sharp cut rates to
activity related to science and
achieve a high growth rate. So, I is
technology, there will be definitely
definitely a challenge for new RBI
some boost up regarding funding in
governor. II is also a challenge as
future. So, both I and II follows.
te

given that to minimize bad debt is a


challenge and to target specified 33. (e); The reason given behind her
Inflation level is also a challenge. resignation in argument I is age
factor which is nothing there in the
29. (d); First one is the main motive behind
ce

statement. Argument II deals with


the setting up of a task force. And
the improper functioning in the
since it is a task force, so the
state which may be the reason
members can be in-house experts
behind her resignation but it cannot
as well as from outside. So I and II
be predicted from the given
are in line in contrast with the
statement. Argument III is absurd.
@

passage. Third one can be the


So, none of the arguments holds
reason behind which this task force
strong.
has been set up. So, III is also in line.
34. (d); From the statement, it is nothing
30. (d); With the above decision taken by
mentioned anywhere that there
the government, there must be
were marches protesting her
focus on quality of education and

20 Adda247 Publications For any detail, mail us at


Publications@adda247.com
A Complete Book of Logical Reasoning

government. So, I is not implicit. It 40. (b); Choice (b) is the correct
is clear that the charges of her assumption. (a) and (d) is incorrect
impeachment were a result of as nothing is mentioned about
illegal bookkeeping manoeuvres to world’s opinion about nuclear
hide a growing deficit of the weapons and about third nation in
country. So, II and III are not
the passage
implicit.

p
35. (e); Officials are being made to retire 41. (b); Energy consumption leads to global
prematurely. The retirement is not warming. (b) is the correct
voluntary. So a) will be ruled out. assumption because developed

ou
Because of the word only b) will not nations have more industries and
be considered. The decision taken more work power who can
by the government will lead to consume energy so the energy
improvement in government consumption is directly more in
services and at the same time any
developed countries. (c) only talk of
negligence at workplace will lead to

gr
premature retirement. So, only c) the affordability. (d) is not an
and d) will follow. assumption.

36. (c); Giving skill training does not 42. (c); This statement clearly shows the
guarantee increase in demand of conclusion of the passage that PM
skilled labour, so (A) does not
m
follow. It can’t be said that giving
can’t be out of the purview of the
Lokpal.
skill training to youth will increase
employment by how much, so (B) 43. (a); This statement shows the hidden
does not follow. Imparting of skill
xa
assumption being taken in this
training in the institutes will boost passage.
the employability of the students
and at the same time encourage 44. (a); this statement gives the inference
entrepreneurship among them. which comes from this passage as
Only (C) and (D) follow. not including PM in lokpal is a flaw.
te

37. (d); All options are positively relevant 45. (d); I is not implicit. II is not implicit
to the increment in production, because the PM only assumes that
improvement in irrigation system
law and order affects the common
with a good income of farmers, so
ce

all are correct. man more than prices do.

38. (a); Govt. promulgate about subsidies 46. (b); Assumption II is implicit as this
and to give benefit to untouched forum provide speedy redressal so
portion of our Population. it will attract more people.
@

39. (d); Increment in CO2 emission and 47. (c); Only Assumption II is implicit
others having this possible effect because government as decided
that by this oceans acidic level means private entities are capable
increase, quality and nutrition will
of constructing highways within
decrease. It may be govt. banned
old industries that emit these gases reasonable time. Assumptions I and
a lot so all are being possible effect. III are not implicit.

21 Adda247 Publications For any detail, mail us at


Publications@adda247.com
A Complete Book of Logical Reasoning

48. (d); America has more than 80,000 Commission. Option (a) properly
dams which can produce electricity. identifies the assumption. Option
Generating electricity from these (b) is beyond the scope of the
dams can avoid million metric tons argument-Option (c) is stated in the
of carbon dioxide emissions every passage in different words. The
year. Option (a) is an easy argument doesn't refer to
assumption; 'flowing water from circumstances when the Centre

p
dams is used to generate electricity. resorts to surcharges and cesses, so
The second and third sentences (d) is not an assumption.
suggest option (b). Option (c) can
50. (c); Granting licenses to people who

ou
be assumed from the last sentence.
will not commit 'crimes can explain
Option (d) mentions the safety of
the situation here. Option (a) does
dams; this is not supported by the
not affect the number of homicides
passage.
committed; carrying guns
49. (a); Clearly, the passage implies that concealed or not does not influence
Finance Commissions in the past

gr
a likely crime. (b) can also be
have recommended increase in the eliminated because one may not
states' percentage share in central need more than seven bullets to
taxes. Since the argument concludes commit a murder. Option (c) can
that fixing the share should not be explain the mystery behind
left to the caprices of the
m
Commission, it assumes that, based
homicide rate not increasing
corresponding to the increase in the
on the past record, the Commission number of guns - licenses are
is likely to increase the share of granted to people with clean
states in the future also. Hence the
xa
background. Option (d) is not
passage shows worrying about the
related to homicides.
wisdom of leaving this power to the
te
ce
@

22 Adda247 Publications For any detail, mail us at


Publications@adda247.com
A Complete Book of Logical Reasoning

p
ou
gr
m
xa
te
ce
@

1 Adda247 Publications For any detail, mail us at


Publications@adda247.com
A Complete Book of Logical Reasoning

Economical Based Logical


08 Reasoning
Introduction: Economical based logical reasoning is majorly concerned with the economic
aspects of a country i.e. GDP of the country, finances, Export-Import, expenditures, taxes etc. The

p
questions based on the financial conditions, prices of commodities, budget, development and
economic policies are also comprised in this topic. The questions based on it may contain some
mathematical data as some fluctuation in prices, growth percentage, allocation of fund etc., these

ou
economic figures may differ from the actual one but you have to consider only the one which are
mentioned in the statement.
Nothing can’t be assumed yourself while solving such questions. The facts are only those
mentioned in the given statement nothing more or less can be considered in it while solving
such question.

gr
Syntax of the Economical Based Logical Question

Example-1: To protect certain fledgling industries, the government of Country Z banned


imports of the types of products those industries were starting to make. As a direct result, the
cost of those products to the buyers, several export-dependent industries in Z, went up, sharply
m
limiting the ability of those industries to compete effectively in their export markets.
Which of the following conclusions about Country Z’s adversely affected export-dependent
industries is best supported by the passage?
(a) Profit margins in those industries were not high enough to absorb the rise in costs
xa
mentioned above.
(b) Those industries had to contend with the fact that other countries banned imports from
Country Z.
(c) Those industries succeeded in expanding the domestic market for their products.
(d) Steps to offset rising materials costs by decreasing labor costs were taken in those
te

industries.
(e) Those industries started to move into export markets that they had previously judged
unprofitable.
Brief for the question: In the given question a situation is described in the statement in which
ce

country Z ban the import of certain type of product that the low margin industries of that
country starting to make which in turn increase the cost of such products to the buyers. We have
to find the conclusion of that statement.
Solution------
Explanation-1: Ans. (a) This statement properly identifies the conclusion that the export-
@

dependent industries were low-margin businesses that could not successfully accommodate the
higher prices of the domestically made products.
(b) No information about other countries’ ban of imports from Country Z is given in the passage.
(c) Not enough information is given in the passage to support this conclusion.
(d) No information about cutting labor costs is given in the passage.
(e) No information about the industries’ moving into different markets is given in the passage.

2 Adda247 Publications For any detail, mail us at


Publications@adda247.com
A Complete Book of Logical Reasoning

Example-2: The Government of India uses both Wholesale Price Index (WPI) and Consumer
Price Index (CPI) to measure the rate of inflation. But CPI is a better indicator as compared to
the WPI.
Which of the following conclusions supports the above statement?
(A) All the countries of the world use CPI to measure the rate of inflation.
(B) Majority of the population of India buy commodities at the retail prices.
(C) Majority of the population of India are very poor.

p
(D) The WPI has a very old base year.
(a) Only A
(b) Only B

ou
(c) Only A, B and C
(d) Only C
(e) Only C and D
Brief for the question: The given question states a comparison between Wholesale price index
(WPI) and Consumer price index (CPI) and indicates that CPI is better than WPI.

gr
Solution------
Explanation-2: Ans.(b) As it is given in the statement that CPI is better indicator than WPI and
we have to find which option best support the given statement. (A) can’t be considered as every
country has different economic conditions and different parameter to measure it. For (B),
m
Consumer price index measure the rate of inflation on the basis of retail prices of goods so
statement B concludes that CPI is a better indicator than WPI. (C) can’t support as having major
poor population does not indicate CPI better to measure the rate of inflation. (D) also not give a
proper reason to support the statement.
xa
Points to Remember
I. Economic Issue based logical questions usually have economic terms so first try to
understand what is given exactly given in the statement regarding it.
II. Economy of country or its economic condition is actually based on many parameters so
keep that in mind while answering the question.
te

III. Do not suppose or assume anything as the answers should be strictly based on the
statement.
IV. Be attentive towards the words as ‘Only’, ‘May’, ‘Always’ etc.
V. The questions may involve Import, Export or fluctuation in commodity prices which get
ce

affected from the International market prices.


@

3 Adda247 Publications For any detail, mail us at


Publications@adda247.com
A Complete Book of Logical Reasoning

Exercise

1. The Deficit of India is increasing day by Which of the following is/are not the
day and we can see that nonplan possible impact(s) of the new 100per
expenditures of the government are cent FDI rule in India?

p
increasing enormously. I. With the advent of new FDI norms,
What courses of action should the there will be creation of
government take in order to decrease employment.

ou
the Deficit of the country? II. It leads to investment in India
(1) Make more strict fiscal policy which results in appreciation of the
(2) Disinvestment Rupee.
(3) Introduction of new bonds III. It helps in better production,
(4) Printing of currency leading to increased GDP.
(a) Only (1) (b) Only (1) and (2)
IV. It would affect our internal security.

gr
(c) Only (3) (d) Only (4)
V. It may exploit the domestic
(e) All of these
resources without giving benefits to
2. The Government of India on the domestic country.
recommendations of a committee led by (a) All except IV
an expert economist started the merger
m
of the SBI associate bank with the SBI by
(b) All except V
(c) All except IV and V
merging State Bank of Saurashtra and (d) Only IV and V
State Bank of Indore, and in due course (e) Only V
other banks will also be merged.
xa
Which of the following could be the 4. Statement: The biggest story of this
reason of the merger? Budget is a major change in the way the
(1) The government wants to decrease Union Government spends its money.
the number of banks. Earlier this week, Finance Minister Arun
(2) The government wants to increase Jaitley announced that the government
the capital of SBI. had accepted the recommendations of
te

(3) The government wants SBI to be at the 14th Finance Commission and raised
par with the leading banks of the the States’ share in the net proceeds of
world. union tax revenues from 32 per cent to
(4) The government is fulfilling the 42 percent. With more of its pie going
ce

Basel norms. directly to States to spend as they like,


(a) Only (1) and (3) (b) Only (2) the Union Government would begin
(c) Only (3) and (4) (d) Only (2) and (3) reducing its allocations to the State plan
(e) None of these and wind up some Centrally Sponsored
3. Statement: The Union government Schemes. The Central Government has
@

permitted 100per cent foreign direct directed the State Governments to


investment (FDI) under government reduce government expenditure in view
approval route for almost every sector, of the serious resource crunch and it
including defence. "With these changes, may not be able to sanction any
India is now the most open economy in additional grant to the states for the next
the world for FDI." six months.

4 Adda247 Publications For any detail, mail us at


Publications@adda247.com
A Complete Book of Logical Reasoning

Assumptions: (II) Data regarding bank account are


I. The State Governments are totally theft using malware at ATMs
dependent on Central Government operated by third-party payment
for its expenditures. services vendor.
II. The Central Government has (III) Information regarding bank
reviewed the expenditure account account are leaked by officials of
of the State Government. Indian Banks to poacher.

p
III. The State Governments will abide (a) Only I and III (b) Only III
by the directives of the central (c) Only II and III (d) Only II
Government. (e) None of these

ou
You have to assume everything in the 6. Which of the following is not in line in
statement to be true, and then decide context of passage?
which of the three given suggested (I) NPCI is trying to find out the reason
assumptions logically follows. behind leakage of information
(a) None is implicit regarding bank account.
(b) Only II and III are implicit (II) Ministry of defense makes alert to

gr
(c) Only III is implicit all his officials regarding chances of
(d) All are implicit larceny.
(e) None of these (III) There is lack of effort by officials of
investigation agency as it has taken
Directions (5-7): The data breach at 19
Indian banks that has led to more than 32
m almost six months for the system to
officially acknowledge the
lakh debit cards being blocked or recalled is a incidents.
wake-up call for the banking industry. While (a) Only II and III (b) Only I and III
the actual number of complaints received so (c) Only II (d) Only I
xa
far, 641, and the sum of money that appears (e) None of these
to have been fraudulently withdrawn, Rs.1.3
crore, are both small relative to the scale of 7. Which course of action should be taken
the potential data theft, it is disconcerting that by investigating officials?
it has taken almost six months for the system (I) Recover money which have been
fraudulently theft by poachers.
te

to officially acknowledge the incidents and


initiate steps to address them. A private bank (II) Make aware to people to not use e-
appears to have been a point of entry for the commerce site for online
purchasing.
data criminals who, according to reports, may
(III) Hang the culprit if it is Indian
ce

have infiltrated using malware at ATMs


banking official who is responsible
operated by a third-party payment services
for leakage of information
vendor. The National Payments Corporation
regarding bank account details.
of India has been coordinating investigations
(a) Only I (b) Only I and III
into the incident, and a forensic audit is (c) Only III (d) Only II and III
expected to reveal preliminary findings soon.
@

(e) None of these


5. Which of the following possible reason 8. Statement: The government is set to
of pilferage of money from Indian banks? overhaul annual targets for public sector
(I) Account of many Indians who are lenders, ending a focus on size that has
using Chinese mobile are hacked by long encouraged banks to inflate their
Chinese hackers. loans and deposits at the year-end to hit

5 Adda247 Publications For any detail, mail us at


Publications@adda247.com
A Complete Book of Logical Reasoning

growth objectives. Banking and Which of the following assumptions is


government sources said that the new implicit in the statement?
targets, to be discussed at meeting with (a) If only assumption I is implicit.
top state bank officials, would focus on (b) If only assumption II is implicit.
efficiency, with objectives set around (c) If either I or II is implicit.
return on assets, or return on equity, and (d) If neither I or II is implicit.
controlling bad debts. (e) If both I and II are implicit

p
Which of the following points can be
inferred from the given information? (An 10. Statement: After rising for several
inference is something that is not months, gold prices are likely to see a

ou
directly stated but can be inferred from gradual decline in the short term on the
the given information.) back of strong global cues. With the US
(a) Loans and deposits of the banks will
economy showing signs of recovery,
go up.
(b) Loans on which there is maximum domestic gold prices are expected to
probability of return will be settle at under Rs. 30000 per 10 grams
encouraged. by March, although the decline may not

gr
(c) Number of bad debts per financial be as steep as international prices.
year will increase. A. Gold prices are determined by
(d) Financial stability of the banks will international market prices.
be lost. B. Prices of gold in local market are on
(e) All can be inferred.
9. Statement: The mayor of Oakland
m a continuous decline.
C. Soaring price of gold is not a
supports a new tax code that would positive sign for an economy.
assess local businesses on the property D. The US economy is very speculative.
value of their site rather than on their
xa
E. There are strong signs of decline in
income or profits, the mayor argues that
this change will not contribute to any the price as it was in the case of
loss in tax revenue for the city. Several petroleum and crude oil five years
city council members disagree, citing ago. Later, the market did decline.
similar changes to tax code that were Which of the following statements
unsuccessful in cities similar to Oakland. (A), (B), (C), (D) and (E) can be a
te

The council members’ argument is conclusion from the above


without merit, though, because property paragraph?
values rise steadily each year, while
(a) Only A (b) Only B
business incomes fluctuate wildly with
(c) Both A and B (d) Both C and D
ce

the national economy.


Assumptions: (e) None of these
(I) The annual percentage rise in real 11. Statement: Foreign lender HSBC has
estate values in Oakland has been
consistently more than the average decided to shut down some of its
annual growth rate percentages branches in the country citing that more
@

across all businesses with customers are availing banking services


properties in Oakland. through the digital medium rather than
(II) The new tax code would not apply going to branches.
to the significant number of online Which of the following may be the
business associated with the city reason(s) for the shutdown of branches
that have no physical presence on a
in India?
piece of property in the city.
6 Adda247 Publications For any detail, mail us at
Publications@adda247.com
A Complete Book of Logical Reasoning

I. The private banking space is hugely Which of the following abrogates the
competitive. Few decades ago, given statement?
foreign banks had an edge in the (a) Only I (b) Only I and III
private banking space because of (c) Only II and IV (d) Only II and III
quality service, global investment (e) None of these
tools and an untapped market.
13. Statement: In a historical move for the
Today, the likes of ICICI Bank and
country, the 122nd Constitutional

p
Axis Bank have stormed the market
Amendment Bill to introduce the Goods
with similar product offerings.
and Services Tax (GST) was unanimously
II. HSBC Bank was in the news due to
passed by Parliament this session.

ou
the black money probe. The bank
Widely believed to be the most
leaked a list of account holders who
important tax reform since 1991, the
illegally stashed funds in the foreign
GST will transform India into a unitary
bank to escape income tax scrutiny.
market, lower transaction and logistics
The Income tax department had
costs, spread the tax net wider and
attached bank accounts worth Rs
propel investments and growth.

gr
2,300 crore of 20 people named in
Is GST bill a panacea for Indian
the HSBC black money list.
economy?
III. The Indian economy is yet to see a
I. Yes, GST is going to create a unified
big pick up despite being in the
India in terms of taxation and thus
bright spot as India is more
consumption-oriented than other
m will improve the ease of doing
business ranking of India.
emerging markets such as China or
II. No, The implementation of GST
Brazil.
could impact the existing processes,
(a) Only I (b) Only I and II
people and technology.
xa
(c) All of the above (d) All except II
III. No, GST would impact negatively on
(e) None of the above
the real estate market.
12. Statement-Indian Oil launches app, SMS IV. Yes, It will help to build a
plan for customers to check daily fuel transparent and corruption-free tax
price. administration.
te

(I) India's largest fuel retailer Indian Which of the following arguments holds
Oil said consumers would be able to strong?
see updated prices of petrol and (a) All of the above (b) All except III
diesel in all cities through the (c) Only I and IV (d) Only I
ce

company's mobile app -- Fuel@IOC (e) Only I, III and IV


-- once the daily price revision kicks
14. The government has decided to instruct
in.
the banks to open new branches in such
(II) The company has taken steps to
a way that there is an e-branch of any of
ensure all its petrol pumps display
the banks in every village of population
the dealer code prominently.
@

1000 an above of a cluster of villages


(III)The move is part of a larger
with population less than 1000 to
effort to work with petrol pump
provide banking services to all the
owners and operators to ensure a
citizens.
smooth rollout of the daily price
Which of the following will weaken the
revision and prevent unscrupulous
step taken by the government?
elements from duping consumers.

7 Adda247 Publications For any detail, mail us at


Publications@adda247.com
A Complete Book of Logical Reasoning

(a) The private sector banks in India (a) People hanker after money ignoring
have stepped up their branch cultural value
expansion activities in rural India. (b) One cannot bear monetary loss
(b) Many government owned banks (c) Traditional culture in country X is
have surplus manpower in its urban degrading day by day
branches. (d) It is human nature to hanker after
(c) All the banks including those in money as well as adopt foreign
culture

p
private sector will follow the
(e) Preserving culture is our utmost
government directive. duty .
(d) Large number of branches of many

ou
government owned banks in the 17. Statement: The relationship between
rural areas are making huge losses income and expenditure is the
every year due to lack of adequate consumption schedule or consumption
business activities. function in economics. When disposable
income rises, consumption increases.
(e) None of the above
Consumption may exceed disposable
15. The principal monetary policy objective income for low-income individuals. As

gr
is to reduce substantially the import the disposable income increases, the
surplus of the coming years while average propensity to consume falls. In
resuming economic growth. Realization other words, the consumer spends a
of this goal entails a market structural smaller percentage of the extra dollars.
Consumption increases with increased
change of the economy, which can be
brought about by freezing the standard
m income, but short-term increases affect
consumption less than long-term
of living (per capita private consumption
increases. For an income increase of a
plus public services) and restricting year or less duration, consumers tend to
investments that do not further exports.
xa
change spending habits less than for
Which of the following assumption is permanent increases in income.
implicit from the above passage? In order to reduce the gap between
(a) Economic growth will result in a income and expenditure, the company
structural change of the economy. has decided to increase the prices of its
(b) Only if people consume less can the product from next month.
te

economy grow. Assumptions:


(c) The export surplus can be reduced I. The rate will remain more or less
if investment is restricted. after the increase.
(d) Only a structural change in the II. The expenditure will more or less
remain the same in near future.
ce

economy can substantially increase


imports. III. The rival companies will also
(e) People will have to be persuaded to increase the price of the similar
products.
give up consumption for the
You have to assume everything in the
national good. statement to be true, and then decide
@

16. “Monetary loss can be made good but which of the three given assumptions
any loss to the country’s culture can logically follows.
never be filled.” –A minister of country X. (a) Only I and II are implicit
If the above statement is true, then (b) Only II and III are implicit
which of the following has been assumed (c) Only III is implicit
in the statement? (d) All are implicit
(e) None of these

8 Adda247 Publications For any detail, mail us at


Publications@adda247.com
A Complete Book of Logical Reasoning

18. Statement: Non-banking entities (c) The demand for fuel decline in this
operating automated teller machines, time period.
known as white label ATM operators, are (d) All are being possible cause
struggling to meet the targets set by the (e) Only b and c
Reserve Bank of India. 21. Read the following passage carefully and
Which of the following is the most answer the question given below.
suitable reason for the above statement? Though focused on inflation, the RBI’s

p
(a) Non-bank entities are unable to find monetary policy can never ignore
sponsor banks of such ATMs in economic growth. The RBI has
small towns. recognized the signs of slowdown but

ou
(b) White label ATM (WLA) operators says that it is essentially confined to
do not fall directly under banking sectors that are sensitive to interest
regulations. rates. There is no evidence yet of a
(c) High transaction fees are charged broad-based slowdown.
by the WLA entities. Which of the following weakens the
(d) The operating cost in the cities is above discussion?

gr
high. (a) Hike in short-term lending and
(e) There is stiff competition in the borrowing rates contain inflation
market from public sector banks. and control price rise.
(b) Measures to control inflation hurt
19. Finance minister announced the fiscal investment across all sectors.
m
deficit target of 3.5% of GDP in 2016-17
which is less than previous year. What
(c) The costly auto, home and
corporate loans do not have overall
will be the possible cause behind this impact on the economy.
less fiscal deficit? (d) The stock markets, bankers and
(a) Due to economy decline, the corporate leaders do not represent
xa
government collects higher taxes. overall economy of the country.
(b) Due to Indian rupees positive (e) None of these
growth against dollar, the deficit 22. Read the following passage carefully and
decrease. answer the question given below.
(c) Government announced new taxes It is a major contradiction in India’s
te

scheme and e-scheme by which healthcare service: even as there has


deficit of revenue decrease. been a mushrooming of huge, well
(d) Indian economy boost by FDI and equipped, multi-discipline hospitals in
investment policies and it collect big cities serving the rich, thousands of
ce

higher export collection. rural India’s poor patients have to go


(e) All except a. without a semblance of medical care.
Which of the following in an assumption
20. Oil prices dropped on Wednesday in the of the above passage?
wake of industry, data that showed a (a) Rural India lacks sufficient trained
huge build in U.S. crude stockpiles. What medical personnel to take care of
@

will be the possible cause behind this oil poor patients.


result? (b) Rural poor bear the brunt of
(a) The price value of U.S. dollar in government apathy towards their
international market increase. health even as rich among them
(b) The global over supply is increasing avail superior healthcare facilities
oil stockpiles. in nearby cities.

9 Adda247 Publications For any detail, mail us at


Publications@adda247.com
A Complete Book of Logical Reasoning

(c) The government wants the rural (a) The average employee in Mexico
people to opt for city-based City works 42 hours a week, slightly
specialty hospitals in case of serious above the national average of 41.
illness. (b) Every day, a typical employee in
(d) The rural poor are themselves to Mexico spends 12 hours commuting
blame for poor medical facilities as and working.
they go for indigenous medical (c) Mexican cuisine has been

p
systems. recognized by UNESCO as an
(e) None of these ‘intangible cultural heritage’.
(d) Surveys have found that few
23. Fossil fuels continue to dominate global

ou
Mexican children exercise regularly.
energy consumption. Oil accounted for (e) None of these
33% of the total in 2014, gas 30% and
natural gas 24%. Nuclear accounted for 25. The rupee has been losing ground
4% of primary energy consumption, and rapidly against the US dollar in recent
renewables 9%. Renewable energy will times, much to the dismay of all. While
have to displace fossil fuels to a much the Indian currency is down close to 6

gr
greater extent in the future to avoid per cent against the dollar since last
unacceptable climate risks. Diwali in 2014, it has gained almost 9
Unfortunately, the current low prices for per cent against the euro and 7 per cent
oil, gas, and coal are the stumbling against the Japanese yen in the same
period. So some economists say that the
blocks.
Which of the following can properly be
m relative weakness of the rupee against
the dollar need not hurt our economy.
inferred from the passage above?
Which of the following, if true, weakens
(a) The present oil prices are unlikely
the argument above?
to bounce back.
xa
(a) The dollar has been racing higher
(b) Low fossil fuel prices discourage
on hopes of a Fed rate hike next
further innovation in, and adoption
month.
of, cleaner energy technologies. (b) Policy makers are planning to
(c) Low fossil fuel prices would increase non dollar invoicing in
necessitate elimination of fossil external trade and to shift
te

subsidies. borrowings to currencies other


(d) Emissions reduction is a disquieting than dollar.
topic. (c) More than 80 per cent of the
(e) None of these invoicing for the country’s imports
ce

24. Between 2000 and 2012, adult obesity and exports is done in US dollars.
has shown a steady upward trend in (d) In the early part of this year the
Mexico City, where it affected 16% of the Reserve Bank of India had a tough
city population in 2000, 19% in 2006, time keeping the rupee from
and 26% in 2012. The city is in the grip appreciating too much.
(e) None of these
@

of an obesity crisis. While the reasons for


this are far from black and white, close 26. lndia‘s strategy at the Paris Climate
analysis suggests that a sedentary life Change Summit is to work with
could be the main contributing factor. emerging economies and press the
Which of the following, if true developed world to concede that
strengthens the argument? responsibility for cutting carbon

10 Adda247 Publications For any detail, mail us at


Publications@adda247.com
A Complete Book of Logical Reasoning

emissions after 2020 cannot be shared (c) Unlike the international market,
equally by rich and poor nations. India India has to pay to Iran in rupees,
says that her national per capita not in dollars which saves the
emissions are very low at 1.56 tonnes foreign exchange.
carbon dioxide equivalent (about a tenth (d) Iran sells petroleum at lower prices
of some developed nations), and its to increase its sale of the petroleum.
share of cumulative global emissions (e) None of these

p
only 3 per cent.
Which of the following, if true, would 28. Which of the following could be the
most strengthen India’s contention? main reason that India cut down oil
imports from Iran in the year 2012-13?

ou
(a) India has been engaging 60
countries ranging from the least (a) India was able to fulfill the demand
developed countries to the from the local market only.
developed nations to evolve a (b) The USA and the European union
consensus imposed ban on Iran due to nuclear
(b) Green cover in India is larger than programme.
(c) Iran stopped the export of oil for

gr
that of developed countries.
(c) There is a concerted attempt to the first half of the year in order to
bring India under pressure by fulfill its local demand.
painting the country as (d) India has agreements with other
‘obstructionist’. countries to buy petroleum.

poverty through
m
(d) Developed nations seek to eliminate
last-paced
(e) None of these
29. Which of the following conclusions can
economic growth.
be drawn from the given statement?
(e) None of these
I. India wants to import petroleum
xa
Direction (27-29): Study the following from Iran as it is cheaper in Iran.
information and answer the questions that II. India has cut down the import of
follow: petroleum from Iran due to
The petroleum minister of India in September international pressure.
2013 wrote a letter to the Prime Minister of III. India wants to import petroleum
India, saying that as the current account from Iran in order to get out of the
te

deficit has widened and the value of the rupee economic crisis. .
has dwindled, India plans to increase crude IV. India wants to import petroleum
oil imports from Iran so as to save $8.5 billion from Iran in order to fulfill the
in foreign exchange. demand of petroleum in India.
ce

27. Which of the following may be the main (a) Only I (b) Only II and IV
reason that buying oil from Iran saves (c) Only III (d) Only II and III
billions of dollars of foreign exchange for (e) Only IV
India? Directions (30-31): Study the following
(a) India has good diplomatic relations information and answer the given questions.
@

with Iran due to which India gets The US has given its clearest signal yet that it
petroleum from Iran at a lower is planning what president Obama has called
price. a ‘limited, narrow’ military attack on Syria as
(b) The distance of India from Iran is the Syrian government is using weapons of
less due to which the transportation mass destruction on the local people, causing
cost is less. mass genocide.

11 Adda247 Publications For any detail, mail us at


Publications@adda247.com
A Complete Book of Logical Reasoning

30. Which of the following could be the Assumptions:


main reason that the USA is planning an I. The countries across the world do
attack on Syria? not make adequate contributions to
(a) The USA wants to end the the UN.
autocratic rule in Syria. II. The state of the UN is a reflection
(b) Syria has used prohibited weapons, upon the state of the world.
breaking international laws. (a) Only assumption I is implicit

p
(c) The USA has economic interest in (b) Only assumption II is implicit
Syria. (c) Either I or II is implicit
(d) The USA wants to create its own (d) Neither I nor II is implicit
military base is Syria.

ou
(e) Both I and II are implicit
(e) None of these
34. Statement: The economy grew by 4.7
31. Which of the following conclusions can
per cent in the quarter ending December,
be drawn from the given statements?
I. Chemical weapons used in Syria which was slightly better than the
have caused mass genocide in the average of 4.6 per cent during the first
half of the year.

gr
country.
II. The rebel group in Syria wants to Which of the following assumptions is
have control on the country implicit in the above statement? (An
III. The USA has decided to attack Syria assumption is something supposed or
to control the violation of human taken for granted.
rights.
IV. The United Nations is forcing the
m (a) Growth figures are released
quarterly.
USA to take necessary actions. (b) Eight core industries grew by just
(a) Only I (b) Only I and II 1.6%.
(c) Only III (d) Only I, II and III (c) There is an unexpected measure of
xa
(e) Only IV uncertainty in the decision making.
32. Statement: Stock prices go up in the (d) Export grew at a slower pace
morning and come down in the evening, during the three months up to
even though there is no policy decision January.
in the intervening period. (e) GDP growth will figure prominently
te

Assumptions: in the General Elections.


I. The market has its own logic why it 35. By fixing a deadline of one year from the
reacts to a situation in a particular
date of framing of charges of criminal
fashion.
activities for the completion of trial
ce

II. Indian stock markets are extremely


involving the members of parliament
safe; they are not volatile.
and legislative assemblies, the Supreme
(a) Only assumption I is implicit
(b) Only assumption II is implicit Court has once again intervened
(c) Either I or II is implicit effectively to give some credibility to the
(d) Neither I nor II is implicit idea of cleaning the polity.
@

(e) Both I and II are implicit Which of the following can be assumed
from the above passage?
33. Statement: It is a shame that the UN, the (a) This will help in reducing the
only institution we have for global scourge of criminalization of
governance, is laboring under severe politics.
financial pressure at the very moment
when we need it most.

12 Adda247 Publications For any detail, mail us at


Publications@adda247.com
A Complete Book of Logical Reasoning

(b) The latest order will help address 38. The prices of food grains and other
this problem by empowering the ssential commodities have decreased for
trial court to refuse routine the second consecutive week. Which of
adjournments. the following can be a possible
(c) To implement the guidelines, the consequence of the facts stated in the
above statement?
government must set up more
I. The consumer price index will come
speedy trail courts, which will help down considerably.

p
them deliver justice in time. II. People will increase their purchase
(d) The steps taken by the apex court of quantity of essential
will help choose the best people commodities and food-grains.

ou
from their constituency and make III. Govt. will increase its taxes on
the Indian democratic system free essential commodities and food-
from criminalization. grains.
(e) None of these (a) Only I and II (b) Only II and III
(c) Only I and III (d) All I, II and III
36. During pre-harvest kharif seasons, the (e) None of these
government has decided to release vast

gr
quantity of food grains from FCI. 39. Statement-The Reserve Bank of India’s
decision to keep the policy interest rate
Which of the following assumptions are
unchanged, and reaffirm its “neutral”
implicit in the above statement. policy stance, clearly indicates that
(a) There may be a shortage of food policymakers at the central bank are
grains in the market during this
season.
m singularly focussed on their primary
remit of ensuring price stability while
(b) The kharif crop may be able to supporting economic growth.
replenish the stock of FCI. What can be the possible reason for
(c) There may be a demand from the RBI’s decision?
xa
farmers to procure kharif crop (I) The RBI remains concerned that a
immediately after harvest. rate cut at this juncture might need
(d) Both a and b are implicit to be reversed out next year if
(e) All are implicit inflation returns to the upper end of
the 2-6 per cent range."
37. An increasing number of farmers prefer (II) Tension had been brewing between
te

to avail loans from local moneylenders the central bank and the
instead of the banks owing to government for some time over
complicated paperwork involved in several issues.
banks. Which of the following actions (III) RBI, however, on Wednesday
ce

lowered its inflation projection to


should be taken by the government?
the 2-3.5% range for the first half of
(a) Local moneylenders who charge the current financial year and 3.5-
interest rates lower than the banks 4.5% in the second half of the
should be punished. current financial year.
(b) Banks should simplify the (IV) The RBI is likely to wait for the July
@

procedure to avail loans so as to 1 roll-out of GST (Goods and


suit the farmers. Services Tax) and assess the impact
(c) The amount of loan should be of the new tax regime on inflation
increased for the farmers. before moving on rates.
(d) Both (a) and (b) (a) Only I and II (b) Only III
(e) Both (b) and (c) (c) Only I and IV (d) Only III and IV
(e) Only II and III
13 Adda247 Publications For any detail, mail us at
Publications@adda247.com
A Complete Book of Logical Reasoning

40. Statement: Should mutual funds be III. Yes. Many big farmers earn much
brought under stricter Government more than the majority of the
control? service earners and they should be
Arguments: taxed to remove the disparity.
I. Yes, that is one of the ways to (a) Only I is strong
protect the interest of the investors. (b) Only I and II are strong
II. No, stricter Government controls (c) Only II and III are strong

p
are likely to be counterproductive. (d) All are strong
(a) Only assumption I is implicit (e) None of these
(b) Only assumption II is implicit
43. On 15th August 2014 during the speech

ou
(c) Either I or II is implicit
(d) Neither I nor II is implicit iven to the nation PM said that the
(e) Both I and II are implicit economy of India is growing at a very
good speed but we should work hard in
41. Statement: The two countries have order to achieve 'Inclusive Growth'.
signed a fragile pact, but the vital Which of the following assumptions is
sovereignty issue remains unresolved. implicit in the statements?

gr
Assumptions: Assumptions:
I. The two countries cannot have (A) India is growing but the growth of
permanent peace pact. the country is not spread in all
II. The two countries may become sections of the society.
hostile again after a short spell of (B) India is growing but society is not
time.
(a) Only assumption I is implicit
m growing.
(C) The economic and social
(b) Only assumption II is implicit infrastructure of the country is not
(c) Either I or II is implicit growing.
xa
(d) Neither I nor II is implicit (D) The growth of females in the
(e) Both I and II are implicit country is not good.
Direction (42): Each question given below (a) Only A (b) Only A and B
consists of a statement, followed by three or (c) Only A, B and C (d) Only D
four arguments numbered I, II, III and IV. You (e) None of these
te

have to decide which of the arguments is/are 44. India has allowed cotton exports, after
'strong' arguments) and which is/are 'weak' producers demanded lifting of a ban.
arguments) and accordingly choose your Which of the following can be a possible
answer from the alternatives given below assumption in the given statement? (An
each question. assumption is something supposed or
ce

Statement: Should the income generated out taken for granted)


of agricultural activities be taxed? (a) The harvest was better than
Arguments: expected this year.
I. No. Farmers are otherwise suffering (b) It will assess cotton availability
from natural calamities and low every two-three weeks.
@

yield coupled with low (c) It will help produce 1500 lakh
procurement price and their tonne cotton more than previous
income should not be taxed. year.
II. Yes. Majority of the population is (d) Cotton export will become No. 1 in
dependent on agriculture and hence India.
their income should be taxed to (e) None of these
augment the resources.
14 Adda247 Publications For any detail, mail us at
Publications@adda247.com
A Complete Book of Logical Reasoning

45. Basmati rice exporters are gearing up to 46. The RBI has raised the Interest rate
top the market in China which recently ceiling on NRI deposits in foreign
decided to allow imports from India. currencies by up to 3%. Which of the
Which of the following can be a possible
following assumption can be possible in
assumption in the given statement? (An
assumption is something supposed or the given statement? (An assumption is
taken for granted) something supposed or taken for

p
(a) Exporters believe, it may take a granted)
couple of years to develop this new (a) The RBI wants to attract inflows in
market, which largely consumes the view of the falling value of rupee.

ou
glutinous sticky rice.
(b) The Indian banks will be able to
(b) China presents a new market for
Indian exporters, who have seen a offer higher interest rates on NRI
surge in volumes of basmati in deposits in foreign currency.
recent years. (c) RBI wants to create perfect
(c) China’s market has monopoly in competition.

gr
competition market.
(d) The Indian banks have more
(d) Basmati rice is high in demand all
over the world. liquidity.
(e) None of these
m (e) None of these

Economical Based Logical Reasoning : Solutions

1. (b); (3) and (4) will only add to the 4. (b); Nothing about the sources of
xa
Deficit of the country but (1) and income of the State Governments is
(2) will help in decreasing deficit of mentioned in the statement. So, I is
the country. not implicit. Since the Central
Government has directed the State
2. (d); The aim is to go for (3), for which Governments to reduce
(2) is a prerequisite. (1) is only expenditure, so II is implicit.
te

incidental while (4) is beside the Further, since the Central


point. Government has refused any
further grant to the State
3. (e); All the given options may be the Governments for the next six
ce

possible impacts of new 100per months, it is quite clear that state


cent FDI rule except the last one Governments will abide by the
directives of central Government.
because it is mentioned there that
So, III is also implicit.
our domestic country will not get
benefitted from this. It is acceptable Directions (5-6):
@

that there may be exploitation of 5. (d); We have to identify possible reason


regarding context of the passage
domestic resources but from this
and reason which is mentioned in
our domestic country will also get (I) and (III) is nowhere discuss in
benefitted. On internal security above passage. So Only II will be the
issue, it may get affected possible reason of pilferage of
somewhere. money from Indian Banks.
15 Adda247 Publications For any detail, mail us at
Publications@adda247.com
A Complete Book of Logical Reasoning

6. (c); As NPCI is investigating into the global cues. Here, note the
matter and trying to reveal relationship with US economy. So,
preliminary findings and it is also only option (a) can be concluded
mentioned that officials takes from the given option.
almost six months to acknowledge
11. (e); As it is mentioned in the statement
the incidents but information of
that more customers are availing
Ministry of defense is nowhere
banking services through the digital

p
mentioned in the above passage.
medium, i.e. digital banking is
7. (a); Only I is effective course of action becoming popular day by day in the
and not using e-commerce site is world and the customers are

ou
not a solution and hanging the making maximum transactions
culprit is a harsh course of action. online. So, none of the options given
can be concluded as a reason from
8. (b); As by focusing on efficiency the
the statement.
chances of loan recovery will
increase. So, Option (b) can be 12. (e); All the given options may be the

gr
inferred from the above statement possible impacts of new 100per
as it is given that the government is cent FDI rule except the last one
looking to overhaul yearly targets because it is mentioned there that
and control bad debts. our domestic country will not get
benefitted from this. It is acceptable
9.
m
(a); Suppose the annual percentage rise
in real estate values, in property
that there may be exploitation of
domestic resources but from this
values, has been consistent less
our domestic country will also get
than the average annual growth
benefitted. On internal security
rate percentages across all
xa
issue, it may get affected
businesses with properties in
somewhere.
Oakland. Thus, even though
individual business may be 13. (d); The present structure of Indirect
fluctuating, when we average over Taxes is very complex in India.
all businesses, the growth rate is There are so many types of taxes
te

higher for that average than is for that are levied by the Central and
property values. Thus, revenue State Governments on Goods &
from taxes on business income Services. So, there is no doubt that
would grow faster than revenue when all the taxes are integrated, it
ce

from taxes on property. If this will improve the ease of doing


negation is true, it devastates the business ranking of India. So, I is a
mayor’s position, and therefore strong argument. It will definitely
devastates the position that the impact the existing processes,
Urban Planner is taking. Because people and technology but we
the negation is so destructive, the cannot say that it will give negative
@

un-negated version that appears in impact. As on real estate market,


choice (A) only be an assumption. GST will have a significant impact.
IV is true but not directly related to
10. (a); As it is mentioned that gold prices
Indian economy. So, II, III and IV are
are likely to see a gradual decline in
not strong arguments.
the short term on the back of strong

16 Adda247 Publications For any detail, mail us at


Publications@adda247.com
A Complete Book of Logical Reasoning

14. (d); Option (d) will weaken the step and demand for fuel decline in that
taken by the government. It shows time, so all are being positive
that the conditions of bank in rural causes behind dropped out price of
areas are not so good and they oil.
suffer huge losses which weakens
21. (b); Measures to control inflation hurt
the above statement.
investment clearly oppose the
15. (e); If the principal monetary policy is passage. So it weakens the above

p
to be attained –reducing the import discussion.
surplus while resuming economic
growth-per capita consumption will 22. (b); Option (b) can be assumed as it is

ou
have to be frozen. Thus, consumers clear from the given statement that
will have to be persuaded to give up the rural poor did not get health
consumption to further national care facilities and bear the brunt of
economic goals. The assumption is government apathy towards their
that people will be willing to put a health even as rich among them
halt to growth in their standard of avail superior healthcare facilities

gr
living. in nearby cities.

16. (c); Traditional culture is degrading day 23. (b); Since the passage states that the
by day is assumed in the above current low prices of fossil fuels are
statement. the obstacle, it is logical to
17. (e); Clearly, the company intends to
m anticipate an inference about the
viability of spending money on
reduce the gap between income and developing cleaner energy
expenditure by increasing the price technologies. Option (a) is
of its product i.e. by keeping the
irrelevant to the argument as the
xa
expenditure unaltered and
focus of the passage is on
increasing the income only. So, II is
developing clean energy sources.
implicit while I is not. However, the
Option b is an inference that can be
rival companies may or may not
drawn from the information in the
follow the same pursuit. So, III is
passage. The passage gives no
not implicit.
te

indication about the need for


18. (c); White label ATM (WLA) charges subsidies, hence c can be
high fees can be the most suitable eliminated. 'Disquieting' topic in
reason for the struggling to meet option d finds no Support in the
the of targets of NBFC.
ce

passage.
19. (e); If any country decreases fiscal 24. (b); The conclusion is that a sitting or
deficit than it is good for their inactive life could be the cause of
economy. It will become less, if obesity. If an average employee
currency improves their position, if spends 12 hours a day commuting
revenue deficit decrease and
@

and working, he is unlikely to get


economy boost, So all except A may time for physical exercise. Even the
be reason for above result. 12 hours spent on working and
20. (d); Crude oil price dropped in U.S. then commuting are likely to be
it may be possible that currency sedentary in nature. Thus, Option
value increase, global over supply (b) is the one that strengthens the

17 Adda247 Publications For any detail, mail us at


Publications@adda247.com
A Complete Book of Logical Reasoning

argument. Working 42 hours a 28. (e); The statement does not provide
week is not as strong as working information regarding the import of
and commuting 12 hours a day, so oil from Iran by India in 2012-13.
option (a) can be rejected. (c) talks
about Mexican cuisine which does 29. (e); The issue of import arises only
not relate to a sedentary life. Option when there is demand. Hence, IV
(d) is about children, but the follows. Rest nothing is mentioned

p
argument relates to adults. about the prices of Oil in Iran and
economic crisis of India and further
25. (c); It should be the one that International pressure on India.
contributes to the weakening of the

ou
Hence I, II and III does not follow.
economy. Option (a) gives a reason
for the strengthening of the dollar; Directions (30-31)
this point doesn’t weaken the 30. (b); Syria has used weapons of mass
argument. (b) is only a proposal, so destruction against the innocent
it is unlikely to weaken the people.
argument. Option (c) is a point that

gr
can weaken the argument: if more 31. (c); The weapons of mass destruction
than 80 per cent of the invoicing for which were used in Syria has
the country’s imports and exports is caused mass genocide, and, in order
done in US dollars, any decline in to control the violation of human
value of rupee against the dollar
will affect the economy. Option (d)
m rights, the USA is planning to attack
Syria.
is totally irrelevant to the argument.
32. (a); I is implicit when you negate
26. (b); The argument is based on the external policies – the factor that
xa
principle that cutting emissions in usually makes the difference. II is
future should be commensurate to not implicit. In fact, the “go up” and
emissions by the country. Any fact “come down” activities point to the
that points to reduced emissions by contrary.
India can be the answer. Engaging
60 countries to evolve a consensus 33. (b); The reason for the UN’s financial
te

doesn’t point to reduced emissions pressure can’t be assumed. Hence I


by India, so option A is incorrect. A is not implicit. II is implicit; that is
large green cover has the potential why “it is a shame”.
to absorb green house gases, so
ce

34. (a); A quarter is the common


option (b) can strengthen the
denomination of the periods
argument. Options (c) and (d) are
totally unrelated to reduced mentioned here so only a can be
emissions by India. assumed rest b, c, d and e can’t be
assumed.
Directions (27-29):
@

27. (c); Unlike the international market, 35. (d); The passage talks about cleaning
India has an agreement with Iran, the Indian politics from
under which India needs to pay in criminalization choice (a) cannot be
rupees rather than in dollars, which considered completely. But (d) can
in turn saves the foreign exchange be considered as it substantiates
reserves of the country. the conclusion fully.

18 Adda247 Publications For any detail, mail us at


Publications@adda247.com
A Complete Book of Logical Reasoning

36. (d); As the government has decided to 40. (a); In government control money of
release vast quantity of food so it is investors will be in safe hand.
clear that there is a demand which
41. (b); From the fact that the present pact
may cause shortage and also by
is not a lasting one, the possibility
Kharif crop the stock of
of a permanent pact cannot be ruled
government’s FCI will be able to get
out. So, I is not implicit. The
filled.
statement mentions that the

p
37. (b); Only (b) is implicit as by simplifying present pact is a 'fragile' one and
the procedure, more number of the vital sovereignty issue still
farmers will prefer to take loans remains unresolved. So the same

ou
from banks as it more secure issue may rise again in the future.
medium. (a) is not appropriate as Thus, II is implicit.
charging less interest rate is not
42. (d); Clearly, the profit and losses both
against law and (c) is also not
are to bear by the farmers which
effective course of action.
occur due to natural calamities. So, I

gr
38. (a); As the price continue to decrease so is strong. Besides, a major part of
there is a possibility that people the population is dependent on
will increase the purchase of agriculture and such a large section,
essential commodity and also if taxed even with certain
consumer price index is directly concessions, would draw in huge

both (I) and (II) follow.


m
related to the price of commodity so funds, into the government coffers.
Also, many big landlords with
substantially high incomes from
39. (c); For I-True, because It can be a
agriculture are taking undue
reason of this constant policy
advantage of this benefit. So, both
xa
interest rate as It might be possible
arguments II and III hold strong.
that if RBI cuts the rate now then it
will affect the next year inflation 43. (a); Inclusive growth means the growth
targets as it is given in the which is not limited only to a
statement that RBI is now focusing certain class of people but spreads
on price stability while supporting to all sections of the society.
te

economic growth.
44. (e); In all the given statements there is
For II- False, because rift between
additional information which
central bank and government
generally can’t be assumed hence
cannot be a reason of unchanged
none of the given statements is a
ce

policy rates.
possible assumption.
For III- False, because it is only the
data of inflation targets for I and II 45. (e); All the statements have the
half of current financial year. additional information which can’t
For IV- True, because it is possible be generally assumed.
that RBI is waiting for GST rollout
@

46. (b); From the statement we can


to see its impact on present generally assume that the Indian
economic condition of the country banks have capacity to pay higher
and then accordingly the policy interest rate on NRI deposits.
rates may be changed by it.

19 Adda247 Publications For any detail, mail us at


Publications@adda247.com
A Complete Book of Logical Reasoning

p
ou
gr
m
xa
te
ce
@

1 Adda247 Publications For any detail, mail us at


Publications@adda247.com
A Complete Book of Logical Reasoning

General Issue Based Logical


09 Reasoning
Introduction: General Issue based logical questions are formed from the daily life issue or

p
problems faced by people. All these types of questions are based on the daily life incidents,
problems of day to day life of people, basic needs and requirement of people etc…So the thing
that should be remembered while solving it is that do not mix your own experience or emotions

ou
and be bit more concentrated towards the statement given in the question. A human life is full of
experiences, surprises, shocks and problems and all these are molded to frame a question.
These types of questions are usually asked in all exams as it seems to be quiet easy to solve but
actually theses are bit tricky one and one should be careful while solving it.
Syntax of the General Issue Based Logical Question

gr
Example-1: A report that many apples contain a cancer-causing preservative called Alar
apparently had little effect on consumers. Few consumers planned to change their apple-buying
habits as a result of the report. Nonetheless, sales of apples in grocery stores fell sharply in
March, a month after the report was issued.
Which of the following if true, best explains the reason for the apparent discrepancy described
above?
m
(a) In March, many grocers removed apples from their shelves in order to demonstrate
concern about their customers’ health.
(b) Because of a growing number of food-safety warnings, consumers in March were
xa
indifferent to such warnings.
(c) The report was delivered on television and also appeared in newspapers
(d) The report did not mention that any other fruit contains Alar, although the preservative is
used on other fruit.
(e) Public health officials did not believe that apples posed a health threat because only minute
traces of Alar were present in affected apples.
te

Brief for the question: The given question states that apples contain cancer causing
preservative Alar so few customers plan to change their apple eating habit. But the sale of apple
fall after this. So we have to find the reason behind it.
ce

Solution
Explanation-1: Ans. (a)
(a) Correct. This would be sufficient to explain why sales fell even though did not plan to stop
buying apples.
(b) This point explains why consumers did not intend to change their apple-buying habits-but
@

not why sales fell.


(c) How consumers may have heard about the report throws no light on the discrepancy
between their response and the decline in sales.
(d) Fruits other than apples are not a part of the discussion.
(e) The health officials’s opinion, if indeed known to consumers, would likely lead to stable
apple sales- so this point does not explain the decline in apple sales.

2 Adda247 Publications For any detail, mail us at


Publications@adda247.com
A Complete Book of Logical Reasoning
Example-2: The growing popularity of computer-based activities was widely expected to result
in a decline in television viewing, since it had been assumed that people lack sufficient free time
to maintain current television-viewing levels while spending increasing amounts of free time on
the computer. The assumption, however, is evidently false: In a recent mail survey concerning
media use, a very large majority of respondents who report increasing time spent per week
using computers report no change in time spent watching television.
Which of the following would it be most useful to determine in order to evaluate the argument?
(a) Whether a large majority of the survey respondents reported watching television regularly

p
(b) Whether the amount of time spent watching television is declining among people who
report that they rarely or never use computers
(c) Whether the type of television programs a person watches tends to change as the amount

ou
of time spent per week using computers increases
(d) Whether a large majority of the computer owners in the survey reported spending
increasing amounts of time per week using computers
(e) Whether the survey respondent’s reports of time spent using computers included time
spent using computers at work
Brief for the question: The given question states about the comparison of time spent on

gr
computer versus time spent on television. It has been assumed that people start spending more
time on computers which reduced their time spending on television. But the report says that the
people start spending time on computer without altering the time spend on watching television.
Solution------
Explanation-2: Ans.(e)
m
(a) The argument is concerned with the change in the amount of television watched by those
whose computer use increased, so whether the survey’s respondents reported watching
television regularly is irrelevant.
(b) The argument is concerned with the change in the amount of television watched by those
whose computer use has increased, so it does not matter whether the amount of time spent
xa
watching television among people who do not use computers is declining, remaining the
same, or increasing.
(c) The argument is concerned with the amount of television watched by those whose
computer use has increased, not the type of television programs such a person does or does
not watch.
(d) The argument here is concerned with people who report spending increasing amounts of
te

time on the computer, what computer owners do is a separate question.


(e) Correct. This statement properly identifies something that would be useful to know in
evaluating the argument.: whether the survey data included time spent using computers at
work – if it did, this would make the data misleading as evidence for the argument’s
conclusion.
ce

Points to Remember----
I. General Issue based logical questions are usually referenced from the daily life activities
so these are quite easy to understand but at the same time require smart approach.
II. These questions mostly raise a general issue or problem, so you should first try to
understand the question completely and then go on with answering it.
@

III. Questions are complicated only when you make it so do not hesitate or scare to attempt
such question.
IV. The thinking and perception of every person is different, but you are expected to not to
put on your own thoughts in the question and only try to understand the thought process
of the examiner.
V. The conclusion and inference can’t be assumed it should be directly or indirectly given in
the statement.

3 Adda247 Publications For any detail, mail us at


Publications@adda247.com
A Complete Book of Logical Reasoning

Exercise
1. The percentage of family income spent Courses of action:
on entertainment has remained almost I. The question should be raised to
the same over the past twenty years – find the number of persons affected
about twelve per cent. When new forms by water-borne diseases during

p
of entertainment become popular, they monsoon period.
do not expand this percentage; instead, II. The Government should
they take consumer spending away from disseminate adequate information

ou
other forms of entertainment. Therefore, regarding the pure drinking water
film producers have observed the video to people.
boom with concern, knowing that every III. All the hospitals in the city should
dollar spent on rental of videos means a
be equipped properly for the
dollar less spent on movie theatre
treatment of patients during
admissions.
monsoon period.
Which of the following, if true, most

gr
forcefully undermines the argument of the (a) All follow
passage above? (b) Only I and II follow
(a) The cost of renting a video is (c) Only II and III follow
generally substantially less than the (d) Only I and III follow
price of a movie theatre admission. (e) None follows
(b) Most film producers receive a
m 3. Statement: Some strains of mosquito
portion of the income from the sale have become resistant to chloroquine —
of video rights to their movies. the widely used medicine for malaria
(c) Fears of some film producers that patients.
xa
videos would completely supersede
Courses of action:
movies have not come to pass.
I. Selling of chloroquine should be
(d) Since the start of the video boom,
stopped.
money spent on forms of
entertainment other than videos II. Researchers should develop a new
and movies has dropped. medicine for patients affected by
te

(e) Some movies that were such mosquitoes.


unprofitable when shown in III. All the patients suffering from
theatres have become successful malaria should be checked for
when released in video form. identification of causal mosquito.
ce

(a) None follows


Directions (2-3): In each question below is (b) Only I and III follow
given a statement followed by some courses
(c) All follow
of action. You have to assume everything in
(d) Only II and III follow
the statement to be true and on the basis of
(e) None of these
the information given in the statement, decide
@

which of the suggested courses of action Direction (4). In the question given below
logically follow (s) for pursuing. consists of a statement, followed by two
2. Statement: There is a considerable arguments numbered I and II. You have to
increase in the number of persons decide which of the arguments is a ‘strong’
affected by water-borne diseases during argument and which is a ‘weak’ argument.
monsoon period. Give answer:

4 Adda247 Publications For any detail, mail us at


Publications@adda247.com
A Complete Book of Logical Reasoning

(a) If only argument I is strong. 6. Which of the following conclusions can


(b) If only argument II is strong. be drawn from the above statements?
(c) If either I or II is strong. I. Growing population in India had
(d) If neither I nor II is strong. divided the land into small
(e) If both I and II are strong. fragments.
II. Small land holdings are impacting
4. Statement: Should reputed private
the farmers’ incomes.
driver training schools be allowed to

p
III. Problems of environment are
issue driving licenses?
overlooked by the people.
Arguments:
IV. Lack of effective storage facilities in
I. Yes, This will enable more and
the country is increasing the

ou
more people to get driving licenses.
problems of agriculture in India.
II. No, This will increase the
(a) Only I and II (b) Only I, II and III
congestion of the vehicles on the
(c) Only II (d) Only II, III and IV
road and will result in traffic jams
(e) All of these
too.
Directions (7-8): Each question given below
Directions (5 –6): Read the following

gr
consists of a statement, followed by two
information carefully and answer the given
arguments numbered I and II. You have to
questions.
decide which of the arguments is a ‘strong’
India has a huge population of over one
argument and which is a ‘weak’ argument.
billion and it is increasing at a very fast rate.
Give answer:
According to 2011 census, the overall density
of population is 382 people per square km.
m (a) If only argument I is strong.
(b) If only argument II is strong.
This is likely to increase further in future. This
(c) If either I or II is strong.
has created great demand for land. Every bit
(d) If neither I nor II is strong.
of land has been brought under the plough.
(e) If both I and II are strong.
xa
Even the hill slopes have been cut into
terraces for cultivation, which on the other 7. Statement: Should system of offering
hand, has disturbed the natural ecosystem of jobs only to the wards of government
the country and given rise to natural employees be introduced in all
calamities. The pressure of increasing government offices in India?
population and the practice of dividing land Arguments:
te

equally amongst the heirs has caused I. No. It denies opportunity to many
excessive sub-divisions of farm holdings. deserving individuals and
Consequently, the holdings have become government may stand to lose in
smaller and fragmented. The small size of the long run.
ce

holdings make farming activity uneconomical II. No. It is against the principle of
and leads to social tension, violence and equality. Does government not owe
discontentment. its responsibility to all its citizens?
5. Which of the following, according to the 8. Statement: Is paying ransom or
passage could be the main reason(s) of agreeing to the conditions of kidnappers
@

the problems of agriculture in India? of political figures, a proper course of


(a) Small and fragmented land of action?
holding of the farmers Arguments:
(b) Lack of adequate irrigation facilities I. Yes. The victims must be saved at
(c) Increase in soil erosion all cost.
(d) Lack of storage of food grains II. No. It encourages the kidnappers to
(e) All of these continue their sinister activities.

5 Adda247 Publications For any detail, mail us at


Publications@adda247.com
A Complete Book of Logical Reasoning

Directions (9): In the given question, a 11. Statement: Should there be a complete
statement is given followed by some ban on manufacture and use of
conclusions. Without resolving anything firecrackers?
yourself, choose the conclusion which Arguments:
logically follows from the given statement. I. No. This will render thousands of
9. Statement: Every man should have his workers jobless.
identity card with him. That card should II. Yes. The firecracker manufacturers

p
mention his blood group, complete use child labour to a large extent.
address and telephone number for III. Yes. This will be a concrete step to
contact, in case, some serious accident reduce noise and air pollution.

ou
takes place. IV. No. Use of firecrackers makes
Conclusions: certain special occasions more
(a) Blood cannot be transfused until its lively and joyful.
group is mentioned in the card. (a) Only I and II are strong
(b) Identity Card contains all necessary (b) Only I and III are strong
information. (c) Only III and IV are strong
(c) In case of emergency, he may forget

gr
(d) Only I, II and III are strong
his address and may need the card (e) Only I, III and IV are strong
to contact his house.
(d) None is supposed to forget his Directions (12): In the given question, a
phone number under any statement is given followed by some
circumstances. assumptions. An assumption is something
m
(e) When the seriously injured person supposed or taken for granted. You have to
is helpless to tell his blood group, consider the statement and the following
this information would suffice to assumptions and decide which of the
indicate the required blood group. assumptions is implicit in the statement.
xa
Direction (10– 11): Each question given 12. Statement: The civic authority has
below consists of a statement, followed by decided that all the factories located
three or four arguments numbered I, II, III inside the city limits be shifted outside to
and IV. You have to decide which of the reduce the level of environmental
arguments is/are ‘strong’ argument (s) and pollution in the city.
which is/are ‘weak’ argument(s) and
te

Assumptions:
accordingly choose your answer from the I. The pollution level in the city in
alternatives given below each question.
future may reduce after these
10. Statement: Should women be given factories are shifted outside the city
equal opportunity in the matter of limit.
ce

employment in every field? II. Enough usable land is available


Arguments: outside the city limit for these
I. Yes. They are equally capable. factories.
II. No. They have to shoulder only III. Many of these factories may shift to
household responsibilities. some other smaller towns to
@

III. Yes. They should also go into the remain profitable.


outside world for opportunities. (a) Only I is implicit
(a) Only I is strong (b) Only I and II are implicit
(b) Only I and II are strong
(c) Only II is implicit
(c) Only II and III are strong
(d) Only II and III are implicit
(d) Only I and III are strong
(e) None of the above
(e) All are strong

6 Adda247 Publications For any detail, mail us at


Publications@adda247.com
A Complete Book of Logical Reasoning

Directions (13–14): Each question given Directions (15–16): In each question below
below consists of a statement, followed by is given a statement followed by two
three or four arguments numbered I, II, III conclusions numbered I and II. You have to
and IV. You have to decide which of the assume everything in the statement together
argument(s) is/are ‘strong’ argument(s) and and decide which of them logically follows
which is/are ‘weak’ argument(s) and beyond a reasonable doubt from the
accordingly choose your answer from the information given in the statement.

p
alternatives given below each question. Give answer
13. Statement: Should all the youngsters (a) If only conclusion I follows
below 21 years of age be disallowed (b) If only conclusion II follows

ou
from going to a beer bar? (c) If either conclusion I or conclusion II
Arguments: follows
I. No. It is not correct to prevent (d) If neither conclusion I nor conclusion II
matured youngsters above 18 years follows
of age who can vote, from having (e) If both conclusions I and II follow
fun. 15. Statements: The government run

gr
II. Yes. The entry fee to such pubs company had asked its employees to
should also be hiked. declare their income and assets but it
III. No. There is no such curb in has been strongly resisted by employees
western countries. union and no employee is going to
IV. Yes. This will help in preventing
youngsters from getting into bad
m declare his income.
Conclusions:
company and imbibing bad habits.
I. The employee of this company does
(a) Only I is strong
not seem to have any additional
(b) Only I and III are strong
undisclosed income besides their
xa
(c) Only III and IV are strong
salary
(d) Only I and IV are strong
II. The employee’s union wants all
(e) None is strong
senior officers to declare their
14. Statement: Should there be compulsory income first.
military training for each college student
16. Statement: The official Secrets Act
te

in India?
Arguments: (O.S.A.) enacted by the XYZ government
I. Yes. This is the only way to build a during the war seems to be the source of
strong and powerful nation. much corruption in the country ‘P’.
II. No. Compulsion always leads to Conclusions:
ce

repulsion. I. The official Secrets Act has to be


III. Yes. This is the only way to impart abolished immediately to stop
discipline to the young generation. corruption in country ‘P’.
IV. No. This goes against the basic II. The XYZ government wanted to
democratic right of an individual to encourage corruption in the
@

choose his/her own programs. government offices.


(a) Only I and III are strong 17. It has been reported in many leading
(b) Only II and IV are strong newspapers that the current year’s
(c) Only I and IV are strong monsoon may be below the expected
(d) Only I, III and IV are strong level as many parts of the country are
(e) All are strong still not getting adequate rainfall.

7 Adda247 Publications For any detail, mail us at


Publications@adda247.com
A Complete Book of Logical Reasoning

Which of the following can be a possible 19. Statement: It is desirable to put the
fallout of the above situation? child in school at the age of 5 or so.
(a) People from those affected areas Assumptions:
with less rainfall may migrate to I. At that age the child reaches
urban areas appropriate level of development
(b) Government may announce ex- and is ready to learn.
gratia payment to all the farmers II. The schools do not take admission
of children after six years of age.

p
affected in these areas
(a) Only assumption I is implicit
(c) Government may declare these
(b) Only assumption II is implicit
areas as drought affected areas
(c) Either I or II is implicit

ou
(d) People may blame the Government (d) Neither I nor II is implicit
and agitate for not getting adequate (e) Both I and II are implicit.
water for cultivation
(e) None of the above 20. Statement: "I have not received
telephone bills for nine months in spite
18. There has been a spate of rail accidents of several complaints" - A telephone
in India in the recent months killing customer's letter to the editor of a daily

gr
large numbers of passengers and newspaper.
injuring many more. This has raised Assumptions:
serious doubts about the railway’s I. Every customer has a right to get
capability of providing safety to bills regularly from the telephone
travelers. company.
Which of the following statement
m II. The customer's complaints point to
defect in the services which are
substantiates the views expressed in the
above statement? expected to be corrected.
(a) Indian Railways has come to be (a) Only assumption I is implicit
(b) Only assumption II is implicit
xa
known to provide best passenger
comfort in the recent years (c) Either I or II is implicit
(d) Neither I nor II is implicit
(b) People have no option other than
(e) Both I and II are implicit
travelling by rail over long
distances 21. Statement: The education of a student at
(c) The railway tracks at many places college level, not taking into account
te

have been found to be stressed due maintenance expenses, costs four


to wear and tear in the recent times hundred rupees a year. College
(d) Local residents are always the first education is thus drawing heavily upon
to provide a helping hand to the the national resources of an
ce

passengers in the event of such impoverished community. So college


education should be restricted to a
disasters.
brilliant few.
(e) None of the above
Assumptions:
Directions (19-21): In each question below I: Our resources are very limited.
is given a statement followed by two II: Only a few students should be
@

assumptions numbered I and II. An admitted to the colleges.


assumption is something supposed or taken (a) Only assumption I is implicit
for granted. You have to consider the (b) Only assumption II is implicit
statement and the following assumptions and (c) Either I or II is implicit
decide which of the assumptions is implicit in (d) Neither I nor II is implicit
(e) Both I and II are implicit
the statement.

8 Adda247 Publications For any detail, mail us at


Publications@adda247.com
A Complete Book of Logical Reasoning

Directions (22-23): Each question given Statement: Should religion be taught in our
below consists of a statement, followed by schools?
two arguments numbered I and II. You have Arguments:
to decide which of the arguments is a 'strong'
I. No. Ours is a secular state.
argument.
Give answer: II. Yes. Teaching religion helps inculcate
moral values among children.
22. Statement: Should all the drugs III. No. How can anyone take such a step

p
patented and manufactured in Western
countries be first tried out on sample when we want the young generation to
basis before giving license for sale to fulfill its role in the 21st century.

ou
general public in India? (a) All are strong
Arguments: (b) None is strong
I: Yes. Many such drugs require (c) Only I is strong
different doses and duration for (d) Only II is strong
Indian population and hence it is
(e) Only I and III are strong
necessary.

gr
II: No. This is just not feasible and Direction (25-28): Study the following
hence cannot be implemented. information carefully to answer the questions
(a) Only argument I is strong given below.
(b) Only argument II is strong
(c) Either I or II is strong Statement: Some preferences in India are
(d) Neither I nor II is strong
(e) Both I and II are strong
m based on patriarchal customs and traditions
and the patrilineal form of inheritance, where
23. Statement: Should there be a maximum the inheritance is obtained through the
limit for the number of ministers in the common male ancestors and the sons inherit
xa
Central Government? more than the daughters. Among the Hindus,
Arguments:
it is largely believed that only a son can light
I: No. The political party in power
should have the freedom to decide the funeral pyre and offer prayers to
the number of ministers to be ancestors, that the son remains a part of the
appointed. family whereas the daughter becomes a part
te

II: Yes. The number of ministers of another family, that Kanyadaan (giving
should be restricted to a certain away of a daughter in marriage) is a
percentage of the total number of necessary spiritual obligation, and that only a
seats in the parliament to avoid
son can provide old-age security.
ce

unnecessary expenditure.
(a) Only argument I is strong 25. Which of the following can be an
(b) Only argument II is strong assumption in the above passage?
(c) Either I or II is strong
(a) India has a patriarchal society.
(d) Neither I nor II is strong
(e) Both I and II are strong (b) In Indian families, females are
@

persecuted.
Directions (24): The question given below (c) All the daughters are not given
consists of a statement, followed by three
arguments numbered I, II and III You have to away in marriage.
decide which of the arguments is/are ‘strong’ (d) Those families which beget no son
argument (s) and accordingly choose your face social opprobrium.
answer from the alternatives given below. (e) None of these

9 Adda247 Publications For any detail, mail us at


Publications@adda247.com
A Complete Book of Logical Reasoning

26. Which of the following statements (d) The Hindus are more traditional
weakens the idea expressed in the than the people belonging to other
above passage? religions.
(a) Today many girls in India are (e) None of these.
opting for love marriage.
Directions (29-31): Each question given
(b) Women in India are largely
below consists of a statement, followed by
uneducated and backward in
three or four arguments numbered I, II, III

p
outlook.
and IV. You have to decide which of the
(c) Girls are highly educated and they
arguments is/are ‘strong’ argument (s) and
have demolished most of the
accordingly choose your answer from the

ou
traditional myths that girls are
inferior to boys. alternatives given below each question.
(d) There are still many pockets of 29. Statement: Should there be complete
India where discrimination against ban on Indian professionals seeking jobs
girls is rampant. elsewhere after getting their education
(e) None of these in India?

gr
27. Which of the following can be an Arguments:
inference from the above passage? I. Yes. This is the only way to sustain
(a) The traditions of the Hindus are present rate of technological
very discriminatory against women development in India.
as compared to what is in practice II. No. The Indians settled abroad send
in other religions.
(b) Kanyadaan is a mandatory process
m huge amount of foreign exchange
and this constitutes a significant
for all Hindu households. part of foreign exchange reserve.
(c) There are many customs and III. No. The practical knowledge gained
by Indians by working in other
xa
traditions among the Hindus which
weaken the position of women in countries help India to develop its
India. economy.
(d) Women are themselves responsible (a) None is strong
for their inferior position in the (b) All are strong
Indian society. (c) Only I and II are strong
te

(e) None of these (d) Only III is strong


(e) Only II and III are strong
28. Which of the following statements
strengthens the idea expressed in the 30. Statement: Should children be
above passage? prevented completely from watching
ce

(a) Despite all legal efforts, the position television?


of women in Indian society is very Arguments:
low. I. No. We get vital information
(b) Besides traditionally bestowed low regarding education through
position to women in India, khap television.
II. Yes. It hampers the study of
@

panchayats and frequent fatwas


issued by Ulemas add to the children.
discomfiture of women. III. Yes. Young children are misguided
(c) While the north-eastern and by certain programmes featuring
southern parts of India are sex and violence.
matrilineal, rest of the country is IV. No. This is the only way to educate
patrilineal. the masses.

10 Adda247 Publications For any detail, mail us at


Publications@adda247.com
A Complete Book of Logical Reasoning

(a) Only, I, II and III are strong Conclusions:


(b) Only I is strong (a) TV viewing promotes criminal
(c) Only I, II and IV are strong behavior.
(d) Only I and II are strong (b) TV viewers are most likely to be
(e) Only I, III and IV are strong victimized than others.
(c) People should not watch TV.
31. Statement: Should admission to all (d) TV promotes a feeling of helpless

p
professional courses be made on the vulnerability in viewers.
basis of past academic performance (e) None of these
rather than through entrance tests? 33. The mushrooming of business schools in

ou
Arguments: Gujarat is a cause for shortage of faculty
I. Yes. It will be beneficial for those with Ph.D qualification. In addition, the
candidates who are unable to bear higher pay and generous fringe benefits
the expenses of entrance tests. given by industry has encouraged
II. Yes. Many deserving candidates qualified people to not seek academic
securing high marks in their positions.

gr
qualifying academic examinations Which of the following statements, if
true, would tend to WEAKEN the
do not perform well on such
argument?
entrance tests.
(a) The average salary for industry
III. No. The standard of examinations positions in Gujarat is more than
and assessment conducted by
different Boards and Universities
m the average salary for faculty
positions in some business schools
are not comparable and hence there by around 30%.
is a need to conduct entrance tests (b) The average salary for industry
xa
to calibrate them on a common positions in Gujarat is less than the
yardstick. average salary for faculty positions
(a) Only I and II are strong in a top business school by around
(b) Only II and III are strong 30%.
(c) The average salary for recent Ph.D.
(c) Only I and III are strong
graduates in the industry is 20%
te

(d) Only III is strong


higher than that in academics.
(e) All are strong (d) The rate of growth of salaries for
Directions (32): In the given question, a the industry positions has been
statement is given followed by some higher than the rate of growth of
ce

conclusions. Without resolving anything salaries for academic positions for


the past three years.
yourself, choose the conclusion which
(e) None of the above.
logically follows from the given statement.
Directions (34-35): Read the following
32. Statement: Television convinces information and the statements I, II, III, IV, V
viewers that the likelihood of their
@

and VI given below it carefully and answer the


becoming the victim of a violent crime is questions.
extremely high; at the same time by its Scientists are worried that using very high
very nature TV persuades viewers to concentration of repellents in fogging agents
passively accept whatever happens to is rendering mosquitoes more and more
them. resistant. This is making the repellent
ineffective over time.
11 Adda247 Publications For any detail, mail us at
Publications@adda247.com
A Complete Book of Logical Reasoning

I. Resistance development among (c) Lions protect their own territories,


mosquitoes is higher than all other whereas men capture other men’s
insects. This shows that mosquitoes are territories.
more adoptive then other insects. (d) Nilgai and Cheetal stay together,
II. If the increase in concentration of whereas men of one race dominate
repellent does not stop, if would end up another.
becoming so toxic that it would (e) None of the above.

p
endanger the health of humans as well as 37. The increase in the number of
the ecosystem as a whole. newspaper articles exposed as
III. Places where increased concentration of fabrications serves to bolster the

ou
mosquito repellents are used, mosquito contention that publishers are more
control is more effective than in other interested in boosting circulation than in
areas. printing the truth. Even minor
IV. While regular concentration mosquito publications have staff to check such
repellents must be sold at a subsidies obvious fraud.
price on government orders, the one Which of the following may be the

gr
with high concentration only helps make assumption of the given argument?
good profits. (a) Newspaper stories exposed as
V. The government should make a policy fabrication are a recent
regarding the limits to concentration of phenomenon.
mosquito repellents and ensure its strict (b) Everything a newspaper print must
implementation.
VI. Development of resistance against
m be factually verifiable.
(c) Fact checking is more comprehend-
repellent drugs is naturally present in sive for minor publications that for
mosquitoes and is not dependent the major ones.
amount of repellent used. (d) The publishers of newspapers are
xa
the people who decided what to
34. Which of the given statements weakens print in their newspapers.
the given information? (e) None of the above.
(a) I (b) IV
(c) V (d) II 38. The rate of violent crime in this state is
(e) VI upto 30% from last year. The fault lies
te

entirely in our system of justice. Recently


35. Which of the following represents a our judges’ sentences have been so
consequence of the increased lenient that criminals can now do almost
concentration of repellents? anything without fear of a long prison
(a) VI (b) IV
ce

term.
(c) I (d) II The argument above would be weakened
(e) III if it were true that
36. Civilization has taught us to be friendlier (a) 85% of the other States in the
towards one another. nation have lower crime rates than
Which of the following would strengthen does this state.
@

the given argument? (b) White-collar crime in this state has


(a) Cats are loyal to their children, also increased by over 25% in the
whereas men are loyal to their last year.
(c) 35% of the police in this state have
communities.
been laid off in the last year due to
(b) Elephants move in a herd, whereas
budget cuts.
men live in nuclear families.

12 Adda247 Publications For any detail, mail us at


Publications@adda247.com
A Complete Book of Logical Reasoning

(d) Polls show that 65% of the 40. Statement: A major railway accident
population in this state opposes involving a mail train was averted due to
capital punishment. the presence of mind of one signal man
(e) None of the above. at a wayside cabin.
Courses of action:
39. During the SARS days, about 23,500
I. The railway track for at least 50 km
doctors who had treated SARS sufferers
should be cleared off any traffic
died and about 23,670 doctors who had

p
ahead of all the mail trains.
not engaged in treatment for SARS
II. The railway signaling systems
sufferers died. On the basis of those
should immediately be made
figures, it can be concluded that it was

ou
automatic.
not much more figures, it can be III. The signal man should be rewarded
concluded that it was not much more so as to encourage others.
dangerous to participate in SARS (a) All follow (b) None follows
treatment during the SARS day than it (c) I and II follow (d) II and III follow
was not to participate in SARS treatment. (e) None of these
Which of the following would reveal

gr
most clearly the absurdity of the 41. Statement: Chewing- tobacco has many
conclusion drawn above? benefits. However, the primary benefits
(a) Counting deaths among doctors occur in the area of mental health. The
who had participated in SARS habit originates in a search for
treatment in addition to addition to contentment. The life expectancy of our
deaths among doctors who had not
m people has increased greatly in recent
years. It is possible that the relaxation
participated is SARS treatment.
(b) Expressing the difference between and contentment and enjoyment
the numbers of deaths among produced by tobacco chewing has
xa
doctors who had treated SARS lengthened many lives. Hence, chewing
sufferers and doctors who had not tobacco is beneficial.
treated SARS suffers as a Which of the following, if true, would
percentage of the total number of weaken the above conclusion?
deaths. (a) The evidence cited in the statement
(c) Separating deaths caused by covers only one example of the
te

accidents during the treatment to effects of tobacco chewing.


SARS suffers from deaths caused by (b) The Government earns millions of
infect of SARS suffers. rupees from the sales of chewing
(d) Comparing death rates per tobacco.
ce

thousand members of each group (c) There is as yet no statistical


rather than comparing total evidence to prove a link between
number of deaths. chewing and longevity.
(e) None of the above. (d) All of the above.
(e) None of the above.
Directions (40): In the given question below,
@

a statement is given followed by some courses 42. Statement: A very large number of
of action. You have to assume everything in technically qualified young Indians are
the statement to be true and on the basis of coming out of colleges every year though
the information given in the statement, decide there are not enough opportunities for
which of the suggested courses of action them to get gainful employment.
logically follow (s) for pursuing. Which of the following contradicts the
views expressed in the above statement?
13 Adda247 Publications For any detail, mail us at
Publications@adda247.com
A Complete Book of Logical Reasoning

(a) Technically qualified persons are 44. Statement: A fundamental faux in the
for superior to those with standard admission of elite Indian engineering
degrees like BA/BSc/BCom etc. colleges has resulted in 3,200 spots lying
(b) The Government has not done vacant at the end of the admission
effective perspective planning for process.
engaging technically qualified Which of the following may be a
personal while authorizing the probable reason for the seat lying

p
setting up of technical colleges. vacant in the college?
(c) All huge gap exists between the (a) A set of rules framed by the joint
level of competence of technically seat allocation authority

ou
qualified graduates and (b) Seat once reserved, not allowed to
requirements of the industry. be withdrawn by the candidates
(d) Majority of the technically qualified after the counseling
persons are migrating from India to (c) Students only looking for admission
developed countries for better to top IITs
opportunities. (d) Less job opportunity in the market

gr
(e) None of the above. with courses offered by the premier
43. Statement: Most car manufacturers institutes
have marginally reduced the price of (e) Ragging by the seniors during the
their products despite higher input costs first year of the course
and increased Government duties and
m Directions (45–46): Read the following
have promised to keep the prices at the information carefully and answer the given
present level for at least the next couple questions:
of months. The crime against woman in India has rapidly
xa
Which of the following may be best increased in the last few years which have
possible reason for the above step by car created fear among the women in India. Due
manufacturers? to this an increasing number of women in the
(a) Car manufacturers have not been city have started training in Mixed Martial Art
able to meet their sales target for (MMA).
te

the current year so far. A) Lack of police force in India.


(b) The Government is planning to B) Self-defense is the new mantra for
review taxes on the raw materials women.
for manufacturing cars. C) People think that women are physically
(c) The car sales market is going
ce

weak.
through a jubilant phase as the D) Criminals have no fear due to lack of
volume of sales has picked up strict laws and its implementation.
considerably in; the recent past and E) The Indian women has been strongly
profit per car has also gone up. influenced by Western culture.
(d) Car buyers are still hesitant to make
@

their purchases and may postpone 45. Which of the following can be the most
for another few months. probable reason behind the increased
(e) Oil marketing companies are crime against women in India?
contemplating reviewing, the petrol (a) Only A (b) Both C and E
and diesel prices in order to decide (c) Only E (d) Only D
future market prices. (e) None of these

14 Adda247 Publications For any detail, mail us at


Publications@adda247.com
A Complete Book of Logical Reasoning

46. Which of the can be a probable reason (c) TB patient should not be permitted
for women started training in MMA? to go to public places.
(a) Only A (b) Both D and E (d) Both (a) & (b)
(c) Only B (d) Only D (e) None of these.
(e) None of these.
50. The given information is followed by two
Direction (47‐48): Study the given statements. Read carefully and answer
information carefully and answer the given the given questions.

p
questions. Many parents have written a plea to the
Civic agencies spent crores of rupees to build administration department of school X to
subways and over bridges, while Delhi police discontinue the rule of wearing ties to

ou
devised a pedestrian friendly approach. school.
However, that did not improve the scenario A. The school has kept different
for pedestrians. On an average, 300 coloured ties for different academic
pedestrians die annually in the city, due to scorers as part of their uniform.
fast cars and faulty roads. Thus the low scoring children of the
A) Often pedestrians die for no fault of school feel discriminated.

gr
theirs. B. The sport uniform of the school
B) Taking a walk in city roads is now does not have a tie which is to be
dangerous. worn only on Wednesdays.
C) City roads have not been designed for (a) Statement A weakens but statement
pedestrians.
D) There should be strict enforcement of
m B strengthens the argument
(b) Both statement A and statement B
speed limit.
weaken the argument
E) Civic agencies did not understand the
(c) Statement B weakens but statement
need of pedestrians.
A strengthens the argument
xa
47. Which of the following can be an (d) Both statement A and statement B
immediate course of action for the safety strengthen the argument
of pedestrians? (e) Statement A strengthens the
(a) Both A and C (b) Only B argument and statement B is a
(c) Both C and D (d) Only D neutral statement
te

(e) None of these


Directions (51-52): In each question below
48. Which of the following can be a possible is given a statement followed by three courses
effect of the fast cars and faulty roads?
of action numbered I, II and III. A course of
(a) Only B (b) Only B, D and E
action is a step or administrative decision to
ce

(c) Both B and D (d) Only D


be taken for improvement, follow up or
(e) None of these
further action in regard to the problem, policy
49. There is a considerable increase in the etc. On the basis of the information given in
number of persons affected from TB in the statement, you have to assume everything
the city. in the statement to be true, then decide which
Which of the following statements can be
@

of the suggested courses of action logically


an effective course of action? follow (s) for pursuing.
(a) All the Dot centres should be made
alert in the city 51. Statement: The members belonging to
(b) All the TB Hospitals and Dot centres two local clubs occasionally fight with
should be equipped properly for the each other on the main road and block
treatment. traffic movement.

15 Adda247 Publications For any detail, mail us at


Publications@adda247.com
A Complete Book of Logical Reasoning

Courses of action Assumptions:


I. The local police station should I. The kith and kin of an accused are
immediately deploy police likely to make an effort to tamper
personnel round the clock on the with the evidence or create a
main road. nuisance to help acquit the accused.
II. Those involved in fighting should II. The capacity of the wrongdoer
be identified and put behind bars. cannot be a yardstick for deciding

p
III. The local administration should punishment for a crime.
disband the management of the two III. The court’s immediate concern is
clubs with immediate effect. the grievance of the victims which

ou
(a) I and II follow (b) II and III follow needs redressal.
(c) I and III follow (d) All of these (a) Only I (b) I and II
(e) None of these (c) I and III (d) All of these
(e) None of these
52. Statement: Many students of the local
school fell ill for the fourth time in row in Directions (54): In the given question, a
statement is given followed by some

gr
the last six months after consuming food
prepared by the school canteen. assumptions. An assumption is something
Courses of action supposed or taken for granted. You have to
I. The school management should consider the statement and the following
immediately terminate the contract assumptions and decide which of the
of the canteen and ask for
m assumptions is implicit in the statement.
compensation. 54. Statement: Instead of burning the
II. The school management should leaves, bury them in compost pits, by
advise all the students not to eat which it gets converted to natural
xa
food articles from the canteen. manure, making it beneficial for the soil.
III. The owner of the canteen should A notice issued in public interest by
immediately be arrested for Department of Environment.
negligence. Assumption:
(a) II and III follow (b) Only II follows I. Whenever leaves are burnt in the
open, the air gets laden with tiny
te

(c) Only III follows (d) I and II follow


(e) None follows particulate matter which raises air
pollution to alarming levels, which
Directions (53): In the given question, a causes severe respiratory disorders
statement is given followed by some and eye infections to those exposed
ce

assumptions. An assumption is something to it.


supposed or taken for granted. You have to II. Benefits gained from ashes of leaves
consider the statement and the following burnt are not as much as the
assumptions and decide which of the benefits gained from the natural
assumptions is implicit in the statement. manure obtained from leaves by
@

53. Statements: The High Court has ordered burring it.


state X to submit the case papers of the (a) Only I is implicit
hit-and-run accident involving actor Mr. (b) Only II is implicit
Y and directed the police not to allow the (c) Either I or II is implicit
actor or his family members and friends (d) Neither I or II is implicit
to meet witnesses or victims. (e) Both I and II are implicit

16 Adda247 Publications For any detail, mail us at


Publications@adda247.com
A Complete Book of Logical Reasoning

Directions (55): In the given question below, 56. Which of the following can be deduced
a statement is given followed by some courses from the above paragraph?
of action. You have to assume everything in (I) Avian influenza virus is menacing
the statement to be true and on the basis of for birds and humans may cause
the information given in the statement, decide threat of death.
which of the suggested courses of action (II) India is proficient enough to tackle
logically follow (s) for pursuing. with deadly Avian influenza virus.

p
55. Statement: Many teachers are leaving (III) High fever with headache are
the teaching profession because of stress symptoms of Avian influenza virus
and discontentment with the workload. which is more dangerous than HIV

ou
Courses of action and DENV virus.
I. Teachers should be paid higher (a) Only II and III (b) Only I and II
salaries. (c) Only II (d) Only I and III
II. Additional teachers should be (e) None of these
appointed. 57. Which of the following substantiates the
III. The teaching curriculum should be fact of threat from bird flu?

gr
changed. (I) Last year Delhi had suffered huge
(a) Either I or II follows loss in the form of death of several
(b) Either II or III follows persons and birds from effect of
(c) Either I or II or III follows bird flu.
(d) Only II follows
(e) All follow
m (II) There was a huge loss to poultry
farmers and there was no
Directions (56-57): Study the given compensation from Indian Govt.
information carefully and answer the given (III) Indian Govt. has implemented an
questions. action plan for preventive check of
xa
Several subtypes and strains of avian bird flu.
influenza viruses are now found around the (a) Only I and II (b) Only III
world, some of them capable of causing death (c) Only II and III (d) Only II
among humans and others inflicting serious (e) All I,II and III
losses on poultry farmers. The latest bird flu
Directions (58-59): Study the given
te

scare in New Delhi and elsewhere has been


triggered by the death of some free living information carefully and answer the given
birds in the city’s A.N. Jha Deer Park, and 15 questions.
painted storks in the Gwalior zoo. Worrying Several cities, including Agra, Ahmedabad,
as it is, early detection and identification of
ce

Patna, Delhi and Varanasi, were choked by


the virus subtypes helps in launching particulate matter pollution on Sunday, when
containment measures. As a major the northern parts of the country celebrated
agricultural nation with a large poultry Deepawali.
industry, India has implemented an action
plan formulated by the Centre’s Department Concentrations of fine particulate matter rose
@

of Animal Husbandry, Dairying and Fisheries to levels that harm respiratory health in
to deal with avian influenza. It incorporates a normal people and severely debilitate those
clear protocol for preventive checks and with illnesses. The Air Quality Index (AQI)
testing, for reporting an outbreak, removing reading for Agra was 384, Ahmedabad 385,
farmed birds from an affected area and and Faridabad and Delhi the worst, at 428
compensating farmers. and 445.

17 Adda247 Publications For any detail, mail us at


Publications@adda247.com
A Complete Book of Logical Reasoning

An AQI of 100 is the limit for good air quality. reserved seats in educational
Bengaluru, Chennai and Mumbai were in the institutions; the Mysore Maharaja and
‘moderate’ to ‘satisfactory’ category, similar the states of Madras and Travancore too
to last year’s Deepawali. Hyderabad improved ensured representation for the very
from ‘poor’ to ‘satisfactory.’ backward in all senses of the term
because of highly stratified social
58. Which of the following can be
structures and the practice of
hypothesized from the above passage?

p
“untouchability” that had left large
I. People of Bengaluru, Chennai and
sections of the population backward for
Hyderabad gives less importance to
centuries. They recognized that it was
Deepawali as compared to people of

ou
only by actively trying to lift up these
Northern part of India.
sections by offering seats in educational
II. Despite of knowing that rise in
institutions and in employment, that
pollution level will affect their
some kind of level playing field could be
health, it’s only negligence of people
established.
of northern part of India which
Should there be reservation of jobs in the
make AQI rise upto worst level.

gr
organizations in the private sector also
III. Govt. of different states of northern
as in the public-sector undertakings in
part of India had no plan to check
India?
worst situation of AQI. Arguments:
(a) Only I and III (b) Only II and III I. Yes, This would give more
(c) Only II
(e) None of these
m
(d) Only I and II opportunities of development to the
weaker sections of the society and
59. Which of the following negates the fact thus help reduce the gap between
which is given in the above passage? the affluent and the downtrodden
xa
I. Cases of people suffering from in India.
respiratory problem increases after II. No, The private sector does not get
country celebrated Deepawali. any government assistance and
II. This year people have decided not therefore they should not be
to use crackers during Deepawali to saddled with such policies.
maintain pollution level as a part of III. No, Nowhere else in the world such
te

Green India revolution. a practice is being followed.


IV. No, The management of the private
III. Manish Sisodia, Deputy CM of Delhi
sector undertaking would not agree
have called high level meeting just a
to such compulsions.
day after Deepawali to analyze the
ce

(a) Only I is strong


situation of increased AQI (Air
(b) Only I and II are strong
Quality Index). (c) Only I, II and IV are strong
(a) Only III (b) Only I and III (d) Only I and IV are strong
(c) Only II (d) Only II and III (e) All are strong
(e) None of these
@

Directions (61): In the given question, a


60. Statement: Reservations have had a statement is given followed by some
place in India for over a century, much assumptions. An assumption is something
before they were written into the supposed or taken for granted. You have to
Constitution as a leg up for socially and consider the statement and the following
educationally backward sections. In assumptions and decide which of the
1902 Pune’s Chhatrapati Maharaj assumptions is implicit in the statement.
18 Adda247 Publications For any detail, mail us at
Publications@adda247.com
A Complete Book of Logical Reasoning

61. Statement: By asking five prominent Courses of action:


hospitals in the national capital to I. India should immediately stop
deposit nearly Rs. 600 crore to supplying root stocks of black
compensate for their failure to treat poor pepper to other countries.
patients, the Delhi government has II. India should adopt modern
drawn attention to the social obligation technology for cultivating black
of healthcare providers in the corporate pepper to complete in the

p
sector as well as the need for timely International market.
enforcement of applicable regulations. III. India should reduce the price of its
According to the Delhi government, black pepper to remain competitive

ou
trusts and registered societies to which in the world market.
public land was allotted to establish You have to assume everything in the
hospitals were required to earmark a statement to be true, and then decide
percentage of their medical facilities and which of the following three given
services for indigent patients. suggested courses of action logically

gr
Assumptions: follows for pursuing?
I. This strengthens the case for (a) All follow (b) Only II follows
private hospitals to dedicate a part (c) Only I follows (d) Only III follows
of their services to those who can (e) None of the above
not afford treatment.
m
II. Social responsibility of hospitals 63. Statement: Govt. has taken decision in
many blocks of country that the
must be monitored and central
government should ensure construction work of highway is given to
compliance by the corporate the private forums of builder operator
xa
hospitals. transfer on the basis of action.
III. There is a great need for the Assumptions:
government to monitor and enforce I. It may be possible that a sufficient
“health services ” so that the poor number of private forums do not
who can not afford modern health answer in the respect of Govt.
te

facilities that are extremely costly- action.


are able to be benefited. II. Many private forums of our country
(a) All are implicit may be capable to complete the
(b) Only III is implicit construction work of highway
ce

(c) Only I is implicit within given time period.


(d) Only I and II are implicit III. It may be that private forums get
(e) None is implicit financial profit due to govt. builder
operated proposed schemes.
62. Statement: India’s pre-eminent position
Which of the following assumptions is
in the world black pepper production
@

implicit in the statement?


and trade is in danger as some of the
(a) Only I and II are implicit.
countries, which recently started
(b) Only II and III are implicit.
production of the ‘king of the spices’
(c) Only II is implicit.
crop from Indian root stocks, are
(d) Only I and III are implicit.
farming better by adopting modern
(e) None of these.
cultivation practices.
19 Adda247 Publications For any detail, mail us at
Publications@adda247.com
A Complete Book of Logical Reasoning

64. Statement: All the single-screen theatre (a) After graduation from college, most
halls in the city have declared indefinite Maths and Science graduates opt
strike and have warned that they will not for jobs in industry rather than in
withdraw strike unless the government teaching.
accepts their demand of reducing (b) Many high schools have been forced
entertainment tax to 50% of the present to lower their standards in hiring
level and also treat these halls at par Maths and Science teachers.

p
with multi-screen halls on all the (c) More scholarship money is already
relevant matters. available for students of Maths and
Assumptions: Science than is available for those in

ou
I. The employees of all the cinema any other field.
halls may disagree with the (d) Population statistics show that the
management and may appeal to the number of high school students is
government to declare the strike expected to decline over the next
illegal. ten years.
II. The government may accept all the
(e) Many experts say that business of

gr
demands of the striking cinema
the future will require relatively
halls to avoid any backlash from the
few highly trained workers, along
public.
with many semi-skilled and purely
III. The general public may support the
technical employees.
cause of the cinema hall owners and
m
put pressure on the government to Direction (66-67): Read the following
accept their demands. information answer the given questions.
Which of the following assumptions is
implicit in the statement? The agitated guardians of grade IX students of
xa
(a) None is implicit a high-profile public school of North Carolina
(b) Only I is implicit protested against the newly introduced
(c) Only III is implicit audio-visual aid of teaching methodology.
(d) Only II is implicit According to them, it is of benefit to a handful
(e) All are implicit of students only, and majority of students will
not be getting any help because of this
te

65. Statement: For our nation to compete technology.


successfully in the high-technology These questions are based on the information
enterprises of the future, workers with given above and the sentences given below
skills in Maths and Science will be labeled as I, II, III, IV and V.
ce

needed. But it is doubtful that they will


I. Change is a gradual process and it takes
be available, since there is a shortage of
some time for people to adjust to it.
high school Maths and Science teachers
II. Guardians are happy with the new
that shows no signs of improving.
teaching methodology, i.e. via the use of
Industry can help alleviate this problem
audio-visual techniques.
by funding scholarship grants and aid to
@

college students who graduate in Maths III. It is not the responsibility of the school
and Science with the hope of pursuing to take care of the weaker students.
teaching careers. IV. This modern technology of teaching
Which of the following, if true, would should be scrapped as soon as possible.
most probably prevent the proposed V. Guardians are unhappy with the new
plan from achieving its intended effect? teaching methodology used of the school.

20 Adda247 Publications For any detail, mail us at


Publications@adda247.com
A Complete Book of Logical Reasoning

66. Which of the following statements above 69. Statement: A sudden cloud burst over
could be a possible conclusion from the the island city resulted into unpredicted
above passage? rainfall causing flood like situation in
(a) Only I (b) Only II entire area. Large number of people was
(c) Only IV (d) Both IV and II caught unaware and was stranded on the
(e) Only V road.
Courses of action:
67. Which of the following could be a

p
I. The local administration should
possible statement that should be used immediately put in place an action
by the school management to defend plan for avoiding such situation in
future.

ou
themselves?
(a) Both II and V (b) Only III II. The local administration should
(c) Only I (d) Only V immediately deploy personnel to
(e) Both IV and III help the stranded people to move to
safer places.
Directions (68-69): In each question below III. The local administration should
is given a statement followed by three courses advise all the citizens not to venture

gr
of action numbered I, II, and III. A course of out on the road till situation
action is a step or administrative decision to improves.
be taken for improvement, follow up or (a) Only I follows (b) Only II follows
further action in regard to the problem, policy (c) Only III follows (d) II and III follow
(e) All of these
m
etc. On the basis of the information given in
the statement, you have to assume everything 70. There is an alarming trend of skewed sex
in the statement to be true, then decide which ratio against women in India during the
of the suggested courses of action logically past decade and situation may get out of
follow (s) for pursuing. hand if adequate steps are not taken to
xa
stop female feticide.
68. Statement: Many school buses have Which of the following can be an
fitted CNG Kit without obeying the safety effective step to reverse the trend?
guidelines properly. This results into I. The Government should
some instances of these buses catching immediately announce strict action
against the culprit.
te

fire due to short circuit and endangering


the lives of the school children. II. The Government should announce
Courses of action: substantial incentive scheme for
I. The regional transport authority couples who have at least one girl
should immediately carry out child.
ce

III. The Government should launch a


checks of all the school buses fitted
nationwide campaign to create
with CNG Kit. awareness against female feticide.
II. The management of all the schools (a) Only I (b) I and II
should stop hiring buses fitted with (c) II and III (d) All of these
CNG Kit. (e) None of these
@

III. The Government should issue a


71. Expressing serious concern over recent
notification banning school buses
reports on use of drugs by Indian
for use of CNG Kit.
athletes at International meets, Prime
(a) Only I follows (b) Only II follows Minister Manmohan Singh has urged
(c) Only III follows (d) I and III follow sportspersons to “keep away from such
(e) None of these temptations”. A news in a daily.
21 Adda247 Publications For any detail, mail us at
Publications@adda247.com
A Complete Book of Logical Reasoning

If this statement is true, then which of The education of the farmer’s children is
the following expresses truly the above often the first casualty of a failed crop. A
statement? large number of students are reported to
(a) Our sportspersons are the most be dropping out of school in villages as
important factor in our strategy to their parents want their children to help
achieve excellence in sports them in farms.
(b) Let no player or athlete take a Courses of action:

p
shortcut in trying to achieve I. The government should
success in sports immediately launch a programme
(c) A single event of malpractice gave a to create awareness among the

ou
major jolt to the country’s pride and farmers about the value of
glory education.
(d) It is necessary for the PM to show II. The government should offer
the athletes an ideal way to success incentives to those farmers whose
in order to earn social repute children remain in schools.
(e) It is not possible for the Indian III. Education should be made

gr
athletes to face competition with compulsory for all children up to
athletes from developed countries. the age of 14 and their employment
banned.
72. Majority of the employees of the ailing You have to assume everything in the
organization opted for voluntary statement to be true, and then decide
retirement scheme and left the
organization with all their retirement
m which of the following three given
suggested courses of action logically
benefits within a fortnight of launching follows for pursuing.
the scheme. (a) Only (i) and (ii) follows
Which of the following can be a probable (b) Only (ii) and (iii) follows
xa
cause of the above effect? (c) Only (i) and (iii) follows.
(a) The company has been making (d) All follows
huge losses for the past five years (e) None of these
and is unable to pay salary to its
74. Some scientists believe that, in certain
employees in time
species of birds, actual particles of metal
te

(b) The management of the company


within the brain react to the Earth’s
made huge personal gains through
magnetic field in the same way as the
unlawful activities
needle in a compass. It is this mechanism
(c) One of the competitors of the that is thought to underlie the birds’
ce

company went bankrupt last year amazing ability to navigate accurately


(d) The company owns large tracts of over distances of thousands of miles by
land in the state which will fetch day and night during migration. To test
huge sum to its owners this theory, researchers surgically
(e) None of the above removed the metal particles from the
@

73. Statement: The life of a farmer is very heads of some birds and then released
tough. With no other revenue source, he them, along with a number of untreated
has to totally depend upon the year’s birds, at the usual time and place of their
crop. And if it happens to fail, an entire annual winter migration.
year is lost,” says, a farmer’s son who Which of the following results would
comes from a very humble background. most seriously weaken the theory being
tested?
22 Adda247 Publications For any detail, mail us at
Publications@adda247.com
A Complete Book of Logical Reasoning

(a) The untreated birds were confused (e) For six months following the day of
by the erratic flight patterns of the the interviews, no official
surgically treated birds and failed complaints were filled by any
to migrate successfully. passenger with the Federal agency
(b) The surgically treated birds were that regulated the airlines.
able to follow their usual flight
patterns successfully by day, but Directions (76-77): Each question given

p
not by night. below consists of a statement, followed by
(c) The surgically treated birds were two arguments numbered I and II. You have
able to migrate about as accurately to decide which of the arguments is a ‘strong’
as the untreated birds.

ou
argument and which is a ‘weak’ argument.
(d) The surgically treated birds were Given answer
able to migrate successfully only (a) if only argument I is strong;
when closely following a group of (b) if only argument II is strong;
untreated birds.
(c) if either I or II is strong;
(e) In addition to being impaired in
(d) if neither I nor II is strong and
their migration ability, the

gr
surgically treated birds suffered a (e) if both I and II are strong.
variety of other disabling effects. 77. Statement: Should the education at all
75. A government survey released today levels be offered only in vernacular
shows that 80% of the people who fly medium?
are satisfied with the service they
m
receive from the airlines in this country.
Arguments:
I. Yes. This is the only way to enhance
Three interviewers stood outside a performance of the students.
major airport and asked people leaving II. No. This will severely affect
the terminal, “Do you have any
xa
acquiring knowledge for want of
complaints about the flight you just got good text books in vernacular
off?” Only 20 percent responded “yes!” medium.
Which of the following, if true, would
most undermine the conclusion of the 78. Statement: Should articles of only
argument above? deserving authors be allowed to be
te

(a) Sixty percent of the people coming published?


out of the airline terminal were not Arguments:
people who had just gotten off a I. Yes. It will save a lot of paper which
flight. is in short supply.
(b) One percent of the people
ce

II. No. It is not possible to draw a line


approached by the interviewers
between the deserving and the
refused to respond to their
undeserving.
inquiries.
(c) The interviewers began their Directions (78-80): In each question below
inquiry just after passengers were is given a statement followed by two courses
@

discharged from a flight that was 40 of action numbered I and II. You have to
minutes late. assume everything in the statement to be true
(d) The interviewers were able to and on the basis of the information given in
speak to only 70 percent of the
the statement, decided which of the suggested
people leaving the terminal, but
courses of action logically follow(s) for
those people were selected at
random. pursuing.

23 Adda247 Publications For any detail, mail us at


Publications@adda247.com
A Complete Book of Logical Reasoning

Give answer Courses of action:


(a) If only I follows; I. The city municipal authority should
(b) If only II follows, take immediate steps to carry out
(c) If either I or II follows; extensive fumigation in ward X.
(d) If neither I nor II follows and II. The people in the area should be
(e) If both I and II follow. advised to take steps to avoid
mosquito bites.
78. Statement: The cinema halls are

p
incurring heavy losses these days as 81. Statements: “The municipal corporation
people prefer to watch movies in home of the city is granting permission to
on TV than to visit cinema halls. builders for new construction of high

ou
Courses of action: rise buildings for the city, despite the
I. The cinema halls should be appeal from the environmentalists to
demolished and residential multi- avoid overcrowding.”
storey buildings should be Assumptions:
constructed there. I. The corporation will take care to
fulfill the minimum requirements

gr
II. The cinema halls should be
converted into shopping malls. for providing environmental
infrastructure in the area for the
79. Statement: Due to substantial reduction newly constructed building.
in fares by different airline services, II. Corporation is expecting good
large number of passengers so far
m
travelling by upper classes in trains have
earnings by way of faces from the
high rise buildings.
switched over to airline services. III. The corporation has taken the
Courses of action: residents of the area into
I. The railways should immediately confidence and assured them of no
xa
reduce the fare structure of the inconvenience to them due to new
upper classes substantially to retain construction.
its passengers. Which of the following assumptions is
II. The railways should reduce the implicit in the statement?
capacity of upper classes in all the (a) Only I is implicit
te

trains to avoid losses. (b) Only II is implicit


80. Statement: A large number of people in (c) Only III is implicit
ward X of the city are diagnosed to be (d) Only I and II are implicit
suffering from a fatal malaria type. (e) None of these
ce

General Issue Based Logical Reasoning : Solutions


1. (d); The author argues for the following Direction (2-3):
@

connection: videos take money 2. (c); Any aspect of health has two factors
away from movies. What choices to tackle with — prevention and
(d) asserts, in effect, is that the treatment. Prevention includes
money spent on videos came from creating awareness among people,
some other source. So, (d) and treatment includes providing
statement undermines the given adequate medical facilities. So, both
passage. II and III follow.
24 Adda247 Publications For any detail, mail us at
Publications@adda247.com
A Complete Book of Logical Reasoning

3. (d); Clearly, chloroquine can still be 8. (b); Only II arguments is strong enough.
used to get rid of the non-resistant All the conditions can’t be agreed
varieties, and new medicines to, in order to save the life of the
developed for the resistant victims, they should not to be
varieties. The patients can then be agreed to, as they encourage the
treated accordingly by performing sinister activities of the kidnappers.
tests for the causal mosquito. So, 9. (b); The blood group, complete address

p
only II and III, follow. and telephone number must be
4. (d); I is weak because it is not desirable. mentioned on the identity card of
each person. This must be useful in

ou
Moreover, is falsely assumes that
case of any serious accidents.
govt authorities can’t cope with the
demand of driving licenses. II is Direction (10-11):
weak because it is not logical. It 10. (d); In present times, women are being
wrongly assumes that such a move imparted education at par with the
will lead to a spurt in the number of men and are capable of competing

gr
vehicles. with them in all professions and
Direction (5 –6): fields. So, argument I holds. Also,
women cannot be confined to the
5. (a); As it is given that the pressure of
household and kept away from the
increasing population and the
challenges of the outside world
practice of dividing land equally
amongst the heirs and the holdings
m against their will. They too have the
right to be self-dependent. Besides,
have become smaller and present-day women are well
fragmented, so (a) is the main looking to outside jobs together
xa
reason. with the household jobs. So,
6. (b); Conclusion I can be inferred from argument III holds while II does
the sentence - “The pressure of not.
increasing population ….. sub- 11. (b); Clearly, banning a product would
divisions of farm holdings.” II can surely render jobless the large
te

be inferred from the fact that the number of workers involved in


“small size of holdings makes manufacturing it. Besides,
farming activity uneconomical.” III firecrackers on burning produce
can be inferred from the way even explosive sounds and immense
poisonous fumes, which cause both
ce

the hill slopes have been cut for


cultivation which had disturbed air and noise pollution. So, both
natural ecosystem. arguments I and III hold. However,
to stop child labour, it is not
Direction (7-8): necessary to close down the
7. (e); Merit, fair selection and equal industry but strict laws against
@

opportunities for all these three child abuse should be enforced and
factors, if taken care of, can help legal actions taken. Similarly, there
government recruit competent are many other ways to make
officials and also fulfil the objectives parties boisterous and special
of the Constitution. Thus, both the events enjoyable. Hence, II as well
arguments hold strong. as IV does not hold strong.

25 Adda247 Publications For any detail, mail us at


Publications@adda247.com
A Complete Book of Logical Reasoning

12. (b); I directly follow from the statement Direction (15-16):


and so is implicit. The decision to 15. (d); Neither of the conclusions follows
shift factories on the outskirts of from the information given in the
the city must have been taken after statement as we don’t know the
taking into account the availability reason behind the oppose of
of land there. So, II is implicit. employee’s union to declare their
Nothing can be assumed as to what income.

p
strategy would the factory owners 16. (d); No government wants or intends to
adopt in future. So, III is not encourage corruption in the
implicit. government offices. Therefore,

ou
conclusion II is not valid. If the
Direction (13-14): Official Secrets Act which was
13. (d); Clearly, our Constitution considers enacted during the war is the
youngsters above 18 years of age, source of corruption, it should be
mature enough to exercise their abolished after the war.
decisive power in Government by

gr
voting. This implies that such 17. (c); Option (c) may be a possible fallout
of the given situation as these
individuals can also judge what is
places are not getting adequate
good or bad for them. Thus,
rainfall else option (a) is not
argument I holds strong. However,
possible as it is not practical, for (b)
m
at such places, youngsters may be
lead astray by certain indecent guys
we don’t know weather it is a
farmer’s place or have farmland
and swayed from the right path into there. (d) is also not possible as it is
bad indulgences. So, IV also holds not government’s fault.
xa
strong. Hiking the entry fees is no
18. (c); Option (c) substantiates the views
way to disallow them, and also the
expressed in the statement, as the
idea of imitating the western
condition of railway tracks are not
countries holds no relevance. So,
upto mark.
neither II nor III holds strong.
Direction (19-21)
te

14. (b); No doubt making military education


compulsory for all college students 19. (a); Since the statement talks of putting
would strengthen India’s forces and the child in school at the age of 5, it
also inculcate strong discipline means that the child is mentally
ce

prepared for the same at this age.


among them, but this is not the only
So, I is implicit. But nothing about
means to achieve these ends. So,
admission after 6 years of age is
neither I nor III holds strong.
mentioned in the statement. So, II is
Besides, the students really willing not implicit.
to join the armed forces and defend
@

the nation can put up a better 20. (e); The customer's eagerness to get the
defence than those compelled to do bills makes I implicit. Besides, the
so. And it is the fundamental right customer has written to the editor
to bring the malfunctioning of the
of every student to pursue a career
department to public notice. So, II is
of his choice. So, II as well as IV
also implicit.
holds strong.

26 Adda247 Publications For any detail, mail us at


Publications@adda247.com
A Complete Book of Logical Reasoning

21. (e); The use of the words 'impoverished 27. (c); There are many customs and
community' in the statement makes traditions among the Hindus which
I implicit while the phrase 'college weaken the position of women in
education should be restricted to a India because India has a
brilliant few' makes II implicit. patriarchal society.
Direction (22-23): 28. (b); Besides traditionally bestowed low
position to women in India, khap

p
22. (a); Clearly, health of the citizens is an
panchayats and frequent fatwas
issue of major concern for the
issued by Ulemas add to the
Government. So, a product like
discomfiture of women. Only (b)

ou
drugs, must be first studied and
strengthen the given statement as it
tested in the Indian context before
describes about the poor condition
giving license for its sale. So, only
of women which is also mentioned
argument I holds strong.
in the given statement but (a) does
23. (b); Clearly, there should be some not follow as nothing is mention
norms regarding the number of about the legal effort taken foe

gr
ministers in the Government, as women while (c), (d) nowhere
more number of ministers would strengthen the statement.
unnecessarily add to the
Directions (29-31):
Government expenditure. So,
argument II holds strong; also,
m
giving liberty to the party in power
29. (a); Clearly, none of the arguments
provides a substantial reason either
could promote extension of for or against the given statements.
unreasonable favour to some So, none of the arguments holds
people at the cost of government strong.
xa
funds. So, argument I does not hold.
30. (b); Clearly, television offers various
24. (d); Ours is a secular state does not educational programs which are of
mean that religion and religious great practical value to the
values should be eradicated. In fact, students. So, it serves as a means
these inculcate moral values. So, (but it is not the ‘only’ means) to
te

argument I is vague while argument educate the masses. Thus, I holds


II is strong. Also, teaching religion strong while IV does not. Besides,
can in no way hinder the students’ the demerits of watching television,
capability to face the challenges of mentioned in II and III, may be done
ce

the 21st century. away with by allowing children to


watch selected programs on
Direction (25-28):
television, according to a set
25. (a); Clearly, India has a patriarchal schedule. So, neither II nor III holds
society. (b), (c), (d) can’t be strong.
@

assumed from the given statement.


31. (d); Clearly, a policy to select deserving
26. (c); Modern girls are highly educated candidates cannot be abolished just
and they are equivalent to boys, this because of the expenditure it
weakens the given statement which entails. So, argument I does not
states about the superiority of boys hold. Also, students who are
over girls. intelligent enough to secure good

27 Adda247 Publications For any detail, mail us at


Publications@adda247.com
A Complete Book of Logical Reasoning

marks in academic exams have no can publish. So, the statement (b) is
reason not to perform well in the correct assumption. Statement
entrance tests. So, II also does not (c) cannot be assumed as
hold. Further, the students passed verification of content does not
out from different universities are depend upon minor or major
assessed on different patterns and publications. Statement (d) is not
hence a common entrance test the correct assumption because it is

p
would put the candidates to given that publishers are more
uniform test and assessment. So, interested in boosting circulation,
only III holds strong. but it cannot be assumed that

ou
Publishers are the persons who
Direction (32):
decide the content of the
32. (d); According to given statement, newspapers.
Television makes a feeling of
38. (c); Statement (c) is the correct option
helpless among viewers and it also
because according to this the
persuades them to accept whatever
increase in crime rate has been

gr
happens to them.
contributed by other factors, not
33. (b); Statement (b) is the correct option leniency in the punishment. So, it
because it clearly weakens the weakens the given passage.
argument as it is given there that
39. (d); Most logically such comparison
the average salary of persons
working in industries is less than
m should reveal mortality rate per
thousand doctors indulged in SARS
that of average salary of persons
treatment and not indulged in
working in business schools which
treatment. So, statement (d) is the
contradicts the given statement.
xa
correct option.
Direction (34-35):
40. (a); Actions I and II will directly
34. (e); VI clearly weakens the given improve the working condition of
statement because according to that railways. Action III though, not
development of resistance amongst directly related with improving
te

mosquitoes depend upon the railway traffic conditions but will


amount of repellent used. encourage other signal men to be
more watchful on their duties,
35. (d); It is clearly given in the statement II
that the increased concentration of which, in turn, will reduce the
ce

probability of accidents.
repellent would endanger the
health of humans as well as the 41. (c); Only (c) weaken the statement as it
ecosystem as a whole. oppose the statement which states
36. (a); Civilization taught us the moral that chewing tobacco will increase
duties regarding the children and the life span.
@

relatives. So, statement (a) is the


42. (d); Option (d) contradicts the views
correct option.
expressed in the statement as it is
37. (b); The exposed articles by the mentioned that there are
publication must be verified so that opportunities for them in
the authentic and original content developed countries.

28 Adda247 Publications For any detail, mail us at


Publications@adda247.com
A Complete Book of Logical Reasoning

43. (a); Option (a) may be a best possible Direction (51-52):


reason as there will be more supply 51. (a); Only courses of action I and II seem
of cars than demand. Hence, to be appropriate for pursuing
manufacturers have not been able course of action III is so harsh to be
to meet their sales target. appropriate.
44. (b); Once students reserve their seats at 52. (d); Only courses of action I and II seem

p
the time of counseling they can’t to be appropriate for pursuing, the
withdraw their admission, this recurrence of an incident is
leads to seats lying vacant in the sufficient to take course of action I.

ou
colleges. So only b) is a valid option. 53. (a); Only assumption I is implicit. III is
Others options will be rules out. not implicit because the reason
behind this decision is that the
Direction (45-46): witnesses or victims can’t be felt
45. (d); Lack of powerful laws to create fear any problem regarding the case.
amongst the criminal is the most They can be pressurized by the

gr
accused to change their witnesses.
probable reason for increasing the
The decision is not given because of
crime against women in recent the court’s immediate concern for
times. the victims.
46. (c); Due to the recent increase in crime 54. (b); The statement is saying about the
against women in India. Women
m benefits for the soil but the
wants to improve their self-defense assumption I describes the benefits
against the crime, criminals and of burying the leaves for reducing
criminal activities. air pollution. Hence, it is not
xa
implicit. But assumption II is
Direction (47-48): implicit. That is why the notice
47. (d); For the safety of pedestrians there stresses on burying the leaves
instead of burning it.
should be strict enforcement of
speed limit for offenders. 55. (b); I does not follow. Salary can’t
te

reduce workload but either II or III


48. (a); Due to fast cars and faulty roads is a way out.
taking a walk in city road is now
dangerous. Direction (56-57):
56. (b); Only I and II can be deduced from
ce

49. (d); To alert the dot centre can be a above paragraph as it is mentioned
better course of action. Also to in second line of paragraph that
equip the TB Hospitals for the Avian influenza virus is capable of
treatment is a right course of action. causing death among human and
Hence both can be effective steps. others and in last part of paragraph
it is clearly mentioned that India
@

50. (e); Classification of wearing tie as per has implemented preventive


academic scores is the cause of measures to check influenza virus
discrimination to the low scorer so but it is nothing mentioned about
statement (a) strengthens the symptoms and nowhere in
argument best. while statement (b) paragraph it is compared with HIV
is neutral in nature. and DENV virus.

29 Adda247 Publications For any detail, mail us at


Publications@adda247.com
A Complete Book of Logical Reasoning

57. (e); I and II is telling about loss which to accommodate such a policy if
had occurred last year which was a implemented. So, neither II nor IV
huge loss to humans and birds so it holds strong. Further, just imitating
substantiates the fact of threat from other countries holds no relevance.
bird flu. In III statement it is So, argument III also does not hold.
mentioned that Indian Govt. has
61. (c); Such strict actions of Delhi
implemented an action plan it
government made private hospitals

p
means it is matter of threat and
Indian Govt. is taking preventive to serve their services to poor
measures. So, All I, II and III people. So I is implicit. But nothing

ou
substantiates the fact of threat from is mentioned about its
bird flu. monitorization in the statement. So,
II and III are not implicit.
Directions (58-59):
58. (b); Only II and III can be hypothesized 62. (b); Only better quality can put India
from the above passage. As, air back in the competitive field of

gr
quality index of Bengaluru, Chennai black pepper production. So, India
and Hyderabad is almost at safe should go for modern technology
level in comparison to northern for cultivating black pepper to
part of the country. It doesn’t mean compete in the present
that people of Bengaluru, Chennai international market.
and Hyderabad gives
m
less
63. (b); Only II & III are implicit. It may be
importance to Deepavali. But we
can assume the negligence of possible that private forums can
people of northern part of country complete the work of highway in a
xa
as all know rise in pollution level given time and also get financial
will affect their health. And as AQI is profit.
at worst level so, we can assume
64. (a); The strike involves only two
that Govt. of northern part had no
plan to check worst situation of AQI. parties: the cinema halls and the
government. The employees or the
te

59. (c); The overall fact in the above public have nothing to do with the
passage is rise in pollution level and strike. Hence I and III are not
worst condition of AQI in northern
implicit. II is not implicit because of
part of country. So, only Statement
the word all.
ce

II nullify the above fact. And


statement I and III support the 65. (a); The correct response to this
given fact in passage. question is a statement that focuses
60. (a); The reservation of jobs in the upon an unforeseen casual
private sector too would surely outcome. What (a) asserts is that
@

increase opportunities for weaker you can pour the money into the
sections to improve their economic system, and you can train students
plight. Thus, argument I is strong in Maths and Science, but they will
enough. Also, private sector not go into teaching. Thus, your
companies work on a good profit plan will not achieve the desired
margin and they can and will have result.

30 Adda247 Publications For any detail, mail us at


Publications@adda247.com
A Complete Book of Logical Reasoning

Direction (66-67): 73. (d); Literacy at basic level is the most


66. (e); Clear from the way the agitated important need to prepare good
guardians have protested against future citizens. So, all children need
this newly introduced audio-visual to be educated. This can be
aid of teaching methodology, they achieved by creating awareness,
are unhappy with the new teaching providing incentives, enforcing
methodology. education and banning employment

p
67. (c); Any new thing takes some time for of children. Thus, all three courses
its adjustment. So, school authority of action follow.

ou
may defend themselves by saying 74. (c); The question stem here asks for the
this statement. evidence that would most weaken
Directions (68-69): the theory that the metal particles
68. (a); Only Ist course of action seems to are important to the bird’s
be appropriate because it is clearly navigational mechanism. The best
mention that buses which have refutation of the theory would be a

gr
fitted CNG Kit without obeying the finding that when the particles are
safety guidelines met with short removed, the birds continue to
circuit resulting of catching fire. So, migrate as effectively as before. (c)
IInd and IIIrd courses of action are is tantamount to saying that this
not appropriate.
m
69. (d); Only II and III follow because in this
was the case.
75. (a); The question was put to people
type of situation, immediate who were coming out of the
courses of action should be taken terminal about the services aboard
xa
which is mentioned in II and III. Ist
the flight. If it was shown that a
will not follow because of word
“immediately”. It is of preventive majority of people coming out of
nature and will take long time. the terminal were not passengers,
Then the conclusion would really be
70. (d); All three steps can be effective steps jeopardized. (a) does just that.
to reverse the trend. The govt.
te

should completely ban and should Direction (76-77):


announce that strict action will be
76. (b); Teaching in vernacular medium
taken against culprit. Nationwide
campaign and incentive scheme will would surely make it easy for
ce

motivate parents to have a girl students to grasp. But the use of


child. ‘only’ in argument I make it invalid.
Also, teaching in international
71. (b); Concern of Indian PM is that no
language would open up more
player should take shortcut in
avenues for students – in procuring
trying to achieve success in sports.
@

books and study material, in going


72. (a); 1st option may be the probable abroad for studies as well as taking
cause of the above effect as up jobs which require interaction
company is not able to pay salary to with people of different
its employees; many of them at
nationalities. So, argument II holds
least want to avail retirement
strong.
benefits.

31 Adda247 Publications For any detail, mail us at


Publications@adda247.com
A Complete Book of Logical Reasoning

77. (b); Clearly, I does not provide a strong railways if there is a marginal
reason in support of the statement. difference between the fares.
Also, it is not possible to analyze the Hence, a considerable gap between
really deserving and not deserving. the two fares is a must for the
So, argument II holds strong. railways. So, course I follows.
Direction (78-80): Following course II would reduce
the volume of passengers. Hence, II

p
78. (d); Doing away with cinema halls is no does not follow.
solution. Hence, none of the course
follows. Instead, certain incentives 80. (e); Clearly, prevention from

ou
and promotional schemes should be mosquitoes and elimination of
awarded to cinema hall owners so mosquitoes are two ways to
that they could manage to draw in prevent malaria. So, both the
crowds. courses follow.
79. (a); Airlines, being convenient and
81. (d); Assumptions I and II are implicit in
faster means of transport, people

gr
would surely prefer it to the the statement.

m
xa
te
ce
@

32 Adda247 Publications For any detail, mail us at


Publications@adda247.com
A Complete Book of Logical Reasoning

p
ou
gr
m
xa
te
ce
@

1 Adda247 Publications For any detail, mail us at


Publications@adda247.com
A Complete Book of Logical Reasoning

Situational Based Logical


10 Reasoning
Introduction: Situational based logical questions are based on the situations which can be an
incident of daily life routine, some business oriented, related to living or nonliving thing

p
etc…These type of questions required practical approach to solve it as the one have to choose
the practical measures not that seems to be easy or pleasant. To solve such questions first

ou
understand the situation given in the question as its type (critical, newly emerged…), its
seriousness, its effects, its importance and most importantly its approach.
One should be practical while solving such questions as hypothetical measures can’t be taken in
it. The things that should be taken in mind while solving it is that if the situation requires
immediate efforts or which measure is more feasible and practically what steps should be taken
etc..

gr
Syntax of the Situational Based Logical Question
Example: India’s decision to approach the International Court of Justice (ICJ) to stall the
possible execution of Kulbhushan Sudhir Jadhav in Pakistan is an unusual move. His case now
m
hinges on an appeal against his conviction on charges of espionage and on petitions for mercy
before Pakistan’s Chief of Army Staff and President. India’s main contention is that Pakistan had
committed “egregious violations” of the Vienna Convention on Consular Relations by repeatedly
denying consular access to Jadhav for over a year.
xa
1. Which of the following can be hypothesized from the above passage?
(I) Indian Govt. is ready to “boil the ocean” to fortify Kulbhushan Jadhav.
(II) International court of justice has prerogative to alter the verdict of Pakistan court.
(III) Indian Govt. is too much concern about the egregious violation of Pakistan.
(a) Only II
te

(b) Only I and III


(c) Only I and II
(d) All of these
(e) None of these
ce

2. Which of the following is not in the line with the passage?


(I) ICJ has considered the point of egregious violation of Pakistan to stall the execution of
Kulbhushan Jadhav.
(II) SAARC nations have supported the major step taken by India.
@

(III) Indian External Affairs has cancelled the trip of Pakistan for major deals.
(a) Only II and III
(b) Only I
(c) Only I and III
(d) Only I and II
(e) All of these

2 Adda247 Publications For any detail, mail us at


Publications@adda247.com
A Complete Book of Logical Reasoning

Solution
Explanation-: Ans. (d); In this question we have to choose that statement which can be
assumed from the passage.
For Statement I: Right, As Indian Govt. has approached ICJ with clear intention it means Govt. is
trying their best to protect Jadhav.
For Statement II: Right, ICJ has stall the execution it means it has power to alter the judgment of
Pakistan court.

p
For Statement III: Right, There is a sentence in a passage “India’s main contention is that
Pakistan had committed “egregious violations”” it clarifies the India’s concern about Pakistan’s
egregious violation.

ou
For Q2. Ans. (a);
In this question, we have to select that statement which is out of the context in regarding to the
above passage.
For Statement I: Wrong, 1st statement is about the consideration of ICJ and egregious violation of
Pakistan, which is same as context of passage.
For Statement II: Right, As in passage there is not anything mentioned about SAARC nations so it

gr
can be considered out of the context statement.
For Statement III: Right, As in passage there is not anything mentioned about External ministers
trip so it can be considered out of the context statement.
m
Points to Remember
I. Each and every situation is different from each other, so every condition is required
different approach and measure for it so you have to keep it in mind while solving.
II. There may be questions related to the fallout of the situation, repercussions, reasons and
course of action so the concepts of all these topics should be clear to you.
xa
III. These questions are recently seen in many exams and can help to score well in exams.
IV. Logical question as the name itself suggest require a logical answer i.e. it has to be exact,
practical and strong enough according to the question.
V. Critical thinking is the main point that should be really taken care of to solve such questions.
te

Exercise
1. All the members of the Student Rights member of the Student Rights
ce

Coalition signed the petition calling for a Coalition.


meeting with the university trustees. (c) Any member of the student body is
Philip must be a member of the Student eligible to sign a petition dealing
Rights Coalition since his signature with university affairs.
appears on the petition. (d) Philip may have resigned from the
Which of the following best states the
@

Student Rights Coalition after


central flaw in the reasoning above?
(a) Some members of the Student signing the petition.
Rights Coalition may not support all (e) Some of those who signed the
of the organization’s positions. petition may not be members of the
(b) It is possible that Philip’s signature Student Rights Coalition.
on the petition was forged by a
3 Adda247 Publications For any detail, mail us at
Publications@adda247.com
A Complete Book of Logical Reasoning

Direction (2): Study the given information (c) The rating scale was a ten-point
and answer the questions that follow. scale with most attractive scoring
India is renowned as the land of tiger, 1-2 and least attractive scoring 8-
elephant and many other wildlife creatures. 10.
But sadly, the equation that existed between (d) Most popular male actors have the
people and wildlife centuries ago has features that the study allocated to
vanished, and our protected areas, which the more masculine category.

p
comprise mere 4% of India's landscape, are (e) The faces with the more masculine
themselves in an alarming condition for the features were all significantly older
government. than those with the feminine

ou
features.
2. Which of the following is the main
threat to wildlife in India at present? 4. Which is the following is an assumption
I. Increasing number of illegal that the researchers apparently made in
poaching of animals this study?
II. Poor quality of forest (a) The women who participated in the
administration study were sufficiently represent-

gr
III. Reckless development in the forest tative of modern women in general.
area (b) Male faces are, in general, attractive
IV. Lack of political will to women.
(a) Only I (b) Only II (c) Visual images are important to
women.
(c) Only III
(e) All of these
m
(d) Only l and Ill
(d) It is impossible to predict what
features an ideal face would have.
Directions-(3-4) Read the following (e) Women in previous ages would
information carefully and answer the have preferred more masculine
xa
questions which follow- men.
In a recent study of responses to visual Directions (5-7): In each question below is
images, researchers found that women most given a statement followed by three
frequently gave the rating ‘most attractive’ to assumptions numbered I, II and III. You have
images of male faces that were more feminine to consider the statement and the following
te

in contour, and rated more masculine faces, assumptions, decide which of the statement
on average, ‘less attractive’. The researchers and choose your answer accordingly.
concluded that modern women prefer men 5. Statement: Use ‘X’ brand shoes. These
who are less obviously masculine in their are durable and available in all sizes-An
ce

facial features. advertisement in the newspaper A.


3. The conclusion would be most severely Assumptions:
weakened if which of the following were I. Normally people like durable shoes.
true? II. Very few people read
(a) Facial features are not the criterion advertisement in a newspaper.
III. Very few people read the
@

that most women use to decide


whether a man is attractive. newspaper A.
(b) The visual images were computer (a) None is implicit
generated composites of (b) Only I and II are implicit
photographs and not pictures of (c) Only I, and either II or III are implicit
actual men. (d) All are implicit
(e) None of the above

4 Adda247 Publications For any detail, mail us at


Publications@adda247.com
A Complete Book of Logical Reasoning

6. Statement: These apples are too cheap Conclusions:


to be good. (a) All male employees are from out
Assumptions: station.
I. When the apple crop is abundant, (b) All male employees are sincere.
the prices go down. (c) 20 per cent of female employees are
II. The lower the selling price, the not sincere.
inferior is the quality of the (d) All female employees are from

p
commodity. Rawalpura.
III. Very cheap apples are also good. (e) None of these.
(a) None is implicit
9. Statement: In a class, three-fourth of the

ou
(b) Only I and III are implicit
boys play football, half of them play
(c) Only II is implicit
cricket, one-fourth of those who play
(d) Only II and III are implicit
cricket do not play football.
(e) All are implicit
Conclusions:
7. Statement: The employees’ association (a) Two-third of the boys plays only
has appealed to the Managers of football.

gr
Company Z to introduce written (b) One-fourth of the boys play neither
examinations for clerical cadre cricket nor football.
recruitment to prevent selection of (c) One-third of the boys play neither
incompetent persons. cricket nor football.
Assumptions:
I.
m
So far the Company Z used to select
(d) One-eighth of the boys play neither
cricket nor football.
candidates without conducting a (e) Two-fifth of the boys plays only
written examination. football
II. A written examination can help to
xa
Directions (10): In the question below is
identify competent persons.
given a statement followed by some
III. At higher level, written examination
assumptions numbered. You have to consider
may not be of much use.
the statement and the following assumptions
(a) Only I and II are implicit
and then decide which of the assumption(s)
(b) Only II and III are implicit
is/are implicit?
te

(c) Only I and III are implicit


(d) Only III is implicit 10. Statement: “Wanted a two bedroom flat
(e) None of the above in the court area for immediate
Directions (8-9): In each of the following possession”. — An advertisement.
ce

questions, a statement/group of statements is Assumptions:


given followed by some conclusions. Without I. Flats are available in court area.
resolving anything yourself, choose the II. Some people will respond to the
conclusion which logically follows from the advertisement.
given statement(s). III. It is a practice to give such an
@

advertisement.
8. Statement: In this company, 60 per cent (a) All are implicit
of the employees are males, 40 per cent (b) Only I and II are implicit
are females, 80 per cent of the (c) None is implicit
employees are sincere and 40 per cent of (d) Only I and III are implicit
the employees are from this city— (e) None of these
Rawalpura.

5 Adda247 Publications For any detail, mail us at


Publications@adda247.com
A Complete Book of Logical Reasoning

Directions (11-12): In each of the following 14. Statement: "A rare opportunity to be a
questions, a statement/group of statements is professional while you are at home." —
given followed by some conclusions. Without An advertisement for computer literate
resolving anything yourself, choose the housewives by a computer company.
conclusion which logically follows from the Assumptions:
given statement(s). I. Some housewives simultaneously
11. Statement: Hitesh told Mohit a ghost desire to become professional.

p
lived by the peepal tree on the outskirts II. Computer industry is growing at a
of the village. fast pace.
Conclusions: III. It is possible to be a professional as

ou
(a) Peepal trees grow on the outskirts well as a housewife.
of the village. (a) Only I and II are implicit
(b) Ghosts live on peepal trees. (b) Only II and III are implicit
(c) Hitesh perhaps believed in the (c) Only I and III are implicit
stories of ghosts. (d) Only II is implicit
(d) Mohit must be afraid of ghosts. (e) None of these

gr
(e) None of these 15. Statement: Everybody loves reading
12. Statement: Every library has books. adventure stories.
Conclusions: Assumptions:
(a) Books are only in library. I. Adventure stories are the only
m
(b) Libraries are meant for books only.
(c) No library is without books.
reading material.
II. Nobody loves reading any other
(d) Some libraries do not have readers. material.
(e) None of these III. All are literate.
(a) None is implicit
xa
Directions (13-15): In each question below
is given a statement followed by three (b) Only I and II are implicit
assumptions numbered I, II and III. You have (c) All are implicit
to consider the statement and the following (d) Only II and III are implicit
assumptions, decide which of the (e) None of these
assumptions is implicit in the statement and Directions (16 –18): In each question below
te

choose your answer accordingly. is given a statement followed by two


13. Statement: "Buy pure and natural honey assumptions numbered I and II. An
of company X."-An advertisement in a assumption is something supposed or taken
newspaper. for granted. You have to consider the
ce

Assumptions: statement and the following assumptions and


I. Artificial honey can be prepared. decide which of the assumptions is implicit in
II. People do not mind paying more for the statement.
pure and natural honey. Give answer
III. No other company supplies pure (a) If only assumption I is implicit
@

honey. (b) If only assumption II is implicit


(a) Only I is implicit (c) If either assumption I or assumption II is
(b) Only I and II are implicit implicit
(c) Only I and III are implicit (d) If neither assumption I nor assumption II
(d) All are implicit is implicit
(e) None of these (e) If both assumption I and II are implicit

6 Adda247 Publications For any detail, mail us at


Publications@adda247.com
A Complete Book of Logical Reasoning

16. Statement: ‘You are expected to be (c) Travel packages offered by other
frank and objective while writing your tour operators are neither cheap
self-appraisal report’. An instruction for nor comfortable
writing self-appraisal report. (d) Many people desire convenience
Assumption: and comfort while going for a
holiday
I. Unless cautioned, people may tend
(e) None of the above
to be little shy and less objective

p
while writing their self-appraisal 20. The State Government’s agency
report. ‘Housewell’ has constructed 500 flats for
II. Every self-appraisal report helps the middle class but inspite of a shortage

ou
the person in his further of houses, it has not even received 100
applications.
development.
Which of the following, if true, could
17. Statement: The higher echelon of any explain this?
organization are expected to be models (a) A private builder’s scheme which
of observational learning and should not has come up on the adjacent plot is
be considered as merely sources of overbooked inspite of higher cost

gr
reward and punishments. and 100% advance payment.
(b) The flats are nearby village area
Assumption:
and not accessible either by bus or
I. Employees are likely to be sensitive by train easily.
enough to learn by observing the (c) The quality of construction of ‘flat’
behavior of their bosses.
II. Normally bosses are considered as
m is reported to be very poor.
(d) The conditions of payment and
sources of reward and punishment. facilities given in flat are not liked
by people.
18. Statement: ‘But, out of A, B, C and D
(e) None of the above.
xa
products, you should buy ‘B’ which alone
is based on the international technology. Directions (21–23): These questions are
A shopkeeper tells a customer’. based on statement given below and the
Assumption: sentences labeled (1), (2), (3) and (4) as given
below.
I. The customers normally accept the
recommendation of the In an attempt to increase sales, XYZ Ltd.
te

shopkeeper. implemented a new customer service policy


II. Use of international technology is that required all sales representatives to
supposed to ensure better quality address all prospective clients by name.
standards. Within six months, sales had risen 15%. The
ce

new policy was therefore a success.


19. If you want a hassle-free holiday package (1) The customers would like to be address
for city M, then join only, our tour. Hurry by name by the sales representatives.
up; only a few seats available ‘An (2) The XYZ Ltd. has launched a new product
advertisement of ABC Tourist Company.’ which has got tremendous response
If the above statements are true, then from the customers.
@

which of the following has been assumed (3) A recent survey shows that customers
feel more connected and show faith in a
while making the statement?
company when the company
(a) No seats may be available with representative address them by name.
other tour operators for city M (4) A regular customer of XYZ told the
(b) Now-a-days people have a lot of branch manager to address him by name
money to spend on their comforts as he likes it.
7 Adda247 Publications For any detail, mail us at
Publications@adda247.com
A Complete Book of Logical Reasoning

21. Which of the above statements Assumptions:


numbered (1), (2), (3) and (4) was an I. When a show is accompanied with
assumption while implementing the latest technology, it has a magical
new policy? effect.
(a) Only 1 (b) Both 1 and 4 II. Bombay people were never
(c) Only 4 (d) Only 2 impressed with performances by
(e) None of these
Indian musici

p
22. Which of the above statements III. Michael Jackson is a super singer.
numbered (1), (2), (3) and (4) if true will (a) Only I is implicit
weaken the given argument? (b) Only II is implicit

ou
(a) Only 2 (b) Both 2 and 3 (c) Only I and III are implicit
(c) Only 3 (d) Both 2 and 4 (d) Either II or III is implicit
(e) Both 1 and 4
(e) Only II and III are implicit
23. Which of the above statements
numbered (1), (2), (3) and (4) if true will Directions (26-27): In each of the following
questions, a statement is given followed by

gr
strengthen the given argument?
(a) Only 1 (b) Both 1 and 3 some conclusions. Without resolving anything
(c) Only 3 (d) Both 1 and 4 yourself, choose the conclusion which
(e) 1, 3 and 4 logically follows from the given statement(s).
Directions (24-25): In each question below 26. Statement: A factory worker has five
is given a statement followed by three
m children. No one else in the factory has
assumptions numbered I, II and III. You have five children.
to consider the statement and the following Conclusions:
assumptions and choose your answer (a) All workers in the factory have five
xa
accordingly.
children each.
24. Statement: Unable to manage with the (b) Everybody in the factory has
present salary, Arun has decided to join children.
another company. (c) Some of the factory workers have
Assumptions: more than five children.
te

I. The new company has better work (d) Only one worker in the factory has
environment. exactly five children.
II. The present company offers
(e) None of these.
moderate pay packets.
III. The new company offers higher 27. Statement: Most dresses in that shop
ce

salary to all its employees. are expensive.


(a) None is implicit Conclusions:
(b) Only II is implicit (a) There are no cheap dresses
(c) All are implicit available in the shop.
(d) Only II and III are implicit
(b) Handloom dresses in that shop are
@

(e) None of the above


cheap.
25. Statement: Bombay people were (c) There are cheap dresses also in that
spellbound, mesmerized and got mad shop.
when they saw the famous pop-singer (d) People prefer expensive dresses.
Michael Jackson’s hi-tech pulsating (e) None of these.
megawatt performance.

8 Adda247 Publications For any detail, mail us at


Publications@adda247.com
A Complete Book of Logical Reasoning

28. Residents of an apartment complex are II. As the super markets have been set up
considering two possible plans for only in bigger cities at present, this step
collecting recyclable trash. would fail to deliver results in the
Plan 1 - Residents will deposit smaller cities.
recyclable trash in municipal dumpsters III. Super markets help the smaller brands
located in the parking lot. The trash will to break into newer markets without
be collected on the first and fifteenth day investing substantially in distribution.

p
of each month. IV. Super markets charge the smaller brands
Plan 2 - Residents will be given 10% higher than the amount charged to
individual containers for recyclable the bigger brands.
trash. The containers will be placed at

ou
V. Being outnumbered by the bigger
the curb twice a week for trash brands, visibility of the smaller brands at
collection. local grocery stores is much lower as
Which of the following points raised at a compared to the super markets.
meeting of the residents, if valid, would VI. Smaller brands are currently making
most favour one of the recycling plans substantial losses in their businesses.
over the other?

gr
(a) Residents will be required to 29. Which of the statements numbered I, II,
exercise care in separating III, IV, V and VI can be assumed from the
recyclable trash from non- facts/information given in the
recyclable trash. statement? (An assumption is something
(b) For trash recycling to be successful, supposed or taken for granted)
residents must separate recyclable
m (a) Only I (b) Only II
bottles and cans from recyclable (c) Both II and III (d) Both V and VI
paper products. (e) Only VI
(c) Penalties will be levied against
30. Which of the statements numbered I, II,
xa
residents who fall to sort their trash
III, V and VI represents a disadvantage of
correctly.
the small grocery stores over the super
(d) Recyclable trash that is allowed to
accumulate for two weeks will markets from the perspective of a
attract rodents. smaller brand?
(e) None of the above. (a) Only I (b) Only III
te

(c) Only V (d) Only VI


Directions (29-31): Read the following (e) Both II and III
information carefully and answer the
questions which follow. 31. Which of the statements I, II, III, IV and V
mentioned above represents a reason
ce

Small brands are now looking beyond local for the shift from local grocery stores to
grocery stores and are tying up with super super markets by the smaller brands?
markets such as Big Bazaar to pull their (a) Only I (b) Only II
business out of troubled water. (c) Only V (d) Both I and IV
These questions are based on the information (e) Both III and V
given above and the sentences labelled I, II,
@

III, IV, V and VI as given below. 32. Statement: The Indian Railways has tied
I. A smaller brand manufacturing a certain up with Tata Consultancy Services to
product of quality comparable with that pilot a system to conduct its recruitment
of a bigger brand, makes much more exams online, as the country’s largest
profit from the local grocery stores than employer looks to introduce
from the super markets. transparency into its operations.

9 Adda247 Publications For any detail, mail us at


Publications@adda247.com
A Complete Book of Logical Reasoning

Which of the following assumptions is II. The employees may now relax and
implicit in the given information? (An slowdown in their day to day work
assumption is something supposed or as there is no immediate threat of
taken for granted). huge deficit.
(a) Government jobs are the ambition Which of the following assumptions is
of every youth in the country. implicit in the given information? (An
(b) Corruption is a major issue in assumption is something supposed or

p
railway recruitment. taken for granted).
(c) Online examination will reduce (a) Only assumption I is implicit
corruption and bring transparency. (b) Only assumption II is implicit

ou
(d) Cheating in examinations will be (c) Either I or II is implicit
wiped out. (d) Neither I nor II is implicit
(e) All examinations for major (e) Both I and II are implicit.
recruitments in the country will be
35. Cocoas and chocolate products have
conducted online.
been used as medicine in many cultures
33. Statement: Next time, when you lose for centuries. Chocolate is made from

gr
your baggage or meet with an accident plants which means it contains many of
while travelling by train, you need not the health benefits of leafy vegetables.
run from pillar to post and wait for years Which of the following statements
before the cheque lands at your door. weakens the above arguments?

from the given information? (An


m
Which of the following can be inferred I. Dark chocolate contains a large
number of antioxidants which
inference is something that is not slowdowns the aging process.
directly stated but can be inferred from II. A small study revealed that regular
the given information.) intake of chocolate increase insulin
xa
(a) There will be quick settlement of sensitivity thus lowering the
claims into the customer’s account. chances of diabetes.
(b) All travellers’ details will be linked III. Green leafy vegetables have
with Aadhaar card. flavonoids which protect skin from
(c) The loss will be covered by any UV rays.
te

insurance company through IV. Chocolates have 3 types of fats one


railways. out of which increase the
(d) There will be direct settlement of cholesterol level.
claim by the IRCTC. V. Cocoas increases blood flow to the
ce

(e) None of these retina thus giving a boost to vision.


(a) Only IV
34. Statement: The head of the organization
(b) I and V
congratulated the entire staff in his
(c) Only III
speech for their sincere effort to bring
(d) None of the given statements
down the deficit and urged them to give
(e) Both III and IV
@

their best for attaining a more profitable


position in future. 36. There has been a considerable drop in
Assumptions: sales of four wheelers during the past six
I. The employees may get motivated months when compared to the number
and maintain and if possible of four wheelers sold during this period
enhance their present level of work. last year.

10 Adda247 Publications For any detail, mail us at


Publications@adda247.com
A Complete Book of Logical Reasoning

Which of the following can be the Assumptions:


probable cause of the above I. The society always indulge in
phenomenon? adverse comments.
I. The Government has imposed II. People’s trust in the police force is
higher excise duty on four wheelers not up to the desired level.
at the beginning of this year. III. Society has the efficiency to judge
II. The petrol prices have risen accurately about the police

p
considerably during the past eight leadership.
months. Which of the following assumptions is
III. The rate of interest on home and implicit in the given information? (An

ou
car loans have been rising for the assumption is something supposed or
past seven months. taken for granted).
(a) Only I (b) I and III (a) I and II (b) II and III
(c) II and III (d) Only II (c) I and III (d) All of these
(e) All of these (e) None of these
37. Statement For the third time in a row Directions (39-41): In each question below

gr
this week, the Indira Gandhi is given a statement followed by some courses
International Airport was put on a full of action. You have to assume everything in
alert today after Air India received an the statement to be true and on the basis of
anonymous message that two terrorists the information given in the statement, decide
with explosives were aboard its
Mumbai-Delhi-Hong Kong flight. – An
m which of the suggested courses of action
logically follow (s) for pursuing.
authority of Indira Gandhi International
39. Statement The annual requirement of
Airport.
blood in our country is 6 million units
Assumptions:
xa
whereas the generation is only 3.5
I. Anonymous message is not likely to
million units per annum at present.
be hoax
Courses of action
II. Anonymous message is likely to be
I. Blood donors should be attracted
a hoax call.
by telling them how the act of
III. Alertness might be helpful in
donating blood would improve
te

reducing the menace.


their health.
Which of the following assumptions is
II. Blood donors should be attracted
implicit in the given information? (An
by providing them monetary
assumption is something supposed or
incentives.
ce

taken for granted).


III. More and more private blood banks
(a) I and III
should be encouraged.
(b) II and III
(a) Either I or II follows
(c) Either I or II and III
(b) I and II follow
(d) Only III
(c) II and III follow
(e) None of these
@

(d) All follow


38. Statement “You should be very careful (e) Only I follow
so that the society does not adversely
40. Statement A survey conducted in eight
comment on the police leadership”. – A
airports has revealed that airports
statement made by a CBI officer to the
continue to be porous and the security as
newly recruited young officers.
bad as ever.

11 Adda247 Publications For any detail, mail us at


Publications@adda247.com
A Complete Book of Logical Reasoning

Courses of action youths in the state. Of the total 368


I. Adequate perimeter fencing should posts, 268 are for general category and
be provided the rest are for SC, ST and other reserved
II. New machines of latest order categories in the age group of 18 to 40
should be deployed for security years. "When we categorized the
purposes. applications viz a viz a qualification, we
III. Politicians and leaders should be were surprised to find that 255

p
asked to keep off the airport so that doctorates have applied for the job," a
crowds can be avoided. senior secretariat administration official
(a) I and II follow (b) II and III follow said.

ou
(c) Only II follows (d) Only I follows Should people with educational
(e) None of these qualification higher than the optimum
41. Statement One million Indians spent requirements be debarred from seeking
much money last year to buy a personal jobs?
computer. For most of them, the biggest Arguments:
attraction is access to the Internet. I. No, It will further aggravate the

gr
Courses of action problem of educated unemploy-
I. Internet service providers should ment.
be encouraged to set up shop. II. Yes, It creates complexes among
II. Personal computer manufacturers employees and affects the work
adversely.
III. Unemployment should be fought
m
should accelerate their production
III. No, This goes against the basic
tooth and nail. rights of the individuals.
(a) None follows (b) Only II follows IV. Yes, This will increase productivity.
(c) Only I follows (d) All follow (a) Only I and III are strong
xa
(e) II and III follow (b) All are strong
(c) Only II and IV are strong
Directions (42-43): Each question given
(d) Only III is strong
below consists of a statement, followed by
(e) None of these
three or four arguments numbered I, II, III
and IV. You have to decide which of the
te

43. Statement: Sociologists and political


arguments is/are 'strong' (arguments) and observers are of the opinion that making
which is/are 'weak' (arguments) and education qualification mandatory for
accordingly choose your answer from the election candidates would be
alternatives given below each question. discriminatory since that would
ce

42. Statement: A record of sort was created naturally deprive citizens of India the
after the UP government, which right to represent their people.
advertised for 368 posts of peon in Institutional qualification, as they say, is
secretariat, got 23 lakh responses. The different from political acumen and
minimum qualification for the post of wisdom. In a country like India where
@

peon is that the applicant should be a about a quarter of 815 million eligible
class five pass out and know how to ride voters are illiterate, it would not be
a bicycle. Grabbing the opportunity, prudent to deny a politically aware and
opposition parties in unison attacked the socially conscious individual the election
SP-led state government over its ticket just because he/she doesn’t have a
promise of providing employment to minimum qualification.

12 Adda247 Publications For any detail, mail us at


Publications@adda247.com
A Complete Book of Logical Reasoning

Should "literacy" be the minimum Which of the following assumptions is


criterion for becoming a voter in India? implicit in the statement?
Arguments: (a) None is implicit
I. No, Mere literacy is no guarantee of (b) All are implicit
political maturity of an individual. (c) All except III is implicit
II. Yes, Illiterate people are less likely (d) Only I and III are implicit
to make politically wiser decisions (e) Only III is implicit

p
of voting for a right candidate or
45. Statement: Beverage and snacks maker
party.
PepsiCo India is pushing to become more
III. No, Voting is the constitutional right
aggressive in the market as it battles

ou
of every citizen.
Coca-Cola in soft drinks and ITC and
(a) None is strong
Parle in foods. The new hierarchy clubs
(b) Only I and II are strong beverages and foods into an integrated
(c) Only III is strong entity, with functions of marketing,
(d) Only II and III are strong operations, HR, finance, legal, corporate
(e) All are strong affairs and R&D being brought together

gr
44. Statement: The median age in under common heads.
developed countries is around 42 years, Which of the following can be a part of
meaning that half the population is possible fallout of the above situation?
below and half the population is above (a) The food business is getting
42 years old. Reports provided by
m
psychologists indicate that 63% of those
increasingly fragmented.
(b) The company is battling growth
who seek psychological help in slowing across foods and
developed countries are below 42 years beverages, fluctuating weather
conditions that make market
xa
old. Such evidence clearly indicates that
in developed countries psychological behaviour unpredictable and
problems are more common among consumers turning to healthier
people who are younger than 42 years. foods and drinks.
Assumptions: (c) Its top eight brands generate a
I. The percentage of the population business of about Rs. 1000 crore
te

with psychological problems is not each.


significantly higher for developed (d) The company has made sweeping
countries than for other states. changes in its senior leadership
II. One of the primary factors structure.
ce

contributing to the development of (e) None of these.


psychological disorders is inability 46. The increase in the number of
to cope with excessive stress newspaper articles exposed as
resulting from professional activity. fabrications serves to bolster the
III. Among those with psychological contention that publishers are more
problems in developed countries,
@

interested in boosting circulation than in


the percentage who seek printing the truth. Even minor
psychological help is not publications have staff to check such
significantly lower for those older obvious fraud.
than 42 than it is for those younger Which of the following may be the
than 42. assumption of the given argument?

13 Adda247 Publications For any detail, mail us at


Publications@adda247.com
A Complete Book of Logical Reasoning

(a) Newspaper stories exposed as II. The people should be made aware
fabrication are a recent about the imminent danger over
phenomenon. radio/television.
(b) Everything a newspaper print must III. The silt and mud from the river bed
be factually verifiable. should be cleared immediately after
(c) Fact checking is more the receding of the water level.
comprehensive for minor (a) None follows

p
publications that for major ones. (b) Only I and ll follow
(d) The publishers of newspapers are (c) Only II and III follow
the people who decided what to (d) Only I and III follow
(e) All follow

ou
print in their newspapers.
(e) None of the above. 49. Statement: Kapil has twelve years of
typing experience behind him; therefore,
47. Statement: Start to think of travelling by
if you are looking for an efficient typist
train for a holiday. A train journey can to enter your data into the new system,
give one a better view of places on the you need look no further.
way which an air journey cannot give.

gr
The speaker assumes that:
You can walk around whenever you (a) Twelve years of practice ensures
want, meet other travelers and locals, typing efficiency.
relax and watch the landscape go (b) The type of typing required for the
peacefully by. Train travel is both a new system is identical to what
m
beautiful and affordable way to see the
country, and no trip is more scenic than
Kapil has been doing.
(c) Kapil’s job profile is the best that
the trip by trains. the new employer is going to get.
Conclusions: (d) Kapil is an outstandingly fast and
I. While going for a holiday, people accurate typist.
xa
want to enjoy the view of the places (e) Kapil will fit well into the new
on the way. office.
II. People should not travel by air 50. Statement: Should only reputed NGOs
when they are going for a holiday. be authorized to distribute the
Which of the following can be concluded commodities to the public under the
te

from the given statement? programme of Public Distribution


(a) Only I follows System (PDS)?
(b) Only II follows Arguments:
(c) Both I and II follow I. Yes, the move will be helpful to
ce

(d) None follows implement the programme more


(e) Either I or II follows effectively and will keep a tab on
various problems like black
48. Statement: Incessant rain for the past marketing of the commodities
several days has posed the problem of supplied under PDS.
overflowing and flood as the river bed is II. Yes, NGOs have helped Government
@

full of silt and mud. on many occasions.


Which of the following course of action (a) If only argument I is strong
follows from the given statement? (b) If only argument II is strong
Courses of Action: (c) If either I or II is strong
I. The people residing near the river (d) If neither I nor II is strong
should be shifted to a safe place. (e) If both I and II are strong

14 Adda247 Publications For any detail, mail us at


Publications@adda247.com
A Complete Book of Logical Reasoning

51. Statement: Should vacation of Central 53. Statement: “Our cricketers who make
Government employees be reduced? millions are granted income-tax
Arguments exemptions while no relief is offered to
I. Yes, increase in number of working the needy,” –Mr. ‘X’
days will speed up the work done Assumptions
by the Central Government I. Income-tax exemption should be
employees, as a result of which given only to the poor.

p
people will get service from these II. The decision to grant tax exemption
employees in reasonable time. to cricketers, who make millions, is
II. Yes, vacation makes the employees an absolute travesty of domestic

ou
lazy and reduces the stamina of justice in a country where millions
employees to work for long hours. are starving and the middle class is
(a) If only argument I is strong struggling to make money.
(b) If only argument II is strong
54. “We have formulated a 10-point
(c) If either I or II is strong
programme to curb molestation cases
(d) If neither I nor II is strong

gr
against woman and we are confident of
(e) If both I and II are strong
restricting the eve-teasing cases with the
Direction (52-53): In each question below is help of our sincere cops.” –Director-
given a statement followed by two General of Police of city ‘X’.
assumptions numbered I and II. An Which of the following, if true, would
assumption is something supposed or taken
m weaken the statement?
for granted. You have to consider the (a) City ‘X’ is known among those cities
statement and the following assumptions and where there are least crime records
decide which of the assumptions is implicit in (b) City ‘X’ is the most populated city in
xa
the statement. the world
Given answer- (c) On several occasions women have
(a) If only assumption I is implicit complained against police cops and
(b) If only assumption II is implicit have charged them of indulging in
(c) If either I or II is implicit eve-teasing
te

(d) If neither I nor II is implicit (d) Literacy rate of city ‘X’ is the highest
(e) If both I and II are implicit amongst all cities
(e) Many NGOs are cooperating with
52. Statement: Although the prospect of the
the police forces in ensuring a
strike of country Y on country X still
ce

peaceful life in city ‘X’


looms large, the government is
considering revising excise and import Directions (55-56): On the basis of given
duties on oil products to cushion the statements answer the following question:
impact of the steep rise in oil prices. All good athletes want win and all athletes
Assumptions who want to win, eat a well-balanced diet.
@

I. New excise and import duties on oil Therefore, all athletes who do not eat a well-
products may cushion the impact of balanced diet are bad athletes.
the strike on oil price.
55. If assumptions of the argument above
II. As a result of the strike price of oil
are true, then which of the following
may increase and put burden on the
statements must be true?
common man.

15 Adda247 Publications For any detail, mail us at


Publications@adda247.com
A Complete Book of Logical Reasoning

(a) No athlete who does not eat a well- 58. If you want a hassle-free holiday package
balanced diet is a good athlete. for city M, then join only, our tour. Hurry
(b) No bad athlete wants to win. up; only a few seats available — an
(c) Every athlete who eats a well- advertisement of XYZ Tourist Company.
balanced diet is a good athlete. If the above statements are true then
(d) All athletes who want to win are which of the following “has been”
good athletes. assumed while making the statement?

p
(e) None of these (a) No seats may be available with
56. Which of the following if true, would other tour operators for city M.
refute the assumptions of argument (b) Nowadays people have a lot of

ou
above? money to spend on their comforts.
(a) Bob, the accountant, eats a well- (c) Travel packages offered by other
balanced diet, but he is not a good tour operators are neither cheap
athlete. nor comfortable.
(b) Ann wants to win, but she is not a (d) Many people desire convenience
good athlete. and comfort while going for a

gr
(c) All the players on the Burros holiday.
baseball team eat a well-balanced (e) None of these
diet.
(d) Cindy, the basketball star, does not Directions (59-62): In each question below
eat a well-balanced diet, but she is a is given a statement followed by three
good athlete.
(e) None of these
m assumptions numbered I, II and III. An
assumption is something supposed or taken
for granted. You have to consider the
Directions (57-58): Attempt these questions
statement and the following assumptions and
based on the information given against each.
decide which of the assumptions is implicit in
xa
57. In order to qualify in an examination the statement.
having 6 subjects, a student has to get at
least 50% and above marks separately in 59. Statements: “One day token strike was
any 4 subjects and 35% and above in called by the employees in Government
each of the 6 subjects. organizations to protest against
privatization of profit making Public
te

If a total of 25% candidates have


qualified in the examination, then which Sector Undertakings.”
of the following is definitely true? Assumptions:
(a) 50% of the students got 50% and I. The Government many favourably
above in 4 subjects but only half of consider the views of the
ce

them could get 35% and above in employees.


all the subject. II. Strike is the most popular tool used
(b) 75% of the students could not get at by people to protest.
least either 35% marks in all the 6 III. The strike may bring pressure on
or 50% and above in subjects taken the Government, forcing them to
@

seperately. reconsider the decision.


(c) 25% of the students have secured (a) Only I is implicit
50% and above in all the 6 subjects. (b) Only III is implicit
(d) Only 25% of the students could get (c) Only I and III are implicit
at least 35% and above marks in (d) All I, II and III are implicit
each of the subjects.
(e) None of these
(e) None of these

16 Adda247 Publications For any detail, mail us at


Publications@adda247.com
A Complete Book of Logical Reasoning

60. Statements: “Buy a variety of items (a) Only I is implicit


from our shop and get upto 20% (b) Only II is implicit
discount.” – An advertisement. (c) Only III is implicit
Assumptions: (d) Only I and II are implicit
I. Many people will visit the shop to (e) All I, II and III are implicit
get the benefit of discount.
II. Customers may go to some other 62. Statements: “Our school provides all

p
shop if the discount is not offered. facilities like school bus service,
III. Offering discount is the best way to computer training, sports facilities. Also
attract customers. gives opportunity to participate in

ou
(a) Only I is implicit various extracurricular activities apart
(b) Only II is implicit
form studies.” – An advertisement by a
(c) Only III is implicit
(d) Only I and II are implicit public school.
(e) None of these Assumptions:
I. Now a day, extracurricular
61. Statements: “Graduates with first class

gr
activities assume more importance
are eligible to apply for the admission to
over the studies.
MBA courses in our Institute.” – An
advertisement by a Management II. Many parents would like to send
Institute. their children to the school as it
provides all the facilities.
Assumptions:
I. Only those who are first class
m III. Overall care of the child has become
graduates can cope up with the the need of the time as many
studies for MBA courses. women are working.
II. There are plenty of first class
xa
(a) Only I is implicit
graduates who are likely to apply (b) Only II is implicit
for admission to MBA. (c) Only I and III are implicit
III. Reputation of the Institute may get
(d) All I, II and III are implicit
affected if students having less than
first class are admitted. (e) None of these
te

Situational Based Logical Reasoning : Solutions


ce

1. (e); In the above question it is asked to then the subjects could be


choose from a given option which responding to that feature. In
weaken the statement or make it statement E, the correct answer, we
imperfect so correct answer will be have the possibility that the
(e). subjects are responding to age, a
2. (e); situation that is very damaging to
@

the conclusion.
Direction (3-4):
4. (a); The conclusion refers to “modern
3. (e); The researchers are testing women” a fact that suggests the
preference for one feature: the authors think their work applies to
masculinity/femininity of the faces. all modern women. Hence, they do
However, if the faces have other
differences, say race or size or age, assume what is stated in A.

17 Adda247 Publications For any detail, mail us at


Publications@adda247.com
A Complete Book of Logical Reasoning

Direction (5-7): advertisements are given with the


expectation of a response which can
5. (e); Any advertisement highlights only
make such a flat available. So, II is
the desirable qualities of the
implicit. Assumption III does not
product. So, I is implicit. The ‘X’
follow from the statement and so is
brand has advertised its product in
not implicit.
newspaper A. This implies that a
large section of people reads Direction (11-12):

p
advertisements in newspapers and
11. (c); From the statement, it is clear that
that news-paper A too has a side
Hitesh somewhere believed in the
circulation. So, neither II nor III is

ou
stories of ghosts because he is
implicit.
telling such stories to Mohit. So,
6. (c); It is mentioned that the apples are option (c) can be concluded. Other
so cheap that they cannot be good. options cannot be predicted
This means that the prices of good directly.
apples are never too low and that
12. (c); Every library has books doesn't

gr
very cheap apples are never good.
mean that books are only in library.
So, neither I nor III is implicit.
It may be at some other places. But
Assumption II clearly follows from
it is quite clear from the statement
the statement that apples are of
that no library is without books. So,
inferior quality because they are
cheap. So, it is implicit.
m option (c) can be concluded. Other
options cannot be concluded
7. (a); An appeal has been made to completely.
‘introduce’ written examination.
Direction (13-15)
This means that so far written
xa
examination was not conducted. So, 13. (a); Artificial honey can be made. That
I is implicit. II follows directly from is why the word ‘natural’ is
the statement and so it is implicit. mentioned in the advertisement. So,
However, nothing can be deduced I is implicit. No comparison is made
about the mode of selection at of the prices of natural and artificial
te

higher level. So, III is not implicit. honey. So, II is not implicit. Nothing
about the quality of honey of other
Direction (8-9):
companies can be deduced. So, III is
8. (e); None of the given options can be also not implicit.
ce

concluded from the given


14. (c); Clearly, a company would highlight
statement.
only that feature in its
9. (d); From all the given conclusions, only advertisement which people crave
option (d) can be concluded from for and which it can make possible.
the given statement. So, both I and III are implicit.
@

Nothing can be deduced about the


Direction (10): growth of computer industry. So, II
10. (b); The advertisement depicts the is not implicit.
requirement, So it can be assumed 15. (a); The statement mentions that
that the flat is available in court adventure stories are liked by
area. So, I is implicit. Such everybody. This does not mean that

18 Adda247 Publications For any detail, mail us at


Publications@adda247.com
A Complete Book of Logical Reasoning

there is no other reading material requirement of people. If in


or nobody loves reading any other information comfort is mentioned,
material. So, neither I nor II is then company must assuming that
implicit. Besides, ‘everybody’ in the people desire convenience and
statement stands for ‘all literate comfort while going for a holiday.
people’ and not for ‘all people’. So,
20. (d); If middle class is looking for
III is also not implicit.
government flats then he must go

p
Direction (16-18) for quality of construction. It’s
moreover doesn’t matter for them
16. (a); Only assumption I is implicit in the
that the flats are nearby village

ou
statement. The instruction was
area, or they will not look for
issued assuming that people tend to desired facilities.
be little shy and less objective while
writing their self appraisal report if Direction (21-23):
not so instructed. Assumption II is 21. (a); 22. (a); 23. (c);
not properly related with the

gr
statement. Direction (24-25)

17. (e); Both assumption I and II are 24. (b); Nothing about the environment in
implicit in the statement. If the new company is mentioned in
employees do not learn by the statement. So, I is not implicit.
Since, Arun is not satisfied with the
observing the behaviour of their
bosses the statement would not
m present salary, it is evident that the
have made. Again, it has been present company offers moderate
highlighted that bosses should not pay packets. So, II is implicit. The
statement talks only of Arun and
be considered as merely sources of
xa
not all the employees of the new
reward and punishment. It implies
company. So, III is not implicit.
that bosses are generally
considered as sources of reward 25. (c); The use of the words ‘hi-tech
and punishment. Hence, pulsating mega-watt performance’
assumption II is implicit. in the statement makes I implicit.
te

Nothing is mentioned about the


18. (e); Both assumptions I and II are performances of Indian musici So, II
implicit in the statement. If is not implicit. The facts that
customers do not accept or follow Michael Jackson is a pop-singer and
the suggestion of the shopkeeper, his performance left people
ce

the latter has not made such a spellbound, make III implicit.
statement. Again, the shopkeeper
rendered his view assuming that Direction (26-27):
customer will consider his view. 26. (d); From given statement, it is clear
Therefore assumption I is implicit. that only one worker has only five
@

The statement clearly indicates that children. Other worker may have
international technology ensures more than or less than five children.
better quality.
27. (c); It is clear that most of dresses in the
19. (d); If a company make advertisement mentioned shop are expensive but
for their company they make there are also some dresses which
according to the desire and are cheap.

19 Adda247 Publications For any detail, mail us at


Publications@adda247.com
A Complete Book of Logical Reasoning

28. (d); The main issue here is the keep up their spirits and strive hard
frequency of trash collection. for the progress of the organization.
Option (d) indicates a serious So, only I is implicit.
problem within the 2-week
35. (a); Chocolates have three types of fats,
frequency plan. It clearly shows
one out of which increase level of
that plan 2 is better than plan 1.
cholesterol thus this statement
Direction (29-31): weakens the argument because it

p
showing negative quality of
29. (d); The assumption behind the small
chocolates.
brands are tying up with

ou
supermarkets such as Big Bazaar 36. (e); All the statements are probable
are statements V and VI which causes of drop in sales of four
describes that visibility of lower wheelers during the past six
brand products is somewhat same months.so the option (e) is the
as compared to the product of correct answer.
bigger brand at the super markets 37. (c); Why was the airport was put on a

gr
but at local grocery stores the full alert? The authority must be
visibility is very less, and as it is assuming assumption I. Hence,
mention that the smaller brands are Assumption I is implicit but II is not.
tying with super markets in order Again, to put the airport on a full
to compensate the business losses alert indicates that the authority
currently the smaller brands are
m must be assuming assumption III
going through. also.
30. (c); Statement V represents a 38. (e); Suggestion or direction to young
disadvantage of the small grocery officers by the officer is aimed at
xa
stores over the super markets from making the new recruits aware of
the perspective of a smaller brand. their responsibilities. Assumption I
can’t be co related with the above
31. (c); Statement IV represents a reason
statement. Assumption II is an
for the shift from local grocery
assumption; this is what leads to
stores to super markets by the
te

adverse comments. Assumption III


smaller brands.
is not implicit.
32. (c); With the implementation of online Direction (39-41):
exam by the Railways, the
39. (e); Only I follow. What is being told is
ce

corruption in entrance examination


can be checked in the years to genuine. On learning this, the blood
come. donors will get attracted by the
win-win proposition, thus
33. (a); Note that the statements conveys to simultaneously meeting the blood
us that the delay in compensation is requirement to a large extent.
@

now a thing of the past. We can’t Statement II is not a practical


infer about the specifics- ‘insurance course of action. Statement III is not
company’ in 3 and ‘IRCTC’ in 4. an immediate course of action as
34. (a); The appreciation received from the blood banks can be created only
head of the organization would after the sufficient blood donors are
surely motivate the employees to available.

20 Adda247 Publications For any detail, mail us at


Publications@adda247.com
A Complete Book of Logical Reasoning

40. (a); Only I and II follow as III course of 44. (e); The whole argument deals with
action cannot be practically developed countries, so there is no
possible. need to consider how these
41. (c); Only I follows because Only I can be statistics relate to those in other
the immediate course of action. III states. Second choice is irrelevant
cannot be linked to the statement. to the conclusion and therefore
can’t support its validity. If a lot

p
Directions (42-43): more of those who are 42+ and
42. (d); The issue discussed in the troubled simply don’t go to see a
statement is nowhere related to psychologist, then it could be that

ou
increase in unemployment, as the an equal or higher percentage of
number of vacancies filled in will 42+ citizens have problems but just
remain the same. Also, in a working don’t seek help.
place, it is the performance of the 45. (d); Change in leadership structure may
individual that matters and that be a part of the aggressive strategy.
makes him more or less wanted,

gr
Rest of the options does not follow
and not his educational because the decision is expected to
qualifications. So, neither I nor II have been after a proper
holds strong. Besides, the needs of a
assessment of the problem.
job are laid down in the desired
qualifications for the job. So,
recruitment of more qualified
m 46. (b); The exposed articles by the
publication must be verified so that
people cannot augment the authentic and original content
productivity. Thus, IV also does not can publish. So, the statement (b) is
hold strong. However, it is the right the correct assumption. Statement
xa
of an individual to get the post for (c) cannot be assumed as
which he fulfills the eligibility verification of content does not
criteria, whatever be his extra depend upon minor or major
merits. Hence, argument III holds publications. Statement (d) is not
strong. the correct assumption because it is
te

43. (e); Clearly, illiterate people lack will given that publishers are more
power and maturity in thoughts. interested in boosting circulation,
They may easily be misled into false but it cannot be assumed that
convictions or lured into Publishers are the persons who
ce

temptations to vote for a particular decide the content of the


group. So, argument II holds. newspapers.
However, a person is literate does 47. (d); I is an assumption. II is a suggestion
not mean that he is conscious of all but not a conclusion. Hence, neither
political movements, which I nor II follows.
@

requires practical awareness of


everyday events. Thus, I also holds 48. (e); All courses of action followed. I and
strong. Besides, Constitution has II have immediate impact for flood
extended the right to vote equally related problems and river bed
to all its citizens. Hence, III also should be cleared after receding of
holds. water level.

21 Adda247 Publications For any detail, mail us at


Publications@adda247.com
A Complete Book of Logical Reasoning

49. (a); All that the speaker implies is that


53. (d); Neither I nor II is implicit because
Kapil is efficient because he has
both the assumptions are not
twelve years of practice, and so
assumed, it is like an action or fact
option (a) is correct. (b) is
which cannot be assumed.
eliminated because the word
‘identical’ is not mentioned in the 54. (c); How can the strategy to curb
paragraph. (c) is eliminated molestation of woman be effective.

p
because we are talking about typing If the responsibility is on the
not the whole ‘job profile’. (d) is shoulders of those cops who
eliminated as nothing is mentioned himself are culprits of eve teasing.

ou
about Kapil’s speed and accuracy
Direction (55-56):
and we cannot assume efficiency as
speed and accuracy. Similarly (e) 55. (a); 56. (d);
cannot be the answer as nothing is Direction (57-58):
implied about fitting into the office.
57. (a); 58. (d);
50. (a); Only I is strong because it will make

gr
the PDS programme more Direction (59-62):
meaningful and purposeful. II is 59. (b); We know that any action has some
weak because it adds nothing effects, but it is difficult to assume
substantial to the statement. that the Government may favorably
51. (a); II is a weak argument because it
m consider the views of the
employees. For similar reason,
wrongly assumes that vacation
makes employees lazy and less assumption II is also not implicit.
hardworking. I is strong because 60. (a); The advertisement was given to
xa
lesser vacation ensures higher attract the people. But this does not
working days and no doubt imply that offering discount is that
increase in the number of working best way to attract people.
days will reduce the work pending. Therefore, only assumption I is
Direction (52-53): implicit.
te

52. (e); Both I and II are implicit. After 61. (b); The advertisement was issued
assuming that strike will make assuming that sufficient number of
increase in price of oil and it may first class graduates are likely to
put burden on common man, it is apply for admission to MBA.
ce

decided to apply new excise and 62. (b); Only assumption II is implicit in the
import duties by assuming that it statement.
will cushion the impact of strike on
oil price.
@

22 Adda247 Publications For any detail, mail us at


Publications@adda247.com
A Complete Book of Logical Reasoning

p
ou
gr
m
xa
te
ce
@

1 Adda247 Publications For any detail, mail us at


Publications@adda247.com
A Complete Book of Logical Reasoning

Previous Year Logical Reasoning


11 Questions
1. Statement: A record of sort was created 2. Statement: There is little that is

p
after the UP government, which surprising about India’s recent refusal to
advertised for 368 posts of peon in allow Google to launch its street view
secretariat, got 23 lakh responses. The service, which gives users a 360-degree

ou
minimum qualification for the post of view of public spaces. The proposal was
peon is that the applicant should be a rejected following objections raised by
class five pass out and know how to ride the Defense Minister.
a bicycle. Grabbing the opportunity, Which of the following objection(s) may
opposition parties in unison attacked the not be a probable reason(s) for the
SP-led state government over its above step taken by the government?

gr
promise of providing employment to I. The decision is said to have come in
youths in the state. Of the total 368 the backdrop of the terror attack on
posts, 268 are for general category and the Pathankot airbase, with
the rest are for SC, ST and other reserved investigators suspecting that
categories in the age group of 18 to 40 terrorists used Google maps to
years. "When we categorized the
m study the topography of the
applications viz. a viz. a qualification, we targeted area.
were surprised to find that 255 II. The concerns of the defense
doctorates have applied for the job," a organizations should be taken into
xa
senior secretariat administration official consideration.
said. III. India’s security and stability should
Arguments: be given prime importance.
I. No, It will further aggravate the IV. We in India are not mature enough
problem of educated to permit such exploratory things
te

unemployment. threatening national security. We


II. Yes, It creates complexes among have more enemies inside than
employees and affects the work outside.
adversely. (a) Only II and III (b) All except III
III. No, This goes against the basic (c) Only IV (d) Only I and IV
ce

rights of the individuals. (e) None of these


IV. Yes, This will increase productivity.
3. Statement: There are two construction
Should people with educational
companies namely X and Y for the
qualification higher than the
construction of the central mall.
optimum requirements be debarred
@

Construction Company X has offering


from seeking jobs?
24x7 Wi-Fi service, 24x7 power and
(a) Only I and III are strong
water service, guarantee of each room
(b) All are strong
for one year in the mall and even an
(c) Only II and IV are strong
insurance scheme. But the owner has
(d) Only III is strong
provided sponsorship to Company Y.
(e) None of these

2 Adda247 Publications For any detail, mail us at


Publications@adda247.com
A Complete Book of Logical Reasoning

What is/are the reason(s) behind this Direction (5): The question given below
decision of the owner? consists of a statement, followed by three
(i) Company has provided an arguments numbered I, II and III. You have to
insurance scheme of 10 years. decide which of the arguments is/are 'strong'
(ii) Company has provided guarantee of (arguments) and which is/are 'weak'
one and half year for each room. (arguments) and accordingly choose your
(iii) Company Y has provided 24x7 answer from the alternatives given below

p
power and water service at the each question.
same cost.
5. Statement: Sociologists and political
(a) All of the above
observers are of the opinion that making

ou
(b) Only (ii) and (iii)
education qualification mandatory for
(c) Only (i) and (ii)
election candidates would be
(d) Only (ii)
discriminatory since that would
(e) None of these
naturally deprive citizens of India the
4. Statement: However, the market right to represent their people.
participants are eagerly waiting for a Institutional qualification, as they say, is

gr
possible change in GDP growth forecast, different from political acumen and
inflation target for FY 17, along with the wisdom. In a country like India where
RBI’s commentary on foreign exchange about a quarter of 815 million eligible
reserves as around $30 billion of foreign voters are illiterate, it would not be

maturing in September.
m
currency non-resident bank deposits are prudent to deny a politically aware and
socially conscious individual the election
In its April review, the RBI had slashed ticket just because he/she doesn’t have a
the repo rate, the rate at which banks minimum qualification.
borrow from the central bank, by 0.25 Should "literacy" be the minimum
xa
percent to 6.50 percent, to a more than criterion for becoming a voter in India?
five-year low. Since January 2015, the Arguments:
RBI has cut its repo rate by 150 basis I. No, Mere literacy is no guarantee of
points or 1.5 percent. political maturity of an individual.
Which of the following negates the above II. Yes, Illiterate people are less likely
te

steps taken by the RBI? to make politically wiser decisions


(a) The Consumer Price Index (CPI), of voting for a right candidate or
which is closely watched by the RBI party.
in order to set interest rate policy, III. No, Voting is the constitutional right
ce

jumped to 5.39 percent in April vs of every citizen.


4.83 percent in March. (a) None is strong
(b) Global Crude Prices hitting a 12- (b) Only I and II are strong
year low of below $27 dollars a (c) Only III is strong
barrel in January 2016 made a (d) Only II and III are strong
strong case for rate cuts for RBI. (e) All are strong
@

(c) Loans and EMI will become cheaper


6. Statement: Reinforcing its commitment
for retail lo
to green initiatives, railways will explore
(d) Rupee will strengthen against other
its future course of action to prevent
currencies.
pollution. Issues related to environment
(e) None of these.
like solid waste management, pollution

3 Adda247 Publications For any detail, mail us at


Publications@adda247.com
A Complete Book of Logical Reasoning

control, water management and energy (c) Number of bad debts per financial
management will be discussed at a year will increase.
workshop on ‘Environmental challenges (d) Financial stability of the banks will
before Indian Railways and solutions’. be lost.
I. Introduction of bio-fuel and solar (e) All can be inferred.
energy in train operation.
8. Statement: Google, which dominates
II. Installation of rain water harvesting
much of life on the Internet, has been

p
facilities at major stations.
trying to expand beyond computers and
III. Waste water recycling plant is also
being planned in which recycled telephones to living rooms, cars and
bodies. It made its way a bit further into

ou
water will be used for washing and
horticulture purpose. people’s homes when it agreed to
IV. Initiatives such as LED light fittings pay $3.2 billion in cash for Nest Labs,
to reduce power consumption and which makes Internet-connected devices
use of five percent bio-diesel in like thermostats and smoke alarms. But
diesel locos. it has agreed that Nest’s privacy policies
are going to be well-respected. For

gr
V. Introduction of water audit to
assess consumption and wastage at Google, gaining visibility into people’s
major stations. habits beyond computers and phones —
Which of the following points will whether watching television using
strengthen the decision taken by the Chrome cast, taking a walk wearing
Railways?
(a) All of the above (b) All except I
m Google Glass or managing their homes
using Nest products — will provide a
(c) All except I and V (d) All except V fuller picture of users.
(e) None of these Which of the following arguments would
most weaken Nest’s beliefs?
xa
7. Statement: The government is set to
(a) Google products can easily be
overhaul annual targets for public sector
integrated into Nest.
lenders, ending a focus on size that has
(b) Unlike many of Silicon Valley’s most
long encouraged banks to inflate their
popular companies that make
loans and deposits at the year-end to hit
software that costs very little to
growth objectives. Banking and
te

build, Nest is capital-intensive


government sources said that the new
targets, to be discussed at meeting with because it builds hardware.
top state bank officials, would focus on (c) To follow the terms and condition,
efficiency, with objectives set around Google make some changes in
ce

return on assets, or return on equity, and Nest’s privacy policies.


controlling bad debts. (d) The deal between two companies
Which of the following points can be has not yet closed.
inferred from the given information? (An (e) None of these.
inference is something that is not 9. Statement: By fixing a deadline of one
@

directly stated but can be inferred from year from the date of framing of charges
the given information.) for the completion of trial involving the
(a) Loans and deposits of the banks will members of parliament and legislative
go up. assemblies, the Supreme Court has once
(b) Loans on which there is maximum again intervened effectively to give some
probability of return will be credibility to the idea of cleaning the
encouraged.
4 Adda247 Publications For any detail, mail us at
Publications@adda247.com
A Complete Book of Logical Reasoning

polity. It gave a jolt to the political class 11. Statement: Prime Minister Narendra
by striking down a provision that Modi on Friday announced the setting up
protected sitting legislators from of a task force after failure of
immediate disqualification on sportsperson in RIO olympics which will
conviction. help to plan for the “effective
Which of the following can be concluded participation” of Indian sportspersons in
from the above statement? the next three Olympics, to be held in

p
(a) To implement the guidelines, the 2020, 2024 and 2028.
government must set up more He made the announcement at the
speedy trial courts, which will help monthly meeting of the Union Council of

ou
them deliver justice in time. Ministers and said the structure of the
(b) The steps taken by the apex court task force would be in place in next few
will help choose the best people days.
from their constituency and make Which of the following is/are not in line
the Indian democratic system free in contrast with the given statement?
I. The task force will prepare an

gr
from criminalization.
overall strategy for facilities,
(c) This will help in reducing the
training, selection procedures and
scourge of criminalization of
other related matters.
politics.
II. The task force will comprise of
(d) The latest order will help address
this problem by empowering the
m members who are in-house experts
as well as those from outside.
trial court to refuse routine III. Poor infrastructure and training
adjournments. facilities for athletes and political
(e) None of these. interference in sports have been
xa
[RRB PO MAINS 2016] flagged as reasons for the poor
performance in current year.
10. Kapil has twelve years of typing (a) All except II (b) Only II
experience behind him; therefore, if you (c) All except I (d) All of the above
are looking for an efficient typist to enter (e) None of these
te

your data into the new system, you need 12. Statement: Faced with infighting in the
look no further. party and huge disenchantment on the
The speaker assumes that: ground, beleaguered Gujarat Chief
(a) Twelve years of practice ensures Minister Anandiben Patel offered to
ce

typing efficiency. resign and asked the BJP to select a new


(b) The type of typing required for the face.
new system is identical to what By offering to quit, she has pre-empted
Kapil has been doing. any move to remove her. Her decision
(c) Kapil’s job profile is the best that comes at a time when the BJP’s long
@

the new employer is going to get. dominance in the State appears to be


(d) Kapil is an outstandingly fast and under strain, just a year before the
accurate typist. Assembly elections.
(e) Kapil will fit well into the new Will Anandiben Patel’s resignation take
office. BJP on back foot?

5 Adda247 Publications For any detail, mail us at


Publications@adda247.com
A Complete Book of Logical Reasoning

I. No, it doesn’t because one of the (d) The cost of energy to the consumer
important factors behind her would rise with increases in the
resignation is age factor. Also, she energy efficiency of home
was not able to tackle the protests appliances.
and violence. Thus, it is neither a (e) None of these
win-win situation nor does it take
14. Statement:- By asking five prominent
BJP on back foot.
hospitals in the national capital to

p
II. Yes, definitely. The instability in the
deposit nearly Rs. 600 crore to
state post the Patel upsurge, the
compensate for their failure to treat poor
deterioration in health and
patients, the Delhi government has

ou
education sectors, the snail-slow
drawn attention to the social obligation
employment rate are few of the
of healthcare providers in the corporate
many factors for her resignation.
sector as well as the need for timely
This will not just affect the BJP in
enforcement of applicable regulations.
Gujarat but also around the
According to the Delhi government,
country.
trusts and registered societies to which

gr
III. Yes, as the opposition especially the
public land was allotted to establish
pseudo seculars would say that
hospitals were required to earmark a
Gujarat BJP lacks Women
percentage of their medical facilities and
Leadership.
services for indigent patients.
Which of the following arguments holds
strong?
m Assumptions:
I. This strengthens the case for
(a) All of the above (b) All except I
private hospitals to dedicate a part
(c) All except II (d) All except III
of their services to those who
(e) None of the above
cannot afford treatment.
xa
[IBPS PO MAINS 2016]
II. Social responsibility of hospitals
13. Twenty percent of all energy consumed must be monitored and central
in the country is consumed by home government should ensure
appliances. If appliances that are twice compliance by the corporate
as energy-efficient as those currently hospitals.
te

available are manufactured, this figure III. There is a great need for the
will eventually be reduced to about ten government to monitor and enforce
percent. “health services” so that the poor
The argument above requires which of who cannot afford modern health
ce

the following assumptions? facilities that are extremely costly-


(a) Home-appliance usage would not are able to be benefited.
increase along with the energy (a) All are implicit
efficiency of the appliances. (b) Only III is implicit
(b) It would not be expensive to (c) Only I and III are implicit
manufacture home appliances that (d) Only I and II are implicit
@

are energy-efficient. (e) None is implicit


(c) Home-appliance manufacturers
15. The ancient Nubians inhabited an area in
now have the technology to
which typhus occurs, yet surprisingly
produce appliances that are twice
few of their skeletons show the usual
as energy-efficient as those
evidence of this disease. The skeletons
currently available.

6 Adda247 Publications For any detail, mail us at


Publications@adda247.com
A Complete Book of Logical Reasoning

do show deposits of tetracycline, an crucial penalty in the shootout against


antibiotic produced by a bacterium Chile after efficient but goal-less play in
common in Nubian soil. This bacterium regulation and extra time. In fact, Messi’s
can flourish on the dried grain used for overall international record with
making two staples of Nubian diet, beer Argentina has been good, if not
and bread. Thus, tetracycline in their spectacular. With him, the team has
food probably explains the low incidence reached the finals of four important

p
of typhus among ancient Nubi tournaments: the World Cup in 2014 and
Which of the following is an assumption the Copa America in 2007, 2015 and
can be drawn from the passage? 2016.

ou
I. Infectious diseases other than According to the given question, you
typhus to which the ancient have to decide which of the statement
Nubians were exposed are is/are strong on the basis of the given
unaffected by tetracycline. passage:-
II. Tetracycline is not rendered After defeating in Copa America Final,
ineffective as an antibiotic by Should Messi continues his international

gr
exposure to the process involved in career for two years more?
making bread and beer. I. Yes, because it is the only way he
III. Typhus cannot be transmitted by can prove himself to the world as a
ingesting bread or beer challenging person.
contaminated with the infectious II. No, It is his personal decision to
agents of this disease.
m play no more for Argentina and as a
IV. Bread and beer were the only items player he gave a lot of contribution
in the diet of the ancients Nubians to his nation. So we should respect
which could have contained his decision.
xa
tetracycline. III. No, because Messi’s record as a
V. Typhus is generally fatal. player of Barcelona is quite better
(a) Only II follows than as a player of Argentina. So he
(b) Only III & V follow should focus on one side rather
(c) Only III follows than playing for both of them.
te

(d) Only II,III,IV and V follow (a) Only II & III are strong
(e) None of these (b) Only I & III are strong
(c) Only II is strong
16. Statement: It is not often that
(d) Only III is strong
professional footballers retire from
(e) None is strong
ce

internationals at the peak of their game.


When the 29-year-old Lionel Messi, 17. Targeted Delivery of Government
widely reckoned to be the most skilled subsidies and benefits to ensure that
footballer today, announced after the they reach the poor and the deserving.
2016 Copa America Final against Chile What would be the cause of above
that he would not wear the Argentinian government budget?
@

shirt again, he took everyone by (a) Some portion is still untouched


surprise. It is not clear if he has made the from new subsidies and other
retirement call in the heat of the scheme.
moment. It came after Messi failed, yet (b) Some urban sector can get subsidy
again, to win a major title with the scheme profit.
Argentinian team; he also missed a

7 Adda247 Publications For any detail, mail us at


Publications@adda247.com
A Complete Book of Logical Reasoning

(c) DBT ensures related information (a) Only III (b) Only I and II
about the poor people for subsidies. (c) All of three (d) Only II
(d) The transparency and clearance (e) None of these
will be increased.
20. The mushrooming of business schools in
(e) None of these
[SBI PO MAINS 2017] the country is a cause for shortage of
faculty with Ph.D qualification. In
Directions (18-19): Read the following addition, the higher pay and generous

p
passage carefully and answer the questions fringe benefits given by industry has
that follow: encouraged qualified people to not seek
The Budget session began on a stormy note as academic positions.

ou
opposition members demanding a debate on Which of the following statements, if
the recent hate crimes against Indians living true, would tend to STRENGTHEN the
in the USA. Congress blamed the Modi argument?
government for remaining silent on the (a) The average salary for industry
attacks against Indians in US. In his reply, positions in Gujarat is more than
Union Home Minister said that the the average salary for faculty

gr
government has taken a serious note and PM positions in some business schools
will reply in next week of Parliament session. in Ahmedabad by around 30%
In recent weeks, at least two Indians have (b) The average salary for industry
been killed in incidents of hate crime in the positions in Gujarat is less than the
US. “Each attack involved a slogan. Go back to
m
your country”. America condemned it but is
average salary for faculty positions
in a top business school in
not taking any steps to address the issue.
Ahmedabad by around 30%
18. What may be the repercussions after (c) The average salary for recent Ph.D
recent attacks on Indians in US? graduates in the industry is 20%
xa
(I) There may be impact on Indian-US higher than that in academics
trade policy. (d) The rate of growth of salaries for
(II) Indian Government will coerce US the industry positions is equal to
Government. the rate of growth of salaries for
(III) American Govt. will give life academic positions for the past
te

imprisonment to accused to make a three years


set mark that America is against (e) None of the above
racism.
(a) Only I (b) Only I and III 21. The Government has appealed to all
(c) Only III (d) Only I and II citizens to use potable water judiciously
ce

(e) None of these as there is an acute shortage in supply.


19. Which of the following substantiates the Excessive use may lead to huge scarcity
laxity of Indian-American Govt.? in future months.
(I) Modi govt. has taken a serious note Which of the assumptions is implicit in
and ready to reply in next week of the above statement?
@

parliament session. An assumption is something supposed or


(II) Condemning the attack but not taken for granted)
taking any steps by American Govt. (a) People may ignore the appeal and
(III) Still there is no action as racism continue using water as per their
slogan “Go back to your country” is consideration
spreading.

8 Adda247 Publications For any detail, mail us at


Publications@adda247.com
A Complete Book of Logical Reasoning

(b) Government may be able to tap (c) All huge gap exists between the
those who do not respond to the level of competence of technically
appeal qualified graduates and
(c) Government may be able to put in requirements of the industry.
place alternate sources of water in (d) Majority of the technically qualified
the event of a crisis situation persons are migrating from India to
(d) Large number of people may developed countries for better

p
positively respond to the opportunities.
Government’s appeal and help tide (e) None of the above
over the crisis Directions (24-25): In each question below

ou
(e) Only poor are going to suffer from are given a statement followed by two
this shortage of water supply. courses of action numbered I and II. You have
to assume everything in the statement to be
22. Statement: Science is a sort of news
true and on the basis of the information given
agency comparable in principle to other in the statement, decide which of the
news agencies. But this news agency suggested courses of action logically follow(s)
gives us information which is reliable to

gr
for pursuing.
an extraordinary high degree due to Give answer
elaborate techniques of verification and (a) If only I follows;
its capacity to survive centuries. So, (b) If only II follows,
science should be read with as much (c) If either I or II follows;
interest as we read news.
Assumptions:
m (d) If neither I nor II follows and
(e) If both I and II follow.
I. Science encourages investigative spirit.
24. Statement: Every year, at the beginning
II. People read news out of interest.
or at the end of the monsoons, we have
(a) If only assumption I is implicit
xa
some cases of conjunctivitis, but this
(b) If only assumption II is implicit year, it seems to be a major epidemic,
(c) If either I or II is implicit witnessed after nearly four years.
(d) If neither I nor II is implicit Courses of action:
(e) Both I and II are implicit I. Precautionary measures should be
23. A very large number of technically taken after every four years to
te

qualified young Indians are coming cut check this epidemic.


of colleges every year though there are II. People should be advised to drink
boiled water during rainy season.
not enough opportunities for them to get
gainful employment. 25. Statement: Researchers are feeling
ce

Which of the following contradicts the agitated as libraries are not equipped to
views expressed in the above provide the right information to the right
statements? users at the right time in the required
(a) Technically qualified persons are format. Even the users are not aware
for superior to those with standard about the various services available for
@

degrees like BA/BSc/BCom etc. them.


(b) The Government has not done Courses of action:
effective perspective planning for I. All the information available to the
engaging technically qualified libraries should be computerized to
personnel while authorizing the provide faster services to the users.
II. Library staff should be trained in
setting up of technical colleges.
computer operations.

9 Adda247 Publications For any detail, mail us at


Publications@adda247.com
A Complete Book of Logical Reasoning

26. The rate of violent crime in this state is (c) Separating deaths caused by
increased up to 30% from last year. The accidents during the treatment to
fault lies entirely in our system of justice. SARS suffers from deaths caused by
Recently our judge’s sentences have infect of SARS suffers
been so lenient that criminals can now (d) Comparing death rates per
do almost anything without fear of a long thousand members of each group
prison term. rather than comparing total

p
The argument above would be weakened numbers of deaths
if it were true that (e) None of the above
(a) 85% of the other States in the
nation have lower crime rates than [RRB PO MAINS 2017]

ou
does this state 28. Television is no longer the medium of
(b) White-collar crime in this state has entertainment at present. Mobile has
also increased by over 25% in the taken the place of it among the
last year youngsters as they used to spend most of
(c) 35% of the police in this state have their time on it.
been laid off in the last year due to Which of the following can be

gr
budget cuts hypothesized from the given statement?
(d) Polls show that 65% of the (a) Television is still most popular
population in this state opposes among kids.
capital punishment (b) People love to play games at mobile
(e) None of the above
27. During the SARS days, about 23,500
m phone instead of involving in
outdoor activities.
doctors who had treated SARS sufferers (c) Mobile has become the addiction
died and about 23,670 doctors who had for the younger generation.
not engaged in treatment for SARS (d) The older generation is not
xa
sufferers died. On the basis of those comfortable in using mobile phone
figures, it can be concluded that it was for their entertainment.
not much more figures, it can be (e) A proper treatment is required by a
concluded that it was not much more doctor to get rid of the addiction of
dangerous to participate in SARS the mobile phone.
te

treatment during the SARS day than it


was not to participate in SARS treatment. 29. Mosquito bites can be more than just
Which of the following would reveal annoying and itchy. They can make you
most clearly the absurdity of the really sick. Protect yourself and your
conclusion drawn above? family from mosquitoes during
ce

(a) Counting deaths among doctors travelling. Because dangerous diseases


who had participated in SARS like Zika, dengue, and chikungunya are
treatment in addition to addition to spread by mosquitoes.
deaths among doctors who had not Which of the following can be assumed
participated is SARS treatment from the given statement?
@

(b) Expressing the difference between (a) Mosquitoes are more dangerous
the numbers of deaths among than any other animals in the
doctors who had treated SARS world.
sufferers and doctors who had not (b) There is no treatment of diseases
treated SARS suffers as a like Zika, dengue, and chikungunya
percentage of the total number of that are spread by mosquitoes.
deaths
10 Adda247 Publications For any detail, mail us at
Publications@adda247.com
A Complete Book of Logical Reasoning

(c) Forty percent of the world’s (c) India needs to embrace internet
population lives in an area at risk and technology if it has to teach all
for dengue spread by mosquito and of its huge population, the majority
an estimated 390 million people per of which is located in remote
year are infected with the viruses. villages.
(d) There are more chances to get (d) Our education system is geared
infected from mosquitoes while towards teaching and testing

p
travelling rather than staying at knowledge at every level as
home. opposed to teaching skills.
(e) All travelers are suffering from (e) We may have the most number of
diseases like Zika, dengue, and

ou
engineering graduates in the world
chikungunya spread by mosquitoes. that has been translated into much
30. Releasing the 'Road Accidents in India - technological innovation here.
2016' report, the Road Transport and 32. Bihar Chief Minister Nitish Kumar on
Highways Minister said more than 400 Monday launched a State-wide campaign
people lost their lives daily on roads in as one of the major step for women

gr
road accidents at night. empowerment to abolish dowry, stated
Which of the following could be the it as major drawback of our society.
appropriate cause of the death in road Is this decision sufficient to get rid from
accidents at night?
this custom, which is running from long
(a) Increase in population lead to more
time in Bihar?
number of accidents.
(b) Many people prefer to travel in
m Which of the following will be best
suited option to get rid from dowry with
night rather than in day.
their explanation?
(c) Lack of proper lightning
(a) Yes, as State govt. Is launching state
arrangement and undignified
xa
installation of traffic signals at wide campaign, it will impact the
many places. whole society of Bihar.
(d) Traffic rules are stricter in day-time (b) No, It will not impact too much
to avoid traffic congestion problem. untill thinking of people will
(e) Traffic violation is not a punishable changed and dowry will be declared
offence in India. as illegal and punishable offence in
te

Bihar.
31. Education has been a problem in our (c) Yes, as many NGO run by women
country and lack of it has been blamed will also come against dowry.
for all sorts of evil for hundreds of years. (d) No, It has been set in nerves of
Half the country does not even today
ce

Bihar’s society as one of the


have access to proper education, and necessities of the marriage.
only a small fraction can go to university. (e) Yes, As other states will also follow
Which of the following substantiates the Mr. Kumar step to abolish dowry
given statement? from society.
(a) Our education system today
@

encourages excellence – in students, 33. Statement-Whole world looking at


in teachers, throughout the system. India, says Modi. India’s growth presents
(b) Our new education system creates a win-win partnership for both India and
entrepreneurs, innovators, artists, the U.S., Prime Minister Narendra Modi
scientists, thinkers and writers who told while adressing the media gathering
can establish the foundation of a ahead of his first meeting with President
knowledge based economy. Donald Trump.
11 Adda247 Publications For any detail, mail us at
Publications@adda247.com
A Complete Book of Logical Reasoning

(I) The talks between the Trump and (c) Statement III is the cause, I and II
Modi would be broad-ranging and are its effects.
seek to advance common priorities. (d) Statement III is the cause and II is
(II) Prime Minister Narendra Modi its effects while statement I is an
would only attend the UN ‘Vesak effect of some independent cause.
Day’ celebrations and no bilateral (e) None of these
agreements will be signed during

p
his visit to Sri Lanka. 35. Statement: Prime Minister Narendra
(III) US President Donald Trump and Modi launched three mega flagship
Prime Minister Narendra Modi schemes aimed at transforming Urban

ou
forging a strong personal bond, and India, including the much talked about
advancing a solid bilateral Smart Cities proposal, with a whopping
relationship. Rs.4 lakh crore kitty having been lined
(IV) The visit provides an opportunity to up for the projects. He also unveiled logo
strengthen the US-India strategic of the housing mission, which has his
partnership, which the President personal touch in the finalization of its

gr
views as being critical in Asia- design.
Pacific and globally.
Speaking at the event, Modi said, “the
Which of the following is not in line with
decision to make cities smart will be
the given statement?
taken not by governments but by the
(a) Only II (b) Only III and IV
(c) Only II and III (d) Only IV
m people of the city, the local
administration. Let there be competition
(e) Only I and III
between cities in development so that
[IBPS PO MAINS 2017] smart cities come up.”
34. In the following question, three
xa
Which of the following will be an effect
statements numbered I, II and III are of the initiative taken by the
given. Read all the statements and
government?
determine the cause and effect
(a) Ensuring water supply to each
relationship between them.
I. In urban areas, the poor bear the household, sewer connections,
te

brunt of environmental effects such sanitation and solid waste


as water or air pollution. management in rural and urban
II. The rapid urban development has governance would be the focus
outpaced the development of under the scheme.
ce

environmental infrastructure in (b) Quality of life will improve in rural


many large cities. and urban areas.
III. Just for the own sake of profit, (c) Provision of roads and public
builders in urban areas are indulge transportation beside
in construction work without implementation of reforms in urban
tendering about the environment governance would be the focus
@

and urban poor, who mostly dies under the scheme.


from chronical diseases. (d) Millions of jobs will be created and
(a) Statement II is the cause, I and III scarcity of houses will be removed
are its effects. in urban and rural areas.
(b) Statement I is the cause, I and III are
(e) All of the above.
its effects.

12 Adda247 Publications For any detail, mail us at


Publications@adda247.com
A Complete Book of Logical Reasoning

36. The difficulty with the proposed high- (c) Drugs XYZ is also used by the
speed train line is that a used plane can sportspersons to enhance their
be bought for one-third the price of the athletic performance.
train line, and the plane, which is just as (d) Drug -XYZ can be used in the
fast, can fly anywhere. The train would treatment of Glaucoma.
be a fixed linear system, and we live in a (e) None of these
world that is spreading out in all
38. In an attempt to check its increasing

p
directions and in which consumers
choose the free-wheel systems (cars, pollution, Litter City has introduced a
buses, aircraft), which do not have fixed levy of Rs. 100 on every private car

ou
routes. Thus a sufficient market for the entering the city. The administration
train will not exist. argues that the imposition of this levy, in
Which of the following, if true, most addition of the existing toll at the
severely weakens the argument entrance of the city, will switch people
presented above? from using their cars to using the public
(a) Cars, buses, and planes require the
transport.

gr
efforts of drivers and pilots to guide
them, whereas the train will be Which of the following, if true, provides
guided mechanically. the best evidence that the city
(b) Cars and buses are not nearly as administration’s argument is flawed?
fast as the high-speed train will be. (a) The city administration is already
(c) Planes are not a free-wheel system
m facing the ire of taxi drivers whose
demand for a rate hike was turned
because they can fly only between
airports, which are less convenient down by the administration.
for consumers than the high-speed (b) A recent survey has shown that
people living in Litter city have
xa
train's stations would be.
(d) The high-speed train line cannot fewer cars than those in the
use currently underutilized train neighboring clean city.
stations in large cities. (c) Hardly 10% of the regular bus
(e) For long trips, most people prefer passengers own their own cars.
(d) The recent hike in the toll for cars
te

to fly rather than to take ground-


level transportation. already makes it more expensive
for people to take a private car into
37. Statement A. City X has recently banned the city than travel in a public
the sale of a drug –XYZ, citing its misuse transport.
ce

by youths for other activities. (e) Both (a) and (d)


Statement B. City Y has recently lifted its
ban on the sale of the drug XYZ, after a Direction (39-40): Read the following
sharp rise in the number of cases of information answer the given questions.
Glaucoma. The agitated guardians of grade IX students of
Which of the following can be inferred a high-profile public school of North Carolina
@

from the above statements? protested against the newly introduced


(a) Drug XYZ is only used in the audio-visual aid of teaching methodology.
treatment of Glaucoma. According to them, it is of benefit to a handful
(b) The number of Glaucoma patients in of students only, and majority of students will
city X is going to increase in the not be getting any help because of this
future. technology.

13 Adda247 Publications For any detail, mail us at


Publications@adda247.com
A Complete Book of Logical Reasoning

These questions are based on the information (d) The US can never convince a
given above and the sentences given below country to give up its nuclear
labeled as I, II, III, IV and V. initiatives.
I. Change is a gradual process and it takes (e) None of these
some time for people to adjust to it.
II. Guardians are happy with the new [CANARA PO 2018]
teaching methodology, i.e. via the use of 42. In the following question a statement is

p
audio-visual techniques. given followed by two more statements
III. It is not the responsibility of the school numbered as I and II. You have to read
to take care of the weaker students. both the statement and decide which of

ou
IV. This modern technology of teaching them strength which of them
should be scrapped as soon as possible.
strengthens or weakens the statement.
V. Guardians are unhappy with the new
Statement: - In the recent 12th boards
teaching methodology used in the school.
examination, school X stood out as one of
39. Which of the following statements above the best performers in the city in terms
could be a possible conclusion from the

gr
of success rate as 93% of its students
above passage? were able to pass the examination.
(a) Only I (b) Only II
I. Schools X provides quality
(c) Only IV (d) Both IV and II
education to its students studying
(e) Only V
in 6th to 8th standard.
40. Which of the following could be a
possible statement that should be used
m II. School X only promotes those
students in class 6th to 10th, who
by the school management to defend perform well in the annual
themselves?
examinations.
(a) Both II and V (b) Only III
xa
(a) Both I and II strengthens
(c) Only I (d) Only V
(e) Both IV and III (b) I strengthen while II weakens
(c) II strengthens while I weaken
41. The US President has downsized his (d) Statement I weaken while
country’s nuclear arsenal, helped to Statement II is neutral.
negotiate a deal to halt Iran’s nuclear-
te

(e) Both I and II weakens.


weapons programme and led a global
initiative to secure radioactive materials. 43. In the following question a statement is
But his legacy on nuclear issues remains given followed by three more statements
uncertain, as the progress in securing numbered as I, II and III. You have to
ce

nuclear materials has been slow. read all the statement and decide which
Which of the following is an assumption of them weakens the argument
of the speaker?
mentioned in statement.
(a) There could be a break in the
Statement: - The population of
president’s initiative at securing
radioactive materials. butterflies in butterfly farms of village A
@

(b) One’s lasting fame depends on the have been declining ever since a rubber
velocity with which one’s initiatives factory was set up. The pollution created
are carried out. by factory must have been the cause for
(c) The deteriorating progress will this decline.
eventually lead to the failure of the Which of the following weakens the
initiative. above argument?
14 Adda247 Publications For any detail, mail us at
Publications@adda247.com
A Complete Book of Logical Reasoning

I. Village B which has more vehicles II. College Y does not have video
than Village A hasn’t observed any lecture facility and yet more
decline in the population of students have enrolled for
butterflies in its farms. admission in college Y than college
II. Large quantities of water required X.
by the factories have created a (a) Both I and II strengthens
shortage of water in the Village A (b) I strengthens while II weakens
due to which most of its butterfly (c) II strengthens while I weakens

p
farms are running dry. (d) I weakens while Statement II is
III. Butterflies are very sensitive to neutral.
pollution and can only thrive in (e) Both I and II weakens.

ou
areas with good air quality.
46. In the following question a statement is
(a) Both I and II (b) Both II and III
given followed by four more statements
(c) Only II (d) Only III
numbered as I, II, III and IV. You have to
(e) None of these
read all the statements and decide which
44. Malaika Arora, a famous actress used to of them can be inferred from the
wear the clothes designed by Losen statement given below.

gr
designers but this time she tried a new Statement:- The chairman of ABC cakes,
designer to design her clothes. Mr. X recently stated –“We are now the
Which of the following cannot be the second largest cake seller in the country.
reason of the given statement? We only sell vegetarian products. If use
(a) Malaika Arora may have some tiff eggs in our products, we could become
with Losen designer so she moved
to the new designer.
m the highest cake seller in the country
within three to four weeks.”
(b) Malaika Arora has to go for a big I. The largest cake seller in the
international event so she wants to country uses only eggs in its
look best. products.
xa
(c) All the given options can be reason. II. The chairman of ABC cakes is a non-
(d) New designer is quite good in vegetarian.
his/her work. III. Eggless cakes are cheaper than the
(e) The designs of new designer are ones with eggs in it.
unique and different. IV. Addition of new variety of cakes
te

with eggs will increase the revenue


45. In the following question a statement is
of ABC Company.
given followed by two more statements
(a) Only I and III (b) Only II and IV
numbered as I and II. You have to read
(c) Only IV, II and III (d) Only I and IV
both the statement and decide which of
(e) Only IV
ce

them strength which of them


strengthens or weakens the statement. 47. In the following question a statement is
Statement: One of the teachers of college given followed by four more statements
X told his chairman that “video lectures numbered as I, II, III and IV. You have to
have no significant effect on the read all the statements and decide which
results/grades of the students. So we can of them can be inferred from the
@

do away with this facility.” statement given below.


I. Number of candidates seeking Survey Report: - Two mobile companies
admission in college X has XYZ and ABC launched a mid-range
increased considerably this year in phone with same price and
comparison to previous years after specifications. However, the sales of ABC
the introduction of video lecture mobile company doubled within three
facility. months after the launch of new phone.
15 Adda247 Publications For any detail, mail us at
Publications@adda247.com
A Complete Book of Logical Reasoning

I. XYZ company has a larger market 50. Statement: - City X has been facing a
share than ABC Company. problem of lack of parking space near its
II. More and more people are getting railway station for the past few years.
addicted to smartphones nowadays. Railway authorities should acquire a
III. The mid-range phone launched by nearby land which is used by vegetable
ABC Company is more popular than vendors.
its other range of products. Which of the following supports the

p
IV. Company ABC has more brand argument mentioned in the above
value than XYZ. statement?
(a) I, II and III (b) II, III and IV I. All the vegetable vendors near the

ou
(c) Only IV (d) Only III station are illegally using the land
(e) Both III and IV which is owned by the government.
48. Statement: - ABC company is working II. Having a shop near the railway
on a long-range wireless charging station reduces the transportation
system that will send power to your cost for the vendors.
device wherever it is in the room, no III. A multilevel parking which is only

gr
cords or careful placement on a charging two kilometers away from the
mat needed. station is under construction.
Which of the following conclusion can be IV. Some residents near the station
drawn from the above statement? reported incidents of cheating by
I. People want more convenient way the vegetable vendors like using
to charge their device.
II. Wireless charging technology will
m false weights and overcharging.
(a) Only IV and I (b) Only I
be a groundbreaking and (c) Only IV (d) Only I, II and III
environmental friendly technology. (e) None of these
xa
III. ABC Company will become a billion
dollar company after the launch of 51. In the following question a statement is
its wireless charging system. given followed by two more statements
(a) Only I (b) Only II numbered as I and II. You have to read
(c) Both I and II (d) Both II and III both the statement and decide which of
(e) Both I and III them strength which of them
te

strengthens or weakens the statement.


49. Statement: - A country named Ausia
The employees of Company A who
imports volcanic ashes from another
resigned were one third of the
country named Puzzolinia and uses it in
employees who resigned from Company
making cement.
ce

Z.
Which of the following assumption is
(I) The Company A has 450 employees
implicit in the above statement?
I. Puzzolinia has surplus amount of whereas the company Z has one
volcanic ashes. lakh employees.
II. Ausia and Puzzolinia are (II) The number of departments in
Company A is 45 whereas in
@

neighbouring countries.
III. Puzzolinia earns a majority of its Company Z is 24.
foreign exchange through the (a) Both statement (I) and statement
export of volcanic ashes. (II) is the neutral statement.
(a) All I, II and III (b) Only I (b) Both statement (I) and statement
(c) Both I and II (d) Both II and III (II) strengthen the given statement.
(e) Both I and III
16 Adda247 Publications For any detail, mail us at
Publications@adda247.com
A Complete Book of Logical Reasoning

(c) Both statement (I) and statement 54. “If we increase our entry fees by 15
(II) are neutral statements. rupees for the visitors, we can do the
(d) Only statement (I) weakens but maintenance work in our park properly
statement (II) strengthen the given to increase the number of visitors in our
statement. park as compared to our only rival
(e) Both statement (I) and statement Gulmohar amusement park,” A
(II) weaken the given statement statement by the owner of Adventure

p
amusement park.
[SYNDICATE PO 2018] Conclusion:
52. Statement- “All the population of City Y (I) Gulmohar amusement do not

ou
relocated to City Z which is near to City require to increase the entry fees to
Y.” perform the maintenance work in
Which of the following can be assumed their park.
from the given statement? (II) Adventure amusement park cannot
Assumption: (I) There is a situation of perform any further maintenance
drought in city Y from past few months work without increasing the entry

gr
and the people of that city are facing lots fees.
of problem due to scarcity of water. (III) Gulmohar amusement park do not
(II) City Z is more developed and rich in want to increase the number of
terms of resources as compared to City visitors in their park
Y. (IV) By proper maintenance the number
(a) Only II follows
m of visitors will increase in
(b) Only I follows Adventure amusement park as
(c) Either I or II follows compared to its rival Gulmohar
(d) Neither I nor II follows amusement park.
xa
(e) Both I and II follows Which of the following can be concluded
from the given statement?
53. Statement- In spite of tremendous
(a) Only II follows
efforts of Sahara NGO of Satava district,
(b) Both II and IV follows
people are still uneducated of that tribal
(c) Both I and III follows
area.
te

(d) Only III follows


Which of the following is the course of
(e) Both I and II follows
action for the given statement?
Course of action: 55. Which of the following arguments is
(I) The people of that tribal area strong in respect of the given statement?
ce

should get shifted to the nearby Statement: Hybrid seeds should also be
distict or city which consist of used in India as it is used in western
adequate resources require for countries for better yields of crops.
education. Arguments:
(II) Government should provide I. It is also used in many other Asian
sufficient support to that NGO for countries and result is better in
@

the education of that tribal people. respect of yielding of crops.


(a) Only II follows II. Weather condition of different
(b) Only I follows countries are different.
(c) Either I or II follows III. Yield of crops will increase in
(d) Neither I nor II follows beginning but fertility of soil will
(e) Both I and II follows reduce further.

17 Adda247 Publications For any detail, mail us at


Publications@adda247.com
A Complete Book of Logical Reasoning

IV. Total costing for Indian farmers will consumption, resulting in severe food
be high if they use hybrid seeds poison, allergies and in extreme cases
instead of domestic seeds. even death.” –A scientist.
(a) I and III (b) II, III and IV Assumptions
(c) All I, II, III and IV (d) II and IV i. Non-biodegradability nature makes
(e) None of these plastic bags an environmental
Direction (56-57): A statement given is hazard.

p
followed by some reason. You have to select ii. Use of recycled coloured plastic
the possible reason for the given statement. bags is harmful and has cascading
effects on human life

ou
56. Statement: New manager of Company X (a) Only (i)
has decided to give allowance for the (b) Both (ii) and (ii)
employees too which has less than 3 (c) Only (ii)
years of experience if they work after (d) Either (i) or (ii)
working hours which was not getting (e) None of these
earlier.
59. It has been reported in recent years that

gr
Possible reason:
I. New Manager has authority to give a very large number of seats in the
allowance without asking to the engineering colleges in the country
higher authority. remain vacant at the end of the
II. There was provision of allowance admission session.
m
for the employee too which has less
than 3 years’ experience in his
Which of the following may be the
probable cause of the above effect?
previous company. (a) There has been a considerable
III. It has been observed that decrease in hiring of engineering
employees who left the company graduates due to economic
xa
since last 1 year had the same slowdown in the recent years
problem. (b) Students have always preferred to
(a) Only I (b) I and III complete graduation in three years
(c) II and III (d) I and II time instead of four years for
(e) None of these engineering
te

57. Statement: Government has increased (c) The Government has recently
10% seats in medical college from this decided to provide post
academic session. qualification professional training
I. Number of seats for women has to all engineering graduates at its
ce

been increased by 20. own cost


II. Hospitality is not better as there is (d) There has always been a very poor
scarcity of doctor in India. success rate among the engineering
(a) Only I (b) Only II students
(c) Either I or II (d) Neither I nor II (e) None of the above
(e) Both I and II
@

60. The condition of the roads in the city has


deteriorated considerably during the
[SBI PO MAINS 2018] first two months of monsoon and most of
58. Statement “Beware! Recycled coloured the roads have developed big pot holes.
plastic bags contain harmful colour Which of the following can be a possible
pigments, which on coming in contact effect of the above cause?
with food particles, make food unfit for

18 Adda247 Publications For any detail, mail us at


Publications@adda247.com
A Complete Book of Logical Reasoning

(a) The municipal corporation had 62. Statement I - Aadhar has become
repaired all the roads in the city mandatory for filing income tax returns
before onset of monsoon with good with PAN card.
quality material Statement II - PAN has become more
(b) A large number of people have and more susceptible to duplicity and
developed spine related injuries fraud.
after regularly commuting long 63. Statement- According to the latest

p
distances by road within the city research-“Few people openly admit to
(c) The municipal corporation has been holding racist beliefs on the basis of
careful in choosing the contractors color but many psychologists claim most

ou
for repairing roads in the past of us are nonetheless unintentionally
(d) People always complain about racist. We hold, what are called "implicit
potholed roads during the monsoon biases".
months One the basis of the information
provided in the above statement, which
(e) None of the above
of the following defines the term
Direction (61-62): In each of the following “implicit bias” most accurately?

gr
questions, two statements numbered I and II (i) A slight automatic preference for
are given. There may be cause and effect white people over black people.
relationship between the two statements. (ii) Preference for white people over
These two statements may be the effect of the black people.
(iii) Discriminating against a particular
same cause or independent causes. These
statements may be independent causes
m community on the basis of colour.
(a) Only (i)
without having any relationship. Read both
(b) Both (ii) and (iii)
the statements in each question and mark (c) Only (ii)
your answer as-
xa
(d) Only (iii)
(a) If statements I is the cause and (e) None of these.
statement II is its effect.
(b) If statements II is the cause and [RRB PO MAINS 2018]
statement I is its effect. 64. Statement: The State Government has
(c) If both the statements I and II are decided to declare ‘Kala Azar’ as a
te

independent causes. notifiable disease under the Epidemics


(d) If both the statements I and II are effects Act. Family members or neighbours of
of independent causes. the patient are liable to be punished in
(e) If both the statements I and II are effects case they did not inform the State
authorities.
ce

of some common cause.


Courses of action:
61. Statement I- Infosys, among India’s I. Efforts should be made to
largest software services companies, effectively implement the Act.
said it plans to hire 10,000 American II. The cases of punishment should be
workers over the next two years. propagated through mass media so
@

Statement II- In filling these jobs, that more people become aware of
Infosys will hire experienced technology the stern actions.
professionals and recent graduates from (a) If only I follow
major universities, and local and (b) If only II follow
community colleges, and the talent pools (c) If either I or II follow
will be created in the future. (d) If neither I nor II follow
(e) If both I and II follow

19 Adda247 Publications For any detail, mail us at


Publications@adda247.com
A Complete Book of Logical Reasoning

65. Statement- Two companies A and B Assumptions:


signed a deal in which both have agreed I. Timely rain is essential for farming.
to not disclose any confidential details of II. Most farmers are generally
their company to the rivals. dependent on rains.
Which of the following could be the
68. Which of the following can be concluded
reason of the legal notice sent by
from the given statement?
company B to company A for breaching
(a) The CMC products are not so

p
the contract?
popular in city Y.
I. Company A is unlawfully receiving
(b) The CMC products with so many
more amount of profit without
health benefits are very famous and

ou
indulging company B in it.
in regular use by the people of city
II. Company A is sharing the private
X.
data and details of company B for
(c) The quality of the products is not
its own profit.
that good as it was initially of the
(a) Only I is implicit
CMC products.
(b) Only II is implicit
(d) A rival company which has come up

gr
(c) Either I or II is implicit
with its product with lower price
(d) Neither I nor II is implicit
are becoming popular among the
(e) Both I and II are implicit
people of city X.
Directions (66-67): In each question below (e) The price of CMC products has been
is given a statement followed by two
assumptions numbered I and II. You have to
m increased significantly in a decade.
69. Which of the following can be inferred
consider the statement and the following
about the decrease in demand of CMC
assumptions and decide which of the
products?
assumptions is implicit in the statement.
xa
(I) The quality of products offered by
Give answer-
CMC reduced which decreases the
(a) If only assumption I is implicit.
demand of the products.
(b) If only assumption II is implicit.
(II) The prices are hiked of CMC
(c) If either I or II is implicit.
products after seeing its popularity
(d) If neither I nor II is implicit.
which in turn reduces its demand.
te

(e) If both I and II are implicit.


(III) The people of city X do not need the
66. Statement: The ‘M’ Cooperative Housing healthy products now.
Society has put up a notice at its gate (IV) The people of city X are now much
that sales persons are not allowed inside more influenced with the foreign
ce

the society. products, which affects the demand


Assumptions: of CMC products.
I. All the sales persons will stay away (a) Only I and II (b) Only III
from the ‘M’ cooperative Housing (c) Only II and IV (d) All of the above
Society. (e) Only I
@

II. The security guard posted at the


gate may be able to stop the sales [IBPS PO MAINS 2018]
persons entering the society. 70. Statement-The Indian Council of
Medical Research issued a warning two
67. Statement: “If it does not rain
years ago, based on studies conducted in
throughout this month, most farmers
hospitals that resistance to antibiotics
would be in trouble this year.”

20 Adda247 Publications For any detail, mail us at


Publications@adda247.com
A Complete Book of Logical Reasoning

was found in 50% of patients. A large 72. Statement- More than 9 million high-
number of infants were dying due to school students across China are taking
infections that did not respond to the biggest exam of their lives known as
treatment. the gaokao, this week. For millions of
Courses of action- teenagers, it's a big source of stress. For
I. The revision of antibiotics classes many businesses, it's a great chance to
by the World Health Organisation in make money.
Which of the following statements/facts

p
its list of essential medicines is a
welcome step in the global initiative substantiates the importance of this test
to push back against antimicrobial for Chinese poeple?
resistance, the phenomenon of (i) The grueling test determines where

ou
bacteria becoming resistant even to the students can go to university
the most potent drugs. and what they'll study.
II. Close scrutiny of these by national (ii) Parents are willing to loosen the
stewardship programmes such as purse strings to try to ensure
those initiated by the ICMR is success, splashing out on private
needed. tutors, good luck charms and even

gr
Which of the following will be the course luxury hotels.
of action? (iii) Chinese students have also begun
(a) Only I (b) Both I and II spending more on drugs and
(c) Either I or II (d) Only II dietary supplements that they
(e) None of these believe will improve their memory
71. Statement- India is buying over 1,000
new planes. However unprecedented
m and attention.
(a) Only (i)
expansion will place immense pressure (b) Both (i) and (ii)
on the aviation system. The industry (c) Only (iii)
currently appears to be underestimating (d) Both (ii) and (iii)
xa
the challenges ahead. (e) All (i), (ii) and (iii)
Which of the following could be the
73. Statement- Thousands of farmers in
reason for the concerns about
“challenges ahead” in the above drought-hit western Madhya Pradesh
statement? have been protesting for over a week
(i) The buying frenzy, led by budget demanding that authorities increase the
te

airlines like SpiceJet and IndiGo, is minimum price for their crops and waive
not without reason. Around 220 their bank lo Madhya Pradesh is one of
million Indians flew last year -- an several states that have suffered
annual increase of 20%. droughts and crop failures in recent
years.
ce

(ii) The country is on track to overtake


the U.K. as the world's third-largest Which of the following courses of action
aviation market by 2026. will help to improve the current
(iii) Only 75 out of 400 airports are scenario?
currently operational in India and (i) Government should punish those
even the biggest hubs like Mumbai, involved in the protests.
@

New Delhi and Chennai are bursting (ii) Cloud seeding should be used in
at the seams. such situations.
(a) Only (i) (iii) Government should constitute a
(b) Both (i) and (ii) team to assess the severity of
(c) Only (iii) drought and convince that
(d) Both (ii) and (iii) appropriate compensation will be
(e) None of these. paid.

21 Adda247 Publications For any detail, mail us at


Publications@adda247.com
A Complete Book of Logical Reasoning

(a) Only (i) (c) Arista Company used to launch its


(b) Both (i) and (ii) new products with the name of
(c) Only (iii) famous action movies so that it
(d) Both (ii) and (iii) became popular among people.
(e) None of these. (d) A famous toothpaste brand used to
74. Statement- Drinking in moderation can add a new ingredient with the
help our health, some research has existing one time to time without

p
showed. Many doctors recommend a changing its packaging so that it
glass of wine or beer a night as part of remains popular among people.
diet plans such as the Mediterranean (e) All are similar types of marketing.

ou
diet and the DASH diet, which have been 76. Statement- A car company ‘Aristo’ was
proven to keep your heart and brain famous car manufacturing company 7
healthy. However, a new study suggests years ago but company’s production has
that even moderate drinking may not be been declining from past 5 years. So,
great for your brain. after all these mis happenings the
Which of the following can be deduced

gr
company is forced to shut down now.
from the above statement? Which of the following could be the most
(i) Scientific researches can contradict possible reason for the above-mentioned
at times. statement?
(ii) Moderate intake of wine is less (a) The company did not have enough
harmful than cigarette.
(iii) Something which is not good for
m funds to buy raw material which
was required for the product.
brain does not mean it cannot be
(b) The employees of the company are
healthy.
not satisfied in their job.
(a) Only (i)
xa
(c) The machinery was too old and was
(b) Both (i) and (ii)
not in proper working condition.
(c) Only (iii)
(d) The demand of the product has
(d) Both (ii) and (iii)
(e) None of these. been declining from past 5 years.
(e) The rival company’s car are fully
75. Statement- Promos Marketing is a kind
te

automatic and is composed with all


of marketing in which people used to new features.
buy those products again and again with
the one they are more familiar with as 77. Statement- The reason for Trump’s
compared to the newly introduced one. victory in USA elections was that the
ce

Which among the following is most Voters believed their political apparatus
similar type of Marketing? was corrupt and Trump was the only one
(a) A well-known brand has not who reliably affirmed that belief and
changed its logo from past 10 years promised to fix it. While Hillary Clinton
and still making profit from its held campaign rallies with Beyoncé and
@

product. Jay-Z, Trump was out talking about the


(b) ‘Wister’ Company has launched a "forgotten" working class, which in turn
new product and advertised it with exacted a "revenge" on the political elite
old theme songs so that everyone is by voting for him.
aware of it and also making profit Which of the following can be inferred
from it. from the given statement?

22 Adda247 Publications For any detail, mail us at


Publications@adda247.com
A Complete Book of Logical Reasoning

(a) Hillary Clinton is a corrupt leader. gets complete salary in an envelope. In


(b) Trump is the only leader who is second group, each of the employee gets
concerned about the working class. salary in two equal parts in two
(c) Only working class voted for envelops. In third group, each employee
Trump. gets salary in an envelop having their
(d) The reason for trump’s success is child’s name on it. All the employees of
the polarization of the people third group spend least among all and

p
towards Trump as he was taking employees of second group spend less
out all the issues and problems of than the first one.
general public. What may be the thought process of the

ou
(e) The reason of Hillary Clinton’s company behind this step?
defeat was the campaign rallies (I) It is a policy of the company as the
with Beyoncé and Jay-Z, which employees are categorized in three
backfired on her. groups based on their performance.
(II) It is human psychology as those
78. Statement- As the festive season is who get salary in two parts spend
round the corner, the country is on high less. Because they would think

gr
alert due to rise in the level of pollution. before spending the second
So, Government has given the orders to envelop. And third group spend
use green crackers in place of least as it contained their child’s
conventional crackers just few days name which itself has some
emotional quotient. So, it will
before Diwali. These green crackers are
environment friendly and cost effective
m promote the saving habits among
employees.
fire-crackers and produces 20-30% less
(III) Company divided the employees in
pollution over conventional one. three groups according to their
Which of the following is incorrect with
xa
saving habits as those who are in
respect to the given statement? second group are more saving
(I) Fire crackers release pollutant oriented as compared to the first
which in turn mixed with air and one.
cause pollution. What could be the reason behind the less
(II) In festive season the level of spending of second group as compared
te

pollution gets increased. to the first one?


(III) The government is taking various (a) I and II (b) II and III
steps to decline the level of (c) I and III (d) only II
pollution. (e) only III
ce

(IV) The green fire crackers are quite 80. Delhi metro commuters may face tough
expensive, so the shopkeepers will time from Saturday as Metro's 9,000-odd
have to bear loss. non-executive employees have
(a) I and II (b) II and III threatened to go on a strike from June 30
(c) I and IV (d) only IV after little headway during talks between
(e) only III the management and the workers. The
@

non-executive workforce including train


79. Statement- There is a company in which operators, station controllers,
there are three different groups of technicians, operations staff,
employees in which each of the maintenance staff, have been protesting
employee gets salary in three different since June 19 at various depots and
ways. In the first group, each employee control rooms of the Delhi Metro for
revision of salaries and other issues.
23 Adda247 Publications For any detail, mail us at
Publications@adda247.com
A Complete Book of Logical Reasoning

The workforce play a critical role in Directions (82): Each question given below
running the Metro network and the consists of a statement, followed by two
strike may totally cripple the services. arguments numbered I and II. You have to
decide which of the arguments is a ‘strong’
Last year too, Metro services had come
argument and which is a ‘weak’ argument.
to the brink of a shutdown when the Give answer-
non-executive staff had threatened to go (a) If only argument I is strong
on strike on similar demands. The crisis (b) If only argument II is strong

p
was, however, averted at the last (c) If either I or II is strong
moment after the DMRC management (d) If neither I nor II is strong
(e) If both I and II are strong

ou
and staff council came to an agreement
following a series of meetings. The 82. Statement: Should all the unauthorized
employees are demanding pay hike, structures in the city be demolished?
payment of arrears, right to form a union Arguments:
I. No. Where will the people residing
and fixed guidelines for sacking an
in such houses live?
employee only in extreme conditions. II. Yes. This will give a clear message

gr
(a) Both Bold statement and to general public and they will
Underlined statement are cause and refrain from constructing
Italic is the effect unauthorized buildings.
(b) Bold statement is the effect while Directions (83): In question below is given a
Italic statement is cause
m
(c) Underlined statement is effect while
statement followed by two assumptions
numbered I and II. An assumption is
Bold statement is cause something supposed or taken for granted. You
(d) Italic Statement is effect while have to consider the statement and the
following assumptions and decide which of
xa
Underlined statement is cause.
the assumptions is implicit in the statement.
(e) Both Bold statement and Give answer-
Underlined statement are effect (a) If only assumption I is implicit
while Italic is cause (b) If only assumption II is implicit
(c) If either I or II is implicit
(d) If neither I nor II is implicit
te

[SBI CLERK MAINS 2018]


81. Statement- Indian Metrological (e) If both I and II are implicit
Department has forecasted that there 83. Statement: “I would like to study the
will be a situation of flood in City A impact of pay revision on job satisfaction
of employees.” — A tells B.
ce

within the next 15 days.


Assumptions:
Course of Action-
I. Job satisfaction can be measured.
I. The population of city A should II. A has necessary competence to
move to city B within next 15 days undertake such study.
to get rid of the problem.
@

II. The people of city A are advised to [IBPS CLERK MAINS 2018]
preserve necessary items for future Direction (84): In the following question a
consumption. statement is given followed by two more
statements numbered as I and II. You have to
(a) Only II (b) Both I and II
read both the statement and decide which of
(c) Only I (d) Neither I nor II them strength which of them strengthens or
(e) None of these weakens the statement.

24 Adda247 Publications For any detail, mail us at


Publications@adda247.com
A Complete Book of Logical Reasoning

The company A used to outsource its II. Teachers started taking more
recruitment process for some years but this holidays after the commencement
year the company has decided to conduct the of the extra classes.
recruitment process within the Organisation.
I. The employees who have been working III. Teachers are already working
in the Organisation are aware of that overtime for checking and
hard work is required to work in the preparing homework for students

p
company. on daily basis.
II. The company has decided to appoint
(a) Both I and II (b) Both II and III
three new persons in its senior level

ou
authority for the smooth recruitment (c) Only II (d) Only III
process. (e) None of these
(a) Both I and II strengthens
(b) I strengthen while II weakens 87. Statement-The Union health ministry
(c) II strengthens while I is neutral has made Aadhaar a compulsory
(d) Statement I and Statement II is neutral. document for tuberculosis patients to be

gr
(e) Both I and II weakens.
able to avail treatment under the
Directions (85-86): Study the following government's Revised National
information carefully and answer the
Tuberculosis Control Programme
questions given below.
The principal of St. Mary school organized (RNTCP).
m
extra classes for week students of X class and (I) A person cannot take treatment of
the concerned teachers are getting paid extra TB without Aadhar card.
amount for the classes. But the average salary (II) Patients suffering from TB will be
of teachers more or less remains the same.
xa
unable to get benefits under a
85. Which of the following can be assumed central government scheme till they
from the given statement?
(I) The students of class X of school Y produce their Aadhaar card.
will score more marks than (III) An individual eligible to receive the
students of St. Mary school. benefit under the scheme, is hereby,
te

(II) Students of class X are not serious required to furnish proof of


about exams.
possession of Aadhaar number or
(III) Extra classes arranged by the
principal for students of class X will undergo Aadhaar authentication.
(IV) The World Health Organisation
ce

help the students to score well in


exams. (WHO) recently said that
(a) Both I and II (b) Both II and III tuberculosis epidemic in India was
(c) Only II (d) Only III
(e) None of these "larger" than what had been
previously estimated.
86. What can be the reason behind the
@

average of the salary remain same of Which of the following could be the
teachers? inference of the given statement?
I. Student of class X attend the extra (a) Only II (b) Only I and III
class two or three days in a week (c) Only II and III (d) Only I
although the teachers provide the
(e) None of these
class regularly.
25 Adda247 Publications For any detail, mail us at
Publications@adda247.com
A Complete Book of Logical Reasoning

88. Statement-The Captain Amarinder introducing a law / scheme / bills


Singh-led government in Punjab has under no circumstance it should not
announced free education for girls in favour any particular gender, if they
government schools and colleges from are really concerned about
Nursery to PhD. education.
(I) Good social welfare law. It is (III) Do not differentiate a poor or
initially for girls as they are average person on gender. A poor

p
comparatively disadvantaged. boy or man should also get free
Hopefully it will soon be followed education as well.
by free education for everyone. Which of the following argument holds

ou
(II) It is gender biased as if a poor or strong for the given statement?
average person wants to educate (a) Only II (b) Only III
his son for higher education he has (c) Only II and III (d) Only I and II
to pay higher bills .If you are (e) All of the above

gr
Previous Year Logical Reasoning : Solutions
1. (d); The issue discussed in the permit such exploratory things
statement is nowhere related to threatening national security. So,
increase in unemployment, as the
m
number of vacancies filled in will
this must not be a probable reason.
3. (d); The time period of insurance
remain the same. Also, in a working
scheme of Company X has not been
place, it is the performance of the
mentioned in the statement. It may
individual that matters and that
xa
also be for 10 years and above 10
makes him more or less wanted,
years. And we cannot compare the
and not his educational
cost of Company X and Y because its
qualifications. So, neither I nor II
mentioning is nowhere in the
holds strong. Besides, the needs of a
passage. So, (i) and (iii) are not the
job are laid down in the desired
reasons behind the decision of the
te

qualifications for the job. So,


owner. But it is quite clear that
recruitment of more qualified
Company Y has provided guarantee
people cannot augment
scheme for more period of time
productivity. Thus, IV also does not
than that of Company X. So (ii) is
ce

hold strong. However, it is the right


the only reason behind the owner’s
of an individual to get the post for
decision.
which he fulfills the eligibility
criteria, whatever be his extra 4. (e); If repo rate goes down, loan rates
merits. Hence, argument III holds will go down. In turn, the value of
strong. rupee will strengthen against other
@

currencies. Consumer Price Index


2. (c); All the options may be the probable
(CPI) measures changes in the price
reasons except option IV because
level of a market basket of
the reason in that option is
consumer goods and services
contradictory to the statement. We
purchased by households; hence it
Indians are matured enough to
should be jumped from its previous

26 Adda247 Publications For any detail, mail us at


Publications@adda247.com
A Complete Book of Logical Reasoning

rates. Reduction in global crude 10. (a); All that the speaker implies is that
prices also plays an important role Kapil is efficient because he has
for the rate cuts by RBI. So, none of twelve years of practice, and so
the options negates the above steps option (a) is correct. (b) is
taken by the RBI. eliminated because the word
5. (e); Clearly, illiterate people lack will ‘identical’ is not mentioned in the
power and maturity in thoughts. paragraph. (c) is eliminated

p
They may easily be misled into false because we are talking about typing
convictions or lured into not the whole ‘job profile’. (d) is
temptations to vote for a particular

ou
eliminated as nothing is mentioned
group. So, argument II holds.
However, a person is literate does about Kapil’s speed and accuracy
not mean that he is conscious of all and we cannot assume efficiency as
political movements, which speed and accuracy. Similarly (e)
requires practical awareness of cannot be the answer as nothing is
everyday events. Thus, I also holds implied about fitting into the office.

gr
strong. Besides, Constitution has
extended the right to vote equally 11. (e); First one is the main motive behind
to all its citizens. Hence, III also the setting up of a task force. And
holds. since it is a task force, so the
6. (d); All the points in the options members can be in-house experts
promote green initiatives except
m
auditing of water consumption as it
as well as from outside. So I and II
are in line in contrast with the
is an examination of system records passage. Third one can be the
and equipment that may be used to reason behind which this task force
xa
identify, quantify and verify how
has been set up. So, III is also in line.
much water passes through the
system and where it goes. 12. (e); The reason given behind her
7. (b); As by focusing on efficiency the resignation in argument I is age
chances of loan recovery will factor which is nothing there in the
increase. So, Option (b) can be statement. Argument II deals with
te

inferred from the above statement the improper functioning in the


as it is given that the government is state which may be the reason
looking to overhaul yearly targets
behind her resignation but it cannot
and control bad debts.
be predicted from the given
ce

8. (c); it is clearly mention that Google has statement. Argument III is absurd.
agreed that Nest’s privacy policies
So, none of the arguments holds
are going to be well- respected
instead of any terms and strong.
conditions. [IBPS PO MAINS 2016]
@

9. (b); The statement talks about cleaning 13. (a); This argument is based on the
the Indian politics from assumption that if the appliances
criminalization. Choice (c) cannot become more efficient, then more
be considered completely. But homes would not be tempted to
choice (b) can be considered as it
increase their utility/consumption
substantiates the conclusion fully.
[RRB PO MAINS 2016] in the household.

27 Adda247 Publications For any detail, mail us at


Publications@adda247.com
A Complete Book of Logical Reasoning

14. (c); Such strict actions of Delhi comparison between the position of
government made private hospitals Messi as a player of Argentina and
to serve their services to poor the position of Messi as a player of
people. So I is implicit. But nothing Barcelona club.
is mentioned about central 17. (a); Govt. promulgate about subsidies
government in the statement II also and benefits to give benefit to
it is a course of action, So, II is not untouched portion of our

p
implicit. Population.
According to statement Delhi Govt. [SBI PO MAINS 2017]
wanted to provide medical facilities
Directions (18-19):

ou
for indigent patients. So we can 18. (d); As stated in statement America is
assume that there is need for the not taking any steps to address the
govt. to monitor and enforce health issue, so to calm down the situation
services to the needy person. Indian Govt. may pressurize
15. (a); Statement II is a valid assumption American Govt. or they may be
impact on Indian-US trade. And as
as tetracycline is produced by a

gr
stated in statement America is not
bacterium which is found in Nubian taking any action despite of racism
soil and the bacterium can develop attack on Indians which results
on the grain which is used in death of two citizens. So it cannot
making bread & beer.so tetracycline be assumed that American Govt.
is formed through the process of
m
making of bread & beer. Statement I
will give life imprisonment
according to the statement.
is not valid because nothing is said 19. (c); All of three supports the negligence
in the passage about the disease of Indian-American Govt. as Modi
other than typhus. Statement III is
xa
Govt. denies replying in same week
also not valid because Typhus is a after racial attack. American Govt. is
disease and the statement talks not ready tackle the issue despite of
about the contamination of a environment of racism is spreading.
disease by another disease which is 20. (a); Statement (a) is the correct option
wrong in the context of this because it clearly strengthen the
te

passage. Statement IV & V is invalid argument that persons with


as No information is given in the sufficient qualification prefer to go
passage that bread & beer were to industrial field rather than to go
their only items in the diet and also in academics as a faculty in
ce

about Typhus is fatal. business schools.


16. (c); Only II is strong argument because 21. (d); Option (d) is an assumption. Any
it clearly explains the freedom appeal has some effects and people
which is given to a person to take generally respond positively to any
appeal.
decisions about his life. So it gives a
@

valid reason. Argument I is a weak 22. (d); Both I and II can’t be assumed from
statement because of the ‘only’ the given statement as statement I
word used in the statement. Messi is vague and II statement is also not
can prove himself by all other ways implicit because it is not clear from
the given statement that whether
also. Argument III is also a weak
people are interested in such news
Statement because there is a or not.

28 Adda247 Publications For any detail, mail us at


Publications@adda247.com
A Complete Book of Logical Reasoning

23. (d); Option (d) contradicts the views For (d)- This can’t be hypothesized
expressed in the statement. from the given statement because
nothing is mentioned about that
Directions (24-25):
older generation is comfortable in
24. (b); The disease occurs at the end of
using mobile phones or not.
monsoons every year. So,
For (e)- This can’t be hypothesized
precautionary measures every four
from the given statement because it
years shall not help. The second

p
is a course of action, it cannot be an
course of action shall be a
assumption.
preventive measure. So, only course
II follows. 29. (d); For (a)-This can’t be assumed from

ou
the given statement because there
25. (e); Clearly, the library needs to be
is no any comparison between
provided with the essential
mosquitoes and other animals in
facilities and trained personnel for
the statement.
better services. So, both the courses
For (b)- This can’t be assumed from
follow.
the given statement because Zika,

gr
26. (c); Statement (c) is the correct option Dengue and Chikungunya are
because according to this, the considered as dangerous diseases
increase in crime rate has been but is not said anything about
contributed by other factors, not treatment of these diseases.
leniency in the punishment.so it
weakens the given passage.
m For (c)- This can’t be assumed from
the given statement there is no
discussion about any stats in the
27. (d); Most logically such comparison
statement.
should reveal mortality rate per
For (d)- This can be assumed from
xa
thousand doctors indulged in SARS
the given statement as it is
treatment and not indulged in
treatment.so statement (d) is the mentioned in the given statement
that Protect yourself and your
correct option.
family from mosquitoes during
RRB PO MAINS 2017]
travelling. so it can be assumed that
te

28. (c); For (a)- This can’t be hypothesized there are more chances to get
from the given statement because infected from mosquitoes while
preference in the sense of travelling .
popularity has been given to mobile For (e)- This can’t be assumed from
ce

instead of T.V. the given statement because there


For (b)- This can’t be hypothesized is only a suggestion that protect
from the given statement because yourself and your family from
there is direct comparison between mosquitoes during traveling , it
T.V and mobile phone in statement. does not mean all travelers are
There is nothing mentioned about suffering from mentioned diseases.
@

outdoor activities in the statement.


30. (c); For (a)- This can’t be the reason
For (c)- This can be hypothesized
behind more number of accidents at
from the given statement as it is
night as population increase could
mentioned in the given statement
that youngsters spend most of their not lead to more number of
accidents during night.
time on mobile phones.

29 Adda247 Publications For any detail, mail us at


Publications@adda247.com
A Complete Book of Logical Reasoning

For (b)- This could not be the education system is geared towards
appropriate reason as preference teaching and testing knowledge
does not matter too much for the whereas given statement states that
reason of death during night time. Education has been a problem in
For (c)- This could be the major our country.
cause behind more number of For (e)- This statement does not
accidents at night as improper substantiates the given statement

p
installation of traffic signal and because it mentioned that We may
lightning arrangement is the major have the most number of
reason behind it. engineering graduates in the world

ou
For (d)- This could not be the major whereas given statement states that
cause of the given statement as the Half the country does not even
traffic rules are same in both day today have access to proper
and night time also. education, and only a small fraction
For (e)- This could not be the major can go to university.
cause behind more number of 32. (b); For (a) It may be the possible

gr
accidents at night as Traffic approach but can’t be the best
violation is a punishable offence in suited option as dowry is like
India. stubborn custom which cannot be
31. (c); For (a)- This statement does not eradicate easily.
For (b) It will be the best suited
m
substantiates the given statement
as it mentioned that Our education option as it is necessary to change
system today encourages excellence the thinking of people but most
whereas given statement states that important thing is to make dowry
Education has been a problem in system as illegal and make it as
xa
punishable offence.
our country.
For (c) It will not make any major
For (b)- This statement does not
changes in the thinking of persons.
substantiates the given statement
For (d) It is right that it has been set
because it mentioned that Our new
in the nerves of people as one of the
education system creates
necessities but it has not suggested
te

entrepreneurs and foundation of a


any action to get out this stubborn
knowledge based economy.
custom.
For (c)- This statement For (e) It will make some impact all
substantiates the given statement over India, as others states will
ce

as it mentioned that India needs follow Mr. Kumar but this step will
internet and technology if it has to not uproot it.
teach all population in remote
villages which is also mentioned in 33. (a); In the above question we have to
the given statement that Half the find which statements do not
country does not even today have support the given statement.
@

access to proper education, and For I-This statement is in line with


only a small fraction can go to the above statement because it
university. supports the given statement as it
For (d)- This statement does not describes that the meeting between
substantiates the given statement Trump and Modi will strengthen
because it mentioned that Our the ties between India and US.

30 Adda247 Publications For any detail, mail us at


Publications@adda247.com
A Complete Book of Logical Reasoning

For II-This statement is not in line (c) Correct. This statement


with the given statement as it states properly identifies the
about Narendra Modi’s visit to Sri weakness in the argument:
lanka whereas the given statement Airplanes are not truly a free-
describes Modi’s first meeting with wheel system because they are
Trump. restricted to traveling between
For III-This statement is also in line airports. Additionally airports
with the given statement as it

p
tend to be less conveniently
describes that the meeting of Modi
and Trump will further make strong located than train terminals,
bilateral relation between both the which has further potential to

ou
countries. weaken the argument in favour
For IV-This is also in line with the of airpl
given statement because this (d) The inability of high-speed
statement states that this meeting trains to use some convenient
will give an opportunity to make train stations strengthens,
strong partnership between India rather than weakens, the

gr
and US. argument in favour of airplanes.
[IBPS PO MAINS 2017] (e) Consumer preference for air
34. (c); Just because of builders which are travel over ground travel on
indulge in profit business without long trips strengthens, rather
caring about environment and poor than weakens, the argument in
m
people which suffers most and dies favor of airpl
due to chronical diseases. And
37. (d); Option (a) cannot be inferred
development of environment has
lack behind due to rapid urban because of the word –“only”. Option
(b) cannot be inferred as we cannot
xa
development. Thus, statement III is
the cause, I and II are its effect. say that the ban was the only factor
that led to an increase in the
35. (c); Note that we are talking about number of glaucoma patients in city
Smart Cities Mission. Thus, Y. Option (c) cannot be inferred as
anything related to rural areas is
the given drug could have been
beyond the scope. Whereas, in
te

option (c), we focus only on urban used for recreational use also.
development. Hence, option (c) will Option (d) can be inferred as the
be an effect of the statement. effectiveness of the given drug in
the treatment of glaucoma could be
36. (c); (a) The method of guidance is
ce

the reason why the ban was lifted.


irrelevant to the argument
about free-wheel versus fixed 38. (d); Litter city has introduced a levy of
linear systems Rs.100 on private cars entering the
(b) The passage compares the city. This levy is in addition to the
speed and system models of toll already being levied. In the
@

airplanes and high-speed trains. passage, the administration argues


The argument does not that this new levy will force people
incorporate buses and cars, from not using their cars and to
which are included only to give take public transport. The
examples of free-wheel system, administration thinks that the
and so this statement is
decisive issue for car owners is
irrelevant.

31 Adda247 Publications For any detail, mail us at


Publications@adda247.com
A Complete Book of Logical Reasoning

saving money. If car owners are 42. (a); Option I strengthen the above
already paying more as toll charges statement as the quality of the
than they would pay in bus fares, education also plays a significant
money is not the reason for their role in the outcome of the results.
using the car. That means car Option II also strengthens as this
owners are unlikely to change their could be a plausible explanation for
way of commuting to save money. high success rate of school X.

p
So, option (d) is the right answer. 43. (d); Statement I is irrelevant as the
Taxi drivers’ demand for rate hike more number of vehicles does not
is irrelevant to the argument, so (a) necessary translate into heavier

ou
is ruled out. Option (b) is inapt pollution. Statement II weakens the
because a comparison with the statement by pointing out that lack
neighboring city doesn’t point to a of water rather than pollution
flaw in the argument. Present bus might be the contributing factor for
riders are irrelevant to the City’s the declining population of
plan. So (c) is also inapt. butterflies in the farm. Statement III

gr
strengthens the statement as it
39. (e); Clear from the way the agitated mentions that butterflies are
guardians have protested against sensitive to pollution.
this newly introduced audio-visual 44. (c); All the given options can be the
aid of teaching methodology, they probable reason as having tiff with
m
are unhappy with the new teaching Losen designer or the impressive
work and quality of the new
methodology.
designer can be the reason of
40. (c); Any new thing takes some time for malaika’s switch to new designer.
xa
its adjustment. So, school authority 45. (d); Option I weakens the argument by
may defend themselves by saying mentioning a point that
this statement. introduction of video lecture might
be beneficial for the college at least
41. (b); The argument in the given text is from the view point of its
that the US president cannot popularity among the students.
te

sustain his legacy because of the Option II strengthens the argument


by mentioning a fact that proves the
slow progress of his initiative, i.e.
popularity of the college in not
‘securing nuclear materials’, which, determined by the availability of
ce

in other words, means that if he can the video lecture facility.


speed up his initiative, he can have
46. (e); It is mentioned in the statement
a lasting glory. So, the underlying that the addition of new variety
assumption here is that whether cake will turn ABC into highest
one has a lasting glory or not seller which means it will increase
@

depends on how fast their the sales of ABC Company and the
revenue is directly related to the
initiatives progress. The
sales so IV can be inferred.
assumption comes out only in Statement II and III cannot be
option (b). Hence, it is the right inferred as it is not related to the
answer. statement. Statement I cannot be
[CANARA PO 2018] inferred because of the word only.
32 Adda247 Publications For any detail, mail us at
Publications@adda247.com
A Complete Book of Logical Reasoning

47. (d); Option I cannot be inferred as due to reduced transportation cost


nothing is mentioned about the of the vegetables. Option III also
overall sales of both the companies. opposes the statement by pointing
Option II cannot be inferred as we out other alternative options.
don’t know if the midrange phone Option IV mentions some incidents
launched by both the companies is a which are unrelated to the
smartphone or a retro phone. statement.

p
Doubling up of sales just after the
51. (a); Both Statement (I) and (II) are the
launch of the phone clearly suggests
neutral statement as more number
that there is a greater demand for
of employees in any company can’t

ou
that phone as compared to other
be the reason of more number of
products sold by ABC Company.
resignations in any company and
Option IV cannot be inferred as we
statement (II) is also the neutral
don’t have any information about
statement as the number of
the sales of XYZ Company after the
department in any company does
launch of the phone, so both the
not relate with the resignation of

gr
companies cannot be compared.
employees.
48. (a); The fact that ABC Company is [SYNDICATE PO 2018]
working on a long-range wireless
52. (b); As per the given in the statement it
charging system clearly suggest
can be assumed that the people of
that there is a demand amongst
m
public for such a device, so I can be
city Y are facing some problem
which is the scarcity of water that is
concluded. Option II cannot be
why the population of City Y
inferred as we don’t know the
relocated to city Z but the second
implications of the technology on
xa
cannot be assumed as the
the environment. Option III cannot
availability of more resources in
be concluded as it is a hypothetical
city Z cannot be the reason of
assumption.
shifting of all the population from
49. (b); The fact that Puzzolinia export its one city to another. Because moving
ashes to other country implies that one city to another of all the
te

she has surplus amount of it. So, I is population is not an easy task.
implicit. Export import activites are
53. (a); From the given statement it is clear
not necessarily carried between
neighbouring countries only. So II is that the efforts of Sahara NGO alone
ce

not implicit. III is not implicit are not sufficient for educating the
because we don’t know if the export tribal people of Satava district so it
of volcanic ashes forms a major or a surely requires the support of the
small part of Puzzolinia’s exports. government so II follows. But
50. (b); The fact that vegetable vendors are shifting the people of that district to
@

illegally using the land provides a another district or city is not the
legal justification proposal of solution of that problem. As the
acquisition by the government. So problem of lack of education will
option I supports the statement. remain the same in that district in
Option II opposes the argument by future also so I does not follow.
pointing out the financial savings

33 Adda247 Publications For any detail, mail us at


Publications@adda247.com
A Complete Book of Logical Reasoning

54. (b); Statement I and III cannot be plastic is biodegradable nor non-
concluded from the given statement biodegradable. Hence, I is not
as nothing is given about it in the implicit. But II is obviously implicit.
given statement. Statement II can That is why the scientist uses the
be concluded from the given word ‘Beware’ in his statement
statement as it is clearly mention in before asserting the negative
the given statement that by features of plastic.

p
increasing the entry fees Adventure
59. (a); Option (a) may be the cause of
amusement park can perform the
vacant seat in the engineering
maintenance work of the park

ou
properly. Statement IV can also be colleges because due to the
concluded from the given statement recession, the number of jobs
as the proper maintenance will lead available are decreasing day by day,
to increase in the number of visitors hence the unemployment is
which is also mentioned in the increasing in the engineering
given statement. sector. So (a) is the correct option.

gr
55. (b); As weather conditions are different 60. (b); Option (b) may be a possible effect
in different countries so it is not of big pig holes developed on the
necessary that crop yield will be roads.
same. As fertility of soil matters a
m
lot so hybrid seeds should not be
used. Costing for farmers also
61. (a); It is clear from the above that
statement I is the cause and II is it’s
matters a lot. effect as this will be the cause that
56. (c); As this provision which is justified Infosys is planning to hire workers
xa
was in previous company of new and for that it will hire graduates
manager, so it may be reason that and will create talent pools will be
they want to apply in his new it’s effect.
company too. As employee which
62. (b); It is clear from the above statement
left since last year had same
that II is cause and I is it’s effect
problem so to avoid such incidents
te

in future, he may want to apply this because the PAN has become more
new provision. vulnerable that is why Aadhar has
become mandatory for filing
57. (e); As number of seats for women has Income Tax.
ce

been increased by 20, so to


maintain balance number of total 63. (a); (i) most accurately defines term
seats has been increased by 10%. “implicit bias” as the word
And to improve hospitality automatic was used which is
condition, number of seats has been similar to the term “unintentionally
increased
@

racist” used by researchers.


[SBI PO MAINS 2018] [RRB PO MAINS 2018]
58. (c); Non –biodegradable nature of
64. (e); The Act is aimed at eradication of
plastic bag can’t be correlated with
the disease and so it needs to be
the statement because the
statement does not say whether proclaimed and promoted. So, both
the courses follow.
34 Adda247 Publications For any detail, mail us at
Publications@adda247.com
A Complete Book of Logical Reasoning

65. (b); Only II is implicit as it is given in the 70. (b); Course of action I follows because
statement that both the companies with a graded approach to the use
signed the contract for not of antibiotics, under which some
revealing the details of their medicines are reserved for the most
company with the rivals but
resistant microbes, the WHO list
company A may have done this and
breached the contract, so the can stop their misuse as broad-
spectrum treatments. Course of

p
company B sent the legal notice to
company A. action II also follows because a
Directions (66-67): close inspection is also needed of

ou
66. (e); As from the given statement we can the national supervisory
assume that the notice on the gate programmes to help people to get
of ‘M’ housing society will be rid out of it.
followed by all the sales persons.
Also it can be assumed that the 71. (c); Statement (i) and (ii) states a fact
security guards are not be able to that supports this buying frenzy

gr
stop so the requirement of notice is being carried out by aviation
emerged. Since both the companies. Whereas (iii) clearly
assumptions follow from the given mentions that lack of infrastructure
statement, so both I and II are will be a major challenge that
implicit.
aviation companies might have to
67. (e); It is mentioned that farmers will be
in trouble without rain. This means
m face in future.

that timely rain is essential. Also, it 72. (e); (i) states a fact that this test will
shows that farmers are dependent determine the future career of the
xa
on rain. So, both I and II are students and thus have a significant
implicit. importance for them. The fact that
68. (b); Only B is concluded from the given parents are spending a lot of money
statement as it is given clearly in to ensure the comfort of their
the statement that the CMC children and the students spending
products are popular in city X but
te

money on supplements signifies the


nothing is mentioned about city Y importance of this test, So (ii) and
or the quality and prices of CMC
(iii) substantiates the statement.
products, so these can’t be
concluded. And also we cannot 73. (c); (i) is not an appropriate course of
ce

conclude anything about the rival action as it not known from the
companies.
statement weather the protest are
69. (a); Both I and II can be inferred from violent or peaceful. (ii) is
the given statement as both the preventive measure not a
quality and price are the factors corrective action. As it is mentioned
@

which can affect the demand so I


in the statement that the given state
and II can be inferred. But III and IV
cannot be inferred as nothing is has faced several drought like
mentioned about the liking of situations over the recent years, so
people of city X for foreign products (iii) is an appropriate course of
or their need of healthy products. action.
[IBPS PO MAINS 2018]
35 Adda247 Publications For any detail, mail us at
Publications@adda247.com
A Complete Book of Logical Reasoning

74. (a); (i) “However a new study” implies 78. (d); Only IV, is incorrect as it is
that there might have been some mentioned in above statement that
studies regarding this in the past green crackers are cost effective
and “proven to be healthy”. So, (i) is whereas the (IV) statement states
definitely true. (ii) cannot be that green fire crackers are quite
deduced as smoking is not expensive.
mentioned in the statement. (iii) is 79. (d); Statement (II) is true as the

p
vague and not in context with the psychological and emotional
statement. quotient of human beings can only
be the reason of less spending of

ou
75. (e); All the given statements are the second group as compared to the
examples of Promos Marketing as first one.
all examples clearly support the fact
that people prefer those products 80. (b); The statement which is bold is the
effect and the one which is italic is
they are more familiar with as they
cause. As the demands of the
built a trust in it so they used to buy

gr
employees of Delhi metro is the
it again. cause and the strike is the effect.
76. (d); For (a), Raw material plays an [SBI CLERK MAINS 2018]
important role in production of a 81. (b); Both course of action should be
product and adequate funds are followed. For I, As the given
also required for it but there are
m statement states that ministry has
alternatives available for it such as forecasted the situation of flood, so
loans to fulfill the requirement so the people of city A should move to
(a) is not the most possible reason. city B as a preventive action. Also as
xa
Employee satisfaction and proper an impact of flood here will be a
machinery plays a vital role in lack of necessities so the people of
growth of the company but again city A should preserve these things.
both of these can be replaced or 82. (b); The demolition of unauthorized
modified. So, (b) and (c) do not buildings would teach a lesson to
te

follow. Statement (e) is vague. So, the unscrupulous builders and also
the most possible reason is (d) as serve as a warning for the citizens
the demand has been declined from not to indulge in such activities in
past 5 years hence the production the future. This is essential, as
unauthorized constructions impose
ce

also declined.
undue burden on the city’s
77. (d); Only (d) can be inferred from the infrastructure. So, only argument II
given statement as the given holds strong.
statement suggest that the victory 83. (e); Clearly, A wishes to study the
of Trump is more a revenge against degree of effect of pay revision on
@

the corrupt ones and moreover job satisfaction of employees. This


Trump was more focusing on means that job satisfaction can be
working class public’s issue measured and A is capable of
whereas Hillary was more making such a study. So, both I and
concerned on fascinating people by II are implicit.
famous personalities. [IBPS CLERK MAINS 2018]

36 Adda247 Publications For any detail, mail us at


Publications@adda247.com
A Complete Book of Logical Reasoning

84. (c); Statement I is neutral as it is For III-This also could be the


nowhere related to the recruitment inference because it is clear from
process. While II strengthen the the given statement that to get
statement as the company is taking benefits of the scheme verification
this step to strengthen the of Aadhar will be required.
recruitment process.
For IV-This is not the inference of
85. (d); Only Statement III is assumed from

p
the given statement because this
the given statement. As from given statement states that cases of TB
statement it can be assumed that patients increased in India now
the extra class will help the

ou
which is not directly related to the
students to score well.
statement.
86. (b); Both II and III can be the reason for
the average salary remain same. 88. (c); For I- This argument does not holds
strong because this initiative is for
87. (c); In the above question we have to
girls to get benefited initially but it
find the inference of the above

gr
statement. is not like that education will be
For I-This cannot be the inference free for everyone.
because if a person does not have For II-This argument also holds
Aadhar, he/she will not get strong because a law/scheme or bill
government help in treatment but
can take treatment of TB without
m should not be for any particular
gender but it should be for the one
Aadhar. who really need it.
For II-This could be the inference as For III-This is also strong because
mentioned in the given statement
xa
gender should not be the criterion
that patients need Aadhar card to
for the free education. It is the poor
get benefits under government
scheme. who should get the benefits.
te
ce
@

37 Adda247 Publications For any detail, mail us at


Publications@adda247.com

You might also like